You are on page 1of 469

DeVita, Heilman, and Rosenberg's

Canr ni
Principles 8c Practice of Oncology

REVIEW
5th edition

Daniel Morgensztern
Ramaswamy Govindan
Siddhartha Devarakonda
Mikolaos A. Trikalinos
flkWoltersKLuwer
DeVita, Heilman, and Rosenberg's

Cancer
Principles & Practice of Oncology

REVIEW

https://t.me/ALGRAWANY33
DeVita, Heilman, and Rosenberg's

Cancer
Principles & Practice of Oncology

REVIEW
5 th Edition
Editors

Daniel Morgensztern, MD Siddhartha Devarakonda, MD


Professor of Medicine Assistant Professor off Medicine
Division ot Oncology Division of Medical Oncology
Alvin |, Siteman Cancer Center Washington University School of Medicine
Washington University School of Medicine .
St, Louis Missouri
St . Louis, Missouri
Nikolaos A. Trikalinos, MD, MS
Ramaswamy Govindan , MD Assistant Professor of Medicine
Professor of \ Icdiciue Division of Medical Oncology
Anlicnscr linscli Chair m Medical Oncology Washington University School of Medicine
Director, Section of Medical Oncology St . Louis, Missouri
Division of Oncology
Washington University School of Medicine
St. Lonis, Missouri

)
^* Wolters Kiuwer
®

Philadelphia * Baltimore * New Yorfc * London


Buenos Aires * Hong Kong Sydney * Tokyo
1
Acquisitions Editor: Nicole Dernoski
Senior Product Development Editor: Stacey Sebring
Editorial Coordinator: Anju Radhakrishnan
Production Project Manager: Bridgett Dougherty
Design Coordinator: Steve Druding
Manufacturing Coordinator: Beth Welsh
Marketing Manager: Kristin Ciotto
Prepress Vendor: S4Carlisle Publishing Services
5th edition
Copyright © 2022 Wolters Kluwer.
4th edition Copyright © 2016 Wolters Kluwer. 3rd edition Copyright © 2012 and 2nd edition
Copyright © 2009 by Lippincott Williams & Wilkins, a Wolters Kluwer business. Copyright © 2005
by Lippincott Williams & Wilkins.
All rights reserved. This book is protected by copyright. No part of this book may be reproduced or
transmitted in any form or by any means, including as photocopies or scanned-in or other electronic
copies, or utilized by any information storage and retrieval system without written permission from the
copyright owner, except for brief quotations embodied in critical articles and reviews. Materials
appearing in this book prepared by individuals as part of their official duties as U.S. government
employees are not covered by the above-mentioned copyright. To request permission, please contact
Wolters Kluwer at Two Commerce Square, 2001 Market Street, Philadelphia, PA 19103, via email at
permissions@lww.com, or via our website at shop.lww.com (products and services).
987654321
Printed in China
Library of Congress Cataloging-in-Publication Data
ISBN-13: 978-1-975151-88-1
ISBN-10: 1-975151-88-7
Library of Congress Control Number: 2021940369

This work is provided “as is,” and the publisher disclaims any and all warranties, express or implied,
including any warranties as to accuracy, comprehensiveness, or currency of the content of this work.
This work is no substitute for individual patient assessment based upon healthcare professionals’
examination of each patient and consideration of, among other things, age, weight, gender, current or
prior medical conditions, medication history, laboratory data, and other factors unique to the patient.
The publisher does not provide medical advice or guidance and this work is merely a reference tool.
Healthcare professionals, and not the publisher, are solely responsible for the use of this work including
all medical judgments and for any resulting diagnosis and treatments.
Given continuous, rapid advances in medical science and health information, independent professional
verification of medical diagnoses, indications, appropriate pharmaceutical selections and dosages, and
treatment options should be made and healthcare professionals should consult a variety of sources.
When prescribing medication, healthcare professionals are advised to consult the product information
sheet (the manufacturer's package insert) accompanying each drug to verify, among other things,

https://t.me/ALGRAWANY33
conditions of use, warnings and side effects and identify any changes in dosage schedule or
contraindications, particularly if the medication to be administered is new, infrequently used or has a
narrow therapeutic range. To the maximum extent permitted under applicable law, no responsibility is
assumed by the publisher for any injury and/or damage to persons or property, as a matter of products
liability, negligence law or otherwise, or from any reference to or use by any person of this work.
shop.lww.com
DEDICATION
To our contributors.

https://t.me/ALGRAWANY33
CONTRIBUTORS

Foluso C. Ademuyiwa, MD, MPH


Associate Professor
Department of Medicine
Washington University School of Medicine
St. Louis, Missouri

Douglas R. Adkins, MD
Professor
Department of Internal Medicine
Washington University School of Medicine
St. Louis, Missouri

Olivia Aranha, MD, PhD


Assistant Professor
Internal Medicine, Division of Oncology
Siteman Cancer Center
St. Louis, Missouri

Matthew Austin, MD
Fellow, Medical Oncology
Medical Oncology Section, Department of Internal Medicine
Yale School of Medicine, Yale University
New Haven, CT

Dhruv Bansal, MD, MBA


Fellow
Department of Oncology
Washington University School of Medicine
St. Louis, Missouri

Ron Bose, MD, PhD


Associate Professor of Medicine
Division of Medical Oncology
Washington University School of Medicine
St. Louis, Missouri

Omar Hameed Butt, MD, PhD


Fellow
Department of Neurology
Washington University School of Medicine
St. Louis, Missouri

Jian L. Campian, MD, PhD


Assistant Professor of Medicine and Neurosurgery
Internal Medicine, Division of Medical Oncology
Washington University School of Medicine
St. Louis, Missouri

Katherine Clifton, MD
Assistant Professor
Department of Medicine
Washington University School of Medicine
St. Louis, Missouri

Siddhartha Devarakonda, MD
Assistant Professor of Medicine
Division of Medical Oncology
Washington University School of Medicine
St. Louis, Missouri

Haley Ellis, MD
Resident Physician
Department of Medicine
Washington University School of Medicine
St. Louis, Missouri

Armin Ghobadi, MD
Associate Professor
Department of Medicine
Washington University School of Medicine
St. Louis, Missouri

Scott R. Goldsmith, MD
Fellow

https://t.me/ALGRAWANY33
Division of Oncology
Washington University School of Medicine
St. Louis, Missouri

Ramaswamy Govindan, MD
Professor of Medicine
Anheuser Busch Chair in Medical Oncology
Director, Section of Medical Oncology
Division of Oncology
Washington University School of Medicine
St. Louis, Missouri

Patrick Grierson, MD, PhD


Assistant Professor
Department of Medical Oncology
Washington University School of Medicine
St. Louis, Missouri

Sasha Haarberg, PharmD, BCOP


Clinical Oncology Pharmacist
Division of Medical Oncology
Washington University School of Medicine
St. Louis, Missouri

Jennifer Hedgecorth, PharmD, BCOP


Clinical Oncological Pharmacist
Division of Medical Oncology
Washington University School of Medicine
St. Louis, Missouri

Leonel Hernandez-Aya, MD
Assistant Professor
Division of Medical Oncology
Washington University School of Medicine
St. Louis, Missouri

Brett H. Herzog, MD, PhD


Fellow
Department of Medicine
Washington University School of Medicine
St. Louis, Missouri
Angela C. Hirbe, MD, PHD
Assistant Professor of Medicine
Assistant Professor of Pediatrics
Division of Medical Oncology
Washington University School of Medicine
St. Louis, Missouri

Shu (Joy) Jiang, PhD


Assistant Professor
Division of Public Health Sciences
Washington University School of Medicine
St. Louis, Missouri

Ramon Jin, MD, PhD


Clinical Fellow
Division of Oncology
Department of Internal Medicine
Washington University School of Medicine
St. Louis, Missouri

Jessica Ley, BS
Clinical Research Specialist
Department of Medical Oncology
Washington University School of Medicine
St. Louis, Missouri

Joshua T. Loesche, PharmD


Clinical Oncological Pharmacist
Division of Medical Oncology
Washington University School of Medicine
St. Louis, Missouri

Esther Lu, MS, PhD


Assistant Professor
Department of Surgery
Washington University School of Medicine
St. Louis, Missouri

Cynthia X. Ma, MD, PhD


Professor of Medicine
Division of Medical Oncology

https://t.me/ALGRAWANY33
Washington University School of Medicine
St. Louis, Missouri

Prabhat Singh Malik, MD, DM


Associate Professor
Department of Medical Oncology
All India Institute of Medical Sciences
New Delhi, India
(Visiting Scholar, Washington University School of Medicine, November 2019)

Janelle E. Mann, PharmD, BCOP


Manager, Clinical Pharmacy Services
Clinical Oncological Pharmacist
Department of Medicine
Washington University School of Medicine
St. Louis, Missouri

Neha Mehta-Shah, MD, MSCI


Assistant Professor of Medicine
Division of Medical Oncology
Washington University School of Medicine
St. Louis, Missouri

Daniel Morgensztern, MD
Professor of Medicine
Division of Oncology
Alvin J. Siteman Cancer Center
Washington University School of Medicine
St. Louis, Missouri

Ashley E. Morton, MSN, ANP-BC


Nurse Practitioner
Division of Oncology, Section of Medical Oncology
Department of Internal Medicine
Washington University School of Medicine
St. Louis, Missouri

David G. Mutch, MD
Vice Chairman of Gynecology
Division of Gynecologic Oncology
Department of Obstetrics and Gynecology
Washington University School of Medicine
St. Louis, Missouri

Peter Oppelt, MD
Assistant Professor
Department of Medicine
Washington University School of Medicine
St. Louis, Missouri

Russell K. Pachynski, MD
Assistant Professor
Division of Oncology
Department of Medicine
Washington University School of Medicine
St. Louis, Missouri

Kevin T. Palka, MD
Assistant Professor
Department of Internal Medicine
Washington University School of Medicine
St. Louis, Missouri

Haeseong Park, MD, MPH


Assistant Professor
Internal Medicine, Division of Oncology
Washington University School of Medicine
St. Louis, Missouri

Katrina S. Pedersen, MD, MS


Assistant Professor of Medicine
Washington University School of Medicine
St. Louis, Missouri

Matthew A. Powell, MD
Associate Professor
Department of Obstetrics and Genecology
Washington University School of Medicine
St. Louis, Missouri

Elizabeth Prsic, MD
Associate Professor

https://t.me/ALGRAWANY33
Department of General Internal Medicine
Yale University
New Haven, Connecticut

William Rafelson, MD
Division of Hematology/Oncology
Department of Medicine
Alpert Medical School, Brown University
Providence, Rhode Island

David Russler-Germain, MD, PhD


Hematology/Oncology Fellow
Department of Internal Medicine
Barnes-Jewish Hospital
St. Louis, Missouri

Benjamin R. Tan, Jr, MD


Associate Professor
Department of Internal Medicine
Washington University School of Medicine
St. Louis, Missouri

Nikolaos A. Trikalinos, MD, MS


Assistant Professor of Medicine
Division of Medical Oncology
Washington University School of Medicine
St. Louis, Missouri

Danielle K. Turlington, PharmD, BCOP


Clinical Oncological Pharmacist
Division of Medical Oncology
Department of Medicine
Washington University School of Medicine
St. Louis, Missouri

Brian A. Van Tine, MD, PhD


Associate Professor of Medicine
Associate Professor of Pediatrics
Division of Medical Oncology
Washington University School of Medicine
St. Louis, Missouri
Ravi Vij, MD, MBA
Professor of Medicine
Division of Medical Oncology
Washington University School of Medicine
St. Louis, Missouri

Fei Wan, PhD


Assistant Professor
Department of Surgery
Washington University School of Medicine
St. Louis, Missouri

Saiama N. Waqar, MBBS, MSCI


Associate Professor
Internal Medicine, Division of Oncology
Washington University School of Medicine
St. Louis, Missouri

Alice Zhou, MD, PhD


Hematology Oncology Fellow
Divisions of Hematology and Oncology
Washington University School of Medicine
St. Louis, Missouri

https://t.me/ALGRAWANY33
PREFACE

The past decade has witnessed numerous advances in cancer therapy. Even
since the publication of the previous edition of Cancer: Principles & Practice
of Oncology (PPO), or simply known as the “DeVita book,” several new
drugs have been approved for cancer therapy. Immunotherapy plays a
significant role in many advanced malignancies. Cancer Genome Sequencing
projects increasingly incorporate proteomics and single-cell sequencing.
Molecular mechanisms that underline the course of several cancer types and
responses to specific therapies are understood better than before. This
companion review book, now in its fifth edition, is an attempt to cull out the
key learning points from the massive tome of the “DeVita book” that
captures all these advances in a timely manner. While these review books are
often seen as “study aids” for last-minute cramming for the board
examinations, we hope this book would serve to highlight key points from
each chapter of PPO. Each chapter in the review book corresponds to one or
more chapters in the main textbook, just as they were in the first edition. We
hope you find this book valuable and informative. Please do not hesitate to
contact us with comments, criticisms, and suggestions. You can reach us by
e-mail at rgovindan@wustl.edu or danielmorgensztern@wustl.edu.
Ramaswamy Govindan, MD
Daniel Morgensztern, MD
PREFACE TO PREVIOUS EDITION

We are pleased to publish the fourth edition of the companion review book
for DeVita, Hellman, and Rosenberg’s Cancer: Principles and Practice of
Oncology, 10th edition (PPO). While these review books are often seen as
study aids for last-minute cramming for the board examinations, we hope this
book will enable readers to learn key points from each chapter of the PPO
textbook. Each chapter in the review book corresponds to one or more
chapters in the main textbook just as they were in the first three editions. We
hope you find this book useful and informative. Please do not hesitate to
contact us with comments, criticisms, and suggestions.
Ramaswamy Govindan, MD
Daniel Morgensztern, MD

https://t.me/ALGRAWANY33
ACKNOWLEDGMENTS

I would like to extend my sincerest thanks to Dr. Govindan for the


opportunity to contribute to this book and his mentorship, my coeditors,
contributors, and publishers for the time and effort they have dedicated to this
project. I am also grateful to my family for their unconditional love and
support.
Siddhartha Devarakonda

I would like to extend my gratitude to Dr. Ramaswamy Govindan for


including me in this ambitious project and to the coeditors Dr. Morgensztern
and Dr. Devarakonda for their guidance and constructive input. Also, many
thanks to the talented physicians/scientists of Washington University in St.
Louis who graciously contributed to the chapters of this book.
Nikolaos A. Trikalinos

I want to thank the contributors for their diligence, time, and patience. My
special thanks to my coeditors for their hard work, dedication, and
commitment to make this project successful. Special thanks to Dr.
Morgensztern for coordinating manuscript submission and revision. I am
grateful to our contributors for their support.

I want to thank the staff at Wolters Kluwer for their support. Needless to say,
these projects take a sizable amount of time away from family. I will always
be grateful to my wife Prabha and my two children, Ashwin and Akshay.
Ramaswamy Govindan

I would like to thank my mentor Ramaswamy Govindan, my colleagues and


new editors Sid Devarakonda and Nikos Trikalinos, and the authors for their
excellent chapters. I am also very grateful to my parents Silvia and Felipe,
my lovely wife Marcela, and my three delightful children Alan, David, and
Michael.
Daniel Morgensztern

https://t.me/ALGRAWANY33
CONTENTS

Contributors
Preface
Preface to Previous Edition
Acknowledgments

1. Molecular Biology of Cancer


David Russler-Germain and Olivia Aranha

2. Design and Analysis of Clinical Trials


Esther Lu, Fei Wan, and Shu (Joy) Jiang

3. Cancer Therapy
Joshua T. Loesche, Janelle E. Mann, and Danielle K. Turlington

4. Central Nervous System Neoplasms and Brain Metastases


Alice Zhou, Omar Hameed Butt, and Jian L. Campian

5. Cancer of the Head and Neck


Peter Oppelt, Kevin T. Palka, Jessica Ley, and Douglas R. Adkins

6. Thoracic Cancers
Brett H. Herzog, Haley Ellis, Prabhat Singh Malik, Saiama N. Waqar, Siddhartha Devarakonda,
Ramaswamy Govindan, and Daniel Morgensztern

7. Breast Cancer
Katherine Clifton, Foluso O. Ademuyiwa, Ron Bose, and Cynthia X. Ma

8. Esophageal and Gastric Cancer


Ramon Jin and Haeseong Park

9. Small Intestine Cancers and Gastrointestinal Stromal Tumors


Katrina S. Pedersen and Nikolaos A. Trikalinos

10. Pancreas and Hepatobiliary Cancer


Patrick Grierson and Katrina S. Pedersen
11. Colorectal and Anal Cancer
Ashley E. Morton and Benjamin R. Tan, Jr.

12. Genitourinary Cancer


Russell K. Pachynski

13. Skin Cancer


Leonel Hernandez-Aya

14. Sarcoma
Brian A. Van Tine and Angela C. Hirbe

15. Endocrine and Neuroendocrine Tumor


Nikolaos A. Trikalinos

16. Gynecologic Cancer


Matthew A. Powell and David G. Mutch

17. Leukemias and Myelodysplastic Syndrome


Dhruv Bansal and Armin Ghobadi

18. Lymphomas
Neha Mehta-Shah

19. Plasma Cell Neoplasms


Ravi Vij and Scott R. Goldsmith

20. Cancer Screening


Olivia Aranha

21. Cancer Pain


Elizabeth Prsic and William Rafelson

22. Oral and Gastrointestinal Complications


Sasha Haarberg and Jennifer Hedgecorth

23. Oncologic Emergencies


Elizabeth Prsic and Matthew Austin

Index

https://t.me/ALGRAWANY33
1 Molecular Biology of Cancer
David Russler-Germain and Olivia Aranha

QUESTIONS

Each of the numbered items below is followed by lettered answers. Select the
ONE lettered answer that is BEST in each case unless instructed otherwise.

Question 1.1 Compared with mutations in tumor suppressor genes,


mutations in oncogenes are most commonly of what type?
A. Nonsense
B. Missense
C. Loss of heterozygosity
D. Frameshift
Question 1.2 What is the estimated range of somatic coding alterations per
cancer genome?
A. 1–5
B. 10–200
C. 1,000–2,000
D. >5,000
Question 1.3 Mutations in which pair of genes would most commonly be
found to co-occur within the same cancer genome?
A. KRAS and BRAF
B. PIK3CA and PTEN
C. IDH1 and IDH2
D. TP53 and RB1
Question 1.4 To be called a “driver” mutation, a specific nonsynonymous
coding deoxyribonucleic acid (DNA) variant must be present in what
proportion of cases of a given cancer?
A. 50%–100%
B. 25%–100%
C. 10%–50%
D. No range applies
Question 1.5 Circulating tumor DNA (ctDNA) typically makes up what
portion of cell-free DNA in a cancer patient’s plasma?
A. <5%
B. 5%–25%
C. 25%–50%
D. >50%
Question 1.6 Which of the following is thought of as the classical example
of a preexisting resistance mutation?
A. FLT3-ITD
B. KIT T670I
C. ABL T315I
D. EGFR T790M
Question 1.7 Chromothripsis is defined by which of the following
characteristics?
A. Massive genomic rearrangement
B. A multistep catastrophic event
C. Release and incorporation of exosomal DNA
D. None of the above
Question 1.8 Chromoplexy is defined by which of the following
characteristics?
A. Absence of DNA translocations and deletions

https://t.me/ALGRAWANY33
B. Two canonical single chromosomal rearrangements
C. Interdependent DNA changes that result in the coordinated disruption
of multiple cancer genes
D. Microsatellite instability
Question 1.9 Which of the following is the canonical consequence of
oncogenic mutations in RAS proteins?
A. Impaired intrinsic GTPase activity leading to a disrupted
autoregulatory negative-feedback mechanism
B. Increased nuclear translocation
C. Increased kinetics of signaling transduction
D. Impaired posttranslational modifications leading to increased protein
abundance
Question 1.10 In what ways inflammation from tumor necrosis is thought
to promote tumor growth?
A. Promote angiogenesis
B. Inhibit recognition of neoantigens on the tumor cell surface
C. Decrease proliferation of the remaining viable tumor cells in the local
vicinity, protecting them from chemotherapy
D. Trigger massive genomic instability in remaining viable tumor cells
Question 1.11 Which of the following is not an antiapoptotic protein?
A. Bcl-2
B. Bcl-xL
C. Mcl-1
D. Bax
Question 1.12 Tumor-associated macrophages (TAMs) are known to do
which of the following?
A. Foster local invasion by supplying matrix-degrading enzymes such as
metalloproteinases
B. Stimulate growth by supplying signaling pathway ligands such as
epidermal growth factor (EGF)
C. Promote intravasation into the circulatory system and metastatic
dissemination of the cancer cells
D. All of the above
Question 1.13 The principle of self-seeding describes what process?
A. The supportive stroma arising in a primary tumor provides a hospitable
site for reseeding and colonization by circulating cancer cells released
from metastatic lesions.
B. DNA containing newly acquired oncogenic mutations can be released
from cancer cells, then taken up and integrated into the genomes of
other nearby cancer cells in a single tumor.
C. Tumors arise from a single cell that acquires multiple oncogenic
mutations that circulate through the bloodstream before identifying an
ideal niche to replicate and produce a tumor.
D. Second primary malignancies are more likely in patients with
metastatic cancers than in those with nonmetastatic cancers.
Question 1.14 Which immune system deficiency confers the greatest risk
of malignancies?
A. Absence of neutrophils and macrophages
B. Absence of macrophages and platelets
C. Absence of T and natural killer (NK) cells
D. Absence of plasma cells and NK cells
Question 1.15 Polymerase chain reaction (PCR)-based methods are best for
detecting which of the following genetic changes in a malignancy?
A. JAK2 V617F and FLT3-ITD
B. Deletion of 5q
C. EWSR translocation
D. t(14:18) translocation
Question 1.16 Which of the following is true regarding ctDNA?

https://t.me/ALGRAWANY33
A. A larger proportion of patients have detectable ctDNA as cancer stage
increases.
B. The average concentration of ctDNA in plasma decreases with
increasing cancer stage.
C. ctDNA detection is not related to cancer stage.
D. ctDNA detection is only possible if there are enough circulating tumor
cells (CTCs) in circulation.
Question 1.17 The Cancer Genome Atlas (TCGA) initial project focused
on the following tumors:
A. Glioblastoma (GBM)
B. Serous cystadenocarcinoma of the ovary
C. Lung squamous carcinoma
D. All of the above
Question 1.18 The idea of genetically analyzing pathways rather than
individual genes has been applied previously, revealing the concept of mutual
exclusivity. Mutual exclusivity has been shown in the following cases:
A. KRAS and BRAF
B. PIK3CA and CDKN2A
C. TP53 and RB1
D. EGFR and TP53

ANSWERS

Question 1.1 The correct answer is B.


Tumor suppressors are genes that normally function to slow down cell
division, repair DNA, or trigger cells to die, such that loss of their
normal function can lead to the development of cancer. Nonsense and
frameshift mutations generate premature stop codons and errors in
translation, respectively, that typically lead to reduced or absent protein
abundance or function. Loss of heterozygosity leads to similar
consequences. In contrast, oncogenes typically contribute to cancer
through aberrantly increased levels of their normal function. They are
frequently affected by activating missense mutations of particular
residues fundamental to their normal functions.
Question 1.2 The correct answer is B.
In a landmark study, the analysis of the protein-coding genome in
several breast and colorectal cancer samples found that individual
tumors accumulate an average of approximately 90 mutant genes.1
Broader analyses as part of TCGA over the next decade used updated
technologies to elucidate that most cancers possess between 1 and 10
driver mutations, with additional nondriver somatic coding mutations
also being present.
Question 1.3 The correct answer is D.
Tumor development is driven by mutations that lead to dysregulation of
cellular proliferation, among the many other hallmarks of cancer.
Mutations that are synergistically advantageous to a tumor will be
selected for evolutionarily. Interestingly, data from cancer genome
sequencing projects revealed that mutations in many cancer driver genes
in the same molecular pathway are often mutually exclusive, thought to
be due to the functional redundancy of these mutations, with a potential
secondary mechanism being that coexpression of two oncogenes can be
harmful to the point of triggering cell cycle exit, senescence, or death.
Of these pairs of genes, only TP53 and RB1 mutations significantly co-
occur in the same cancer genome.
Question 1.4 The correct answer is D.
Large-scale genomic profiling studies in cancer have produced extensive
sets of validated nonsynonymous DNA sequence variants. An ongoing
challenge is how to distinguish “driver” mutations that are responsible
for malignant transformation from nonfunctional “passenger” mutations
that arise in an already-malignant clone, accumulating during cell
division. Statistical models exist to call “driver” genes based on genomic
data alone, and functional studies of specific mutations have found that

https://t.me/ALGRAWANY33
even alleles present in <1% of cases of a given malignancy can have
“driver” potential in vitro and in vivo.2
Question 1.5 The correct answer is A.
The detection of ctDNA is challenging in part due to the need to
discriminate it from DNA released by normal tissues, especially by
circulating blood cells. Even using ultrasensitive approaches, the median
variant allele frequency of driver mutations in lung cancer patients was
found to be <1% across plasma DNA samples.3
Question 1.6 The correct answer is D.
Targeted therapy in the treatment of cancer almost always leads to
acquired drug resistance. Using non-small-cell lung cancer as an
example, gefitinib and erlotinib demonstrate upfront effectiveness that
wanes often within 6 to 12 months after the initiation of therapy. The
underlying reason for this resistance was identified as a secondary
mutation in EGFR exon 20, T790M, which is detectable in 50% of
patients who progress on treatment. Some studies have shown the
mutation to be present before the patient was treated with these drugs,
suggesting exposure to the drug selected for these cells. Other drug-
resistance mutations include T315I in ABL, T670I in KIT, and L1196M
in ALK, and it remains an area of active research on whether these other
mutations are present in any patients before receiving targeted therapy.
Question 1.7 The correct answer is D.
Chromothripsis was first used to describe the genomic profile of a
chronic lymphocytic leukemia tumor. DNA sequencing identified 55
chromosomal rearrangements in the long arm of chromosome 8, in
addition to numerous rearrangements in regions of chromosomes 7, 12,
and 15. Similar patterns were identified in other cancers, such as
melanomas, sarcomas, and lung cancers. Based on the large numbers of
complex rearrangements in localized regions of single chromosomes, it
is hypothesized that chromosomes must be condensed (such as during
mitosis) for chromothripsis to occur. Furthermore, the number of copy
number states following chromothripsis is predominantly restricted to
two, suggestive of these rearrangements taking place over a short period
of time in a one-step event. It is estimated that chromothripsis occurs in
2% to 3% of cancers across all subtypes.
Question 1.8 The correct answer is C.
Studies of prostate cancer genomes identified complex rearrangements
of DNA containing genes coregulated by specific transcription factors. It
is proposed that these genomic loci are locally concentrated in nuclear
transcription hubs, and then, this DNA sustains multiple transient breaks
during transcription followed by religating in an incorrect, although
tumorigenic configuration. Chromoplexy often involves segments of
DNA from multiple chromosomes, often five or more, compared with
those from chromosomes affected by chromothripsis.
Question 1.9 The correct answer is A.
Negative-feedback loops that normally dampen signaling to ensure
homeostatic regulation can be disrupted during oncogenesis. One
prototypical example of this is how the oncogenic effects of mutant RAS
proteins do not result from a hyperactivated downstream signaling
activity. Rather, oncogenic RAS mutations impair the intrinsic GTPase
activity of RAS that normally serves to turn its activity off. In turn, RAS
mutations generate chronic proliferative signals due to disrupted
autoregulatory negative feedback.
Question 1.10 The correct answer is A.
Necrotic cell death leads to the release of proinflammatory signals into
the surrounding cancer microenvironment, leading to recruitment of
immune cells to survey tissue damage and clear debris. These
inflammatory cells can be actively tumor-promoting by fostering
angiogenesis, cancer cell proliferation, and invasiveness.
Question 1.11 The correct answer is D.
Apoptosis is regulated through counterbalancing pro-apoptotic and
antiapoptotic proteins. Bcl-2 and its close relatives Bcl-xL, Bcl-w, and
Mcl-1 are inhibitors of apoptosis. They function in part by binding to

https://t.me/ALGRAWANY33
and suppressing two pro-apoptotic proteins: Bax and Bak, which are
embedded in the mitochondrial outer membrane and promote apoptosis
through the disruption of the outer mitochondrial membrane, leading to
the release of cytochrome C and other pro-apoptotic signaling proteins
into the cytosol.
Question 1.12 The correct answer is D.
TAMs can promote local invasion by supplying matrix-degrading
enzymes such as metalloproteinases and cysteine cathepsin proteases.
These TAMs have been shown to be activated by interleukin (IL)-4
produced by the cancer cells. TAMs can also supply EGF to tumor cells,
and the tumor cells can reciprocally stimulate the TAMs through the
production of colony-stimulating factor-1 (CSF-1). This synergistically
can promote cancer cell intravasation into the circulatory system and
thus metastatic spread.
Question 1.13 The correct answer is A.
It is believed that once cancer cells successfully metastasize, they
possess tissue-specific colonizing abilities that also facilitate their
further dissemination. This includes dissemination not only to new sites
in the body but also back to the primary tumors in which their ancestor
cells arose. An elegant study showed that CTCs can colonize their
tumors of origin, modeled with breast cancer, colon cancer, and
melanoma tumors in mice.4 Tumor-derived cytokines such as IL-6 and
IL-8 act as CTC attractants. Self-seeding could explain the relationships
between anaplasia, tumor size, vascularity, and prognosis, as well as
local recurrence seeded by disseminated cells following what was often
presumed to be complete primary tumor excision.
Question 1.14 The correct answer is C.
It has been known for many years that mice with genetically engineered
deficiencies of various immune system components are more likely to
develop carcinogen-induced tumors. In particular, deficiencies in the
development or function of either CD8+ cytotoxic T lymphocytes (CTL),
CD4+ Th1 helper T cells, or NK cells led to demonstrable increases in
tumor incidence. Mice with combined immunodeficiencies in both T
cells and NK cells were even more susceptible to cancer development.
These findings indicate that, at least in certain experimental models, the
innate and adaptive cellular arms of the immune system contribute to
tumor immune surveillance and eradication.
Question 1.15 The correct answer is A.
PCR methods utilize cycles of repeated primer binding, DNA synthesis,
and denaturation to achieve amplification of a specific region within a
strand of DNA. In turn, genomic alterations that change the size of a
given DNA locus (such as the FLT3-ITD mutation) or point mutations
that can be exploited by allele-specific primers (such as the JAK2 V617F
mutation) are amenable to study by PCR. Metaphase karyotyping and
fluorescence in situ hybridization are favored for studying structural
chromosomal changes, such as deletions of entire arms or translocations.
Question 1.16 The correct answer is A.
In a study using digital PCR-based technologies, the percentage of
patients with detectable ctDNA was higher in patients with more
advanced stage compared to those with localized disease.5 Similarly, the
concentration of ctDNA in the plasma increased with stage. ctDNA was
often present in patients without detectable CTCs, suggesting that these
two biomarkers are distinct entities.
Question 1.17 The correct answer is D.
TCGA began in 2006 in the United States as a comprehensive program
in cancer genomics supported by the U.S. National Institutes of Health
(NIH). The initial project focused on three tumors: GBM, serous
cystadenocarcinoma of the ovary, and lung squamous carcinoma. The
TCGA has now expanded to produce genomic data sets for at least 20 to
25 cancers during the next few years.
Question 1.18 The correct answer is A.
Mutual exclusivity has been shown elegantly in the case of KRAS and
BRAF, where a KRAS-mutated cancer generally does not also harbor a

https://t.me/ALGRAWANY33
BRAF mutation, because KRAS is upstream of BRAF in the same
pathway.

References
1. Sjöblom T, Jones S, Wood LD, et al. The consensus coding sequences of human breast and
colorectal cancers. Science. 2006;314:268–274.
2. Fröhling S, Scholl C, Levine RL, et al. Identification of driver and passenger mutations of FLT3 by
high-throughput DNA sequence analysis and functional assessment of candidate alleles. Cancer
Cell. 2007;12:501–513.
3. Newman AM, Bratman SV, To J, et al. An ultrasensitive method for quantitating circulating tumor
DNA with broad patient coverage. Nat Med. 2014;20:548–554.
4. Kim MY, Oskarsson T, Acharyya S, et al. Tumor self-seeding by circulating cancer cells. Cell.
2009;139:1315–1326.
5. Bettegowda C, Sausen M, Leary RJ, et al. Detection of circulating tumor DNA in early-and late-
stage human malignancies. Sci Transl Med. 2014;6:224ra24.
___________
Corresponding chapters in DeVita, Hellman, and Rosenberg’s Cancer: Principles & Practice of
Oncology, Eleventh Edition: 1 (The Cancer Genome), 2 (Molecular Methods in Cancer), and 3
(Hallmarks of Cancer: An Organizing Principle for Cancer Medicine).
2 Design and Analysis of Clinical Trials
Esther Lu, Fei Wan, and Shu (Joy) Jiang

QUESTIONS

Each of the numbered items below is followed by lettered answers. Select the
ONE lettered answer that is BEST in each case unless instructed otherwise.

Question 2.1 Once a new compound with therapeutic potential is identified,


laboratory tests are conducted to study efficacy and identify associated
toxicities using animal models. This is related to
A. Preclinical trial
B. Phase I trial
C. Phase II trial
D. Phase III trial
E. Phase IV trial
Question 2.2 Which of the following is the most commonly used endpoint
of phase I trials?
A. Maximum tolerated dose (MTD)
B. Overall response rate (ORR)
C. Progression-free survival (PFS)
D. Overall survival (OS)
Question 2.3 Which of the following is the main purpose of
randomization?
A. To select subjects at random
B. To achieve baseline comparability between the intervention and control

https://t.me/ALGRAWANY33
groups with respect to known and unknown confounders
C. To avoid sampling bias
D. To enhance the generalizability of the study
Question 2.4 What does “compliance” of randomization mean in a
randomized clinical trial (RCT)?
A. Flexibility in assigning study subjects to different treatment arms.
B. Study subjects receive the treatment or intervention to which they were
randomly allocated.
C. An interinstitutional agreement for a multicenter study.
D. The research team should follow the protocol.
Question 2.5 Which of the following applies to phase III trials?
A. A type of trial design with safety and efficacy estimation as a primary
objective
B. Small study of efficacy intended to inform the decision to conduct a
larger trial, if it is favorable, to inform its planning
C. Evidence of activity, efficacy, and safety at a fixed dose
D. Prove whether the new intervention is more effective than a standard
control intervention
Question 2.6 Which of the following is TRUE with regard to noninferiority
trials?
A. Noninferiority and equivalence trials are regarded as the same.
B. Superiority can be tested after noninferiority is claimed where sample
size and power will not be affected.
C. Superiority can be tested after noninferiority is claimed, but sample
size and power will be affected.
D. None of the above.
Question 2.7 One of the primary purposes of the interim analysis is to:
A. Determine whether the trial should be stopped before its planned
termination time
B. Modify the data once the statistical results are unexpected
C. Reveal the findings in public
D. Discuss whether the stopping rule is reasonable
Question 2.8 The primary purpose of a phase I trial is to:
A. Monitor the effectiveness of a newly approved drug in the general
population
B. Determine the safety of investigational treatment
C. Determine the efficacy of investigational treatment
D. Compare the investigational treatment to the placebo
Question 2.9 Which of the following statements about “blinding” in RCTs
is correct?
A. Blinding means we should not have any preconceived ideas before we
perform a statistical analysis of the data.
B. Blinding refers to the uncertainty regarding whether the investigational
treatment is effective.
C. Blinding means that the subjects and/or investigators are unaware of
which treatment arm the subjects are assigned to. The purpose is to
reduce bias in assessing the outcome.
D. Blinding means the study results differ from the previously published
studies and we need to reevaluate the data.
Question 2.10 Which of the following is TRUE about the response rate and
survival endpoint in a phase II trial?
A. A larger response rate will lead to a smaller survival probability.
B. A larger response rate will lead to a greater survival probability.
C. There is no inherent relationship between the response rate and
survival.
D. A smaller response rate will lead to a greater survival probability.
Question 2.11 Which of the following statements correctly describes the
interim analysis?

https://t.me/ALGRAWANY33
A. If statistical significance tests are performed repeatedly, the probability
of finding a statistically significant outcome increases due to type I
error.
B. The tests at different stages are independent.
C. Type I error increases in the same way as it would for independent
tests.
D. All of the above.
Question 2.12 If no interim analysis was planned, but one decides to
overlook the interim data in the trial and report on the intermediate results,
which of the following is TRUE?
A. The trial can be terminated if the intermediate results were found to be
significant.
B. The trial should continue as planned as if no interim data were
available.
C. The trial should be modified accordingly.
D. None of the above.
Question 2.13 Which of the following statements is TRUE about
identifying the relevant studies in a meta-analysis?
A. Potential studies are retrieved based on personal knowledge, searches
of literature databases, reference lists from selected publications, and
so on.
B. There are some limitations of retrieval.
C. In order to avoid the publication bias, the prospective registration
database of trials or complete reporting of trial results is proposed.
D. All of the above.
Question 2.14 Which of the following is TRUE regarding a phase I trial?
A. Phase I trials are designed to determine a dose that is appropriate for
the use in phase II trials.
B. Phase I trials usually involve a small sample size and do not require
statistical justification.
C. Phase I trials are targeted at treatment comparison for community use.
D. All of the above.
Question 2.15 When designing a RCT, the design should be a balanced
design mainly
A. Due to computational ease
B. Because we obtain the greatest power to detect the effect size if the
alternative hypothesis is true
C. Because we obtain the greatest power to detect the effect size if the null
hypothesis is true
D. Due to comparable treatment effect in all arms
Question 2.16 Which of the following is TRUE about noncompliance in an
RCT?
A. Confounding will be introduced.
B. Noncompliance will not bias the final estimate of the treatment effect.
C. Blinding will not have any effect on the compliance with the treatment
assigned.
D. If compliance is poor, the two groups will become more alike and the
statistical power to detect true differences in outcome will be
diminished.
Question 2.17 Which of the following is TRUE regarding sample size
calculations when testing for treatment difference that is specified under the
alternative hypothesis?
A. If one specifies a larger treatment difference under the alternative
hypothesis while holding all other parameters constant, the required
sample size will decrease.
B. If one specifies a smaller treatment difference under the alternative
hypothesis while holding all other parameters constant, the required
sample size will decrease.
C. The sample size will remain the same regardless of the treatment
difference specified if all other parameters are kept the same.

https://t.me/ALGRAWANY33
D. None of the above.
Question 2.18 Which of the following is TRUE about intention-to-treat
(ITT) analysis of a clinical trial?
A. Only study subjects who really intend to take the investigational
treatment should be enrolled into the trial.
B. All study subjects who are enrolled and randomly allocated to
treatment are included in the analysis and are analyzed in the groups to
which they were randomized.
C. ITT analysis classifies the study subjects according to the treatment
that they actually received rather than according to the treatment that
they were randomly assigned to.
D. All subjects must agree to be randomized and take the treatment they
were assigned to.
Question 2.19 Which of the following is TRUE about a noninferiority trial?
A. The primary objective of a noninferiority trial is to show that the
response to the investigational treatment is superior to an established
treatment.
B. The primary objective of a noninferiority trial is to show that two
treatments differ by an amount that is clinically unimportant.
C. The primary objective of a noninferiority trial is to show that the
response to the investigational treatment is not clinically inferior to an
established treatment.
D. Hypothesis testing for a noninferiority trial is always two-sided.
Question 2.20 The development of most new drugs, from discovery to
marketing approval, usually takes:
A. <5 months
B. 9 years or more
C. 5 years
D. 1–3 years
ANSWERS

Question 2.1 The correct answer is A.


Laboratory tests are part of a preclinical trial. Once patients are
involved, the trials may be classified as phases I to IV.
Question 2.2 The correct answer is A.
Phase I studies mainly focus on the safety of the study drug, with
maximum tolerated dose (MTD) being one of the most commonly used
endpoints. In contrast, the response rate and survival are more
commonly used endpoints in phase II and III trials.
Question 2.3 The correct answer is B.
The purpose of randomization is to make sure that the intervention and
control groups are comparable with respective to known and unknown
confounders.
Question 2.4 The correct answer is B.
Randomization compliance is when study subjects receive the treatment
or intervention to which they were randomly allocated.
Question 2.5 The correct answer is D.
The main purpose of a randomized phase III trial is to evaluate whether
the new intervention is more effective than the standard control. Phase II
trials are usually smaller, evaluate for efficacy, and inform the decision
regarding the conduct of a larger trial.
Question 2.6 The correct answer is B.
For noninferiority trials, the goal is to stop the trial if inferiority was
evidenced. It is plausible to test for superiority after noninferiority was
claimed and the power will not be affected in doing so. It is not the same
for equivalence trials, where once equivalence is claimed, superiority
cannot be claimed simultaneously.

https://t.me/ALGRAWANY33
Question 2.7 The correct answer is A.
The main purpose of the interim analysis is to evaluate whether the trial
should be stopped before the planned termination time. Once the study
is stopped and the analysis is done, the researchers need to reveal the
findings in public. The data should not be modified, and the stopping
rule should be prespecified in the protocol, before a trial starts.
Question 2.8 The correct answer is B.
The primary purpose of a phase I trial is to determine the clinical safety
of the investigational treatment.
Question 2.9 The correct answer is C.
Blinding means that the subjects and/or investigators are unaware of
which treatment arm the subjects are assigned to. The analysis plan must
be determined before a randomized trial starts. The purpose of
randomized trials is to test whether investigational treatment is effective,
and the study results may differ from the previously published studies.
However, they are not the meaning of blinding.
Question 2.10 The correct answer is C.
There is no biologically inherent relationship between the response and
survival.
Question 2.11 The correct answer is A.
Repeated performance of statistically significant tests increase the
probability of type I error at some point. Since subsequent analyses
include the previous ones, they are not independent.
Question 2.12 The correct answer is B.
If no interim analysis was planned in the design stage, there should be
no statistical tests done before the planned date. Bias could occur if one
decides to report on the intermediate results or modify the trial and lead
to insufficient power if the trial was terminated.
Question 2.13 The correct answer is D.
Meta-analyses require retrieval of studies from databases with
prospective registration to avoid publication biases despite the
limitations of study retrieval.
Question 2.14 The correct answer is A.
Traditional phase I trials are designed to determine the appropriate dose
for further studies.
Question 2.15 The correct answer is B.
In calculating the treatment difference between the arms, the distribution
of the pooled variance will not be affected by the balance of the sample
size if null hypothesis is true. However, if the alternative is true, a
balanced design has the greatest power to detect the treatment
differences.
Question 2.16 The correct answer is A.
The study subjects who are not compliant with an intervention may
differ from those who are compliant, and in ways, those noncompliers
can be expected to have a worse outcome. Therefore, compliance is a
confounding factor. Ignoring noncompliance in a randomized trial may
cause biased results. Blinding can help to reduce noncompliance
particularly in the placebo control group because these subjects are not
aware that they are not getting an active treatment.
Question 2.17 The correct answer is A.
As the specified treatment difference gets larger under the alternative
hypothesis while keeping all other parameters the same, the required
sample will be smaller.
Question 2.18 The correct answer is B.
Intention-to-treat (ITT) analysis includes all patients enrolled and
allocated to treatment.

https://t.me/ALGRAWANY33
Question 2.19 The correct answer is C.
Noninferiority trials are designed to show that the investigational agent
is not clinically inferior to the standard therapy. Superiority trials are
designed to evaluate whether the investigational therapy is more
effective than the standard of care. The hypothesis testing in a
noninferiority trial is one-sided.
Question 2.20 The correct answer is B.
The first step in the drug development process allows investigators to
evaluate the drug using tissue cultures and animal models. This research
is referred to as preclinical studies. The drug’s activity, its
pharmacokinetics, and its bioavailability are also established in animal
models. The outcome of these tests allows the sponsor to determine
whether the drug appears to be safe and shows promise of effectiveness,
thereby warranting an additional study. Preclinical studies take 3 to 6
years. Clinical trials of investigational drugs are conducted through a
systemic approach and take 6 to 7 years.
___________
Corresponding chapter in DeVita, Hellman, and Rosenberg’s Cancer: Principles & Practice of
Oncology, Eleventh Edition: 39 (Design and Analysis of Clinical Trials).
3 Cancer Therapy
Joshua T. Loesche, Janelle E. Mann, and Danielle K. Turlington

QUESTIONS

Each of the numbered items below is followed by lettered answers. Select the
ONE lettered answer that is BEST in each case unless instructed otherwise.

Question 3.1 A 55-year-old male is currently taking erlotinib for treatment


of his lung cancer. He is admitted for uncontrolled epigastric pain, and the
admitting team places orders for the pantoprazole. What effect may
concurrent administration have on erlotinib concentrations?
A. Increased erlotinib serum concentrations
B. Decreased erlotinib serum concentrations
C. No change in erlotinib serum concentrations
D. Separation by 6 h or more is required to ensure adequate erlotinib
serum concentrations
Question 3.2 Which of the following therapy–adverse effects is
appropriately associated?
A. Palbociclib—constipation
B. Ribociclib—hyperkalemia
C. Abemaciclib—diarrhea
D. Tamoxifen—corrected QT (QTc) prolongation
Question 3.3 A 65-year-old female with advanced colorectal cancer is in
clinic to receive cycle 5 of mFOLFOX (oxaliplatin 85 mg/m2, 5-fluorouracil
[FU] bolus 400 mg/m2 intravenously [IV], leucovorin 400 mg/m2 IV, and 5-
FU 2,400 mg/m2 continuous IV infusion [CIVI] over 46 h every 14 days).

https://t.me/ALGRAWANY33
She reports experiencing nausea and vomiting requiring admission after her
last cycle (cycle 4) and increased intolerance to cold temperatures with
residual grade 2 peripheral neuropathy. Which of the following would be
appropriate to adjust in her mFOLFOX treatment today?
A. Decrease the oxaliplatin dose to 65 mg/m2 secondary to grade 3
gastrointestinal (GI) toxicities and grade 2 neuropathy
B. Decrease the 5-FU CIVI dose to 1,920 mg/m2 over 46 h and add in
additional fluids today for dehydration
C. Discontinue the 5-FU bolus and leucovorin secondary to grade 2
neuropathy
D. Discontinue oxaliplatin secondary to persistent grade 2 neuropathy
Question 3.4 Which of the following would be appropriate
recommendations for a patient who experiences extravasation from
vincristine?
A. Cold compress and dexrazoxane
B. Cold compress and hyaluronidase
C. Warm compress and sodium thiosulfate
D. Warm compress and hyaluronidase
Question 3.5 A patient returns to clinic following her fifth cycle of
docetaxel 75 mg/m2 IV every 21 days for her recurrent non–small-cell lung
cancer (NSCLC). She reports increased bilateral lower-extremity swelling
and periorbital edema. Which of the following is most appropriate for the
management of the reported adverse effect?
A. Ensure compliance with dexamethasone premedication and start
furosemide.
B. Initiate spironolactone.
C. Obtain lower-extremity Doppler ultrasound.
D. Increase the duration of dexamethasone premedication from 3 days
starting the day before chemotherapy to 5 total days of therapy.
Question 3.6 DM is a newly diagnosed NSCLC patient who is found to
have RET fusion–positive disease. He is started a selpercatinib 160 mg by
mouth twice daily. Which of the following would be appropriate to monitor
upon the initiation of therapy?
A. Hypotension
B. Nausea and vomiting
C. Hepatotoxicity
D. Acneiform rash
Question 3.7 Which of the following would be recommended to obtain at
baseline before the initiation of alpelisib?
A. Electrocardiogram
B. Echocardiogram
C. Hemoglobin A1c (HbA1c)
D. Lipid panel
Question 3.8 A 35-year-old female with triple-negative breast cancer is
starting her treatment today with sacituzumab govitecan 10 mg/kg IV days 1
and 8 every 21 days. Which of the following would be appropriate supportive
care recommendations?
A. Dexamethasone, ondansetron, acetaminophen, famotidine, and
diphenhydramine
B. Atropine, dexamethasone, and ondansetron
C. Acetaminophen and diphenhydramine
D. Dexamethasone, famotidine, and diphenhydramine
Question 3.9 SW is a 62-year-old female with newly diagnosed metastatic
NSCLC. She presents to clinic to start treatment with carboplatin,
pemetrexed, and pembrolizumab. Programmed death-ligand 1 (PD-L1)
expression is 20%. Which of the following is TRUE regarding SW’s
treatment?
A. Use of pembrolizumab with chemotherapy is not appropriate if the
tumor is PD-L1–negative.
B. Due to recent pharmacokinetic data, pembrolizumab should be dosed
using weight-based dosing rather than flat dosing.

https://t.me/ALGRAWANY33
C. The addition of pembrolizumab to frontline chemotherapy
demonstrated a survival benefit regardless of PD-L1 expression level
for patients with advanced or metastatic NSCLC.
D. Pembrolizumab monotherapy is an alternative option regardless of the
PD-L1 expression.
Question 3.10 Which of the following statements regarding the use of
leucovorin is TRUE?
A. Leucovorin is required when administering pemetrexed to help prevent
renal toxicity.
B. When used following the administration of high-dose methotrexate,
leucovorin enhances antitumor activity of methotrexate.
C. When used following the administration of high-dose methotrexate,
leucovorin helps to prevent renal toxicity.
D. When used with 5-FU, leucovorin enhances its antitumor activity.
Question 3.11 Which of the following statements regarding cytarabine is
TRUE?
A. Standard 5- to 7-day continuous dosing of cytarabine is associated with
a higher incidence of myelosuppression when compared to high-dose
cytarabine.
B. Neurotoxicity is not a common adverse effect associated with
cytarabine because it does not cross the blood–brain barrier.
C. Conjunctivitis is an adverse effect associated with cytarabine and can
be treated with topical corticosteroid eye drops.
D. Cytarabine has documented clinical activity against a wide range of
hematologic malignancies and solid tumors.
Question 3.12 A 67-year-old female with relapsed peripheral T-cell
lymphoma, type II diabetes mellitus, atrial fibrillation, bipolar disorder, and
coronary artery disease is evaluated for treatment with romidepsin as the next
line of therapy. Which of the following monitoring parameters should be
obtained before initiating the treatment with romidepsin?
A. Echocardiogram
B. Tuberculosis (TB) test
C. Electrocardiogram
D. Cytomegalovirus (CMV) immunoglobulin M (IgM)/immunoglobulin G
(IgG)
Question 3.13 Which of the following is an advantage to administering
bortezomib subcutaneously versus IV?
A. Subcutaneous bortezomib is administered at less frequent intervals than
IV bortezomib.
B. Subcutaneous bortezomib is associated with a lower incidence of
peripheral neuropathy.
C. Subcutaneous bortezomib is associated with a lower incidence of
mucositis.
D. Subcutaneous bortezomib is associated with increased inhibition of
proteasome activity.
Question 3.14 Which of the following statements is TRUE regarding the
management of immune-related adverse events (IRAEs) associated with
toxicity of immune checkpoint inhibitors?
A. Use of glucocorticoids should be avoided in management of severe
immune-mediated toxicity because the mechanism of action of immune
checkpoint inhibitors relies on T-cell activation.
B. IRAEs can affect any organ, but most commonly affect the colon, lung,
skin, liver, and endocrine glands.
C. IRAEs are the most common with the first administration of an
immune checkpoint inhibitor.
D. Following resolution of mild toxicity to an immune checkpoint
inhibitor, treatment should not be resumed.
Question 3.15 Which is TRUE regarding the cytokine release syndrome
(CRS) associated with chimeric antigen receptor (CAR) T-cell therapy?
A. Tocilizumab should be avoided in patients who have received CAR T-
cell therapy as it can suppress T-cell function.
B. CRS occurs most commonly within 4 weeks following CAR T-cell

https://t.me/ALGRAWANY33
infusion.
C. Encephalitis often occurs in conjunction with CRS and can occur up to
4 weeks after treatment.
D. Hypertension is a common symptom of CRS.
Question 3.16 Dose reductions of irinotecan are recommended for a patient
with all of the following EXCEPT:
A. Bilirubin >1.5 mg/dL
B. Creatinine clearance <40 mL/min
C. Absolute neutrophil count (ANC) <1,500/mm3
D. Homozygosity for the UGT1A1*28 allele
Question 3.17 A 55-year-old female is diagnosed with rhabdomyosarcoma
and will begin treatment with doxorubicin and ifosfamide. Which of the
following supportive care medications along with aggressive IV hydration is
recommended to prevent hemorrhagic cystitis associated with ifosfamide?
A. Mesna
B. Furosemide
C. Dexamethasone
D. Methylene blue
Question 3.18 A 30-year-old female with fallopian tube cancer who has
experienced a partial response to platinum-based chemotherapy will now be
initiated on maintenance niraparib. Which of the following parameters will
need to be monitored while the patient is receiving niraparib?
A. Blood glucose and HbA1c
B. Left ventricular ejection fraction
C. Blood pressure and heart rate
D. Bone mineral density
Question 3.19 Which of the following medication/targetable mutation pairs
is correctly matched?
A. Lorlatinib/anaplastic lymphoma kinase (ALK)
B. Osimertinib/BCR-ABL
C. Ruxolitinib/mitogen-activated protein kinase (MAPK)
D. Vemurafenib/mitogen-activated extracellular kinase (MEK)
Question 3.20 Which of the following medications is an example of an
antibody–drug conjugate?
A. Axicabtagene ciloleucel
B. Ipilimumab
C. Enfortumab vedotin
D. Ramucirumab

ANSWERS

Question 3.1 The correct answer is B.


Erlotinib requires an acidic environment for adequate absorption.
Erlotinib maximum serum concentration and area under the curve
(AUC) decreased by an average of 61% and 46%, respectively, when
coadministered with omeprazole. It is recommended to avoid concurrent
administration of proton pump inhibitors with erlotinib therapy as we
would expect decreased erlotinib serum concentrations. Separating the
administration time between erlotinib and proton pump inhibitors is
unlikely to be an adequate means of avoiding the interaction.
Question 3.2 The correct answer is C.
Cyclin dependent kinase (CDK) 4/6 inhibitors are used in estrogen
receptor–positive metastatic breast cancer. Palbociclib and ribociclib
have similar toxicities including myelosuppression and fatigue. Both are
associated with febrile neutropenia; however, neutropenic fever is
infrequent. Abemaciclib has unique adverse effects including diarrhea,
while ribociclib and palbociclib commonly cause QT interval
prolongation. Tamoxifen has not been reported to cause
electrocardiogram changes.

https://t.me/ALGRAWANY33
Question 3.3 The correct answer is A.
The reported symptoms of persistent grade 2 peripheral neuropathy and
increased intolerance to cold temperatures as well as increased nausea
and vomiting requiring admission are likely associated with oxaliplatin
and may improve with dose reduction. Decreasing the 5-FU CIVI would
not improve her persistent grade 2 peripheral neuropathy. Holding the 5-
FU bolus and leucovorin is most commonly done for patients who have
increased myelosuppression. Discontinuation of oxaliplatin is not yet
warranted as she is still receiving full dose therapy, and the peripheral
neuropathy is not a grade 3.
Question 3.4 The correct answer is D.
Vinca alkaloids are potent vesicants, and drug extravasation can lead to
significant complications. Cold compresses are used for nearly all drug
extravasations; however, management of vinca alkaloid extravasation is
unique as initial compression with warm compresses is preferred to limit
tissue damage. Additionally, the use of hyaluronidase can minimize
discomfort and latent cellulitis. Dexrazoxane and sodium thiosulfate are
used for anthracycline and cisplatin extravasations, respectively.
Question 3.5 The correct answer is A.
Docetaxel causes a unique fluid retention syndrome that is often seen
with patients reported edema, weight gain, and third-space fluid
collections. This is cumulative in nature and is thought to be related to
increased capillary permeability. It is essential to provide early treatment
with diuretics. Additionally, the use of prophylactic corticosteroids has
been found to reduce the incidence of fluid retention. Starting an
antidiuretic alone does not address the importance of prophylactic
dexamethasone therapy. As the edema is bilateral, deep vein thrombosis
is less likely and a Doppler ultrasound is not indicated at this time.
Question 3.6 The correct answer is C.
Selpercatinib was U.S. Food and Drug Administration (FDA)-approved
for metastatic RET fusion–positive NSCLC as well as thyroid cancer
and for RET-mutant medullary cancer.1 Hepatotoxicity with aspartate
transaminase (AST) and alanine aminotransferase (ALT) elevations
occurred in about half of patients with the median time to first onset of
4.1 weeks. Hypertension has been more commonly associated with this
drug as it inhibits vascular endothelial growth factor receptors. Skin rash
has been observed with this therapy; however, it is not acneiform in
nature. Although nausea and vomiting has been reported, the incidence
is <25%.
Question 3.7 The correct answer is C.
Alpelisib was approved for metastatic triple-negative breast cancer in
combination with fulvestrant.2 Before the initiation of therapy, it is
recommended to obtain a baseline HbA1c as severe hyperglycemia,
including ketoacidosis, has been reported. Monitoring a baseline HbA1c
along with fasting plasma glucose is recommended to appropriately
address and intervene on patients developing severe hyperglycemia.
Electrocardiogram and echocardiogram are not warranted for this
therapy, and there is no utility in obtaining a baseline lipid panel for
adverse effect monitoring for alpelisib.
Question 3.8 The correct answer is A.
Sacituzumab govitecan is an antibody–drug conjugate that consists of
anti-trophoblastic cell-surface antigen 2 (Trop-2) coupled with the
topoisomerase 1 inhibitor SN-38.3 Hypersensitivity reactions were
reported in over one-third of patients warranting premedication with
antipyretics and H1/H2 antagonists prior to infusions. Additionally, the
rates of nausea and vomiting were 69% and 45%, respectively. It is
recommended to provide prophylactic antiemetics with either a 2- or 3-
drug regimen following moderately emetogenic chemotherapy
guidelines. The most appropriate answer would include dexamethasone,
a 5-HT3 antagonist, acetaminophen, famotidine, and diphenhydramine as
premedication to prevent against infusion-related reactions and
chemotherapy-induced nausea and vomiting.
Question 3.9 The correct answer is C.

https://t.me/ALGRAWANY33
The combination of platinum-based chemotherapy and pembrolizumab
is appropriate for first-line treatment regardless of PD-L1 positivity
according to the results of the KEYNOTE-189 trial, which showed
survival benefit compared with chemotherapy alone.4 Additionally,
recent pharmacokinetic data have justified flat dosing of
pembrolizumab. Single agent pembrolizumab would be a reasonable
choice for initial therapy if the patient was PD-L1-positive, particularly
if >50%.
Question 3.10 The correct answer is D.
While pemetrexed is an antifolate, there is no role for coadministration
of leucovorin. Leucovorin reduces myelosuppression associated with the
administration of high-dose methotrexate and does not enhance
antitumor effect. When used with 5-FU, leucovorin acts as a
biomodulator to enhance antitumor activity of 5-FU.
Question 3.11 The correct answer is C.
Cytarabine is an antimetabolite medication that can be given for a
variety of hematologic malignancies as either continuous infusion over 5
to 7 days or high-dose cytarabine, defined as doses of 1 g/m2 and above.
Side effects commonly found with cytarabine vary with the
administration schedule, with myelosuppression being more common
with high dose. As cytarabine crosses the blood–brain barrier, frequent
monitoring for neurotoxicity is required. While cytarabine has clinical
activity against several hematologic malignancies, it does not have
documented clinical activity against solid tumors. A common side effect
of cytarabine is conjunctivitis for which prophylactic corticosteroid eye
drops are often used.
Question 3.12 The correct answer is C.
Romidepsin is a histone deacetylase (HDAC) inhibitor used for
cutaneous and peripheral T-cell lymphomas. Due to the possibility of
causing QT prolongation, it is recommended to perform baseline and
periodic electrocardiograms. It is not recommended to perform an
echocardiogram before initiating romidepsin. Romidepsin carries a
warning for infection, but there is no specific requirement for baseline
TB test or CMV testing.
Question 3.13 The correct answer is B.
Bortezomib is a proteasome inhibitor that can be administered either IV
or subcutaneously using the same schedule. Subcutaneous bortezomib
has demonstrated similar response rates with a lower incidence of
neuropathy when compared to IV bortezomib. However, there is no
significant decrease in the incidence of mucositis or stronger inhibition
of proteasome activity when compared to IV administration.
Question 3.14 The correct answer is B.
Toxicities of immune checkpoint inhibitors are related to aberrant
activation of autoreactive T cells against host tissues. Use of
glucocorticoids is indicated for moderate-to-severe IRAEs in dose of 0.5
to 1 mg/kg of prednisone or equivalent. IRAEs may occur at any time
point, and timing may differ depending on the affected organ. Treatment
with an immune checkpoint inhibitor may be reinitiated following mild
IRAEs once the toxicity has resolved to grade 1 or baseline. IRAEs can
take place in any organ but most commonly affect the colon, lung, skin,
liver, and endocrine glands.
Question 3.15 The correct answer is C.
CRS and encephalitis are common adverse events of CAR T-cell
therapy. The interleukin (IL)-6 inhibitor tocilizumab is often required for
the management of CRS and does not impact T-cell function. CRS
occurs most commonly within a few days of treatment with CAR T
cells, making hospitalization for the first week following treatment
usually necessary. One of the most common symptoms of CRS is
hypotension, not hypertension. Encephalitis often occurs in conjunction
with CRS, but can occur up to 4 weeks after treatment. This toxicity is
not only caused initially by the release of cytokines but also results from
trafficking of CARs into the central nervous system.
Question 3.16 The correct answer is B.

https://t.me/ALGRAWANY33
Irinotecan is a prodrug that generates the active metabolite SN-38. SN-
38 is responsible for the two most common toxicities related to
irinotecan, diarrhea and myelosuppression. It is inactivated in the liver
by glucuronidation by UGT1A1. Exposure to SN-38 is increased in
patients with hepatic impairment, and dose reductions are recommended
for patients with a bilirubin >1.5 mg/mL. Additionally, genetic
polymorphisms of UGT1A1 increase the risk of diarrhea and
myelosuppression. Dose reductions are recommended for patients who
are homozygous for the UGT1A1*28 allele. If myelosuppression occurs,
irinotecan should be held until absolute neutrophil count recovery and
subsequent doses of irinotecan should be reduced. There is no need for
dose adjustments based on the renal function.
Question 3.17 The correct answer is A.
Mesna is required to prevent hemorrhagic cystitis for all patients
receiving ifosfamide, patients receiving high-dose cyclophosphamide,
and patients who have a history of drug-induced cystitis. Mesna has a
free sulfhydryl group that conjugates with toxic metabolites (particularly
acrolein) of ifosfamide and cyclophosphamide. The dose of mesna
should be 60% of the alkylating agent dose, given in divided doses every
4 h.
Question 3.18 The correct answer is C.
Niraparib is a poly(ADP-ribose) polymerase (PARP) inhibitor indicated
for the maintenance treatment of patients with recurrent epithelial
ovarian, fallopian tube, or primary peritoneal cancer who are in a
complete or partial response to platinum-based chemotherapy.5
Hypertension and hypertensive crisis have been reported in patients
treated with niraparib. Therefore, patients should have the blood
pressure and heart rate monitored monthly for the first year of treatment
and periodically thereafter during treatment. Management with
antihypertensive medications as well as dose adjustments of niraparib
may be necessary.
Question 3.19 The correct answer is A.
Lorlatinib is a tyrosine kinase inhibitor that reversibly binds to both
ALK and ROS1. Osimertinib and vemurafenib target EGFR and BRAF
V600E mutations, respectively, whereas ruxolitinib is a Janus kinase
(JAK) inhibitor.
Question 3.20 The correct answer is C.
Enfortumab vedotin is an antibody–drug conjugate directed at the cell-
surface adhesion protein Nectin-4. It contains an IgG1 anti-Nectin-4
antibody conjugated to monomethyl auristatin E (MMAE), which is a
microtubule inhibitor. When enfortumab vedotin binds to the Nectin-4
protein, a complex is formed and internalized into the cell. MMAE is
then released and binds to the microtubules, thereby disrupting the
cellular microtubule network that ultimately induces cell cycle arrest and
apoptosis. It is indicated for the treatment of adult patients with locally
advanced or metastatic urothelial cancer who have previously received a
programmed cell death receptor-1 or PD-L1 inhibitor and a platinum-
containing chemotherapy in the neoadjuvant/adjuvant, locally advanced
or metastatic setting.6

References
1. Drilon A, Oxnard GR, Tan DSW, et al. Efficacy of selpercatinib in RET fusion-positive non-small-
cell lung cancer. N Engl J Med. 2020;383:813–824.
2. Andre F, Ciruelos E, Rubovszky G, et al. Alpelisib for PIK3CA-mutated, hormone receptor-
positive advanced breast cancer. N Engl J Med. 2019;380:1929–1940.
3. Bardia A, Mayer IA, Vahdat LT, et al. Sacituzumab govitecan-hziy in refractory metastatic triple-
negative breast cancer. N Engl J Med. 2019;380:741–751.
4. Gandhi L, Rodriguez-Abreu D, Gadgeel S, et al. Pembrolizumab plus chemotherapy in metastatic
non-small-cell lung cancer. N Engl J Med. 2018;378:2078–2092.
5. Mirza MR, Monk BJ, Herrstedt J, et al. Niraparib maintenance therapy in platinum-sensitive,
recurrent ovarian cancer. N Engl J Med. 2016;375:2154–2164.
6. Rosenberg JE, O’Donnell PH, Balar AV, et al. Pivotal trial of enfortumab vedotin in urothelial
carcinoma after platinum and anti-programmed death 1/programmed death ligand 1 therapy. J Clin
Oncol. 2019;37:2592–2600.
___________
Corresponding chapters in DeVita, Hellman, and Rosenberg’s Cancer: Principles & Practice of
Oncology, Eleventh Edition: 18 (Pharmacokinetics and Pharmacodynamics of Anticancer Drugs), 19
(Pharmacogenomics), 20 (Alkylating Agents), 21 (Platinum Analogs), 22 (Antimetabolites), 23
(Topoisomerase-Interacting Agents), 24 (Antimicrotubule Agents), 25 (Kinase Inhibitors as Anticancer
Drugs), 26 (Histone Deacetylase Inhibitors and Demethylating Agents), 27 (Proteasome Inhibitors), 28

https://t.me/ALGRAWANY33
(Poly[ADP-Ribose] Polymerase Inhibitors for Tumors with Defects in DNA Repair), 29
(Miscellaneous Chemotherapeutic Agents), 30 (Hormonal Agents), 31 (Monoclonal Antibodies) and 32
(Immunotherapy Agents).
4 Central Nervous System Neoplasms
and Brain Metastases
Alice Zhou, Omar Hameed Butt, and Jian L. Campian

QUESTIONS

Each of the numbered items below is followed by lettered answers. Select the
ONE lettered answer that is BEST in each case unless instructed otherwise.

Question 4.1 Mutation of which tumor suppressor gene is associated with a


reduced survival rate in medulloblastoma?
A. TP53
B. Rb
C. MYC
D. Epidermal growth factor receptor (EGFR)
Question 4.2 Activation of which pathway is the least common in
medulloblastoma but carries the best prognosis?
A. SHH
B. MYC
C. WNT
D. Transforming growth factor (TGF)-β
Question 4.3 What genomic element/mechanism has emerged as genomic
drivers in group 3 and group 4 medulloblastoma?
A. Super-enhancer
B. Microsatellite instability
C. Field defect

https://t.me/ALGRAWANY33
D. Loss of heterozygosity
Question 4.4 Aberrations in which molecular pathways have helped stratify
medulloblastomas into subgroups?
A. Isocitrate dehydrogenase (IDH) mutation
B. WNT, SHH
C. EGFR
D. 1p/19q codeletion
Question 4.5 A 30-year-old male who presents with a persistent positional
headache and unsteady gain is found to have a posterior fossa mass. He
undergoes surgical resection, and pathology reveals medulloblastoma. A pan-
spine magnetic resonance imaging (MRI) does not demonstrate any
abnormalities. What is the next best management plan?
A. Radiotherapy followed by adjuvant chemotherapy
B. Posterior fossa irradiation only
C. Chemotherapy only
D. No further interventions indicated
Question 4.6 What is the standard of care for newly diagnosed
ependymoma?
A. Surgery and radiation
B. Chemotherapy only
C. Surgery and chemotherapy
D. Radiation and chemotherapy
Question 4.7 Somatic mutations in which of the following pathways in
glioblastoma (GBM) are frequently found in younger patients and are
associated with better outcomes?
A. IDH1/2
B. PTEN
C. TP53
D. BRAF
Question 4.8 Mutations in TP53 and ATRX as well as loss of
heterozygosity on chromosome 17 are common in what glioma subtype?
A. GBM
B. Oligodendroglioma
C. Astrocytoma
D. None of the above
Question 4.9 Oligodendrogliomas, which generally have a favorable
prognosis, are characterized by what genetic mutations?
A. BRAF
B. O6-methylguanine-DNA methyltransferase (MGMT) methylation
C. 1p/19q codeletion and IDH mutation
D. ATRX
Question 4.10 What somatic gene alteration is found in mesenchymal
subgroup of GBM and is associated with a particularly poor outcome?
A. PDGFR
B. Neurofibromatosis type 1 (NF1)
C. EGFR
D. BRAF
Question 4.11 What epigenetic finding is a favorable prognostic factor and
is associated with increased response to alkylating agents in GBMs?
A. MGMT promoter methylation
B. KDM5A overexpression
C. 1p/19q codeletion
D. IDH1/2 wild-type
Question 4.12 Genetic variants of which tumor suppressor gene are
associated with a twofold relative risk increase in gliomas compared with the
general population?
A. TP53

https://t.me/ALGRAWANY33
B. RB
C. APC
D. NF1
Question 4.13 GBM is most commonly associated with alterations in
which pathway?
A. EGFR
B. RB
C. P53
D. All of the above
Question 4.14 What is the most common malignant adult primary central
nervous system (CNS) tumor in the United States?
A. Astrocytoma
B. Meningioma
C. GBM
D. Medulloblastoma
Question 4.15 What environmental exposure is associated with an increase
in primary CNS tumors?
A. Benzene
B. Aflatoxin
C. Radiation
D. Betel nut
Question 4.16 Which primary CNS tumor is most commonly associated
with Epstein–Barr virus infection?
A. Primary CNS lymphoma
B. Astrocytoma
C. Pituitary adenoma
D. Meningioma
Question 4.17 Which primary CNS tumor is often found to have
calcification on MRI?
A. Meningioma
B. Oligodendroglioma
C. GBM
D. Anaplastic astrocytoma
Question 4.18 Radiographic changes early on in the treatment course
should always be responded with prompt management changes:
A. True. Radiographic changes early on reflect refractory disease and
should always prompt immediate treatment change.
B. False. Pseudo-progression is common within the first 3 months
postchemoradiation. Patients do not need to change management if
pseudo-progression is suspected. Novel imaging techniques can be
tried. Short-term follow-up MRIs are often performed.
Question 4.19 Intraoperative tumor tissue identification for surgical
margins can be done with this compound in tumors that enhance on imaging:
A. Methylene blue
B. 5-Aminoleuvulinic acid
C. OTL38
D. Bevacizumab-IRDye 800CW
Question 4.20 Which radiotherapy mode has been shown to minimally
disrupt neurocognitive outcomes in benign or low-grade brain tumors?
A. Whole brain radiation therapy (WBRT)
B. High-dose radiotherapy
C. X-ray radiation
D. Proton therapy
Question 4.21 A 55-year-old male has a resected low-grade glioma.
Pathologic determination of which molecular findings will help assess his
prognosis and risk?
A. IDH1/2 mutation status

https://t.me/ALGRAWANY33
B. 1p/19q deletion
C. TP53 mutation status
D. All of the above
Question 4.22 1p/19q codeletion is most commonly associated with which
low-grade glioma subtype and does it confer a better prognosis?
A. Oligodendroglioma
B. Ependymal tumors
C. Diffuse astrocytoma
D. Midline glioma
Question 4.23 Which of the following features is associated with favorable
prognosis in patients with low-grade gliomas?
A. Age < 40
B. Diameter ≥ 6 cm
C. Tumor crossing the midline
D. Astrocytoma histology
Question 4.24 A 45-year-old man presents with seizures. MRI reveals an
enhancing 6.5-cm tumor; biopsy reveals a non-1p/19q-deleted anaplastic
glioma. The addition of which adjuvant therapy to radiotherapy has been
shown to improve the overall survival (OS)?
A. Temozolomide (TMZ)
B. Nitrosourea
C. Bevacizumab
D. No adjuvant therapy needed
Question 4.25 In a patient with anaplastic oligodendrogliomas harboring
IDH1/2 mutation and 1p/19q codeletion, what is the best postoperative
treatment strategy that has been shown to prolong survival in phase III
clinical trials?
A. Radiation alone
B. Chemotherapy alone
C. Radiation and chemotherapy
D. No adjuvant therapy needed
Question 4.26 The antiangiogenic agent bevacizumab is approved for
recurrent GBM. What is the benefit of irradiation in addition to bevacizumab
treatment compared with bevacizumab alone?
A. Improved progression-free survival (PFS) and OS
B. Improved PFS but not OS
C. No difference in PFS or OS
D. Inconclusive
Question 4.27 What is the standard of care for elderly patients with newly
diagnosed GBM?
A. TMZ
B. Hypofractionated radiotherapy with concurrent TMZ
C. Short-course radiotherapy
D. Hypofractionated radiotherapy with concurrent TMZ followed by up to
six cycles of adjuvant TMZ
Question 4.28 A 76-year-old patient with newly diagnosed GBM with
multiple medical comorbidities, not eligible for combined chemoradiation
therapy, presents to you for treatment options. What is the next best step?
A. Longer course radiation alone, such as 60 Gray (Gy) in 30 daily
fractions over 6 weeks
B. Shorter course radiation (40 Gy in 15 fractions over 3 weeks) if
MGMT unmethylated
C. TMZ alone if MGMT methylated
D. B or C
Question 4.29 Which targeted therapies have been approved for second-
line therapy in GBM based on study on improved PFS and OS?
A. Bevacizumab
B. EGFR targeted therapies

https://t.me/ALGRAWANY33
C. Mechanistic target of rapamycin (mTOR) inhibitors
D. Dabrafenib
Question 4.30 What local treatment utilizes electrical current and has
prolonged survival when used concurrently with adjuvant TMZ in newly
diagnosed GBM?
A. Tumor-treating fields (TTFs)
B. Electron radiotherapy
C. Extracorporeal lithotripsy
D. High-intensity focused ultrasound
Question 4.31 Which factors affect the efficacy of TTFs?
A. Gender
B. Location of the tumor
C. Duration of use of the device
D. Prior radiation status
Question 4.32 The mechanism of which TTFs work on GBM is markedly
different from conventional therapies. What are the main adverse side
effects?
A. Nausea/vomiting
B. Pancytopenia
C. Local mild-moderate dermatitis
D. Hypertension and thromboembolic events
Question 4.33 A 49-year-old patient presents with altered mental status,
and MRI identifies a frontal mass with associated dural thickening. She
undergoes complete mass resection along with the involved dura. Histology
shows the World Health Organization (WHO) grade I meningioma. What is
the next step?
A. Radiotherapy alone
B. Watchful waiting
C. Adjuvant chemotherapy
D. Combination chemoradiation
Question 4.34 Early-onset, bilateral vestibular schwannoma is often
associated with which germ line mutation in which gene?
A. NF1
B. NF2
C. TSC
D. VHL
Question 4.35 What targeted therapy has been shown to treat bilateral
vestibular schwannomas associated with NF2?
A. Dabrafenib
B. Bevacizumab
C. Sorafenib
D. Tofacitinib
Question 4.36 A 67-year-old male with a history of stage IIc melanoma
treated surgically presents with altered mental status and is found to have
multiple brain lesions suggestive of metastatic disease. What is NOT
considered a routine practice?
A. BRAF mutation detection of the resected tissue
B. Empiric levetiracetam for seizure prophylaxis
C. Temporary use of steroids if cerebral edema is seen on imaging
D. Radiation oncology referral
Question 4.37 WBRT is associated with neurocognitive effects. Which
innovation in radiation delivery has been shown to minimize the
neurocognitive decline associated with WBRT?
A. Hippocampal avoidance
B. Hypofractionated therapy
C. Concurrent memantine
D. A and C

https://t.me/ALGRAWANY33
Question 4.38 For patients with limited-stage small-cell lung cancer and
complete response to induction therapy, which of the following adjuvant
therapy can decrease the likelihood of brain metastases and prolong OS?
A. Memantine
B. Prophylactic cranial irradiation
C. Intrathecal methotrexate
D. Serial lumbar punctures with cerebrospinal fluid (CSF) cytology

ANSWERS

Question 4.1 The correct answer is A.


TP53 mutations are almost universally a poor prognostic factor in
malignancies and are associated with a reduced survival rate in
medulloblastomas (5-year survival rate of 40%). RB is also a tumor
suppressor gene often mutated in malignancies but is not associated with
a reduced survival rate in medulloblastomas. MYC and EGFR both are
oncogenes. MYC amplification, not mutation, is found in group 3
medulloblastomas and carries a poorer prognosis.
Question 4.2 The correct answer is C.
The WNT-activated subgroup of medulloblastoma is least commonly
present in approximately 10% of medulloblastomas and is associated
with the best prognosis with a 5-year survival rate of >95%.1 The SHH
pathway is associated with an intermediate prognosis. MYC
amplification and TGF-β dysregulation are found predominately in
group 3 medulloblastoma and are associated with a poor prognosis.
Question 4.3 The correct answer is A.
Genomic rearrangements that juxtapose enhancer elements near
normally quiescent proto-oncogenes, termed “super-enhancers,” have
emerged as an important mechanism of oncogenesis in group 3 and
group 4 medulloblastoma. Microsatellite instability and loss of
heterozygosity are important drivers in many hereditary cancer
syndromes but are not significant in medulloblastoma. Field defect
refers to an area of cells that were exposed to the same carcinogenic
process, all with an increased risk of transformation, and is found with
carcinogen-associated cancers such as bladder cancer.
Question 4.4 The correct answer is B.
Genomic studies have reclassified medulloblastomas into four
subgroups with different behaviors and prognosis. Abnormalities in
SHH and WNT pathways independently drive medulloblastoma with
differing behaviors. IDH mutations and 1p/19q codeletion are important
in gliomas with a significant prognostic value. Alterations in the EGFR
pathway are found in many cancer types including gliomas but do not
significantly contribute to medulloblastoma oncogenesis.
Question 4.5 The correct answer is A.
Radiotherapy followed by adjuvant (often platinum-based)
chemotherapy is the standard of care after surgical debulking in
medulloblastoma. Given its classic midline or paramedian cerebellar
location, there is a high risk for ventricular entrapment and/or
progressive hydrocephalus often requiring the placement of a
ventriculoperitoneal shunt. Its location increases the risk of further CSF
seeding and spread throughout the entire neuraxis. As a result,
management following surgery traditionally consists of dose-reduced
craniospinal radiation with boost targeting the posterior fossa, followed
by adjuvant chemotherapy. With this approach, toxicity remains an issue
even with dose reduction. Therefore, a referral to a tertiary care center
with modern stereotactic proton-based radiation therapy (RT) is always
recommended as per the most current National Comprehensive Cancer
Network (NCCN) guidelines.
Question 4.6 The correct answer is A.
The standard of care for an ependymoma is surgery and radiation.
Surgery can be curative, and maximal safe resection is the goal.
Adjuvant radiotherapy improves recurrence-free survival. Outside of

https://t.me/ALGRAWANY33
investigational studies, chemotherapy is not a standard for newly
diagnosed ependymomas.
Question 4.7 The correct answer is A.
Gliomas are marked by multiple genetic alterations. IDH1/2 mutations
and MGMT methylation status are two independent prognostic factors
for a better outcome. IDH1/2 mutations are generally found in younger
patients. PTEN, TP53, and BRAF mutations are found to be commonly
altered in gliomas but do not portend a favorable prognosis.
Question 4.8 The correct answer is C.
Mutations in TP53 and ATRX, as well as loss of heterozygosity of
chromosome 17 (where TP53 is located), are commonly found in
astrocytomas. Oligodendrogliomas are characterized by IDH1/2
mutations and 1p/19q codeletions. GBMs have multiple oncogenic
drivers.
Question 4.9 The correct answer is C.
Oligodendrogliomas are characterized by IDH mutations and 1p/19q
codeletions. BRAF and ATRX mutations and MGMT methylation can be
found in multiple glioma subtypes.
Question 4.10 The correct answer is B.
Four distinct GBM subgroups have emerged based on molecular
signatures.2 The mesenchymal subgroup, which expresses mesenchymal
markers such as chitinase-3-like protein-1 and MET, is marked by
somatic alterations in NF1 and is associated with a particularly poor
outcome. PDGFR amplification is often found in proneural subtype of
GBM. EGFR and BRAF alterations are found to play a role in multiple
subtypes of GBMs.
Question 4.11 The correct answer is A.
MGMT promoter methylation is associated with an increased response to
alkylating agents. Methylation of MGMT results in decreased expression
of DNA repair genes, resulting in accumulation of permanent DNA
damage. MGMT promoter methylation is an important favorable
prognostic factor in GBM. Overexpression of the histone demethylase
KDM5A is associated with TMZ resistance. IDH1/2 wild-type is
associated with poorer prognosis than IDH1/2 mutation. 1p/19q
codeletion is a positive prognostic factor but is not an epigenetic finding.
Question 4.12 The correct answer is A.
Recent studies in genome-wide association studies have helped define
the genetic susceptibility of glioma development. Ten independent
inherited variants in eight chromosomal regions were associated with
20% to 40% increased risk for adult glioma. The rs78378222 variant of
TP53 is associated with a twofold increase in risk.
Question 4.13 The correct answer is D.
Genetic studies have identified recurrent alterations in key signaling
pathways such as receptor tyrosine kinase (67.3% including EGFR,
PDGFRA, and FGFR3), TP53 (85.3%), RB1 (78.9%), and PI3K
(25.1%).
Question 4.14 The correct answer is C.
In the United States, the most common malignant adult CNS cancer is
GBM. In children, the most common malignant CNS cancer is
medulloblastoma. The most common benign CNS tumors are
meningiomas.
Question 4.15 The correct answer is C.
There is a strong association between ionizing radiation and the risk of
developing primary CNS tumors. Benzene exposure is associated with
hematologic malignancies. Aflatoxin is a potent fungal-derived
carcinogen, usually associated with hepatocellular carcinoma (HCC).
Betel nut chewing is associated with head and neck cancers.
Question 4.16 The correct answer is A.

https://t.me/ALGRAWANY33
Primary CNS lymphoma is associated with Epstein–Barr virus infection,
often in the setting of immunosuppressive states such as human
immunodeficiency virus (HIV) and posttransplant immunosuppression.
Primary CNS lymphoma is an acquired immune deficiency syndrome
(AIDS)-defining disease in patients with HIV, along with Kaposi’s
sarcoma, high-grade non-Hodgkin’s lymphoma, and invasive cervical
carcinoma.
Question 4.17 The correct answer is B.
Calcifications are typical of oligodendroglioma. MRI findings,
otherwise, are unable to discern between different histologic subtypes.
The widely held convention that enhancing tumors are high-grade
tumors and nonenhancing tumors are mostly low-grade tumors has been
put into question recently.
Question 4.18 The correct answer is B.
As many as 25% to 40% of tumors after multimodal treatment will have
radiographic changes that may not reflect disease change. In the setting
of pseudo-progression, surgical excision of these lesions can show large
areas of necrosis. Treatment with antiangiogenic agents can cause rapid
resolution of tumor enhancement on imaging that does not correlate with
disease progression, which is termed “pseudo-response.”3
Question 4.19 The correct answer is B.
5-Aminolevulinic acid is a prodrug that is converted into fluorescent
porphyrins by glioma cells and can be visualized intraoperatively.
Methylene blue is often used for sentinel lymph node detection in breast
cancer surgery. OTL38 is a fluorophore conjugated to a folate receptor
α–targeting ligand and binds selectively to folate receptor α; its use for
defining surgical margins in folate expressing lung tumors is currently
being investigated. Bevacizumab-IRDye 800CW is an antibody–
fluorophore conjugate being investigated for gastrointestinal cancers.
Question 4.20 The correct answer is D.
For primary CNS low-grade gliomas, defined radiation fields can be
more precisely delineated with proton therapy. A prospective study from
Massachusetts General Hospital with 20 patients undergoing proton
therapy for low-grade gliomas showed stable cognitive function and
quality of life.4
Question 4.21 The correct answer is D.
Genome-wide analysis of low-grade gliomas has determined three
prognostically significant subtypes based on molecular abnormalities;
patients with IDH mutations and 1p/19q codeletions had the most
favorable prognosis. Patients with IDH mutations without 1p/19q
codeletion that were associated with TP53 mutation and ATRX
inactivation had intermediate prognosis. Patients without IDH mutations
but with mutations in PTEN, EGFR, NF1, TP53, PIK3CA, PTPN11, and
PLCG1 had the worst prognosis.
Question 4.22 The correct answer is A.
The 2016 update of the WHO classification of CNS tumors defines
oligodendroglial tumors as harboring both IDH1/2 mutation and
codeletion of chromosome 1p/q19. These molecular findings are
associated with a good prognosis.
Question 4.23 The correct answer is A.
Pignatti et al. developed a scoring system to identify patients with low-
grade gliomas at varying levels of risk for mortality.5 Negative risk
factors include age 40 or greater, astrocytoma histology, maximum
diameter of 6 cm or greater, tumor crossing the midline, and the
presence of neurologic deficits. Patients with up to two factors were
considered at low risk (median survival = 7.7 years), and patients with
three or more factors were considered at high risk (median survival = 3.2
years).
Question 4.24 The correct answer is A.
In the EORTC 26053 CATNON trial, 751 patients with newly
diagnosed 1p/19q non-codeleted anaplastic glioma were randomized to
RT alone, concurrent RT and TMZ, concurrent RT and TMZ with

https://t.me/ALGRAWANY33
additional adjuvant TMZ, and RT followed by adjuvant TMZ.6 An
interim analysis found that adjuvant TMZ confers a survival benefit. The
role of concurrent TMZ is still undetermined, and a longer follow-up is
needed. There are no trials comparing nitrosourea to TMZ.
Bevacizumab is approved for recurrent, but not newly diagnosed
anaplastic astrocytoma.
Question 4.25 The correct answer is C.
Adjuvant chemoradiation is beneficial for patients with anaplastic
oligodendroglioma, especially with IDH1/2 mutation and 1p/19q
codeletion. These tumors are radiation-sensitive, and there is no level 1
evidence for chemotherapy alone. Further studies have also shown that
chemoradiation is superior to radiation alone for these patients. The
choice of adjuvant chemotherapy is currently being investigated in the
CODEL study.
Question 4.26 The correct answer is B.
The phase II, multi-institutional RTOG 1205 trial in recurrent GBM
found that combination irradiation and bevacizumab improved PFS, but
not mean survival time.7
Question 4.27 The correct answer is D.
The EORTC 26062-22061 phase III trial randomized newly diagnosed
GBM patients aged 65 or older to hypofractionated radiotherapy (40
Gy/15 fractions) with concurrent and adjuvant TMZ to radiotherapy
alone. Combined modality therapy with TMZ showed significant
prolonged OS.8 Therefore, the current standard of care for newly
diagnosed elderly GBM is a combination of hypofractionated
radiotherapy and TMZ chemotherapy. In addition, MGMT methylation
is associated with better prognosis and better treatment response with
TMZ.
Question 4.28 The correct answer is D.
Studies have shown that TMZ alone or hypofractionated radiotherapy is
superior to conventional radiotherapy, that short-course radiotherapy is
noninferior to long-course radiotherapy, and that chemoradiation is
superior to radiation alone. MGMT methylation status is a useful marker
for benefit from TMZ.
Question 4.29 The correct answer is A.
Bevacizumab was approved for recurrent GBM in 2009 when it was
shown to significantly improve PFS compared with historical controls,
either alone or in combination with irinotecan. Subsequent studies have
continued to demonstrate its utility in recurrent GBM.
Question 4.30 The correct answer is A.
TTF therapy was approved for newly diagnosed GBM in 2015 based on
EF-14 study.9 Electron radiotherapy mechanism is based on ionizing
radiation. Extracorporeal lithotripsy utilizes sound wave to treat kidney
stones. High-intensity focused ultrasound is an experimental technique
for laparoscopic surgery
Question 4.31 The correct answer is C.
Recent studies have shown that the effectiveness of TTF is directly
related to how long the patient wears the device. The electrical fields are
programmed to be configured to the target location. Therefore, tumor
location does not affect efficacy. There is no evidence that gender
affects response to TTF. There is no evidence that prior radiation status
affects response to TTF.
Question 4.32 The correct answer is C.
TTF can cause local mild-to-moderate dermatitis at the site of the
transducers, usually treated with topical steroids. It is very well
tolerated, and compliance is high. Conventional cytotoxic
chemotherapies can cause nausea/vomiting. TMZ can be
myelosuppressive, and bevacizumab can cause hypertension and
thromboembolic events.
Question 4.33 The correct answer is B.

https://t.me/ALGRAWANY33
A WHO grade I meningioma with complete tumor resection and
resection of the involved dura constitutes Simpson grade 2 resection, for
which watchful waiting is appropriate. Risk for recurrence of resected
meningioma is guided by the five grades of resection. Simpson grade 5
is biopsy only and is associated with higher rates of progression.
Simpson grade 4 is partial tumor resection and is associated with 44%
recurrence rate. Simpson grade 3 resection is gross tumor resection of
the tumor without addressing the bone or dural attachments and is
associated with 29% of relapse. Simpson grade 2 resection includes
removal of the dural attachments, and the relapse rate is 19%. In
Simpson grade 1 resection, the hyperostotic bone is also removed, and
the relapse rate is 9%.
Question 4.34 The correct answer is B.
Bilateral schwannomas are almost always associated with NF2 mutation
and are one of the criteria for diagnosis. Additionally, patients are also at
risk of developing meningiomas and ependymomas. NF1 mutations are
not associated with bilateral schwannomas. Tuberous sclerosis complex
(TSC) is associated with subependymal giant cell astrocytomas. Von
Hippel–Lindau (VHL) is associated with hemangioblastomas, other
angiomas, and renal cell carcinomas.
Question 4.35 The correct answer is B.
Bevacizumab has been shown to shrink NF2-associated bilateral
schwannomas. A recent study showed that high-dose bevacizumab is not
superior to the standard-dose bevacizumab. Dabrafenib is a BRAF
inhibitor used for melanoma. Sorafenib is a receptor tyrosine kinase
(RTK) inhibitor used for HCC. Tofacitinib is a Janus kinase 3 (JAK3)
inhibitor and is used for rheumatoid arthritis.
Question 4.36 The correct answer is B.
Empiric seizure prophylaxis is not indicated in patients with brain
metastases and no prior history of seizures. Evaluation by radiation
oncology and steroids for cerebral edema are common management
strategies of brain metastasis. BRAF and MEK inhibitors have been
shown to have intracranial activity; therefore, BRAF testing for
melanoma is appropriate.
Question 4.37 The correct answer is D.
WBRT is easy to deliver but is associated with significant toxicity.
RTOG 0933 demonstrated that hippocampal avoidance during WBRT
resulted in less neurocognitive decline compared with historical
controls.10 The RTOG 0614 found that patients on memantine plus
WBRT arm had significantly longer time to cognitive decline than those
on the WBRT arm alone.11 Hypofractionated radiotherapy does not
confer a survival benefit.
Question 4.38 The correct answer is B.
Small-cell lung cancer has a high propensity for intracranial
micrometastases, even at a limited stage. A meta-analysis of
prophylactic cranial irradiation in small-cell lung cancer found that in
patients who achieved a complete systemic response, prophylactic
cranial irradiation resulted in a survival benefit. Intrathecal methotrexate
is used for prophylaxis against brain recurrence in acute lymphocytic
leukemia. There is no role for serial lumbar punctures to monitor CSF
cytology in small-cell lung cancer.

References
1. Ellison DW, Dalton J, Kocak M, et al. Medulloblastoma: clinicopathological correlates of SHH,
WNT, and non-SHH/WNT molecular subgroups. Acta Neuropathol. 2011;121:381–396.
2. Louis DN. Molecular pathology of malignant gliomas. Annu Rev Pathol. 2006;1:97–117.
3. Wen PY, Macdonald DR, Reardon DA, et al. Updated response assessment criteria for high-grade
gliomas: response assessment in neuro-oncology working group. J Clin Oncol. 2010;28:1963–1972.
4. Shih HA, Sherman JC, Nachtigall LB, et al. Proton therapy for low-grade gliomas: results from a
prospective trial. Cancer. 2015;121:1712–1719.
5. Pignatti F, van den Bent M, Curran D, et al. Prognostic factors for survival in adult patients with
cerebral low-grade glioma. J Clin Oncol. 2002;20:2076–2084.
6. van den Bent MJ, Baumert B, Erridge SC, et al. Interim results from the CATNON trial (EORTC
study 26053-22054) of treatment with concurrent and adjuvant temozolomide for 1p/19q non-co-
deleted anaplastic glioma: a phase 3, randomised, open-label intergroup study. Lancet.
2017;390:1645–1653.
7. Tsien C, Pugh S, Dicker AP, et al. Randomized phase II trial of re-irradiation and concurrent

https://t.me/ALGRAWANY33
bevacizumab versus bevacizumab alone as treatment for recurrent glioblastoma (NRG
oncology/RTOG 1205): initial outcomes and RT plan quality report. Int J Radiat Oncol Biol Phys.
2019;105:S78.
8. Perry JR, O’Callaghan CJ, Ding K, et al. A phase III randomized controlled trial of short-course
radiotherapy with or without concomitant and adjuvant temozolomide in elderly patients with
glioblastoma (NCIC CTG CE.6, EORTC 26062-22061, RTOG 08.02, NCT00482677). Neuro
Oncol. 2014;16:iii46.
9. Stupp R, Taillibert S, Kanner AA, et al. Maintenance therapy with tumor-treating fields plus
temozolomide vs temozolomide alone for glioblastoma: a randomized clinical trial. JAMA.
2015;314:2535–2543.
10. Gondi V, Pugh SL, Tome WA, et al. Preservation of memory with conformal avoidance of the
hippocampal neural stem-cell compartment during whole-brain radiotherapy for brain metastases
(RTOG 0933): a phase II multi-institutional trial. J Clin Oncol. 2014;32:3810–3816.
11. Brown PD, Pugh S, Laack NN, et al. Memantine for the prevention of cognitive dysfunction in
patients receiving whole-brain radiotherapy: a randomized, double-blind, placebo-controlled trial.
Neuro Oncol. 2013;15:1429–1437.
___________
Corresponding chapters in DeVita, Hellman, and Rosenberg's Cancer: Principles & Practice of
Oncology, Eleventh Edition: 93 (Molecular Biology of Central Nervous System Tumors), 94
(Neoplasms of the Central Nervous System).
5 Cancer of the Head and Neck
Peter Oppelt, Kevin T. Palka, Jessica Ley, and Douglas R. Adkins

QUESTIONS

Each of the numbered items below is followed by lettered answers. Select the
ONE lettered answer that is BEST in each case unless instructed otherwise.

Question 5.1 A 65-year-old man with a 40 pack-year history of smoking


presented with hoarseness and a neck mass. The patient did not aspirate and
had not lost weight. He was found to have T4N2cM0 squamous cell
carcinoma (SCC) of the glottic larynx. T4 classification was based on limited
cortical erosion of the thyroid cartilage. A total laryngectomy and bilateral
neck dissections followed by adjuvant radiation therapy were recommended
by the otolaryngologist. However, the patient preferred a chance to preserve
the larynx. Which treatment approach offers the best chance to preserve the
patient’s larynx and prevent disease recurrence?
A. Radiation therapy alone
B. Induction chemotherapy followed by radiation therapy
C. Concurrent chemotherapy and radiation therapy
D. Chemotherapy alone
Question 5.2 Which of the following statements about the epidermal
growth factor receptor (EGFR) in head and neck SCC (HNSCC) is TRUE?
A. The EGFR inhibitor cetuximab with concurrent radiation improved
overall survival in locally advanced human papillomavirus (HPV)-
related oropharyngeal SCC compared with cisplatin with concurrent
radiation.
B. Activating EGFR mutations are frequent in HNSCC.

https://t.me/ALGRAWANY33
C. The EGFR inhibitor cetuximab improved overall survival when added
to definitive radiation therapy compared with radiation alone.
D. High EGFR expression is associated with sensitivity to radiation
therapy.
Question 5.3 Which of the following are risk factors for HNSCC?
A. Tobacco use
B. Alcohol use
C. Fanconi anemia
D. All of the above
Question 5.4 Which of the following characteristics are typical features of
HPV-related HNSCC?
A. Most patients have limited or no smoking history and have primary
tumors that involve the oropharynx (base of tongue or palatine tonsils).
B. Survival outcomes are substantially better than in patients with
smoking-associated HNSCC.
C. The role of the HPV in subsites other than the oropharynx is not clear.
D. All of the above.
Question 5.5 A 56-year-old man presented to his primary care physician
with a 3-month history of right ear pain and two masses on the right side of
his neck. On examination, he was found to have a 3-cm right palatine tonsil
mass and two enlarged lymph nodes under the upper part of the right
sternocleidomastoid muscle. One lymph node was 2 cm and the other was 1
cm in diameter. A fine-needle aspiration (FNA) of one of the lymph nodes
was performed, and it was consistent with p16+ SCC. Neck and chest
computed tomography (CT) showed no additional lymphadenopathy or
distant metastasis. Which of the following is TRUE about his curative
treatment options?
A. Given the overall excellent prognosis of HPV-related HNSCC, he can
safely be treated with radiation alone.
B. Cisplatin dosed at 30 mg/m2 weekly with radiation provides equivalent
local–regional control compared with cisplatin 100 mg/m2 every 3
weeks with radiation.
C. Cisplatin given with radiation improves his chance of overall survival
compared with cetuximab given with radiation.
D. HPV-related SCCs tend to be resistant to radiation-based treatments
and should preferentially be treated surgically.
Question 5.6 A 56-year-old man presents with a 3-month history of
epistaxis and nasal congestion. Over the past month, he has noted enlarging,
firm bilateral masses in his lower neck. CT scan of his neck shows a 6-cm
mass in the nasopharynx with apparent invasion of the skull base, along with
multiple enlarged cervical lymph nodes, the largest of which is 6 cm.
Magnetic resonance imaging (MRI) of the base of skull confirms invasion.
Staging positron emission tomography (PET) scan reveals no evidence of
distant metastasis. Biopsy of the largest cervical lymph node shows
nonkeratinizing SCC, which is positive for Epstein–Barr virus (EBV).
Complete blood count (CBC) and comprehensive metabolic profile are
unremarkable. Which of the following treatment approaches has been shown
to provide the greatest benefit for overall survival?
A. Surgery followed by adjuvant radiation with concurrent cisplatin (100
mg/m2 for three cycles)
B. Induction therapy with concurrent radiation and three cycles of
cisplatin followed by surgery
C. Concurrent radiation and cisplatin (100 mg/m2 for three cycles)
D. Induction chemotherapy with three cycles of gemcitabine and cisplatin,
followed by concurrent radiation with cisplatin (100 mg/m2 for three
cycles)
E. Induction chemotherapy with three cycles of gemcitabine and cisplatin,
followed by surgery
Question 5.7 A 64-year-old man with extensive chewing tobacco history
presented with an ulcerated mass of the left oral tongue. Imaging of the neck
and chest confirms a 3-cm left oral mass with bilateral cervical adenopathy
and pulmonary nodules measuring up to 2 cm in size. Biopsies of the tongue
mass and pulmonary metastases confirm moderately differentiated SCC. He
was treated with first-line pembrolizumab, but had clear clinical and

https://t.me/ALGRAWANY33
radiographic progression. Which of the following statements is TRUE?
A. The combination of cisplatin and 5-fluorouracil (5-FU) improves
overall survival in comparison with single-agent methotrexate in
patients with metastatic HNSCC.
B. Cetuximab has no activity in metastatic or recurrent HNSCC.
C. The combination of cisplatin and 5-FU results in a higher tumor
response rate than single-agent methotrexate in patients with metastatic
HNSCC.
D. Higher chemotherapy doses improve overall survival.
Question 5.8 A 66-year-old man presented to his otolaryngologist with a 4-
cm left floor of mouth SCC. A 3-cm left neck node was felt on examination.
CT of the neck also showed that the primary tumor invaded into the
mandible. There was no evidence of distant metastasis on CT of the chest.
The patient’s largest concern is overall survival. What is the most appropriate
initial therapy for this patient?
A. Surgery
B. Radiation therapy
C. Concurrent chemotherapy and radiation therapy
D. Chemotherapy
Question 5.9 A 54-year-old woman recently underwent resection of an
SCC of the supraglottis with bilateral neck dissections. The pathology
showed a T2 lesion completely excised to negative margins, but
lymphovascular invasion and perineural invasion were present. Two of the 28
lymph nodes contained SCC with extracapsular extension. What is the most
appropriate adjuvant therapy for this patient?
A. Observation
B. Radiation alone
C. Concurrent chemotherapy and radiation therapy
D. Chemotherapy alone
Question 5.10 What is the benefit of cetuximab in patients with HNSCC?
A. Concurrent cetuximab and radiation therapy resulted in improved
overall survival compared with radiation therapy alone in stage I to IV
disease.
B. Cetuximab is equally effective as cisplatin when given concurrently
with radiation therapy.
C. The addition of cetuximab to concurrent cisplatin and radiation therapy
improved overall survival and reduced relapse risk.
D. Concurrent cetuximab and radiation therapy resulted in improved
overall survival compared with radiation therapy alone in
nonmetastatic stage III to IV HNSCC.
Question 5.11 Which of the following statements about the treatment of
laryngeal SCC is TRUE?
A. Radiation therapy alone resulted in poorer overall survival compared
with concurrent chemotherapy and radiation therapy in patients with
locally advanced laryngeal SCC.
B. Concurrent chemotherapy and radiation therapy resulted in better local
tumor control and had a higher laryngeal preservation rate than
radiation therapy alone for locally advanced SCC of the larynx.
C. A total laryngectomy is required for the treatment of locally advanced
SCC of the larynx.
D. Large-volume laryngeal SCC is defined as tumor that extends more
than 1 cm onto the base of tongue and/or extension through the thyroid
cartilage into adjacent soft tissue and is best treated with nonsurgical
therapy.
Question 5.12 There is an overall survival benefit with adjuvant
chemotherapy given after resection of an SCC of the larynx.
A. True
B. False
Question 5.13 In patients with metastatic HNSCC who are treated with
immunotherapy (pembrolizumab or nivolumab), what is the most common
immune-mediated toxicity?

https://t.me/ALGRAWANY33
A. Colitis
B. Pneumonitis
C. Hypothyroidism
D. Hypophysitis
E. Hepatitis
Question 5.14 A 56-year-old woman presented to her primary care
physician with left-sided facial swelling and facial drooping. The patient had
weakness of the lower portion of her face and a palpable left parotid mass. A
biopsy was performed and showed high-grade mucoepidermoid carcinoma.
There were no palpable neck lymph nodes. What is the best treatment for this
patient?
A. Radiation alone
B. Concurrent chemotherapy and radiation therapy
C. Surgery alone
D. Surgery followed by adjuvant radiation therapy
Question 5.15 Which of the following statements is TRUE about locally
advanced HNSCC?
A. The most common long-term complication of radiation therapy is
xerostomia.
B. Once the patient is able to receive nutrition via a G-tube, there is no
benefit from having the patient continue to swallow during radiation
therapy.
C. Intensity-modulated radiation therapy (IMRT) increases the risk of
xerostomia.
D. The severity of the acute side effects of radiation therapy is similar
when chemotherapy is given concurrently.
Question 5.16 Which of the following statements are TRUE regarding
second primary cancers in patients with HNSCC?
A. Patients with HPV-related oropharyngeal SCC have a similar risk of
second primary cancers as do patients with smoking-induced HNSCC.
B. Patients with resected HNSCC who quit smoking have a lower risk of
second primary cancers than those who continue to smoke.
C. Treatment with α-tocopherol decreases the incidence of second primary
cancers.
D. Treatment with isotretinoin decreases the incidence of second primary
cancers.
Question 5.17 Which of the following statements regarding induction
chemotherapy in the treatment of HNSCC is TRUE?
A. Treatment with concurrent chemotherapy and radiation therapy
resulted in improved overall survival compared with induction
chemotherapy followed by radiation therapy for locally advanced
laryngeal SCC.
B. The addition of docetaxel to cisplatin and 5-FU was associated with
similar overall survival when compared to cisplatin and 5-FU in
patients with locally advanced HNSCC subsequently treated with
radiation therapy alone or concurrent carboplatin and radiation therapy.
C. Induction chemotherapy may facilitate organ preservation in patients
with locally advanced laryngeal or hypopharyngeal SCC.
D. Induction chemotherapy followed by concurrent cisplatin and radiation
therapy results in improved overall survival compared with concurrent
cisplatin and radiation therapy.
Question 5.18 A 35-year-old nonsmoking female with a history of chronic
oral lichen planus presented with a painful right oral tongue lesion measuring
2.5 cm. Biopsy revealed well-differentiated SCC. Ultrasound revealed that
the oral tongue lesion was 0.6 cm thick. CT showed no abnormal neck nodes.
The lesion was widely excised, and a right selective neck dissection (SND)
was performed. Pathology revealed a 2.6-cm SCC, negative surgical margins,
no perineural or lymphovascular invasion, and all 15 lymph nodes without
SCC. The pathologic stage was II (T2N0M0). The most appropriate next
therapy is:
A. Postoperative adjuvant radiation therapy
B. Postoperative adjuvant concurrent chemotherapy and radiation therapy

https://t.me/ALGRAWANY33
C. Close monitoring
D. Left SND
Question 5.19 A 65-year-old man is treated with surgery and adjuvant
radiation for stage IVA SCC of the hypopharynx, with a complete response to
treatment. Two years later, he is noted to have five new pulmonary nodules in
both lungs. The largest nodule was biopsied, and pathology returns as SCC,
consistent with metastatic disease from his prior hypopharyngeal cancer. His
combined positive score (CPS) was noted to be 0. PET scan shows no other
sites of disease, and his laboratory studies are unremarkable. What regimen
has been shown to provide the greatest benefit for overall survival?
A. Pembrolizumab monotherapy
B. Carboplatin, 5-FU, and pembrolizumab
C. Carboplatin, 5-FU, and cetuximab
D. Carboplatin and cetuximab
E. Carboplatin, paclitaxel, and bevacizumab
Question 5.20 A 65-year-old male presented with a large hard palate mass.
Biopsy showed a salivary ductal adenocarcinoma. The mass was completely
excised, and the pathology revealed a 4-cm tumor with positive surgical
margins. Postoperative adjuvant radiation therapy was given. Nine months
later, a surveillance CT showed several pulmonary nodules. A biopsy of one
of the pulmonary nodules showed metastatic salivary ductal adenocarcinoma.
The patient was referred to medical oncology for management. The most
appropriate next step in the management of this patient is:
A. Weekly cetuximab
B. Carboplatin, 5-FU, and cetuximab
C. Paclitaxel
D. Perform immunohistochemistry stain for estrogen and progesterone
receptors
E. Perform immunohistochemistry stain for the androgen receptor

ANSWERS
Question 5.1 The correct answer is C.
Radiation Therapy Oncology Group (RTOG) 91-11 was a three-arm
randomized trial of patients with laryngeal SCC who were candidates
for total laryngectomy but wanted to preserve their larynx. These
patients were randomly assigned to receive radiation alone, induction
chemotherapy followed by radiation, or concurrent chemotherapy and
radiation therapy. Concurrent chemotherapy and radiation therapy
resulted in the highest chance for laryngeal preservation and disease
control. However, overall survival was similar in the three treatment
arms.1
Question 5.2 The correct answer is C.
EGFR plays a key role in the development of HNSCC. EGFR
overexpression leads to activation of multiple signaling pathways that
lead to cell growth and resistance to apoptosis. Activating EGFR
mutations are infrequent in HNSCC. The addition of cetuximab to
definitive radiation therapy improved the survival compared with
radiation therapy alone in a randomized clinical trial.2 The analysis of
EGFR expression in archived tumor tissue from a prospective RTOG
clinical trial demonstrated that EGFR overexpression is associated with
resistance to radiation therapy. In contrast to monoclonal antibody
inhibitors of EGFR such as cetuximab, no randomized trial has been
performed to show clinical benefit of an oral tyrosine kinase inhibitor
(TKI) of EGFR such as erlotinib in HNSCC.
Question 5.3 The correct answer is D.
Several risk factors for the development of HNSCC have been
identified. The two most important are tobacco and alcohol use, which
seem to be synergistic. Other risk factors include occupational exposure
to chemicals and irritants, such as aromatic hydrocarbons and wood
dust, and viruses, such as HPV and EBV. In addition, patients with
Fanconi anemia are also at increased risk.
Question 5.4 The correct answer is D.

https://t.me/ALGRAWANY33
It has been recognized over the last decade that HPV is the cause of an
increasing proportion of SCC of the oropharynx. Today, most cases of
oropharyngeal SCC are due to HPV. However, smoking can also
contribute to the biology of HPV-related SCC. These tumors are often
described as having a basal cell–like or poorly differentiated appearance
and are frequently nonkeratinizing. These tumors are more commonly
found in never-smokers or those patients with a limited smoking history.
HPV-related oropharyngeal SCC has a much better prognosis than other
oral mucosal HNSCC. The overall survival is higher, and the risks of
primary cancer relapse and secondary cancer development are much
lower in comparison with HPV-unrelated HNSCC. Patients with HPV-
related HNSCC have a better tumor response to chemotherapy and
radiation therapy than patients with smoking-associated HNSCC.
Question 5.5 The correct answer is C.
Given the markedly improved outcomes for HPV-related oropharyngeal
carcinoma, attempts to deintensify curative treatments have been an area
of active research. However, two randomized phase III studies, the
RTOG 10163 and the De-ESCALaTE (Determination of Cetuximab
Versus Cisplatin Early and Late Toxicity Events in HPV+ OPSCC)
trial,4 demonstrated that concurrent high-dose cisplatin with concurrent
radiation was associated with improved overall survival compared with
cetuximab with radiation. During concurrent radiation therapy, low-dose
weekly cisplatin (30 mg/m2) was associated with higher locoregional
failure rates compared with high-dose bolus cisplatin (100 mg/m2) given
every 3 weeks in a randomized clinical trial.5 While primary surgery can
be considered for HPV-related oropharyngeal carcinoma, radiation-
based approaches offer a high cure rate.
Question 5.6 The correct answer is D.
In a large, phase III, randomized controlled trial, the addition of
induction chemotherapy to chemoradiotherapy resulted in a significant
improvement in outcomes compared with chemoradiotherapy alone
including increases in recurrence-free survival at 3 years (85% vs. 77%)
and overall survival at 3 years (95% vs. 90%).6 Surgery is not
recommended for initial treatment of locally advanced nasopharyngeal
carcinoma, particularly with base of skull invasion.
Question 5.7 The correct answer is C.
Many chemotherapeutic agents including cisplatin, carboplatin, 5-FU,
taxanes, and methotrexate have activity against metastatic HNSCC.
Combinations of conventional cytotoxic chemotherapy and higher doses
of chemotherapy result in higher tumor response rates but have not
shown an overall survival benefit compared with single-agent cytotoxic
chemotherapy. The anti-EGFR antibody cetuximab has shown modest
activity against HNSCC. The EXTREME (Erbitux in First-Line
Treatment of Recurrent or Metastatic Head and Neck Cancer) trial
demonstrated that the addition of cetuximab to platinum-based
chemotherapy improved overall survival compared with chemotherapy
alone in patients with incurable HNSCC.7 Cetuximab is also approved
for the treatment of platinum-refractory incurable HNSCC.
Question 5.8 The correct answer is A.
This patient has a stage IVA (T4aN1M0) SCC of the oral cavity. The
primary therapy that is most effective in most cases of oral cavity SCC
is surgery. Primary radiation therapy alone, chemotherapy alone, or
concurrent chemotherapy would not be considered to be as effective,
particularly with invasion of the mandible. The patient will likely
require postoperative adjuvant radiation therapy alone or with
chemotherapy, depending on the pathologic findings.
Question 5.9 The correct answer is C.
After gross tumor resection, adjuvant therapy with either radiation
therapy or concurrent chemotherapy and radiation therapy is
recommended for most cases of locally advanced HNSCC. Pathologic
findings determine which adjuvant therapy should be recommended.
The major adverse pathologic risk factors for local recurrence are
positive surgical margins or extracapsular nodal extension. Minor
adverse pathologic risk factors for recurrence include T3 or T4 primary
tumors, N2 or N3 nodal disease, and perineural or lymphovascular

https://t.me/ALGRAWANY33
invasion. If the patient has any major risk factor, adjuvant concurrent
chemotherapy and radiation therapy is recommended based on the
results of two randomized trials.8 If the patient has any minor risk factor,
adjuvant radiation therapy is recommended. If the patient has no adverse
risk factors, observation is recommended.
Question 5.10 The correct answer is D.
Cetuximab is an EGFR inhibitory antibody that has been used alone or
combined with chemotherapy for the treatment of metastatic disease and
combined with radiation therapy to treat locally advanced stage III-IV
HNSCC. In a randomized phase III trial that compared concurrent
radiation therapy with cetuximab to radiation therapy alone for
nonmetastatic stage III/IV HNSCC of the oropharynx, larynx, or
hypopharynx, cetuximab improved both local disease control and overall
survival. In patients receiving cetuximab with radiation therapy, the
incidence of acne and infusion reactions were increased, but the
incidence of mucositis was similar.2 The RTOG 0522 trial showed the
lack of benefit and increased acute toxicity with the addition of
cetuximab to concurrent cisplatin and radiation therapy.9
Question 5.11 The correct answer is B.
The treatment of larynx cancers has changed significantly over the last
30 years. Historically, the traditional treatment approach for locally
advanced SCC of the larynx was a total laryngectomy and neck
dissection followed by adjuvant radiation therapy. This resulted in loss
of speech, even if the quality of the speech was acceptable before
therapy. The Veterans Administration Laryngeal Study Group Trial was
a randomized trial that compared total laryngectomy followed by
adjuvant radiation therapy with induction chemotherapy followed by
radiation therapy. In the latter treatment arm, salvage total laryngectomy
was performed in patients with persistent or recurrent local disease. This
pivotal trial established that overall survival was similar between the two
treatment arms; however, approximately two-thirds of patients had
laryngeal preservation with the nonsurgical treatment approach.
Subsequently, RTOG 91-11 was a three-arm randomized trial that
compared radiation therapy alone, induction chemotherapy followed by
radiation therapy, and concurrent chemotherapy with radiation therapy
for the treatment of patients with locally advanced laryngeal SCC.1 This
trial excluded patients with large-volume primary tumors who were felt
to benefit from a primary surgical approach. RTOG 91-11 demonstrated
that the best chance for laryngeal preservation was when patients were
treated with concurrent chemotherapy and radiation therapy, even
though overall survival was the same in each of the three treatment
arms. However, subsequent improvements in surgical techniques
including transoral endoscopic laser resection and transoral robotic
surgery have also provided surgical approaches to preserve the larynx.
Question 5.12 The correct answer is B.
Adjuvant chemotherapy has been tested in several trials of HNSCC
without resulting in survival improvement. Currently, adjuvant
chemotherapy is only used in the treatment of patients with locally
advanced nasopharyngeal SCC following concurrent chemotherapy and
radiation therapy based on the result of the Intergroup 0099 trial.10 Even
in this setting, the benefit of adjuvant chemotherapy is unclear.
Question 5.13 The correct answer is C.
The KEYNOTE-048 was a large trial where patients with recurrent or
metastatic HNSCC were randomized to pembrolizumab alone,
pembrolizumab plus chemotherapy, or cetuximab plus chemotherapy.
Hypothyroidism was one of the most common adverse events, occurring
in 18% of patients treated with single-agent pembrolizumab and 16% of
those treated with pembrolizumab combined with chemotherapy.11
Question 5.14 The correct answer is D.
The treatment of choice for malignant tumors of the parotid gland is
surgery. High-grade tumors and tumors with perineural invasion are also
treated with adjuvant radiation therapy. There is no clear role for
adjuvant chemotherapy in salivary gland cancers that have been resected
or in those treated with definitive or postoperative radiation therapy.

https://t.me/ALGRAWANY33
Question 5.15 The correct answer is A.
Xerostomia is the most common long-term complication of radiation
therapy for the treatment of HNSCC. Xerostomia impairs swallowing
function and sleeping and results in poor dentition. Radiation to the head
and neck can cause acute and late toxicities. The degree of toxicity from
radiation varies from patient to patient and depends on the dose of
radiation, the radiation port, the radiation technique, and whether
chemotherapy is given concurrently. The addition of chemotherapy to
radiation therapy increases the acute toxicity. IMRT allows varying
doses of radiation to be given to different parts of a radiation field.
IMRT permits the parotid gland and other major salivary glands to be
spared from treatment, resulting in a lower risk of xerostomia. To
decrease the risk of long-term swallowing dysfunction, patients should
be strongly encouraged to continue to swallow to exercise the
pharyngeal muscles even if they are receiving the majority of nutrition
via a gastrostomy tube.
Question 5.16 The correct answer is B.
Prospective trials on secondary chemoprevention with isotretinoin and
α-tocopherol have shown no significant benefit in preventing second
primary cancers in patients with resected HNSCC. Smoking status is an
important risk factor for the development of second primaries, and
never-smokers have lower risk for second primaries compared with
former- and current-smokers. Patients who quit smoking have a lower
risk of second primary cancers than those patients who continue to
smoke. The risk of second cancers is lower in patients with HPV-related
oropharyngeal SCC than in those with smoking-induced HNSCC.
Question 5.17 The correct answer is C.
Induction chemotherapy followed by definitive radiation therapy has the
potential for organ preservation in patients with locally advanced SCC
of the larynx or hypopharynx. However, there has been no improvement
in overall survival. The addition of docetaxel to cisplatin and 5-FU was
associated with improved overall survival when compared to cisplatin
and 5-FU in patients with locally advanced HNSCC subsequently
treated with radiation therapy alone in the TAX 323 trial12 or concurrent
carboplatin and radiation therapy in the TAX 324 trial.13 Nevertheless,
when compared to concurrent chemoradiotherapy alone, two
randomized trials, PARADIGm14 and DeCIDE,15 showed no
improvement in the overall survival with the addition of induction
chemotherapy.
Question 5.18 The correct answer is C.
The most appropriate management of a patient with pathologic stage II
(T2N0M0) oral cavity SCC is close monitoring. Approximately 80% of
these patients will be cured with appropriate surgery alone, which
includes resection of the primary tumor site to achieve wide negative
margins, and a selective ipsilateral neck dissection if the tumor thickness
exceeds 4 to 6 mm. There is no established role for adjuvant radiation
therapy or concurrent chemotherapy and radiation therapy in such
patients. Furthermore, a lateralized oral tongue SCC would not require a
contralateral neck dissection as the risk of contralateral nodal
involvement by SCC would be low (<15%). This patient should be
followed closely by an experienced otolaryngologist for the possibility
of a local–regional recurrence. Also, patients with lichen planus are at
risk for the development of oral mucosal SCC and must be followed to
monitor for this potential problem.
Question 5.19 The correct answer is B.
The KEYNOTE-048 trial demonstrated that patients with recurrent or
metastatic HNSCC had improved 24-month overall survival when
treated with the carboplatin, 5-FU, and pembrolizumab regimen when
compared to carboplatin, 5-FU, and cetuximab (30% vs. 19%).11
Pembrolizumab monotherapy is approved for patients with CPS of
programmed death-ligand 1 (PD-L1) greater than or equal to 1, and
bevacizumab is not approved for metastatic HNSCC.
Question 5.20 The correct answer is E.
Salivary gland cancers are rare and heterogeneous in pathology,

https://t.me/ALGRAWANY33
prognosis, and treatment. The most common histologic types include
adenoid cystic carcinoma, mucoepidermoid carcinoma, adenocarcinoma,
and salivary ductal carcinoma. Salivary ductal carcinomas are
aggressive cancers that are treated with surgery and adjuvant radiation
therapy. Incurable salivary ductal carcinoma often metastasizes to the
lungs and bone, and cytotoxic chemotherapy has limited activity.
Several reports demonstrated frequent expression of the androgen
receptor (67% to 89%) in salivary ductal carcinomas and, in these
patients, high tumor response to androgen blockade. Androgen blockade
can include gonadotropin-releasing hormone (GnRH) agonists
(leuprolide) with or without androgen receptor blockade (bicalutamide),
similar to strategies used to treat prostate cancer.16 Of note, salivary
ductal carcinomas and mucoepidermoid carcinomas may also express
human epidermal growth factor receptor 2 (HER2) and have responded
to HER2-directed therapies. Estrogen/progesterone receptor expression
and related treatment are not standard for salivary gland carcinomas.

References
1. Forastiere AA, Goepfert H, Maor M, et al. Concurrent chemotherapy and radiotherapy for organ
preservation in advanced laryngeal cancer. N Engl J Med. 2003;349:2091–2098.
2. Bonner JA, Harari PM, Giralt J, et al. Radiotherapy plus cetuximab for squamous-cell carcinoma of
the head and neck. N Engl J Med. 2006;354:567–578.
3. Gillison ML, Trotti AM, Harris J, et al. Radiotherapy plus cetuximab or cisplatin in human
papillomavirus-positive oropharyngeal cancer (NRG Oncology RTOG 1016): a randomised,
multicentre, non-inferiority trial. Lancet. 2019;393:40–50.
4. Mehanna H, Robinson M, Hartley A, et al. Radiotherapy plus cisplatin or cetuximab in low-risk
human papillomavirus-positive oropharyngeal cancer (De-ESCALaTE HPV): an open-label
randomised controlled phase 3 trial. Lancet. 2019;393:51–60.
5. Noronha V, Joshi A, Patil VM, et al. Once-a-week versus once-every-3-weeks cisplatin
chemoradiation for locally advanced head and neck cancer: a phase III randomized noninferiority
trial. J Clin Oncol. 2018;36:1064–1072.
6. Zhang Y, Chen L, Hu GQ, et al. Gemcitabine and cisplatin induction chemotherapy in
nasopharyngeal carcinoma. N Engl J Med. 2019;381:1124–1135.
7. Vermorken JB, Mesia R, Rivera F, et al. Platinum-based chemotherapy plus cetuximab in head and
neck cancer. N Engl J Med. 2008;359:1116–1127.
8. Bernier J, Cooper JS, Pajak TF, et al. Defining risk levels in locally advanced head and neck
cancers: a comparative analysis of concurrent postoperative radiation plus chemotherapy trials of
the EORTC (#22931) and RTOG (# 9501). Head Neck. 2005;27:843–850.
9. Ang KK, Zhang Q, Rosenthal DI, et al. Randomized phase III trial of concurrent accelerated
radiation plus cisplatin with or without cetuximab for stage III to IV head and neck carcinoma:
RTOG 0522. J Clin Oncol. 2014;32:2940–2950.
10. Al-Sarraf M, LeBlanc M, Giri PG, et al. Chemoradiotherapy versus radiotherapy in patients with
advanced nasopharyngeal cancer: phase III randomized Intergroup study 0099. J Clin Oncol.
1998;16:1310–1317.
11. Burtness B, Harrington KJ, Greil R, et al. Pembrolizumab alone or with chemotherapy versus
cetuximab with chemotherapy for recurrent or metastatic squamous cell carcinoma of the head and
neck (KEYNOTE-048): a randomised, open-label, phase 3 study. Lancet. 2019;394:1915–1928.
12. Vermorken JB, Remenar E, van Herpen C, et al. Cisplatin, fluorouracil, and docetaxel in
unresectable head and neck cancer. N Engl J Med. 2007;357:1695–1704.
13. Posner MR, Hershock DM, Blajman CR, et al. Cisplatin and fluorouracil alone or with docetaxel in
head and neck cancer. N Engl J Med. 2007;357:1705–1715.
14. Haddad R, O’Neill A, Rabinowits G, et al. Induction chemotherapy followed by concurrent
chemoradiotherapy (sequential chemoradiotherapy) versus concurrent chemoradiotherapy alone in
locally advanced head and neck cancer (PARADIGM): a randomised phase 3 trial. Lancet Oncol.
2013;14:257–264.
15. Cohen EE, Karrison TG, Kocherginsky M, et al. Phase III randomized trial of induction
chemotherapy in patients with N2 or N3 locally advanced head and neck cancer. J Clin Oncol.
2014;32:2735–2743.
16. Boon E, van Boxtel W, Buter J, et al. Androgen deprivation therapy for androgen receptor-positive
advanced salivary duct carcinoma: a nationwide case series of 35 patients in The Netherlands. Head
Neck. 2018;40:605–613.
___________
Corresponding chapters in DeVita, Hellman, and Rosenberg’s Cancer: Principles & Practice of
Oncology, Eleventh Edition: 44 (Molecular Biology of Head and Neck Cancers), 45 (Cancer of the
Head and Neck), and 46 (Rehabilitation after Treatment of Head and Neck Cancer).

https://t.me/ALGRAWANY33
6 Thoracic Cancers
Brett H. Herzog, Haley Ellis, Prabhat Singh Malik, Saiama N. Waqar,
Siddhartha Devarakonda, Ramaswamy Govindan, and Daniel
Morgensztern

QUESTIONS

Each of the numbered items below is followed by lettered answers. Select the
ONE lettered answer that is BEST in each case unless instructed otherwise.

Question 6.1 A 40-year-old never-smoker woman was diagnosed with


adenocarcinoma of the lung in 2015. Molecular testing revealed exon 19
deletion in EGFR. She was started on erlotinib and had a partial response.
However, the most recent scans showed significant tumor progression with
multiple new liver and lung metastases. One of the liver lesions was biopsied.
Which of the following mutations in EGFR is most likely to explain the
tumor’s resistance to erlotinib?
A. L858R
B. Exon 20 insertion
C. T790M
D. C797S
Question 6.2 A 57-year-old female and never smoker is diagnosed with
metastatic lung adenocarcinoma. Molecular analysis of her tumor tissue
reveals a ROS1 gene rearrangement. Her performance status (PS) is excellent
and she has no other comorbid conditions. Which of the following treatments
would you recommend?
A. Osimertinib
B. Crizotinib
C. Carboplatin, pemetrexed, pembrolizumab
D. Alectinib
Question 6.3 A 55-year-old man with a 30 pack-year history of smoking
presents to the emergency room with shortness of breath. Chest radiograph
demonstrates a right upper lobe opacity. Computed tomography (CT) scan
reveals a 3.5-cm spiculated mass in the peripheral right upper lobe, which is
suspicious for malignancy, without any hilar or mediastinal
lymphadenopathy. What is the next best step in management?
A. Bronchoscopy and biopsy of the mass
B. CT-guided biopsy of the mass
C. Brain magnetic resonance imaging (MRI)
D. Refer to thoracic surgeon for resection
Question 6.4 A 65-year-old man with a 40 pack-year history of smoking
with no medical comorbidities presents to the emergency room with
shortness of breath. Chest radiograph demonstrates a left lower lobe mass and
left pleural effusion. CT reveals a 3-cm left lower lobe mass, left hilar
fullness, and a moderate left pleural effusion. Biopsy findings of the lung
mass and thoracentesis are both positive for adenocarcinoma. Molecular
analysis revealed no targetable alterations. Programmed death-ligand 1 (PD-
L1) expression was seen on 1% of tumor cells. Staging studies do not reveal
any distant metastases. Which of the following is the next best step in his
management?
A. Referral to a thoracic surgeon
B. Radiation to the chest
C. Concurrent chemotherapy and radiation
D. Platinum-based doublet combined with pembrolizumab therapy
Question 6.5 A 66-year-old man with a 30 pack-year history of smoking
presents to your office for consultation regarding chemotherapy options for
metastatic non–small cell lung cancer (NSCLC), squamous histology. He has
no significant medical problems, and the Eastern Cooperative Oncology
Group (ECOG) PS is 0. PD-L1 was observed on 20% of tumor cells.

https://t.me/ALGRAWANY33
Laboratory studies reveal normal blood counts, liver enzymes, and kidney
function. Which of the following treatment regimens would you recommend?
A. Cisplatin, pemetrexed, pembrolizumab
B. Carboplatin, paclitaxel, and pembrolizumab
C. Single-agent pemetrexed
D. Carboplatin and pemetrexed
Question 6.6 A 55-year-old Asian woman with systemic lupus
erythematosus, who is a never smoker, completed four cycles of frontline
carboplatin and paclitaxel, for metastatic lung adenocarcinoma. Imaging
studies done after completion of therapy show stable disease. She is very
active, and has continued to work as a nurse throughout her treatment. Her
ECOG PS is 0 and she has tolerated the treatment well, other than grade I
neuropathy. Her tumor EGFR status is wild type. She wants “the best
treatment possible” and desires further treatment. What would you
recommend?
A. Treatment break, erlotinib at the time of disease progression
B. Stopping carboplatin, continuing paclitaxel until disease progression
C. Continuing carboplatin and paclitaxel for four additional cycles
D. Pemetrexed maintenance therapy
Question 6.7 A 58-year-old man with a 40 pack-year history of smoking is
referred to you by a radiation oncologist for management of limited-stage
small cell lung cancer (SCLC). What treatment would you recommend?
A. Cisplatin and etoposide chemotherapy
B. Cisplatin and etoposide chemotherapy with concurrent thoracic
radiation
C. Cisplatin and etoposide chemotherapy, followed by prophylactic
cranial irradiation (PCI) if responding to chemotherapy
D. Cisplatin and etoposide chemotherapy with concurrent thoracic
radiation, followed by PCI if responding to treatment
Question 6.8 A 57-year-old man presents to you after recently being
diagnosed with clinical stage IA lung adenocarcinoma of the right upper lobe.
He underwent lobectomy; however, a detailed sampling of mediastinal lymph
nodes was performed at the time of surgery, and reveals disease in a 4R
lymph node with negative margins and no involvement of the lymph node
capsule. He asks you what treatment gives him the best chance of survival?
A. Observation
B. Adjuvant chemotherapy with cisplatin and pemetrexed
C. Adjuvant radiation therapy
D. Adjuvant chemotherapy with single-agent cisplatin
Question 6.9 Which of the following statements is CORRECT regarding
pulmonary carcinoid tumors?
A. Patients usually present with carcinoid syndrome.
B. Surgery has a curative potential for resectable localized tumors.
C. Adjuvant chemotherapy is the standard of care following complete
surgical resection.
D. Most pulmonary carcinoids are atypical carcinoids.
Question 6.10 A 50-year-old male with multiple sclerosis currently treated
with recombinant interferon beta-1alpha is diagnosed with metastatic lung
adenocarcinoma harboring a BRAF V600E mutation and PD-L1 score is
60%. Which of the following is the best treatment option?
A. Dabrafenib plus trametinib
B. Carboplatin and pemetrexed
C. Pembrolizumab
D. Treatment with combined dabrafenib and trametinib and
pembrolizumab
Question 6.11 A 60-year-old man with a 45 pack-year history of smoking
presents with chest pain. Chest x-ray reveals a right upper lobe mass. CT scan
of the chest demonstrates a 4-cm right upper lobe lung mass, with right hilar
and multiple ipsilateral enlarged mediastinal lymph nodes. Bronchoscopy and
biopsy of the mass reveals NSCLC. Positron emission tomography (PET)
scan demonstrates increased fluorodeoxyglucose (FDG) uptake in the lung
mass, right hilar, and mediastinal lymph nodes, but no other site of metastatic

https://t.me/ALGRAWANY33
disease. CT of the abdomen and brain MRI are unremarkable.
Mediastinoscopy and mediastinal lymph node biopsy reveal NSCLC. His
cancer is staged as T2aN2M0, stage IIIA NSCLC. His PS is 1, and he is
otherwise in good health. Which of the following is the best management for
this patient?
A. Definitive radiation to the chest
B. Radiation to the chest, followed by platinum-based chemotherapy
C. Concurrent radiation to the chest and platinum-based chemotherapy
followed by durvalumab consolidation
D. Platinum-based chemotherapy
Question 6.12 A 47-year-old female presents to the clinic with metastatic,
ALK-rearranged adenocarcinoma of the lung. She has been treated with
alectinib for 11 months, but, unfortunately, her most recent CT scan
demonstrates disease progression. She tolerated alectinib well with only mild
myalgias, and her PS remains excellent. Tumor biopsy reveals
adenocarcinoma histology with ALK G1202R mutation. Which of the
following is the optimal treatment option for this patient?
A. Treatment with lorlatinib
B. Treatment with brigatinib
C. Treatment with certinib
D. Treatment with crizotinib
Question 6.13 A 45-year-old Asian woman, who is a never smoker,
presents to your office for consultation regarding systemic therapy for
metastatic adenocarcinoma of the lung. Her tumor has an EGFR exon 19
deletion. Which of the following is a valid first-line treatment option for this
patient?
A. Cetuximab
B. Osimertinib
C. Necitumumab
D. Crizotinib
Question 6.14 A 65-year-old Caucasian man, who is a never smoker,
presents to your office for consultation regarding therapy for metastatic
adenocarcinoma of the lung. Genotyping of his tumor revealed a gene fusion
involving ROS1. The patient is interested in knowing if he would qualify for
treatment with any of the currently approved tyrosine kinase inhibitors
(TKIs). Which of the following treatment options is CORRECT?
A. Carboplatin and pemetrexed
B. Afatinib
C. Entrectinib
D. Alectinib
Question 6.15 A 65-year-old man, who is a former smoker, presents for
consultation regarding management of unresectable stage IIIB
adenocarcinoma of the lung. He has a PS of 0 and adequate blood counts,
hepatic, and renal function. Which of the following would you recommend?
A. Concurrent definitive cisplatin and etoposide therapy, no consolidation
therapy, no PCI
B. Concurrent definitive cisplatin and etoposide therapy, consolidation
docetaxel therapy, no PCI
C. Concurrent definitive cisplatin and etoposide therapy, consolidation
durvalumab therapy, no PCI
D. Concurrent definitive cisplatin and etoposide therapy, consolidation
gefitinib therapy, followed by PCI
Question 6.16 A 44-year-old woman presents with a newly diagnosed
adenocarcinoma of the lung located at the apex of the left lung and involving
the brachial plexus and first rib. She undergoes a PET scan that reveals no
other nodal involvement or distant sites of FDG uptake. Her PS is excellent.
What would the preferred treatment strategy in this situation be?
A. Sequential chemotherapy followed by radiation therapy
B. Surgical resection followed by adjuvant chemotherapy
C. Palliative chemotherapy
D. Concurrent chemoradiation followed by surgical resection
Question 6.17 Which of the following are characteristic genomic feature(s)

https://t.me/ALGRAWANY33
of SCLC?
A. Recurrent alterations in EGFR and ALK
B. Recurrent alterations in TP53 and RB1
C. Recurrent mutations in DDR2 and BRAF
D. Recurrent EGFR and KRAS mutations
Question 6.18 A 45-year-old male was diagnosed with a 4-cm biopsy-
proven adenocarcinoma of the right lower lobe lung. Staging revealed no
locoregional or distant metastatic disease. He has no significant comorbidities
and his preoperative laboratories were unremarkable. He underwent a right
lower lobe lobectomy with pathologic peribronchial lymph node
involvement. Postoperative recovery was uneventful. Which is the next
preferred step in the management of his disease?
A. No further treatment, observation only
B. Adjuvant chemotherapy with a cisplatin-based doublet
C. Adjuvant chemotherapy with single-agent cisplatin
D. Adjuvant chemotherapy with single-agent vinorelbine
Question 6.19 What is the most common histologic subtype of malignant
mesotheliomas?
A. Epithelial
B. Sarcomatoid
C. Poorly differentiated
D. Biphasic
Question 6.20 Which of the following immunohistochemistry markers is
present in malignant mesotheliomas?
A. CEA
B. TTF-1
C. Moc-31
D. Calretinin
Question 6.21 Which of the following is an indicator of poor prognosis in
malignant mesothelioma?
A. Anemia
B. Epithelial histology
C. Female gender
D. High c-MET expression
Question 6.22 Which of the following finding(s) indicate an unresectable
pleural mesothelioma?
A. Direct extension into the spine
B. Extension into the internal surface of the pericardium without pleural
effusion
C. Direct transdiaphragmatic extension to the peritoneum
D. All of the above
Question 6.23 Frequent alteration of which of the following tumor
suppressor genes is seen in familial risk of mesothelioma?
A. RBL1
B. APC
C. RB1
D. BAP1
Question 6.24 A 47-year-old female with a history of ALK-positive
metastatic lung adenocarcinoma is referred to your office after progressing on
alectinib. Other than her lung cancer diagnosis, she has a history of
hypertension and hyperlipidemia. Her family history is remarkable for
myocardial infarctions in multiple family members on her paternal side and
Hodgkin lymphoma in a maternal aunt. You decide to initiate lorlatinib.
Development of which side effect of lorlatinib is most concerning in her?
A. Edema
B. Fatigue
C. Nausea
D. Hypercholesterolemia

https://t.me/ALGRAWANY33
Question 6.25 A 60-year-old man was recently diagnosed with metastatic
pleural mesothelioma. Assuming that this patient has no other significant
comorbidities, which of the following chemotherapy regimens would be most
preferable in the first-line setting?
A. Cisplatin and gemcitabine
B. Carboplatin and gemcitabine
C. Single-agent pemetrexed
D. Cisplatin and pemetrexed, bevacizumab
Question 6.26 A 72-year-old woman with a 50 pack-year smoking history
presented with shortness of breath, productive cough, fatigue, anorexia, and
20-lb weight loss. Chest x-ray revealed a hilar mass and 5.4-cm right mass.
CT chest/abdomen/pelvis redemonstrated the right lung mass, along with
mediastinal lymphadenopathy, and suspicious liver lesions. Liver biopsy
showed SCLC. What is preferred frontline therapy in this scenario?
A. Platinum-etoposide chemotherapy alone
B. Immunotherapy alone
C. Platinum-etoposide chemotherapy plus immunotherapy
D. Thoracic radiation therapy
Question 6.27 Which statement is CORRECT regarding the role of PCI in
extensive-stage small cell carcinoma?
A. PCI increases overall survival in patients who responded to initial
treatment.
B. PCI increases overall survival and reduces the incidence of brain
metastases in patients who responded to initial treatment.
C. PCI reduces the incidence of symptomatic brain metastases in patients
who respond to initial therapy.
D. PCI should be avoided in small cell carcinoma due to associated acute
and long-term toxicities.
Question 6.28 A 67-year-old male referred to an oncologist after chest x-
ray with a right upper lobe lung mass and bronchoscopy revealed an
extensive endobronchial tumor with biopsy positive for small cell carcinoma.
The patient notes becoming short of breath when he leans over, with
associated dizziness and head pain. His examination is notable for facial
plethora and dilated, engorged blood vessels across his chest. SpO2 98% on
room air. What is the appropriate treatment?
A. Chemotherapy
B. Endovascular therapy
C. Radiation therapy
D. Corticosteroids
Question 6.29 A 56-year-old man with a 35 pack-year smoking history and
extensive-stage small cell carcinoma was brought to the emergency
department due to becoming suddenly disoriented with generalized muscle
weakness. On admission, vitals notable were temperature 36.7°C (98°F),
blood pressure 102/62 with positive orthostatic signs, heart rate 102 bpm,
respiratory rate 28/min, and SpO2 100% on room air. On physical
examination, the patient was in no acute distress, but appeared clinically
dehydrated. No focal neurologic deficits. Initial laboratories included Na 126
mmol/L, K 3.3 mmol/L, Cl 84 mmol/L, bicarbonate 30 mmol/L, blood urea
nitrogen (BUN) 8 mg/dL, Cr 0.54 mg/dL, glucose 125 mg/dL, and lactate 2.3
mmol/L. Laboratory tests 1 month before admission were normal. What is the
suspected diagnosis?
A. Cushing syndrome
B. Lambert-Eaton myasthenic syndrome
C. Subacute cerebellar degeneration
D. Syndrome of inappropriate antidiuretic hormone secretion (SIADH)
Question 6.30 A 45-year-old female has presented with an anterior
mediastinal mass. Which of the following would be the least likely
differential diagnosis?
A. Thymoma
B. Non-Hodgkin lymphoma
C. Teratoma
D. Seminoma

https://t.me/ALGRAWANY33
Question 6.31 What is true about small cell carcinoma?
A. Receptor kinase–PI3K signaling is the most commonly affected
pathway in SCLC.
B. SCLC is characterized by alterations in cell cycle regulation and
NOTCH signaling.
C. Basal cells of the bronchial epithelium are the cells of origin for SCLC.
D. Mutations in CDKN2A are the commonest alterations in SCLC.
Question 6.32 A 49-year-old male sees you after undergoing a
thymectomy. Postoperative histopathology shows type B2 thymoma with
microscopic capsular invasion and involvement of perithymic fat. What
further advice would you give?
A. Surveillance with CT chest every 6–12 months for 2 years
B. Postoperative radiotherapy
C. Concurrent chemoradiation
D. Adjuvant chemotherapy
Question 6.33 Which of the following is not a mandatory investigation in
evaluation of all patients with SCLC?
A. MRI brain
B. PET CT or contrast-enhanced CT chest and abdomen
C. Bone marrow biopsy
D. Thoracentesis in case of pleural effusion

ANSWERS

Question 6.1 The correct answer is C.


Acquisition of a secondary EGFR T790M gatekeeper mutation is the
most common mechanism of resistance, and occurs in about 50% of all
cases. MET amplification, activation of other bypass signaling pathways
and transdifferentiation to SCLC represent other, less prevalent
mechanisms of acquired resistance.
Question 6.2 The correct answer is B.
Frontline treatment of ROS1-rearranged metastatic NSCLC should be
with a ROS1 TKI such as crizotinib.1 Osimertinib, an EGFR TKI, or
alectinib, an ALK TKI, has no established efficacy in ROS1-rearranged
NSCLC. Chemotherapy is not preferred for frontline therapy in patients
with NSCLC harboring a targetable molecular aberration such as ROS1
gene rearrangement.
Question 6.3 The correct answer is B.
The right upper lobe mass is suspicious for malignancy. However, a
tissue diagnosis is needed in this case. Because the location of the mass
is peripheral, the best way to approach it would be through CT-guided
biopsy, rather than through bronchoscopy and biopsy. Once a diagnosis
of lung cancer is established, staging studies, such as CT of the
abdomen, MRI of the brain, PET, or bone scan may be performed. In
case of NSCLC, if there is no evidence of distant metastases on staging
studies, the patient should be referred to a thoracic surgeon.
Question 6.4 The correct answer is D.
This patient has stage IV disease due to the presence of malignant
pleural effusion, and should receive treatment with platinum-based
doublet chemotherapy and pembrolizumab.2,3 Surgery, or radiation alone
in selected cases, would be appropriate only for early-stage disease,
whereas chemoradiation would be the treatment of choice for fit patients
with locally advanced disease.
Question 6.5 The correct answer is B.
Cisplatin or carboplatin combined with pemetrexed are possible
combinations to consider in patients with advanced NSCLC with
nonsquamous histology. For patients with squamous histology,
pemetrexed is less effective; and the best option would be with
combination therapy including carboplatin, paclitaxel (or nab-
paclitaxel), and pembrolizumab.4
Question 6.6 The correct answer is D.

https://t.me/ALGRAWANY33
Several studies have demonstrated that there is no survival advantage to
continuing platinum doublet therapy beyond four to six cycles. Agents
used in the second-line setting for the treatment of NSCLC include
docetaxel and pemetrexed. Patients whose tumors have an activating
mutation in the EGFR–tyrosine kinase domain, have the most durable
benefit from erlotinib. Maintenance chemotherapy is an option for
patients with good PS and stable disease following frontline
chemotherapy, who do not experience significant treatment-related
toxicities. Maintenance chemotherapy with either pemetrexed or
bevacizumab has been shown to improve overall survival in patients
with NSCLC. Therefore, pemetrexed maintenance would be an
appropriate option for this patient.
Question 6.7 The correct answer is D.
In a patient with limited-stage SCLC, the optimal treatment would
comprise concurrent chemotherapy and radiotherapy including cisplatin
and etoposide, followed by PCI.
Question 6.8 The correct answer is B.
Adjuvant platinum-based chemotherapy doublets have been shown to
improve survival in patients with stage II and III NSCLC who undergo
surgical resection. In contrast, there is no established benefit from
single-agent cisplatin or postoperative radiation, and observation is
suboptimal.
Question 6.9 The correct answer is B.
Only approximately 2% of patients with pulmonary carcinoids present
with carcinoid syndrome, and approximately 30% of patients are
asymptomatic at presentation. Ninety percent of pulmonary carcinoids
are typical, whereas 10% are atypical. Surgery is the treatment of choice
for carcinoid tumors. There is no established role for adjuvant
chemotherapy, following complete resection of carcinoid tumors.
Question 6.10 The correct answer is A.
On the basis of a phase II clinical trial, combined BRAF and MEK
inhibition with dabrafenib and trametinib was efficacious, and is the
favored frontline therapy for this patient with the a V600E alteration in
BRAF despite the lack of randomized trials comparing this approach to
platinum-based chemotherapy.5 Although other alterations in BRAF
have been identified in NSCLC, the efficacy of dabrafenib and
trametinib is unknown in these cancers. There is no evidence of benefit
in combining TKIs and immunotherapy in BRAF-mutated patients.
Furthermore, multiple sclerosis was an exclusion criterion for the trials
in the clinical trials evaluating the use of immune-checkpoint inhibitors.
Question 6.11 The correct answer is C.
This patient has locally advanced NSCLC. In this setting, concurrent
chemoradiation followed by durvalumab is better than is radiation
therapy alone or sequential chemotherapy followed by radiation.6
Chemotherapy alone would be indicated for palliation of metastatic
disease, and is not appropriate in this setting.
Question 6.12 The correct answer is A.
Acquired alectinib resistance in adenocarcinomas of the lung can be due
to secondary mutations in ALK. The only ALK inhibitor with
meaningful clinical activity against ALK G1202R mutation is
lorlatinib.7,8
Question 6.13 The correct answer is B.
In patients with EGFR mutations, initial therapy with an EGFR TKI
results in better response rates and longer time to tumor progression
compared with chemotherapy. Osimertinib was associated with
improved overall survival when compared with first-generation EGFR
TKIs in the FLAURA trial.9 Osimertinib, erlotinib, dacomitinib, and
afatinib are currently approved EGFR TKIs. Cetuximab and
necitumumab are EGFR-directed monoclonal antibodies that have been
studied in NSCLC. However, the latter have no established role in the
management of patients with EGFR-mutated NSCLC at the current time.
Crizotinib is a TKI active against ALK-, ROS1-, and MET-altered

https://t.me/ALGRAWANY33
tumors.
Question 6.14 The correct answer is C.
The frequency of ROS1 translocations in all patients with NSCLC is
approximately 1% to 2%. In patients with actionable gene alterations,
TKIs are usually preferred in the first-line setting, with chemotherapy
used at the time of tumor progression. The main treatment options for
these patients include crizotinib and entrectinib.10 Afatinib is used for
EGFR mutations and alectinib for ALK rearrangements.
Question 6.15 The correct answer is C.
The standard of care for locally advanced unresectable NSCLC is
concurrent definitive chemoradiation with cisplatin and etoposide
followed by durvalumab consolidation. Consolidation docetaxel is
associated with increased toxicity and no survival improvement
compared with concurrent chemoradiation alone, and the use of the
EGFR TKI gefitinib after consolidation docetaxel is associated with a
possible decrease in survival. Prophylactic cranial radiation decreases
the incidence of brain metastases in patients with stage III disease, but
does not improve overall survival. At present, there is no sufficient
evidence to recommend PCI in patients with stage III disease.
Question 6.16 The correct answer is D.
This patient has a Pancoast tumor of the lung. On the basis of data from
multiple phase II studies, the preferred management approach in these
patients is trimodality therapy with concurrent chemoradiation (typically
with a platinum doublet such as cisplatin-etoposide) followed by
surgical resection. Curative-intent concurrent chemoradiation alone is
recommended in patients not deemed to be surgical candidates.
Radiotherapy for symptom relief is an option in patients treated with a
palliative intent.
Question 6.17 The correct answer is B.
Genomic characterization of SCLCs has consistently revealed co-
alteration of TP53 and RB1 across several different studies. Recurrent
EGFR and ALK alterations are more prevalent among patients with
adenocarcinoma of the lung, particularly never or former smokers.
KRAS mutations are present in nearly a third of all lung
adenocarcinomas, and DDR2 mutations in roughly 4% of squamous cell
lung cancers.
Question 6.18 The correct answer is B.
This patient has stage T2aN1M0 (IIB) NSCLC. The Lung Adjuvant
Cisplatin Evaluation (LACE) meta-analysis indicated an absolute 5-year
overall survival benefit of 5% with adjuvant cisplatin-based
chemotherapy regimens in patients with stage II and III NSCLC when
compared with observation.11 There is no data on adjuvant therapy with
single-agent cisplatin or vinorelbine.
Question 6.19 The correct answer is A.
Epithelial is the most common subtype of malignant mesotheliomas,
comprising approximately 50% to 60% of cases. Biphasic and
sarcomatoid mesotheliomas are less frequent. Uncommon tumors that
cannot be categorized morphologically are considered to be poorly
differentiated.
Question 6.20 The correct answer is D.
Mesothelioma cells are diffusely positive for pankeratin, keratin 5/6,
calretinin, and WT1. CEA, a nonspecific marker, Moc-31, which is
commonly seen in adenocarcinomas, and TTF-1, which is seen in
primary tumors of the lung and thyroid, are usually negative in
mesotheliomas.
Question 6.21 The correct answer is A.
The established poor prognostic factors in malignant mesothelioma are
poor PS, male gender, anemia, thrombocytosis, leukocytosis, elevated
lactate dehydrogenase (LDH), and nonepithelial histology including
sarcomatoid and biphasic subtypes. Increased c-MET expression has not
been associated with worse overall survival.

https://t.me/ALGRAWANY33
Question 6.22 The correct answer is D.
Locally advanced and technically unresectable tumor (T4) is defined by
a tumor involving all the ipsilateral pleural surfaces and extension of the
tumor to the peritoneum, contralateral pleura, mediastinal organs, spine,
chest wall, or internal surface of the pericardium with or without
effusion. Involvement of the endothoracic fascia describes a locally
advanced but potentially resectable disease.
Question 6.23 The correct answer is D.
Mutations in the BRCA-associated protein 1 (BAP1) tumor suppressor
gene have been described in nearly 20% of mesotheliomas. These
mutations are more frequent among smokers. Germline mutations in
BAP1 have also been reported in families with a high incidence of
mesotheliomas.
Question 6.24 The correct answer is D.
Hypercholesterolemia is one of the most common side effects among
patients treated with lorlatinib, occurring in up to 65% of patients and
often a reason for temporary discontinuation of the treatment.12 Edema,
fatigue, and nausea are less common.
Question 6.25 The correct answer is D.
On the basis of the Mesothelioma Avastin Cisplatin Pemetrexed Study
(MAPS) trial, one of the standard therapies for mesothelioma is the
combination of cisplatin and pemetrexed along with bevacizumab.13
Another option is the combination cisplatin and pemetrexed. Patients
who are unlikely to tolerate cisplatin may be treated with carboplatin,
whereas gemcitabine regimens may be used in patients unable to be
treated with pemetrexed.
Question 6.26 The correct answer is C.
Platinum-etoposide chemotherapy plus immunotherapy for four cycles
followed by maintenance with immunotherapy is now the recommended
first-line treatment for patients with newly diagnosed extensive-stage
small cell carcinoma. The IMpower133 trial showed improved survival
with the addition of the anti-PD-L1 antibody atezolizumab to
carboplatin and etoposide.14 More recent data from the CASPIAN trial
demonstrated increased survival with the addition of anti-PD-L1
antibody durvalumab to platinum and etoposide chemotherapy
compared with chemotherapy alone.15
Question 6.27 The correct answer is C.
Although PCI in extensive-stage SCLC following a response to
chemotherapy has demonstrated a decrease in the incidence of
symptomatic brain metastases by more than 50%, the impact on overall
survival is uncertain. MRI surveillance can be considered as an
alternative in patients who do not receive PCI. These studies have all
been conducted before the incorporation of immunotherapy in the first-
line management of small cell carcinoma. The role of PCI in patients
with extensive-stage small cell carcinoma, therefore, continues to remain
unclear. In limited-stage small cell carcinoma, multiple clinical trials
have shown an improvement in overall survival with PCI.
Question 6.28 The correct answer is A.
Small cell carcinoma is a chemosensitive malignancy at diagnosis. For
patients with small cell carcinoma and non–life-threatening symptomatic
superior vena cava (SVC) obstruction, the clinical response to
chemotherapy is usually rapid. Chemotherapy relieves symptoms of
SVC obstruction in approximately 75% of patients with small cell
carcinoma. Endovascular intervention or radiation therapy may be
required for patients with life-threatening SVC obstruction or those who
do not respond to systemic therapy.
Question 6.29 The correct answer is D.
Paraneoplastic syndromes represent a constellation of clinical signs and
symptoms from a primary malignancy, which can be mediated by
hormones secreted by the malignant cells. Ectopic production of
antidiuretic hormone (ADH) by a tumor is usually due to small cell
carcinoma, and develops in up to 16% of these patients. SIADH is a

https://t.me/ALGRAWANY33
euvolemic hypo-osmolar hyponatremic state in which the patient can
present with symptoms such as nausea, vomiting, decreased oral intake,
and fatigue.
Question 6.30 The correct answer is D.
The approach to an anterior mediastinal mass depends on several
clinical factors such as age at presentation, gender, course of illness, and
associated clinical findings. Ancillary laboratory studies include
complete hemogram, serum LDH, AFP (alpha fetoprotein) and β-HCG
(beta subunit of human chorionic gonadotropin). In terms of frequency,
the most common cause of anterior mediastinal mass in a woman in the
fourth decade of life would be thymic malignancies followed by
lymphoma. Mediastinal germ cell tumors in females are usually
teratomas or non–seminomatous germ cell tumors. Mediastinal
seminoma in a female is rare.
Question 6.31 The correct answer is B.
Recurrent inactivating mutations in RB1 and TP53 are the most common
genomic alterations in SCLC. Receptor kinase–PI3K signaling and
transcription regulation pathways are more commonly affected in lung
adenocarcinoma and only in a minority of small cell carcinoma. SCLC
arises from neuroendocrine cells. Basal cells in bronchial epithelium
give rise to squamous cell carcinoma.
Question 6.32 The correct answer is B.
Capsular invasion in resected thymoma is considered a high-risk factor
for recurrence. This would make it Masaoka stage II. Postoperative
radiotherapy is recommended. There is no established role for adjuvant
chemotherapy in the management of patients with completely resected
thymomas. Concurrent chemoradiation may be considered in patients
with an unresectable thymoma.
Question 6.33 The correct answer is C.
Bone marrow examination is performed only in selective cases of
otherwise limited-stage small cell carcinoma in the presence of
hematologic findings suggesting marrow invasion including nucleated
red blood cells (RBCs) in peripheral blood smear, neutropenia, or
thrombocytopenia.

References
1. Shaw AT, Ou SH, Bang YJ, et al. Crizotinib in ROS1-rearranged non-small-cell lung cancer. N
Engl J Med. 2014;371:1963–1971.
2. Gandhi L, Rodriguez-Abreu D, Gadgeel S, et al. Pembrolizumab plus chemotherapy in metastatic
non-small-cell lung cancer. N Engl J Med. 2018;378:2078–2092.
3. West H, McCleod M, Hussein M, et al. Atezolizumab in combination with carboplatin plus nab-
paclitaxel chemotherapy compared with chemotherapy alone as first-line treatment for metastatic
non-squamous non-small-cell lung cancer (IMpower130): a multicentre, randomised, open-label,
phase 3 trial. Lancet Oncol. 2019;20:924–937.
4. Paz-Ares L, Luft A, Vicente D, et al. Pembrolizumab plus chemotherapy for squamous non-small-
cell lung cancer. N Engl J Med. 2018;379:2040–2051.
5. Planchard D, Smit EF, Groen HJM, et al. Dabrafenib plus trametinib in patients with previously
untreated BRAF(V600E)-mutant metastatic non-small-cell lung cancer: an open-label, phase 2 trial.
Lancet Oncol. 2017;18:1307–1316.
6. Antonia SJ, Villegas A, Daniel D, et al. Overall survival with durvalumab after chemoradiotherapy
in stage III NSCLC. N Engl J Med. 2018;379:2342–2350.
7. Gainor JF, Dardaei L, Yoda S, et al. Molecular mechanisms of resistance to first-and second-
generation ALK inhibitors in ALK-rearranged lung cancer. Cancer Discov. 2016;6:1118–1133.
8. Shaw AT, Felip E, Bauer TM, et al. Lorlatinib in non-small-cell lung cancer with ALK or ROS1
rearrangement: an international, multicentre, open-label, single-arm first-in-man phase 1 trial.
Lancet Oncol. 2017;18:1590–1599.
9. Ramalingam SS, Vansteenkiste J, Planchard D, et al. Overall survival with osimertinib in untreated,
EGFR-mutated advanced NSCLC. N Engl J Med. 2020;382:41–50.
10. Drilon A, Jenkins C, Iyer S, et al. ROS1-dependent cancers—biology, diagnostics and therapeutics.
Nat Rev Clin Oncol. 2021;18:35–55.
11. Pignon JP, Tribodet H, Scagliotti GV, et al. Lung adjuvant cisplatin evaluation: a pooled analysis
by the LACE Collaborative Group. J Clin Oncol. 2008;26:3552–3559.
12. Solomon BJ, Besse B, Bauer TM, et al. Lorlatinib in patients with ALK-positive non-small-cell
lung cancer: results from a global phase 2 study. Lancet Oncol. 2018;19:1654–1667.
13. Zalcman G, Mazieres J, Margery J, et al. Bevacizumab for newly diagnosed pleural mesothelioma
in the Mesothelioma Avastin Cisplatin Pemetrexed Study (MAPS): a randomised, controlled, open-
label, phase 3 trial. Lancet. 2016;387:1405–1414.
14. Horn L, Mansfield AS, Szcze˛sna A, et al. First-line atezolizumab plus chemotherapy in extensive-
stage small-cell lung cancer. N Engl J Med. 2018;379:2220–2229.
15. Paz-Ares L, Dvorkin M, Chen Y, et al. Durvalumab plus platinum-etoposide versus platinum-
etoposide in first-line treatment of extensive-stage small-cell lung cancer (CASPIAN): a
randomised, controlled, open-label, phase 3 trial. Lancet. 2019;394:1929–1939.
___________
Corresponding chapters in DeVita, Hellman, and Rosenberg's Cancer: Principles & Practice of

https://t.me/ALGRAWANY33
Oncology, Eleventh Edition: 47 (Molecular Biology of Lung Cancer), 48 (Non–Small Cell Lung
Cancer), 49 (Small Cell and Neuroendocrine Tumors of the Lung), and 109 (Benign and Malignant
Mesothelioma).
7 Breast Cancer
Katherine Clifton, Foluso C. Ademuyiwa, Ron Bose, and Cynthia X.
Ma

QUESTIONS

Each of the numbered items below is followed by lettered answers. Select the
ONE lettered answer that is BEST in each case unless instructed otherwise.

Question 7.1 What percentage of breast cancers is caused by germline


mutations?
A. 20%
B. 10%
C. 15%
D. 2%
Question 7.2 BRCA1 mutations are most commonly associated with:
A. Triple-negative breast cancers.
B. Estrogen receptor (ER)-positive breast cancers.
C. Pancreatic cancer and melanoma.
D. Autosomal recessive inheritance.
Question 7.3 Li–Fraumeni syndrome is characterized by which clinical
features?
A. Lobular breast cancer, gastric cancer
B. Breast cancer, soft tissue sarcoma, central nervous system tumors,
adrenocortical cancer, leukemia, prostate cancer
C. Breast cancer, hamartoma, thyroid cancer, oral mucosa cancer,

https://t.me/ALGRAWANY33
endometrial cancer, brain tumors
D. Male breast cancer, pancreas cancer, gall bladder cancer, pharynx
cancer, gastric cancer, melanoma, prostate cancer
Question 7.4 Which of the following statements about luminal A subtype is
TRUE?
A. It is more common in premenopausal black women.
B. Luminal A breast cancers frequently carry TP53 mutations.
C. It is characterized by high-expression levels of ER-related genes and
low expression of the HER2 cluster and proliferation-associated genes.
D. It has a worse prognosis than do other molecular subtypes of breast
cancer.
Question 7.5 A 42-year-old female undergoes mastectomy and sentinel
lymph node biopsy. Final pathology reveals a 1.5-cm invasive ductal
carcinoma that is ER positive, progesterone receptor positive and HER2 1+
by immunohistochemistry (IHC), with intermediate grade. Two sentinel
lymph nodes are negative. Treatment recommendations include:
A. Adjuvant radiation followed by 5–10 years of endocrine therapy.
B. 5–10 years of letrozole alone.
C. Oncotype DX testing and 5–10 years of endocrine therapy.
D. HER2-based therapy followed by 5–10 years of tamoxifen.
Question 7.6 Which of the following factors is a risk factor for developing
breast cancer?
A. Cowden syndrome
B. Early age at first full-term pregnancy
C. Breast density <10%
D. Simple breast cysts
Question 7.7 American Cancer Society (ACS) guidelines for magnetic
resonance imaging (MRI) screening are supported for which of the following
patients?
A. A 47-year-old woman with a deleterious mutation in BRCA1
B. A 60-year-old woman with biopsy-proven lobular carcinoma in situ
(LCIS)
C. A 55-year-old woman with a personal history of ER-positive breast
cancer treated 8 years ago
D. A 65-year-old woman who underwent chest radiation at age 60 years
Question 7.8 Which of the following scenarios are considered a
contraindication to the use of tamoxifen?
A. Major surgical procedure within the previous 6 months
B. A history of deep vein thrombosis, stroke, pulmonary embolism, or
transient ischemic attack
C. A 60-year-old woman with bilateral asymptomatic cataracts
D. A 65-year-old woman currently on a selective serotonin reuptake
inhibitor (SSRI) for hot flashes
Question 7.9 A 44-year-old woman presents to your office with a new
palpable breast mass. A biopsy has been recommended on the basis of the
results of mammogram and ultrasound. In general, the most appropriate way
to diagnose suspected invasive carcinoma is:
A. Circulating tumor DNA.
B. An excisional biopsy.
C. Diagnosis by core needle biopsy.
D. A fine-needle aspiration biopsy.
Question 7.10 You are advising a 42-year-old premenopausal woman with
a history of atypical hyperplasia on prevention strategies. Which of the
following is CORRECT?
A. Tamoxifen reduces the risk only in premenopausal women.
B. Oophorectomy reduces the risk by 50%–65%.
C. Bilateral mastectomy completely eliminates her risk of breast cancer.
D. Raloxifene provides similar benefit compared with tamoxifen.

https://t.me/ALGRAWANY33
Question 7.11 The Gail model includes which of the following risk factors?
A. Second-degree relatives with breast cancer
B. Previous breast biopsies
C. Age at first pregnancy conception
D. Age at menopause
Question 7.12 Which of the following statements about ductal carcinoma
in situ (DCIS) is TRUE?
A. DCIS accounts for 15%–30% of mammographically detected cancers.
B. DCIS is most common in women aged 40–49 years.
C. Younger women have a lower rate of local recurrence after local
therapy.
D. Sentinel lymph node biopsy should be performed routinely during
breast-conservation surgery for nonpalpable DCIS.
Question 7.13 The use of radiation therapy (RT) after lumpectomy in
patients with DCIS:
A. Improves overall survival (OS).
B. Reduces the risk of recurrent DCIS, but not invasive disease, in the
treated breast.
C. Does not decrease local recurrence when tamoxifen therapy is
administered.
D. None of the above.
Question 7.14 According to the AJCC (American Joint Committee on
Cancer) eighth edition, which factor is not included in breast cancer staging
in addition to traditional TNM (tumor, node, metastasis) staging:
A. ER expression
B. Tumor grade
C. Ki 67
D. Oncotype DX if applicable
Question 7.15 Which of the following statements concerning local
recurrences after breast-conserving therapy is FALSE?
A. The underlying molecular subtype is the most significant determinant
of the likelihood of local recurrence after breast-conserving therapy
and mastectomy, particularly when negative margins are achieved.
B. Women initially treated with breast-conserving surgery may be offered
salvage mastectomy.
C. Local recurrences do not require systemic staging because very few
patients with local recurrences have concurrent metastatic disease.
D. Chemotherapy after local recurrence improves survival in ER-negative
tumors.
Question 7.16 Which of the following is an absolute contraindication to
breast-conserving surgery requiring RT?
A. RT during pregnancy
B. Diffuse suspicious or malignant-appearing microcalcifications
C. Widespread disease that cannot be incorporated by local excision
through a single incision that achieves negative margins with a
satisfactory cosmetic result
D. All of the above
Question 7.17 Identify the CORRECT statement.
A. Patients aged 70 and older with stage 1, ER-positive breast cancer
treated by lumpectomy and tamoxifen alone have no difference in
locoregional recurrence rates relative to those who receive the addition
of whole-breast radiation.
B. Preoperative chemotherapy has a long-term survival advantage relative
to adjuvant chemotherapy.
C. Concurrent use of pertuzumab with trastuzumab and chemotherapy is
appropriate in women receiving preoperative chemotherapy for HER2-
positive breast cancer.
D. All of the above.
Question 7.18 The gains associated with adjuvant tamoxifen:

https://t.me/ALGRAWANY33
A. Are independent of patient age.
B. Are less in patients who also receive adjuvant chemotherapy.
C. Are dependent on patient menopausal status.
D. Decline 10 years after diagnosis.
Question 7.19 Identify the patient most appropriate for ovarian
suppression.
A. A 31-year-old female with T2N1 triple-negative breast cancer
B. A 38-year-old female with T1cN1 ER-positive HER2 negative-breast
cancer
C. A 47-year-old female with a T1cN0 ER-positive HER2 negative-breast
cancer with an Oncotype DX score of 11
D. A 75-year-old female with T2N0 ER-positive HER2 negative-breast
cancer with a history of osteoporosis with prior hip fracture
Question 7.20 Identify the CORRECT statement regarding the use of
taxanes in the adjuvant setting.
A. Concurrent administration of docetaxel/doxorubicin/cyclophosphamide
chemotherapy is better tolerated than is sequential dose-dense AC
(Adriamycin and Cyclophosphamide) followed by paclitaxel due to
lower rates of neutropenia.
B. Weekly paclitaxel is associated with increased grades 3 and 4
neutropenia relative to dose-dense paclitaxel.
C. Weekly paclitaxel is associated with increased grades 3 and 4
neuropathy relative to dose-dense paclitaxel.
D. Incorporation of gemcitabine into adjuvant anthracycline- and taxane-
based chemotherapy improves efficacy.
Question 7.21 Risk factors for cardiac dysfunction with trastuzumab
include:
A. Preexisting cardiac disease, age older than 65 years.
B. Non–anthracycline-based chemotherapy, age older than 60 years.
C. Age older than 65 years, current aspirin use for cardio protection,
hyperthyroidism.
D. None of the above.
Question 7.22 A 57-year-old woman has been taking anastrozole as
adjuvant therapy for a moderately differentiated T1N1 stage II breast cancer
for 3 years. During a routine follow-up visit you recommend:
A. History and physical examination and complete blood count and liver
function tests.
B. History and physical examination, complete blood count and liver
function tests, and cancer antigen 15-3.
C. History and physical examination.
D. History and physical examination, complete blood count and liver
function tests, cancer antigen 15-3, and yearly computed tomography
scans.
Question 7.23 Regarding inflammatory breast cancer (IBC), which of the
following is TRUE?”:
A. Patients presenting with clinical features of IBC but no discrete
palpable breast mass may forego systemic staging.
B. The presence of dermal lymphatic involvement indicates IBC
regardless of the clinical features.
C. Adjuvant radiation is not necessary in patients with IBC who achieve a
pathologic complete response to neoadjuvant systemic therapy.
D. None of the above.
Question 7.24 With regard to the clinical and pathologic characteristics of
male breast cancer, which of the following is TRUE?
A. Male breast cancer is found, more often than is female breast cancer, to
be estrogen receptor negative.
B. Liver cirrhosis and mumps orchitis are associated with a decreased risk
of male breast cancer.
C. The median age of onset is 10 years younger than is the median age of
onset for females.

https://t.me/ALGRAWANY33
D. Sentinel node biopsy is the preferred treatment for clinically node-
negative patients.
Question 7.25 Regarding bone metastases, which of the following is
TRUE?
A. The optimal duration of receptor activator of nuclear factor-κβ
(RANK)-ligand inhibition is 2 years.
B. Intravenous bisphosphonates and RANK-ligand inhibitors lessen the
pain associated with bone metastases.
C. Bone-directed therapies in patients with widespread bone metastases
improve OS.
D. Only patients with asymptomatic sclerotic bone metastases benefit
from bone-directed therapies.
Question 7.26 A 58-year-old woman has a history of T2N1 ER-positive
HER2 negative-breast cancer and underwent mastectomy followed by
adjuvant chemotherapy. She has been on letrozole for the past 3 years.
During an evaluation for mild hip pain, she was found to have bone-only
metastatic disease. First-line treatment includes:
A. Anastrozole alone.
B. Palbociclib and fulvestrant.
C. Paclitaxel.
D. Everolimus and exemestane.
Question 7.27 What is the most common side effect associated with
alpelisib?
A. Neutropenia
B. Diarrhea
C. Hyperglycemia
D. Fatigue
Question 7.28 What is a rare toxicity of poly (ADP-ribose) polymerase
(PARP) inhibitors?
A. Myelodysplastic syndrome/acute myeloid leukemia (MDS/AML)
B. Squamous cell carcinomas of the skin
C. Hepatitis
D. Hypophysitis
Question 7.29 Which patient is eligible for immunotherapy?
A. A 31-year-old female with T3N1 triple-negative breast cancer
undergoing neoadjuvant chemotherapy
B. A 65-year-old female with metastatic ER-positive breast cancer who
has progressed on CDK 4/6 inhibitors, PD-L1 >1%
C. A 55-year-old female with unresectable triple-negative breast cancer
with PD-L1 >1%
D. A 32-year-old female with metastatic triple-negative breast cancer who
has progressed on multiple lines of chemotherapy, PD-L1 is negative
Question 7.30 A 44-year-old female presents with isolated right axillary
lymphadenopathy. No breast mass is palpable on examination. Mammogram
is negative. The next step involves:
A. Next-generation sequencing of axillary biopsy.
B. Upper endoscopy.
C. Breast MRI.
D. Positron emission tomography (PET) scan.

ANSWERS

Question 7.1 The correct answer is B.


Approximately 10% of all breast cancers are associated with germline
mutations, whereas the rest occur sporadically.
Question 7.2 The correct answer is A.
BRCA1-associated breast cancers usually are characterized by a “triple-
negative” phenotype (ER, PR, and HER2 negative). BRCA2 is

https://t.me/ALGRAWANY33
associated with pancreatic cancer and melanoma as well as ER-positive
breast cancer. BRCA1 and BRCA2 mutations are inherited in an
autosomal dominant manner.
Question 7.3 The correct answer is B.
Li–Fraumeni syndrome is characterized by TP53 mutations leading to
breast cancer, soft tissue sarcoma, central nervous system tumors,
adrenocortical cancer, leukemia, and prostate cancer.
Question 7.4 The correct answer is C.
Luminal A breast cancer is characterized by high-expression levels of
ER-related genes and low expression of the HER2 cluster and
proliferation-associated genes. It rarely has mutations in TP53, and has
an overall better prognosis than do other forms of breast cancer.
Question 7.5 The correct answer is C.
Clinical practice guidelines support consideration for multigene testing
to guide the addition of combination chemotherapy to standard hormone
therapy for hormone receptor–positive patients with T1–3 tumors who
are node negative based on the TAILORx trial (Trial Assigning
Individualized Options for Treatment).1 Because the HER2 is negative,
HER2-based therapy would not be recommended. There is no indication
for postmastectomy radiation in this patient.
Question 7.6 The correct answer is A.
Cowden syndrome, which is associated with germline mutations in
PTEN, confers a 25% to 50% lifetime risk of breast cancer. Other risk
factors include late age (after age 30) at first full-term pregnancy, >75%
breast density, and proliferative breast lesions.
Question 7.7 The correct answer is A.
The ACS guidelines for MRI screening of breast cancer include
individuals with BRCA mutations; untested first-degree relative of BRCA
carrier; those with lifetime risk of breast cancer from 20% to 25%;
radiation to the chest between ages 10 and 30 years; Li–Fraumeni,
Cowden, or Bannayan–Riley–Ruvalcaba syndromes; and first-degree
relatives of patients with breast cancer. A recent study showed that the
addition of MRI screening in women with dense breast tissue resulted in
significantly fewer interval cancers than did mammography alone,
although it is unclear how this will impact clinical practice.2
Question 7.8 The correct answer is B.
History of deep vein thrombosis, stroke, pulmonary embolism, or
transient ischemic attacks are considered contraindications to the use of
tamoxifen and raloxifene. Tamoxifen has been reported to increase the
risks of cataracts. Some SSRIs, particularly paroxetine, fluoxetine, and
bupropion, are strong CYP2D6 inhibitors that may decrease the
conversion of tamoxifen to its active form.
Question 7.9 The correct answer is C.
Core needle biopsy is the most appropriate way to diagnose suspected
invasive disease. Fine-needle aspiration does not reliably distinguish
invasive cancer from DCIS. Excisional biopsy as a diagnostic technique
should be reserved for patients with imaging abnormalities that cannot
be targeted by core biopsy. Circulating tumor DNA in screening and
diagnosis of breast cancer remains investigational.
Question 7.10 The correct answer is B.
Both tamoxifen and raloxifene have been associated with a decrease in
the risk of breast cancer in patients with atypical hyperplasia. However,
although the risk reduction for tamoxifen may be seen in both
premenopausal and postmenopausal women, the data with the raloxifene
is restricted to postmenopausal women. The risk reduction by tamoxifen
is 84%. Oophorectomy before menopause decreases the risk of breast
cancer by 50% to 65% depending on age at the time of surgery.
Although bilateral mastectomy decreases the risk by more than 90%, the
risk is not completely eliminated.
Question 7.11 The correct answer is B.

https://t.me/ALGRAWANY33
The Gail model includes first-degree relatives with breast cancer,
previous breast biopsies, age at menarche, age at first live birth, race,
and ethnicity.
Question 7.12 The correct answer is A.
DCIS accounts for 15% to 30% of mammographically detected cancers,
and is most common among women aged 49 to 69 years. Several studies
have reported an increased risk of local recurrence in younger women.
The routine use of sentinel lymph node biopsy is not recommended due
to the low rates of axillary involvement (1%–2%) and recurrence despite
the lack of axillary surgery (<0.1%). Nevertheless, selected patients at
high risk for axillary lymph node involvement may benefit from sentinel
node biopsy.
Question 7.13 The correct answer is D.
The use of RT after breast-conserving therapy for DCIS reduces both
invasive and noninvasive recurrences, but does not alter OS.
Question 7.14 The correct answer is C.
According to the AJCC eighth edition, in addition to traditional TNM
staging, ER, PR, and HER2 expression, as well as tumor grade and
Oncotype DX are incorporated into breast cancer staging.
Question 7.15 The correct answer is C.
The underlying molecular subtype is the most significant determinant of
the likelihood of local recurrence after breast-conserving therapy or
mastectomy, particularly when a negative margin is achieved. Patients
with local regional recurrences warrant restaging, and more than 60%
will eventually develop metastatic disease. Women initially treated with
breast-conserving therapy may be offered salvage mastectomy.
Chemotherapy reduces the risk of subsequent cancer recurrence and
improves OS, especially in ER-negative tumors.
Question 7.16 The correct answer is D.
All of the above are absolute contraindications to breast-conserving
therapy followed by RT.
Question 7.17 The correct answer is C.
In 2013, the U.S. Food and Drug Administration (FDA) approved
concurrent use of pertuzumab with trastuzumab and chemotherapy in
women receiving preoperative chemotherapy for HER2-positive breast
cancer. Patients aged 70 and older with stage I, ER-positive breast
cancer treated by lumpectomy and tamoxifen alone have no difference in
OS relative to those who receive the same therapy with the addition of
whole-breast radiation, although the latter approach is associated with an
improvement in locoregional recurrence. Preoperative chemotherapy has
no long-term survival advantage relative to adjuvant chemotherapy.
Question 7.18 The correct answer is A.
Adjuvant tamoxifen results in an improvement in OS for at least 15
years. The benefits are independent of age, menopausal status, and the
use of chemotherapy.
Question 7.19 The correct answer is B.
According to the trials SOFT3 (Suppression of Ovarian Function Trial)
and TEXT (Tamoxifen and Exemestane Trial),4 younger premenopausal
patients with ER-positive, node-positive breast cancers who need
adjuvant chemotherapy should receive ovarian suppression with either
tamoxifen or an aromatase inhibitor. The addition of ovarian
suppression to tamoxifen improves OS. Premenopausal women with
low-risk breast cancer may receive tamoxifen without ovarian
suppression.
Question 7.20 The correct answer is B.
Weekly paclitaxel is associated with increased grades 3 and 4
neutropenia relative to dose-dense paclitaxel because pegfilgrastim
support is not administered with the former, whereas dose-dense
paclitaxel is associated with increased grades 3 and 4 neuropathy.

https://t.me/ALGRAWANY33
Incorporation of gemcitabine into adjuvant anthracycline- and taxane-
based chemotherapy does not improve efficacy.
Question 7.21 The correct answer is A.
Preexisting cardiac disease, age older than 65 years, anthracycline-based
chemotherapy, borderline left ventricular ejection fraction, and
hypertension are all risk factors for cardiac dysfunction with adjuvant
trastuzumab.
Question 7.22 The correct answer is C.
Complete blood counts and liver function tests, cancer antigen 15-3, and
yearly computed tomography scans are not indicated in routine
surveillance for patients with early-stage breast cancer.
Question 7.23 The correct answer is D.
All patients presenting with clinical features of IBC should undergo full-
staging workup due to the substantial risk of metastatic disease.
Involvement of dermal lymphatics in the absence of clinical findings
does not indicate IBC because it is a clinical diagnosis. Due to the high
risk of local recurrence, all patients with IBC should undergo
postmastectomy radiation regardless of the response to neoadjuvant
therapy.
Question 7.24 The correct answer is D.
Male breast cancer is found, more often than is female breast cancer, to
be ER positive. Chronic liver disorders, Klinefelter syndrome (XXY),
BRCA1 or 2 mutations, mumps orchitis, undescended testis, testicular
injury, and feminization are all associated with an increased risk of male
breast cancer. The median age of onset is later than that in females. The
same considerations regarding nodal surgery pertain for men as for
women.
Question 7.25 The correct answer is B.
The optimal duration of intravenous bisphosphonates or RANK-ligand
inhibitors is not well characterized. Intravenous bisphosphonates and
RANK-ligand inhibitors lessen the pain associated with bone metastases
and prevent skeletal-related complications, such as hypercalcemia and
fractures. Bone-directed therapies in patients with widespread bone
metastases do not impact OS.
Question 7.26 The correct answer is B.
Frontline therapy for hormone-positive metastatic breast cancer includes
a CDK 4/6 inhibitor. Because this patient had a recurrence while on
endocrine therapy with an aromatase inhibitor, treatment includes a
CDK 4/6 inhibitor with fulvestrant.
Question 7.27 The correct answer is C.
In the SOLAR-1 trial, the most common grade 3 toxicity from alpelisib
was hyperglycemia.5
Question 7.28 The correct answer is A.
A rare (<1%–1.5%), but potentially fatal, side effect of PARP inhibitors
is the development of MDS or AML. Patients should be monitored
closely for hematologic toxicity.6
Question 7.29 The correct answer is C.
The only current approval for immunotherapy in breast cancer is in
patients with unresectable locally advanced or metastatic triple-negative
breast cancer that is PD-L1 positive.7
Question 7.30 The correct answer is C.
Isolated axillary metastases can be an uncommon presentation of breast
cancer. Initial evaluation includes breast MRI if mammogram finding is
negative.

References
1. Sparano JA, Gray RJ, Makower DF, et al. Adjuvant chemotherapy guided by a 21-gene expression
assay in breast cancer. N Engl J Med. 2018;379:111–121.

https://t.me/ALGRAWANY33
Bakker MF, de Lange SV, Pijnappel RM, et al. Supplemental MRI screening for women with
2.
extremely dense breast tissue. N Engl J Med. 2019;381:2091–2102.
3. Francis PA, Regan MM, Fleming GF, et al. Adjuvant ovarian suppression in premenopausal breast
cancer. N Engl J Med. 2015;372:436–446.
4. Pagani O, Regan MM, Walley BA, et al. Adjuvant exemestane with ovarian suppression in
premenopausal breast cancer. N Engl J Med. 2014;371:107–118.
5. André F, Ciruelos E, Rubovszky G, et al. Alpelisib for PIK3CA-mutated, hormone receptor–
positive advanced breast cancer. N Engl J Med. 2019;380:1929–1940.
6. Litton JK, Rugo HS, Ettl J, et al. Talazoparib in patients with advanced breast cancer and a
germline BRCA mutation. N Engl J Med. 2018;379:753–763.
7. Schmid P, Adams S, Rugo HS, et al. Atezolizumab and nab-paclitaxel in advanced triple-negative
breast cancer. N Engl J Med. 2018;379:2108–2121.
___________
Corresponding chapters in DeVita, Hellman, and Rosenberg’s Cancer: Principles & Practice of
Oncology, Eleventh Edition: 78 (Molecular Biology of Breast Cancer), 79 (Malignant Tumors of the
Breast), and 80 (Genetic Testing in Breast Cancer).
8 Esophageal and Gastric Cancer
Ramon Jin and Haeseong Park

QUESTIONS

Each of the numbered items below is followed by lettered answers. Select the
ONE lettered answer that is BEST in each case unless instructed otherwise.

Question 8.1 Which of the following statements about the incidence of


esophageal cancer is TRUE?
A. Esophageal cancer is relatively uncommon in the United States, and the
lifetime risk of being diagnosed with the disease is <1%.
B. Incidence rates among white men have increased to between 7 and 8
per 100,000 person-years, and reflect the marked increase in the
incidence of adenocarcinoma of the esophagus of more than 400% over
the past two decades.
C. Although the incidence of adenocarcinoma in Caucasian females (1.6
per 100,000) is lower than that in white men, rates of adenocarcinoma
have increased in women by more than 300% over the past 20 years.
D. All of the above.
Question 8.2 Which of the following statements is TRUE?
A. Blood group B has been associated with gastric cancers.
B. Most patients with Helicobacter pylori infection will develop gastric
cancer.
C. Epstein–Barr virus (EBV) infection has not been identified in
association with gastric carcinoma.
D. The World Health Organization (WHO) classifies H. pylori infection
as a class I carcinogen.

https://t.me/ALGRAWANY33
Question 8.3 Which of the following are risk factors for development of
esophageal adenocarcinoma?
A. Mediterranean diet
B. Low body mass index (BMI)
C. Barrett esophagus
D. H. pylori infection
Question 8.4 A 65-year-old man is diagnosed endoscopically with early-
stage gastric cancer and is treated with endoscopic resection. Pathology of the
resection specimen shows a 1.7-cm adenocarcinoma with mucosal depth of
invasion and active H. pylori infection. Which of the following interventions
has been shown to decrease rates of metachronous gastric cancer and improve
baseline stomach atrophy?
A. Proton pump inhibitor use
B. H. pylori eradication
C. Repeat endoscopic resection
D. Repeat endoscopic surveillance in 3 months
Question 8.5 Which of the following is TRUE regarding the molecular
subtypes of gastroesophageal cancer?
A. Distal esophageal adenocarcinoma is predominantly the chromosomal
instability (CIN) subtype.
B. There are three predominant gastric cancer molecular subtypes.
C. Proximal squamous cell carcinomas resemble distal adenocarcinomas
of the esophagus depending on molecular characterization.
D. These molecular subtypes are based solely on RNA transcriptional
expression analyses.
Question 8.6 Which of the following is TRUE about Barrett esophagus
with high-grade dysplasia?
A. Esophageal cancer incidence per year is 30%.
B. Esophagectomy is the only available treatment option.
C. Endoscopic mucosal resection (EMR) can be used as a therapeutic
option.
D. Surveillance with annual endoscopy is sufficient.
Question 8.7 A 57-year-old Caucasian woman with long-standing reflux
symptoms presented with hematemesis to a local emergency department.
Endoscopy revealed a distal esophageal malignant ulcer with the epicenter at
3 cm proximal to the esophagogastric junction (EGJ). Biopsy revealed
intestinal metaplasia at the gastroesophageal junction (GEJ), and confirmed a
diagnosis of adenocarcinoma. Computed tomography (CT) showed thickened
distal esophagus and proximal stomach, hiatal hernia, and enlarged celiac
nodes. Which statement most accurately describes her cancer?
A. She has a Siewert type I cancer: adenocarcinoma of the distal
esophagus, which usually arises from an area with specialized
intestinal metaplasia of the esophagus (Barrett esophagus) and may
infiltrate the EGJ from above.
B. She has a Siewert type II cancer: adenocarcinoma of the cardia, which
arises from the epithelium of the cardia or from short segments with
intestinal metaplasia at the EGJ.
C. She has a Siewert type III cancer: adenocarcinoma of the subcardial
stomach, which may infiltrate the EGJ or distal esophagus from below.
D. The endoscopic description does not allow for the accurate Siewert
type classification.
Question 8.8 A 48-year-old man with a long-standing history of
gastroesophageal reflux disease (GERD) presented to the clinic with
dysphagia and weight loss. The patient had esophagogastroduodenoscopy
(EGD) with biopsies that showed a friable circumferential lesion in the lower
part of the esophagus. Endoscopic ultrasound (EUS) revealed a cT3 lesion
with one regional lymph node. CT and positron emission tomography (PET)
scan for evaluation did not show any distant sites of metastasis. Initial
treatment with preoperative chemoradiation using weekly carboplatin and
paclitaxel was suggested for the patient. Which of the following is TRUE for
this treatment modality?
A. The rate of pathologic complete response is 5%.

https://t.me/ALGRAWANY33
B. Chemoradiation followed by surgery can be used only for localized
esophageal adenocarcinoma but not for squamous cell carcinoma.
C. Chemoradiation followed by surgery improves progression-free
survival (PFS) but not overall survival (OS) compared with surgery
alone.
D. Chemoradiation followed by surgery improves both PFS and OS
compared with surgery alone.
Question 8.9 A 54-year-old male has a 10-year history of untreated GERD.
He presents to his primary care physician with melena. The patient underwent
EGD which revealed a mass in the GEJ that is biopsy proven to be
adenocarcinoma. EUS and PET-CT scan showed a mass that is abutting the
diaphragm, but no evidence of distant metastases. The patient has no
comorbid conditions and has very good performance status. Which of the
following responses is TRUE about using perioperative chemotherapy for
this patient?
A. The lesion is not resectable and hence only chemotherapy should be
offered.
B. The FLOT regimen (5-fluorouracil [5FU], leucovorin, oxaliplatin, and
docetaxel) is associated with a significant improvement in median OS
compared with ECF (epirubicin, cisplatin, and 5FU) or ECX
(epirubicin, cisplatin, and capecitabine) regimen.
C. The ECF/ECX regimen has significantly lower rates of perioperative
complications compared with the FLOT regimen.
D. Postoperative cycles of chemotherapy may be omitted if a pathologic
complete response is seen.
Question 8.10 A 49-year-old female with a long-standing history of GERD
presents to her gastroenterologist with worsening abdominal pain, weight
loss, and heartburn. EGD with biopsy shows a tumor in the lower third of the
esophagus. Biopsy reveals high-grade adenocarcinoma. EUS shows cT3 and
one paraesophageal lymph node. CT and PET scans done for staging shows
multiple liver lesions consistent with metastasis. The patient has no
comorbidities, and her Eastern Cooperative Oncology Group (ECOG)
performance status is 0. Which of the following is the most appropriate next
step?
A. Chemotherapy with cisplatin and 5FU
B. Docetaxel, cisplatin, and 5FU
C. Epirubicin, cisplatin, and 5FU
D. Check for HER2 overexpression on the esophageal tumor tissue
Question 8.11 In the question above, HER2 was overexpressed on the
tumor tissue. Which treatment do you offer?
A. Chemotherapy
B. Chemotherapy plus cetuximab
C. Chemotherapy plus trastuzumab
D. Supportive care
Question 8.12 Which of the following cases would be sent for genetic
testing?
A. A patient with gastric cancer histopathologically confirmed as diffuse
and older than 50 years with a first-degree relative with stomach cancer
at the age of 55
B. A patient with diffuse gastric cancer, who has an uncle with stomach
cancer diagnosed at the age of 45 years and an aunt with lobular breast
cancer
C. A 55-year-old man diagnosed with diffuse gastric cancer, and no
family history of gastric cancer
D. A 62-year-old male smoker diagnosed with gastric cancer and history
of nasopharyngeal cancer
Question 8.13 Which of the following syndromes is usually associated with
intestinal type of gastric cancer?
A. Lynch syndrome
B. Li–Fraumeni syndrome
C. Familial adenomatous polyposis syndrome
D. Peutz–Jeghers syndrome

https://t.me/ALGRAWANY33
Question 8.14 Which of the following gene mutations is associated with
the highest risk for gastric cancer?
A. BRCA1/2
B. P53
C. CDH1
D. APC
Question 8.15 A 22-year-old woman is diagnosed with metastatic gastric
cancer, with diffuse involvement of the stomach and linitis plastica. Her
father died of the same cancer at the age of 42 years. Her mother is concerned
about familial gastric cancer, and is asking for information about hereditary
gastric cancer and appropriate screening for her other children. Which of the
following would be the most appropriate recommendation to this family?
A. Hereditary gastric cancer is rare and unlikely. She may just have some
environmental exposures or DNA mismatch repair gene mutations that
cannot be screened.
B. She may very well have hereditary early-onset diffuse gastric cancer,
but no surveillance or workup is recommended because this has yet to
be confirmed in larger studies.
C. E-cadherin mutation testing should be considered, prophylactic
gastrectomy should be considered strongly for her siblings if a
germline E-cadherin mutation is confirmed, and mucosal abnormality
can be documented by endoscopic examination of the stomach.
D. E-cadherin mutation testing should be considered, and prophylactic
gastrectomy should be strongly considered for her siblings if a
germline E-cadherin mutation is confirmed, even if no mucosal
abnormalities are seen by endoscopic examination of the stomach.
Question 8.16 Which of the following is TRUE about the use of diagnostic
modalities in patients with gastric cancer?
A. PET scan is less useful in assessing metastasis of signet cell subtype.
B. The accuracy rate of CT scan for staging of advanced gastric cancer is
approximately 96%.
C. There is no role for laparoscopy if staging CT scan finding was
negative.
D. Brain magnetic resonance imaging (MRI) should be routinely done as
part of the initial workup.
Question 8.17 A 48-year-old male who was diagnosed with
adenocarcinoma of the stomach underwent partial gastrectomy. PET-CT
before surgery shows no evidence of distant disease. He had 1 of 18 lymph
nodes removed involved with cancer (D1 resection), and margins were
negative for cancer. The pathologic stage is III (T3 N1 tumor). He was
referred to you to discuss adjuvant treatment options. He has an ECOG
performance status of 1. What is the best treatment modality?
A. Chemotherapy with carboplatin and paclitaxel.
B. There is no evidence for benefit from adjuvant treatment in this case.
C. 5FU and leucovorin, followed by 45 Gray (Gy) in 25 fractions plus
concurrent 5FU and leucovorin.
D. Check for HER2 overexpression, and if positive, treat with
cisplatin/5FU plus trastuzumab.
Question 8.18 A 48-year-old man presents to the clinic with abdominal
pain and weight loss. CT scan of the abdomen and pelvis showed diffuse
gastric wall thickening as well as liver lesions suspicious for metastasis. CT-
guided biopsy of one of the lesions confirmed metastatic adenocarcinoma of
the stomach. HER2 was not overexpressed on the tumor tissue. Which of the
following is TRUE regarding combination chemotherapy for the treatment of
metastatic gastric cancer?
A. Cisplatin is superior to oxaliplatin with a better OS.
B. 5FU is superior to capecitabine with better OS.
C. Oxaliplatin is noninferior to cisplatin in terms of OS.
D. Capecitabine is superior to 5FU in terms of OS.
Question 8.19 A 57-year-old man presented to your clinic for a second
opinion on treatment of metastatic HER2-negative gastric cancer. The patient
has received first-line treatment with epirubicin, cisplatin, and 5FU. On his

https://t.me/ALGRAWANY33
last evaluation, his CT scan of the chest, abdomen, and pelvis showed
progression of the disease in the lungs and liver. The patient remains
asymptomatic, except for fatigue. The weight is stable and ECOG
performance status is 1. Blood work included complete blood count (CBC)
that showed mild anemia, and his chemistry panel including liver function
test and bilirubin was normal. Which of the following is the best
recommendation for this patient?
A. Refer to hospice
B. Erlotinib single agent
C. Ramucirumab/paclitaxel
D. Trastuzumab/cisplatin/capecitabine
Question 8.20 A 68-year-old female with metastatic gastric cancer who has
progressed on frontline chemotherapy is currently being treated with
ramucirumab and paclitaxel. Her ECOG performance status is 1. Restaging
CT scan of the chest, abdomen, and pelvis shows enlarging hepatic lesions
and worsening peritoneal disease with minimal ascites. Further molecular
testing of her tumor shows programmed death-ligand 1 (PD-L1) of 5% by
combined positive score (CPS). All of the following would be appropriate
treatment options for this patient (select two correct responses):
A. Pembrolizumab
B. Trifluridine/tipiracil
C. Bevacizumab
D. Docetaxel

Answers

Question 8.1 The correct answer is D.


Esophageal cancer is relatively uncommon in the United States, and the
lifetime risk of being diagnosed with the disease remains <1%.
Incidence rates among Caucasian men increased up until the year 2000,
reflecting the marked increase in the incidence of adenocarcinoma of the
esophagus of more than 400% in the past two decades, but now have
stabilized between 7 and 8 per 100,000 person-years. Although the
incidence of esophageal cancer in Caucasian females (1.6 per 100,000)
is lower than that in Caucasian males, rates of adenocarcinoma have
increased in white women by more than 300% during the past 20 years.
Question 8.2 The correct answer is D.
As a commensal organism, H. pylori infection is widely prevalent
throughout the world. Despite its classification by the WHO as a class I
carcinogen, infection with H. pylori does not typically lead to gastric
cancer. This underscores the importance of other factors, such as
virulence, environmental factors, and host factors, as well as genetic
polymorphisms (e.g., interleukin-1 beta, a potent inhibitor of acid
secretion). The blood group A phenotype has been reported to be
associated with gastric cancers. Although EBV infection has been noted
in lymphoepithelioid-type gastric cancer, the importance of this finding
is still unclear.
Question 8.3 The correct answer is C.
GERD has been implicated as one of the strongest risk factors for the
development of adenocarcinoma of the esophagus. Chronic reflux is
associated with Barrett esophagus, the premalignant precursor of
esophageal adenocarcinoma. Population-based case-control studies that
examined the relationship between symptomatic reflux and risk of
adenocarcinoma of the esophagus have demonstrated that increased
frequency, severity, and chronicity of reflux symptoms are associated
with a 2- to 16-fold increased risk of adenocarcinoma of the esophagus,
regardless of the presence of Barrett esophagus. Increased BMI is a risk
factor for adenocarcinoma of the esophagus, and individuals with the
highest BMI have up to a sevenfold greater risk of esophageal cancer
than do those with a low BMI. Barrett esophagus remains the single
most important risk factor for developing esophageal adenocarcinoma.
Patients with Barrett esophagus are 11-fold more likely to develop
esophageal adenocarcinoma than are individuals without Barrett
esophagus. Infection with H. pylori, and particularly with cagA-positive

https://t.me/ALGRAWANY33
strains, is inversely associated with the risk of adenocarcinoma of the
esophagus. The mechanism of action is unclear, although an H. pylori
infection can result in chronic atrophic gastritis, leading to decreased
acid production and potentially reducing the development of Barrett
esophagus. The Mediterranean diet has not been associated with
increased risk of esophageal adenocarcinoma.
Question 8.4 The correct answer is B.
In patients with early gastric cancers limited to the mucosa or
submucosa, H. pylori eradication after endoscopic resection has been
shown to decrease rates of metachronous gastric cancer and improve
baseline gastric corpus atrophy. In a landmark study, 396 patients with
early-stage gastric cancer treated with endoscopic resection and found to
be H. pylori positive were randomized to eradication treatment versus
placebo.1 After a median follow-up of 5.9 years, 7.2% of patients in the
treated group compared with 13.4% of patients in the placebo group
developed metachronous gastric cancer (hazard ratio [HR] = 0.50; 95%
CI 0.26–0.94; p = 0.03). Improvement in baseline atrophy at the corpus
was seen in 48.4% of patients in the treated group compared with 15.0%
of patients in the placebo group.
Question 8.5 The correct answer is A.
The Cancer Genome Atlas has provided detailed genomic
characterization of gastric and esophageal carcinomas. Squamous cell
carcinomas are unique from distal esophageal adenocarcinomas.
Squamous cell carcinomas frequently exhibit CCND1 amplifications,
TP63/SOX2 amplifications, and KDM6A deletions. Distal esophageal
adenocarcinomas closely resemble and can be classified as the CIN
subtype of gastric cancers, characterized by ERBB2 amplification,
VEGFA amplifications, and TP53 mutations. In addition, to the CIN
subtype, the three additional subtypes of gastric cancers include EBV,
microsatellite instability (MSI), and genomically stable (GS). Multiple
platforms were used to define these subtypes, including genomic,
transcriptomic, microRNA, DNA methylation, and proteomic analyses.
Question 8.6 The correct answer is C.
High-grade dysplasia in Barrett esophagus is the most powerful
predictor of subsequent invasive adenocarcinoma, and is associated with
a per-year cancer incidence of 6%, thereby warranting therapeutic
intervention. The rationale for esophagectomy is that resection
completely eradicates the mucosa at risk, which prevents progression to
invasive carcinoma. This is supported by older surgical series reporting
previously unidentified invasive cancer, which was present in up to 40%
of resected specimens. The argument against esophagectomies is that
most patients with high-grade dysplasia do not develop invasive
carcinoma in their lifetimes and, in the era of endoscopic resection, that
early cancers can be effectively addressed without an esophagectomy.
Those supporting endoscopic methods, ranging from surveillance to
mucosal ablative and resection techniques, argue that this allows for the
identification of patients with an early invasive lesion that is readily
amenable to cure or elimination of the mucosa at risk, thus preventing
progression. EMR is now considered an essential diagnostic, staging,
and therapeutic option available for patients with either high-grade
dysplasia or superficial esophageal cancers (T1a). Annual endoscopy is
recommended for those patients with low-grade dysplasia, whereas more
frequent screening (every 3 months) is recommended for those patients
with high-grade dysplasia if eradication therapy has not been instituted.
Question 8.7 The correct answer is A.
Adenocarcinomas of the GEJ present a unique challenge because
appropriate management of these tumors as either esophageal or gastric
cancers has been uncertain. The Siewert classification system is based
on demographics, histopathologic variables, and patterns of lymphatic
spread that provides clarity, which is well established, and has been
generally accepted worldwide.2 In this classification scheme, type I
tumors are considered adenocarcinomas of the distal esophagus located
between 5 and 1 cm proximal to the EGJ; type II tumors are located in
the cardia, between 1 cm proximal and 2 cm distal to the EGJ; and type
III tumors are located in the subcardial area, between 2 and 5 cm distal
to the EGJ. This classification system allows for a tailored and

https://t.me/ALGRAWANY33
consistent surgical approach to these tumors, as well as consistency in
reporting outcome results associated with therapeutic interventions.
Question 8.8 The correct answer is D.
The CROSS (Chemoradiotherapy for Oesophageal Cancer Followed by
Surgery Study) trial has, for many practitioners, established a new
standard of care for preoperative chemoradiotherapy.3 This study
randomized 366 patients with squamous cell carcinoma or
adenocarcinoma of the esophagus or GEJ to treatment with (1)
preoperative carboplatin at an area under the curve of 2 mg/mL per
minute and paclitaxel 50 mg/m2 once weekly for 5 weeks, and
concurrent radiotherapy (1.8 Gray [Gy] daily to 41.4 Gy) in 23 fractions,
followed by transthoracic esophagectomy or transhiatal esophagectomy
for GEJ cancers, or (2) immediate surgery. Unlike earlier trials, this
more modern trial staged all patients by EUS and CT scan, and patients
were required to have either T1N1 or T2-3N0-1 disease and no evidence
of metastases. The majority of patients treated had adenocarcinoma
(75%), and most tumors involved the distal third of the esophagus
(58%). The majority of patients were node positive (65%), and slightly
more patients on the chemoradiotherapy arm had T3 tumors (84%)
compared with the surgery-alone arm (78%). At a median follow-up of
45 months, the trial showed a significant survival benefit for
chemoradiotherapy added to surgery, with a median survival increased
from 24 to 49 months (HR, 0.0657; p = 0.003), and improvement in 2-
and 5-year OS (67% and 47% vs. 50% and 34%; HR, 0.665). The rate of
pathologic complete response was 25%.
Question 8.9 The correct answer is B.
The phase II/III FLOT4-AIO trial compared the FLOT regimen to the
ECF/ECX regimens for perioperative systemic treatment of resectable
gastric or GEJ tumors.4 A total of 716 patients were enrolled and
randomized to four preoperative and four postoperative 2-week cycles of
FLOT versus three preoperative and three postoperative 3-week cycles
of ECF/ECX. Pre- and postoperative treatments were completed
regardless of resection pathology findings. Median follow-up was 43
months, and the FLOT regimen showed significantly greater median OS
(50 vs. 35 months, HR 0.77, 95% CI 0.63–0.94) and 3-year OS (57% vs.
48%). In terms of toxicity, the rates of perioperative complications were
similar, with 50% seen for the FLOT regimen and 51% seen for the
ECF/ECX regimens. The rates of pathologic complete responses were
analyzed in the phase II portion of the study, and the FLOT regimen was
shown to have higher rates (16% vs. 8%) compared with the ECF/ECX
regimen.
Question 8.10 The correct answer is D.
HER2 testing is now recommended for all patients with metastatic EGJ
adenocarcinoma at the time of diagnosis.
Question 8.11 The correct answer is C.
The combination of chemotherapy, including cisplatin and either
capecitabine or 5FU, with trastuzumab has been associated with
significant improvement outcomes compared with chemotherapy alone,
with increased antitumor response (47.3% vs. 34.5%), PFS (6.7 vs. 5.5
months), and OS (13.8 vs. 11.1 months) (HR, 0.74; p = 0.0046).
Question 8.12 The correct answer is B.
At the second meeting of the International Gastric Cancer Linkage
Consortium (IGCLC) in 2010, hereditary diffuse gastric cancer (HDGC)
guidelines were extended to recommend CDH1 genetic testing to
families with the following: (1) cases of gastric cancer in which one case
is histopathologically confirmed as diffuse and younger than 50 years;
families with both lobular breast cancer and diffuse gastric cancer, with
one diagnosed younger than 50 years; and probands diagnosed with
diffuse gastric cancer and younger than 40 years, with no family history
of gastric cancer.
Question 8.13 The correct answer is A.
Lynch syndrome–associated gastric cancers predominantly show
intestinal histology (more than 90% of the cases). This correlation
echoes the strong association between MSI tumor phenotype and

https://t.me/ALGRAWANY33
intestinal gastric cancer.
Question 8.14 The correct answer is C.
Mutations in the CDH1 gene are associated with a more than 60% risk
of developing gastric cancer.
Question 8.15 The correct answer is D.
Large families with an autosomal-dominant, highly penetrant inherited
predisposition for the development of gastric cancer is rare. However,
early-onset diffuse gastric cancers have been described and linked to the
E-cadherin/CDH1 locus on 16q and associated with mutations in this
gene, which may be the case in this family. This seminal finding has
been confirmed in other studies, with gastric cancers at a relatively high
(67%–83%) penetrance rate. Thus, E-cadherin mutation testing should
be considered in the appropriate clinical setting. In fact, prophylactic
gastrectomy should be considered strongly in families with germline E-
cadherin mutation even without gross mucosal abnormalities on
endoscopic examination of the stomach. Hereditary nonpolyposis colon
cancer (HNPCC) involves germline mutations of DNA mismatch repair
genes. Gastric adenocarcinoma may be observed in families with
HNPCC.
Question 8.16 The correct answer is A.
Whole-body 2-[18F]-fluoro-2-deoxyglucose (FDG) PET is being
applied increasingly in the evaluation of gastrointestinal malignancies.
In gastric cancer, approximately half of the primary tumors are FDG
negative. The diffuse (signet cell) subtype is most likely to be non-FDG
avid, likely because of decreased expression of the glucose transporter-1
(Glut-1). Laparoscopic staging may detect CT-occult metastatic disease
in approximately 40% of patients and spare nontherapeutic operations in
approximately one-third of patients with gastric cancer. Staging
laparoscopy with or without cytology should be considered only if
therapy will be altered consequent to information obtained by
laparoscopy.
Question 8.17 The correct answer is C.
The Intergroup Trial (INT 0116) randomized patients to receive surgery
alone or surgery plus postoperative 5FU-based chemotherapy and
radiation.5 The trial included patients with stages IB–IVA nonmetastatic
adenocarcinoma of the stomach or GEJ. After en bloc resection (D0
resection 54%, D1 resection 36%, and D2 resection 10%), 556 patients
were randomized to either observation alone or postoperative combined-
modality therapy consisting of one monthly 5-day cycle of 5FU and
leucovorin, followed by 45 Gy in 25 fractions plus concurrent 5FU and
leucovorin (4 days in week 1, 3 days in week 5) followed by two
monthly 5-day cycles of 5FU and leucovorin. Nodal metastases were
present in 85% of the cases. With 5 years of median follow-up, 3-year
relapse-free survival was 48% for adjuvant treatment and 31% for
observation (p = 0.001); 3-year OS was 50% for treatment and 41% for
observation (p = 0.005). The median OS in the surgery-only group was
27 months, compared with 36 months in the chemoradiotherapy group;
the HR for death was 1.35 (95% CI = 1.09–1.66; p = 0.005). The HR for
relapse in the surgery-only group as compared with the
chemoradiotherapy group was 1.52 (95% CI = 1.23–1.86; p < 0.001).
The median duration of relapse-free survival was 30 months in the
chemoradiotherapy group and 19 months in the surgery-only group.
Subsequent studies including the ARTIST (Adjuvant Chemoradiation
Therapy in Stomach Cancer) and CRITICS (ChemoRadiotherapy after
Induction chemoTherapy In Cancer of the Stomach) trials have shown
similar survival metrics using adjuvant chemoradiotherapy compared
with chemotherapy alone. Currently, adjuvant chemoradiotherapy can be
considered standard of care for patients who have had completely
resected stage IB or more advanced gastric adenocarcinoma where
adjuvant chemotherapy without radiation can be considered for patients
who have had a D2 resection.
Question 8.18 The correct answer is C.
In the REAL-2 (Randomized ECF for Advanced and Locally Advanced
Esophagogastric Cancer 2) trial, 1,002 patients were randomized to one
of four treatment groups: a control arm of ECF and three investigational

https://t.me/ALGRAWANY33
arms including EOF (epirubicin, oxaliplatin, and 5FU), ECX
(epirubicin, cisplatin, and capecitabine), and EOX (epirubicin,
oxaliplatin, and capecitabine).6 The primary end point was in OS. The
study was powered to show noninferiority for capecitabine compared
with 5FU, and oxaliplatin compared with cisplatin. There were
approximately 250 patients per arm. A total of 40% of patients had
primary gastric cancer, and the remainder had either EGJ or esophageal
cancers, with 10% of patients having squamous cell cancer of the
esophagus. There were no significant differences in median OS among
the arms. The authors concluded the oxaliplatin could be substituted for
cisplatin, and capecitabine could be substituted for 5FU in the palliative
setting.
Question 8.19 The correct answer is C.
The REGARD (Ramucirumab monotherapy for previously treated
advanced gastric or gastro-oesophageal junction adenocarcinoma) study
was a placebo-controlled, double-blind, phase III international trial
conducted in the second-line setting in patients with metastatic gastric or
EGJ adenocarcinoma.7 Median OS was 5.2 months for ramucirumab and
3.8 months for placebo (HR = 0.776; 95% CI = 0.603–0.998; p =
0.0473). The significance of this study is that it provides a proof of
principle that antiangiogenic therapy has activity in gastroesophageal
malignancies. The RAINBOW (Ramucirumab plus paclitaxel versus
placebo plus paclitaxel in patients with previously treated advanced
gastric or gastro-oesophageal junction adenocarcinoma) trial showed
that weekly paclitaxel with ramucirumab significantly improved median
OS compared with weekly paclitaxel with placebo (9.6 vs. 7.4 months,
HR 0.807, 95% CI 0.678–0.962).8 PFS (4.4 vs. 2.9 months) and
objective response rates (41% vs. 19%) were also improved.
Question 8.20 The correct answer is A.
Given the patient’s preserved performance status, there are several third-
line treatment options available for this patient. In the United States,
pembrolizumab is approved for third-line treatment of PD-L1-
overexpressing adenocarcinomas after failure of two separate
chemotherapy regimens. This was based on the KEYNOTE-059 trial, a
multicenter, nonrandomized, open-label multicohort trial that included
259 patients with gastric or GEJ adenocarcinoma whose disease
progressed on at least two prior systemic treatments for advanced
disease.9 An overall response rate of 11.6% was observed, higher in the
PD-L1–positive patients (15.5%) versus the PD-L1–negative patients
(6.4%). It is notable that in the subsequent KEYNOTE-061 trial, 592
patients with advanced gastric or GEJ cancers with progression on first-
line chemotherapy were randomized to pembrolizumab or paclitaxel.10
Although pembrolizumab did not significantly prolong OS (median 9.1
vs. 8.3 months, HR 0.82, 95% CI 0.66–1.03) and the objective response
rates were similar (16% vs. 14%) in patients with positive PD-L1
expression (1% or greater by CPS), it was associated with better toxicity
profile. The TAGS (Trifluridine/tipiracil versus placebo in patients with
heavily pretreated metastatic gastric cancer) trial showed efficacy of
trifluridine/tipiracil in 507 heavily pretreated gastric or GEJ
adenocarcinomas with significantly improved OS over placebo (median
5.7 vs. 3.6 months).11 Once approved, it should be another option for
previously treated patients with gastric cancer. There are currently no
studies showing efficacy of bevacizumab in previously treated advanced
gastric or GEJ adenocarcinomas; and docetaxel is not indicated for this
patient.

References
1. Choi IJ, Kook M-C, Kim Y-I, et al. Helicobacter pylori therapy for the prevention of metachronous
gastric cancer. N Engl J Med. 2018;378:1085–1095.
2. Siewert JR, Stein HJ. Classification of adenocarcinoma of the oesophagogastric junction. Br J Surg.
1998;85:1457–1459.
3. van Hagen P, Hulshof MCCM, van Lanschot JJB, et al. Preoperative chemoradiotherapy for
esophageal or junctional cancer. N Engl J Med. 2012;366:2074–2084.
4. Al-Batran SE, Homann N, Pauligk C, et al. Perioperative chemotherapy with fluorouracil plus
leucovorin, oxaliplatin, and docetaxel versus fluorouracil or capecitabine plus cisplatin and
epirubicin for locally advanced, resectable gastric or gastro-oesophageal junction adenocarcinoma
(FLOT4): a randomised, phase 2/3 trial. Lancet. 2019;393:1948–1957.
5. Macdonald JS, Smalley SR, Benedetti J, et al. Chemoradiotherapy after surgery compared with
surgery alone for adenocarcinoma of the stomach or gastroesophageal junction. N Engl J Med.
2001;345:725–730.
6. Cunningham D, Starling N, Rao S, et al. Capecitabine and oxaliplatin for advanced esophagogastric

https://t.me/ALGRAWANY33
cancer. N Engl J Med. 2008;358:36–46.
7. Fuchs CS, Tomasek J, Yong CJ, et al. Ramucirumab monotherapy for previously treated advanced
gastric or gastro-oesophageal junction adenocarcinoma (REGARD): an international, randomised,
multicentre, placebo-controlled, phase 3 trial. Lancet. 2014;383:31–39.
8. Wilke H, Muro K, Van Cutsem E, et al. Ramucirumab plus paclitaxel versus placebo plus paclitaxel
in patients with previously treated advanced gastric or gastro-oesophageal junction adenocarcinoma
(RAINBOW): a double-blind, randomised phase 3 trial. Lancet Oncol. 2014;15:1224–1235.
9. Fuchs CS, Doi T, Jang RW, et al. Safety and efficacy of pembrolizumab monotherapy in patients
with previously treated advanced gastric and gastroesophageal junction cancer: phase 2 clinical
KEYNOTE-059 trial. JAMA Oncol. 2018;4:e180013.
10. Shitara K, Özgürog˘lu M, Bang YJ, et al. Pembrolizumab versus paclitaxel for previously treated,
advanced gastric or gastro-oesophageal junction cancer (KEYNOTE-061): a randomised, open-
label, controlled, phase 3 trial. Lancet. 2018;392:123–133.
11. Shitara K, Doi T, Dvorkin M, et al. Trifluridine/tipiracil versus placebo in patients with heavily
pretreated metastatic gastric cancer (TAGS): a randomised, double-blind, placebo-controlled, phase
3 trial. Lancet Oncol. 2018;19:1437–1448.
___________
Corresponding chapters in DeVita, Hellman, and Rosenberg’s Cancer: Principles & Practice of
Oncology, Eleventh Edition: 51 (Molecular Biology of the Esophagus and Stomach), 52 (Cancer of the
Esophagus), and 53 (Cancer of the Stomach).
9 Small Intestine Cancers and
Gastrointestinal Stromal Tumors
Katrina S. Pedersen and Nikolaos A. Trikalinos

QUESTIONS

Each of the numbered items below is followed by lettered answers. Select the
ONE lettered answer that is BEST in each case unless instructed otherwise.

Question 9.1 A 60-year-old previously healthy woman noted abdominal


distension and discomfort for 6 months, associated with nausea and vomiting.
Computed tomography (CT) scan shows a 20 × 25 cm abdominal mass, and
exploratory laparotomy demonstrated a pedunculated mass arising from the
stomach. No other metastases were found. A partial gastrectomy was done,
and pathology revealed a gastrointestinal stromal tumor (GIST) that strongly
stains for CD117 and CD34. Sixty mitoses were seen per 50 high-power field
(HPF). Which of the following is TRUE regarding GISTs?
A. The most common mutation associated with GIST involves the
inactivation of a tumor suppressor gene.
B. Both tumor size and mitotic index predict response to imatinib therapy.
C. Gastric GISTs are associated with relatively worse outcomes compared
with small intestinal GISTs.
D. Patients with metastatic GIST tumors harboring exon nine mutations in
KIT have a worse prognosis and response to imatinib compared with
those with exon 11 mutation.
Question 9.2 After the patient in Question 9.1 has recovered from surgery,
what would you recommend for this patient on the basis of current data?
A. Observation with serial scans

https://t.me/ALGRAWANY33
B. Imatinib 400 mg per os (PO) daily for 1 year
C. Imatinib 400 mg PO daily for at least 3 years
D. Sunitinib 50 mg 4 weeks on/2 weeks off therapy for 5 years
E. Pembrolizumab 200 mg intravenous (IV) every 3 weeks for 3 years
Question 9.3 A 70-year-old male with a past medical history of
gastroesophageal reflux disease (GERD) is diagnosed with gastric GIST after
a routine endoscopy with biopsy. Full-body CT shows multiple liver
metastases, and fluorodeoxyglucose (FDG) positron emission tomography
(PET)/CT shows that the lesions are FDG avid with a mean standardized
uptake value (SUV) of 3.5. He is offered imatinib at 400 mg PO daily and
tolerates it well. Repeat PET/CT 2 months later shows that the metastatic
burden is unchanged, with perhaps a slight increase in two liver lesions, but
now none of the tumors are FDG avid. Patient feels well. What is the next
best step in treatment?
A. Increase imatinib to 800 mg PO daily.
B. Urgent biopsy of the liver lesions.
C. Switch to sunitinib.
D. Continue current treatment and repeat imaging in 3 months, paying
attention to contrast enhancement and density of the lesions.
Question 9.4 A 52-year-old woman with metastatic gastric GIST had an
initial complete response to daily imatinib 400 mg with resolution of her
hepatic and peritoneal metastases after 6 months of therapy. Imatinib was
continued for 18 months when her CT scan showed recurrent hepatic lesions.
Imatinib was increased to 800 mg daily. However, subsequent scans revealed
progressive disease. You recommend starting sunitinib for this patient. Which
of the following statements is TRUE?
A. Acquired resistance to imatinib therapy may be associated with the
development of secondary KIT or PDGFRA mutations.
B. Sunitinib therapy for patients with imatinib-resistant GIST did not
improve progression-free survival compared with placebo.
C. Patients with GIST-harboring exon 11 mutation have a higher response
to sunitinib than do those with exon nine mutation.
D. Patients with the wild-type (WT) GIST are resistant to both imatinib
and sunitinib therapy.
Question 9.5 A 75-year-old female with metastatic GIST involving the
liver and omentum progresses on imatinib 400 mg PO daily and then 800 mg
PO daily. She is started on sunitinib 50 mg daily for 28 days followed by a 2-
week break. After two cycles, repeated CT scans showed a decrease in the
measurable lesions. In addition to hypopigmentation of her hair, she also
noted progressive generalized fatigue. Patient denies any dyspnea on
exertion, diarrhea, or pedal edema. Physical examination reveals an
erythematous rash in the hands, clear lungs, no cardiac gallops or rubs, and
no focal neurologic deficits. Pertinent laboratory tests are as follows:
White blood cell (WBC) 5.6 × 103 cells/μL
Hemoglobin 11.8 g/dL
Sodium 145 mmol/L
Potassium 4.5 mmol/L
Creatinine 0.8 mg/dL
Total bilirubin 0.5 mg/dL
Alkaline phosphatase 118 μ/L
Which test would you order next?
A. Magnesium level
B. Magnetic resonance imaging of the brain
C. Thyroid function tests
D. 25-Hydroxycholecalciferol level
Question 9.6 After 6 months of sunitinib therapy, the patient in Question
9.5 was noted to have progression of the liver and omental lesions. The
Karnofsky performance status is 90%, and hematologic, renal, and hepatic
functions are all adequate. She is interested in further therapy. What would be
your best recommendation?
A. No further therapy, and proceed with hospice because there is no

https://t.me/ALGRAWANY33
standard therapy after second-line sunitinib.
B. Sorafenib.
C. Regorafenib.
D. Rechallenge imatinib 800 mg PO daily.
Question 9.7 Familial and genetic syndromes associated with GIST are:
A. Cowden syndrome.
B. Li–Fraumeni syndrome.
C. Carney triad.
D. Turcot syndrome.
Question 9.8 Which of the following statements about GIST is TRUE?
A. Half of GIST tumors demonstrate mutations in either KIT or PDGFRA,
whereas half are WT.
B. Most GIST cases arise from the small bowel, followed by the stomach
and esophagus.
C. Succinate dehydrogenase (SDH)-deficient GISTs usually occur in
younger females.
D. Small GISTs can be followed if <4 cm in size.
Question 9.9 What is the most commonly diagnosed type of small bowel
tumor?
A. Metastasis
B. Adenocarcinoma
C. Lymphoma
D. Carcinoid
Question 9.10 A 28-year-old female presents to the emergency room (ER)
with acute-onset abdominal pain and 2 days of complete intolerance to food
and liquid, with emesis following all attempts at oral intake. She has not had
bowel movements during this time. CT of the abdomen and pelvis with
contrast contains findings of small intestinal obstruction with concern for
small bowel cancer. Where is this tumor most likely to arise?
A. Duodenum
B. Jejunum
C. Ileum
D. Cecum
Question 9.11 The patient in Question 9.10 undergoes
esophagogastroduodenoscopy. The biopsy findings are consistent with small
bowel adenocarcinoma arising from a hamartomatous polyp. In addition to a
full staging workup for treatment planning purposes, you refer for genetic
counseling due to the patient’s young age and polyp histology. You anticipate
a potential germline mutation in which predisposing gene?
A. MUTYH
B. APC
C. STK11
D. MLH1
Question 9.12 Which of the following are characteristic of primary
intestinal mucosal-associated lymphoid tissue (MALT) lymphoma?
A. Association with Hashimoto thyroiditis
B. The majority of patients present with stages III and IV
C. Most common in women
D. Associated with the translocation t(11;14)
Question 9.13 A 50-year-old male presented with bloating and tarry stools.
Upper endoscopy revealed a mass in the third portion of the duodenum, with
the biopsy showing moderately differentiated adenocarcinoma. Resection
demonstrated three periduodenal lymph nodes involved with carcinoma. CT
scan showed no distant metastases. Which of the following statements
regarding small bowel adenocarcinoma is TRUE?
A. Pancreaticoduodenectomy is the preferred surgical resection modality
for duodenal adenocarcinomas.
B. Jejunal and ileal tumors are associated with better outcomes compared
with duodenal adenocarcinoma.

https://t.me/ALGRAWANY33
C. Stage III small bowel adenocarcinoma is associated with a 63%
survival.
D. Adjuvant therapy with irinotecan, 5-fluorouracil, and leucovorin is
standard for stage III duodenal adenocarcinoma.
Question 9.14 A 68-year-old male patient presents with newly diagnosed
neuroendocrine tumor (NET), incidentally found in the terminal ileum after
receiving a CT urogram for voiding concerns. He denies symptoms of
flushing or diarrhea, but notes, in retrospect, that he has had mild right lower
quadrant discomfort for several years. No other lesions were noted on this
scan. He is taken for segmental resection with lymph node dissection. The
tumor is well differentiated and has invaded through the muscularis and into
the adjacent cecum. Of the 18 lymph nodes, 10 harbor tumor cells that stain
strongly for chromogranin and synaptophysin. One 2.5-cm tumor deposit is
found in the mesentery. Ki-67 stains 2% of tumor cells. Margins are clear of
disease. What is the stage of this cancer?
A. Stage III, pT3N1M0
B. Stage III, pT4N1M0
C. Stage III, pT4N2M0
D. Stage IV, pT4N1M1b
Question 9.15 Which of the following is a criterion to differentiate between
primary intestinal and secondary lymphomas?
A. No superficial adenopathy
B. No evidence of splenic involvement except through direct extension of
the primary tumor
C. No evidence of peripheral blood or bone marrow involvement
D. All the above
Question 9.16 A 45-year-old male has experienced several years of
progressive postprandial bloating, nonbloody loose stools, diffuse abdominal
pain, and gradual 55-pound unintentional weight loss. More recently, he
began noting daily fevers and more rapid weight loss. Due to lack of medical
insurance, he has not visited a physician, relying instead on taking up to eight
calcium carbonate tablets, maximum loperamide tablets, and daily
acetaminophen, although without relief. He has become progressively
dyspneic in the month before presenting to the ER for acute-onset emesis and
no bowel movements for the past week. Complete blood count (CBC) shows
hemoglobin of 4.9 g/dL and WBC 6.8 × 109 cells/L. CT scans of the
abdomen and pelvis are obtained and show a mid-jejunal mass with marked
proximal small intestinal and gastric distension. He is emergently taken to the
operating room (OR) for diversion and tumor sampling. Tumor tissue
contains lymphocytic infiltrates with positive immunohistochemical staining
for CD3+, CD8+, and CD103+. The tumor also likely exhibits:
A. Flattened villi.
B. t(8;22) translocation.
C. Cyclin D1 overexpression.
D. Crypt abscesses, transmural fissures, and surrounding dysplasia.
Question 9.17 The risk of progressive disease for a patient with small
intestinal GIST measuring <2 cm with more than five mitoses/50 HPF is:
A. 0%.
B. 4.3%.
C. 24%.
D. 50%.
Question 9.18 A 20-year-old student presented to the emergency
department with a 2-day history of right lower abdominal pain associated
with fever. His abdomen was slightly distended, with diffuse tenderness but
without guarding or rebound tenderness. Rectal examination showed no
masses, and result was negative for occult blood. CBC count showed a
slightly elevated WBC at 11,000. CT scan showed a mass in the terminal
ileum with no evidence of free peritoneal air. Colonoscopy revealed a
terminal ileum mass, the biopsy of which showed sheets of monotonous
round nucleated cells with abundant basophilic cytoplasm with numerous
macrophages. Ki-67 index was 100%. What is the most likely diagnosis?
A. MALT lymphoma
B. Burkitt lymphoma

https://t.me/ALGRAWANY33
C. Peripheral T-cell lymphoma
D. Medullary carcinoma of the small intestine

ANSWERS

Question 9.1 The correct answer is D.


In the majority of malignant GISTs, a gain-of-function mutation results
in the constitutive activation of the KIT proto-oncogene. Approximately
80% of GISTs harbor KIT mutations, whereas another 5% to 7% have
activating PDGFRA mutations. The most common mutation is identified
in the KIT juxtamembrane domain, exon 11. Other mutations in exon 9,
13, 17, and 18 have been described. Exon 11 mutations are associated
with better response rates and median time to progression compared
with the other mutations. The higher prevalence of a specific exon nine
mutation among GISTs originating in the small intestine may explain in
part the observation of worse prognosis of these tumors compared with
gastric GISTs. Although tumor size and mitotic index carry prognostic
significance for patients with GIST, neither has been associated with
response to tyrosine kinase inhibition.
Question 9.2 The correct answer is C.
On the basis of the tumor size and mitotic rate, this patient is at high risk
for recurrence. Thus, observation and serial scans will not be
appropriate. Adjuvant imatinib for 3 years is associated with increased
recurrence-free and overall survival compared with 1 year. There are no
randomized studies on the use of sunitinib in the adjuvant setting.
Checkpoint inhibitors, such as pembrolizumab, are not effective in
GIST, and are not given in the adjuvant setting.
Question 9.3 The correct answer is D.
The patient has an FDG-positive metastatic GIST on first-line imatinib
treatment, and is probably responding, as evidenced by the loss of FDG
avidity. Favorable metabolic response on tyrosine kinase inhibitors
(TKIs) can be rapid; hence, the “cold” lesions on PET/CT, tumor size
might not change immediately, and may even continue to grow in the
early stages of treatment. There is no reason to suspect imatinib failure
in the absence of new or rapidly enlarging lesions. Therefore, increasing
the dose or switching to sunitinib is not appropriate. A biopsy of the
liver tumors could be indicated in the case of discordant growth or with
newly emergent FDG-avid lesions, but at this point, it is premature to
suspect tumor resistance. Sometimes a CT scan can indicate treatment
response if lesions show decreased density or contrast enhancement.
Question 9.4 The correct answer is A.
Primary resistance to imatinib, manifested by continued tumor growth
within the first 6 months of imatinib therapy, occurs in a minority of
GISTs. Secondary resistance generally occurs after a median of 24
months of continued tyrosine kinase inhibition. Acquired mutations in
KIT or PDGFRA have been implicated in the development of drug
resistance to imatinib. The emergence of KIT-independent genotypes
may also result in imatinib resistance. Sunitinib was approved by the
U.S. Food and Drug Administration for the treatment of GIST after
disease progression or intolerance to imatinib. The pivotal phase III
study demonstrated a superior time to progression of 27.3 weeks for
patients treated with sunitinib compared with 6.4 weeks for patients
treated with placebo. Six-month survival rates also favor sunitinib at
79.4% compared with 56.9% for placebo. With the crossover design, the
advantage for survival benefit on longer follow-up diminished. A subset
analysis showed improvement in response rates and outcomes for
patients with GIST-harboring exon nine mutation compared with those
harboring exon 11 mutations. Responses were also observed among
patients with WT genotype.
Question 9.5 The correct answer is C.
Biochemical and clinical hypothyroidism have been associated with
sunitinib therapy. Routine monitoring of thyroid function test is
warranted, and thyroid hormone replacement therapy may alleviate the
symptoms. The precise mechanism for sunitinib-associated

https://t.me/ALGRAWANY33
hypothyroidism is unclear. Destructive thyroiditis through follicular
apoptosis has been postulated for sunitinib-associated thyroid
dysfunction based on the observation of transient thyroid-stimulating
hormone suppression and subsequent absence of visualized thyroid
tissue. Other causes of fatigue associated with sunitinib include adrenal
insufficiency or heart failure.
Question 9.6 The correct answer is C.
Regorafenib, a TKI targeting KIT, PDGFR, and vascular endothelial
growth factor receptor (VEGFR) 1, 2, and 3, is the standard third-line
treatment for GIST refractory to imatinib and sunitinib. Although
sorafenib appears to have activity against GIST, it is not approved as a
standard third-line therapy for GIST due to lack of confirmatory phase
III studies. There are no current data on the use of bevacizumab, an anti-
vascular endothelial growth factor (VEGF) monoclonal antibody for the
treatment of GIST. Although rechallenging with imatinib might be a
reasonable option in select cases, this patient has a clear third-line
option, and has progressed through both 400 and 800 mg PO daily.
Question 9.7 The correct answer is C.
Familial paraganglioma and neurofibromatosis type 1 are associated
with GIST. These syndromes, along with pediatric GIST, can be
characterized by the WT KIT gene. Carney triad, which encompasses
multifocal GIST with pulmonary chondromas and extra-adrenal
paraganglioma is another syndrome associated with GIST. Cowden
syndrome is an autosomal dominant subtype of the PTEN hamartoma
syndrome associated with a lifetime increased risk for various cancers
including breast, colorectal, endometrial, renal cancers, and melanoma.
Li–Fraumeni syndrome is an autosomal dominant hereditary disorder
characterized by TP53 mutation and the development of multiple
malignancies including sarcoma, breast cancer, colorectal and pancreatic
cancers, lymphoma, melanoma, and germ cell tumors. Turcot syndrome
is associated with the development of colonic polyps and central
nervous system (CNS) tumors.
Question 9.8 The correct answer is C.
Only a minority (10%–15%) of GISTs are WT. Most others show KIT
and PDGFRA mutations. The stomach is the most common location of
GISTs (more than 50%) and microscopic GISTs can be found
incidentally in as many as 25% of stomachs because not all are
aggressive disease forms. Other common origins include the small
bowel in approximately 25% of cases, and a minority arise in the
esophagus or rectum. SDH-deficient GISTs are more common in young
females, and can be associated with predisposing conditions such as the
Carney triad, neurofibromatosis type 1 and the Carney–Stratakis
syndrome. Although observation can be considered for some GIST
tumors <2 cm, most tumors >2 cm are considered malignant and should
ideally be excised.
Question 9.9 The correct answer is A.
Metastases are 2.5 times more likely to occur than are primary small
bowel cancers. They tend to arise in the submucosal and muscularis
layers of the intestine. According to the National Cancer Database
(NCDB), the most common primary tumors of the small bowel are
carcinoids. Carcinoids arise from the neuroendocrine cells of Kulchitsky
and have increased fourfold in incidence during the past two decades.
Carcinoids are followed by adenocarcinomas and lymphomas as the
most common small intestinal malignancies. Sarcomas are less likely,
and other tumors are rare.
Question 9.10 The correct answer is A.
Over half of all small intestinal malignancies arise in the duodenum. In
this location, there is a preponderance of adenocarcinoma histology.
Approximately 30% of tumors arise in the jejunum, where there is a
higher incidence of lymphomas, and 20% in the ileum, where carcinoids
may be found more frequently.
Question 9.11 The correct answer is C.
Peutz–Jeghers is a syndrome caused by an autosomal dominant germline

https://t.me/ALGRAWANY33
mutation of the STK11 (LKB1) gene in affected families that causes the
development of hamartomatous and adenomatous polyps. Family
members are more likely to develop gastric, small and large intestinal,
and exocrine pancreatic cancers, although they may arise anywhere
throughout the gastrointestinal (GI) tract. Affected individuals with
Peutz–Jeghers are 15 times more likely to develop small bowel
adenocarcinoma than is the general population. Even greater risk is
conferred in affected individuals with germline mutations of MLH1,
MSH2, and other genes mutated in hereditary nonpolyposis colorectal
cancer (HNPCC, Lynch syndrome) who are 100 times more likely to
develop SBA than are unaffected individuals. The highest known
germline-associated risk is conferred with inherited APC gene
mutations, as seen in familial adenomatous polyposis (FAP), where
affected individuals have a risk of developing small intestinal cancer at
rates 300 times higher than does the general population. This occurs via
the adenoma–carcinoma sequence, similar to what has been well
described in colorectal carcinogenesis. Although seen at lower rates than
FAP, probands with MUTYH-associated polyposis (MAP) still carry a
4% lifetime risk of developing cancer in the duodenum. This patient’s
young age makes MAP less likely as cancers tend to arise in later
decades of life.
Question 9.12 The correct answer is A.
MALT lymphomas are predominantly seen in men, with a peak
incidence in the sixth decade. These malignancies may be associated
with chronic inflammatory disorders such as Hashimoto thyroiditis and
Sjögren disease. The majority of patients present with stages I or II. The
translocation t(11;14) causing overexpression of cyclin D1 is
characteristic of mantle cell lymphoma.
Question 9.13 The correct answer is B.
Small bowel adenocarcinomas are rare malignancies that are slightly
more common in men. The duodenum represents with most common
location, and a pancreaticoduodenectomy has been the traditional gold
standard approach for definitive resection. Because of greater morbidity
and mortality associated with Whipple procedures, segmental intestinal
resection with lymph node dissection has been explored and shown to be
sufficient for some tumors arising in the third and fourth portions of the
duodenum. Compared with more distal lesions, tumors of the duodenum
tend to occur in older patients, and are associated with worse outcomes
with higher risk for recurrence. Overall survival for patients with small
bowel adenocarcinomas for patients with stages I, II, III, and IV disease
are approximately 63%, 48%, 32%, and 4%, respectively. Despite the
high risk of recurrence for patients with stages II and III disease, there
are no standard adjuvant therapy regimens.
Question 9.14 The correct answer is C.
Per the staging guidelines from the eighth edition of the American Joint
Committee on Cancer Staging of Small Intestinal Neuroendocrine
Tumors, this patient has a stage III tumor. This is characterized by a T4
tumor growing through the wall and into adjacent organs (cecum).
Although the lymph nodes involved are fewer than 12, which would
typically qualify for N1 disease, the large single mesenteric tumor
deposit >2 cm increases the stage to N2. A single mesenteric deposit
does not qualify as distant metastatic disease, and thus his M0 stage;
although if a number of peritoneal or mesenteric deposits were seen, the
stage would be M1b. NET staging differs from the staging of small
bowel carcinomas in that not only are T stages defined by depth of
invasion but also in some size considerations of the primary tumor. N
staging permits a much greater number of involved lymph nodes in its
definitions of N1 and N2 disease. Metastatic disease is stratified by
which organs are involved (liver vs. extrahepatic) in NET staging,
whereas any visceral involvement outside the small intestine qualifies as
M1 disease in carcinomas. Ki-67 does not currently play any role in
NET staging, although is prognostic and a predictive biomarker.
Question 9.15 The correct answer is D.
The main criteria to distinguish between primary intestinal lymphomas
and secondary involvement include no superficial palpable
lymphadenopathy, no mediastinal adenopathy, no evidence of peripheral

https://t.me/ALGRAWANY33
blood or bone marrow involvement, no involvement of the liver or
spleen unless by direct extension of the primary tumor, and disease
confined to the affected small bowel and regional draining mesenteric
lymph nodes.
Question 9.16 The correct answer is A.
Enteropathy-associated T-cell lymphoma (EATL) or adenocarcinomas
develop in nearly 40% of individuals with celiac disease who are
nonadherent to gluten elimination or have refractory disease in spite of
exposure minimization. The presence of celiac disease would be evident
in the denuded intestinal villi due to chronic inflammation.
Immunohistology stains in EATL specimens are positive for CD3+,
CD7+, CD8+, and CD103+, and negative for B-cell markers, including
CD19+ and CD20+ that would be expected in marginal zone, diffuse
large cell, and mantle cell lymphomas. Typically, patients with EATL
will undergo resection followed by anthracycline-based adjuvant
chemotherapy. c-Myc translocations t(8;X) are a feature of Burkitt
lymphomas and associated with Epstein–Barr virus (EBV) infections.
Cyclin D1 is overexpressed in mantle cell lymphomas, which are a B-
cell type lymphoma. Crypt abscesses and transmural fissures are a
hallmark of Crohn disease, which has been associated with higher rates
of carcinomas, particularly those arising in the terminal ileum, but not
lymphomas.
Question 9.17 The correct answer is D.
GISTs arising from the small intestine appear to have a more malignant
behavior at a smaller size compared with those found in the stomach or
in the colon. For small tumors <2 cm, those with less than five mitoses
per 50 HPF have a 0% risk of progression, according to one study,
whereas those with more than five mitoses per 50 HPF have a 50% risk
for progression. GISTs larger than 10 cm with more than 10 mitoses per
50 HPF have a 90% risk of progression.
Question 9.18 The correct answer is B.
This patient has Burkitt lymphoma arising from the terminal ileum
presenting with appendicitis-like symptoms. This type of lymphoma
accounts for <5% of all lymphomas in the small intestine. A “starry-
eyed” pattern interspersed throughout monomorphic cells with abundant
basophilic cytoplasm is characteristic and related to numerous
macrophages with ingested apoptotic tumors within. A high proliferative
index is also characteristic of this malignancy. Treatment consists
primarily of aggressive chemotherapy.
___________
Corresponding chapters in DeVita, Hellman, and Rosenberg’s Cancer: Principles & Practice of
Oncology, Eleventh Edition: 59 (Small Bowel Cancer) and 60 (Gastrointestinal Stromal Tumor).

https://t.me/ALGRAWANY33
10 Pancreas and Hepatobiliary Cancer
Patrick Grierson and Katrina S. Pedersen

QUESTIONS

Each of the numbered items below is followed by lettered answers. Select the
ONE lettered answer that is BEST in each case unless instructed otherwise.

Question 10.1 Which of the following factors are associated with an


increased risk of pancreatic cancer?
A. Cigarette smoke
B. Chronic pancreatitis
C. African-American ethnicity
D. All of the above
Question 10.2 Which of the following statements regarding pancreatic
cancer is TRUE?
A. At diagnosis, 31% have evidence of distant metastases.
B. Pancreatic cancer decreases in incidence later in life.
C. Activation of the KRAS oncogene plus inactivation of tumor suppressor
genes (TP53, SMAD4, CDKN2A, and BRCA2) are associated with the
development of pancreatic cancer.
D. Pancreatic intraepithelial neoplasms (PanINs) are intraductal
proliferative epithelial lesions that will not progress to pancreatic
cancer.
Question 10.3 A 56-year-old man is evaluated for a 1-month history of
gradually worsening painless jaundice and a 10-lb weight loss. Computed
tomography (CT) scan of the abdomen and pelvis revealed a 3.3-cm
pancreatic head mass adjacent to the superior mesenteric vein with no
intervening fat plane and encasing both the superior mesenteric vein and
artery. The common bile duct was dilated, and the pancreatic body and tail
were atrophied with dilatation of the pancreatic duct. Portal lymphadenopathy
measuring 2 cm was present. The patient underwent endoscopic retrograde
cholangiopancreatography (ERCP), and a biliary stent was placed. Biopsy
tissue of the mass was consistent with moderately differentiated pancreatic
adenocarcinoma. What stage is this cancer according to the TNM (tumor-
node-metastasis) staging system?
A. Stage I
B. Stage II
C. Stage III
D. Stage IV
Question 10.4 A 67-year-old woman presents to her local emergency
department with a 2-month history of right upper quadrant pain, jaundice,
dyspepsia, and 20-lb weight loss. CT of the abdomen with contrast reveals a
2.5-cm, ill-defined soft tissue density within the head of the pancreas and
adenopathy of the celiac axis, porta hepatis, and portacaval nodes, with the
largest measuring 2.0 × 1.8 cm within the celiac axis region. Following
admission, ERCP with biliary stent placement and biopsy is done. Pathology
reports poorly differentiated adenocarcinoma. What test should be ordered at
this point to help establish her stage?
A. Multiphase multidetector helical CT
B. Magnetic resonance imaging (MRI)
C. Ultrasonography
D. Endoscopic ultrasonography
Question 10.5 Which of the following statements regarding adjuvant
therapy for pancreatic adenocarcinoma is TRUE?
A. In Radiation Therapy Oncology Group (RTOG) 9704, administering
gemcitabine before and after adjuvant 5-fluorouracil (5FU)-based
chemoradiotherapy (CRT) for resected pancreatic head
adenocarcinoma demonstrated a trend toward improved median overall

https://t.me/ALGRAWANY33
survival (OS) compared with 5FU before and after 5FU CRT.
B. Charite Onkologie (CONKO) 001 demonstrated that gemcitabine,
when compared with observation, improves disease-free survival but
not OS.
C. Gastrointestinal Tumor Study Group (GITSG) 9173 clearly showed no
benefit with 5FU-based chemoradiation followed by chemotherapy
compared with observation.
D. European Study Group for Pancreatic Cancer (ESPAC) 1 trial showed
that those who received CRT did better than did those treated with
chemotherapy alone.
Question 10.6 A 71-year-old woman undergoes Whipple resection for a
pT3N1M0 pancreatic adenocarcinoma with negative margins. Postoperative
recovery was uneventful, excepting for pancreatic insufficiency requiring the
use of replacement enzymes. Seven weeks after surgery, she is now fully
ambulatory and participating in baseline activity with minimal disruption.
What treatment course do you recommend at this point in time?
A. Adjuvant gemcitabine
B. Adjuvant gemcitabine and capecitabine
C. Adjuvant FOLFIRINOX (5FU, leucovorin, irinotecan, and oxaliplatin)
D. Surveillance for disease recurrence
Question 10.7 The patient in Question 10.6 starts adjuvant therapy with
modified FOLFIRINOX given intravenously every other week. In a follow-
up visit after her first cycle, she reports a 5-lb weight loss, nausea, decreased
appetite, and diarrhea with floating, greasy stools. What should be done next?
A. Increase pancreatic enzyme supplementation.
B. Admit the patient for small bowel obstruction.
C. Hold chemotherapy for 1 week and follow up on symptoms.
D. CT scan of the chest, abdomen, and pelvis to rule out metastatic
disease.
Question 10.8 A 64-year-old woman is diagnosed with locally advanced
unresectable pancreatic cancer. After 2 months of a gemcitabine-based
chemotherapy regimen, CT scans demonstrate a decrease in the size of the
lesion and the CA19-9 decreased from 854 to 201. What should be done
now?
A. Switch to 5FU-based chemoradiation.
B. Continue current gemcitabine-based chemotherapy for another 2
months.
C. Add radiation to her current regimen.
D. Add 5FU to her chemotherapy regimen.
Question 10.9 A 52-year-old woman is evaluated for chronic right upper
quadrant abdominal pain and is diagnosed with pancreatic adenocarcinoma
with metastasis to the liver. She is distraught and wants to do everything
possible to prolong her life. Which of the following regimens has been shown
to prolong OS compared with gemcitabine alone?
A. Gemcitabine and oxaliplatin
B. Gemcitabine and capecitabine
C. Gemcitabine and nab-paclitaxel
D. Gemcitabine, cisplatin, and bevacizumab
E. Gemcitabine and cetuximab
Question 10.10 A 45-year-old man with unintentional weight loss is
diagnosed with metastatic adenocarcinoma of the pancreas. Which of the
following chemotherapy regimens would be an appropriate first-line
treatment choice in this otherwise healthy patient with normal organ
function?
A. Gemcitabine
B. FOLFIRINOX
C. 5FU with radiation
D. FOLFOX (5FU + leucovorin + oxaliplatin)
Question 10.11 Mutation of which tumor suppressor gene is most
frequently associated with familial pancreatic cancer?
A. BRCA2

https://t.me/ALGRAWANY33
B. PALB2
C. KRAS
D. TP53
Question 10.12 Which one of the following statements regarding risk
factors for pancreatic cancer is CORRECT?
A. People with blood types A, B, and AB are more likely than those with
type O to develop pancreatic cancer.
B. Testing for KRAS mutations in the pancreatic juice of patients is an
effective screening test for pancreatic cancer.
C. Patients with hereditary nonpolyposis colorectal cancer (HNPCC)
syndrome do not have an increased risk of pancreatic cancer.
D. Hereditary pancreatitis is not a significant risk factor for pancreatic
cancer.
Question 10.13 A 65-year-old male diagnosed with metastatic pancreatic
cancer received treatment with single-agent gemcitabine. He now has disease
progression and is interested in pursuing further systemic therapy. Which one
of the following should be offered as a second-line treatment choice?
A. 5FU and oxaliplatin
B. Erlotinib
C. Paclitaxel
D. 5FU/nanoliposomal irinotecan
E. Supportive care alone
Question 10.14 Which of the following is TRUE about pancreatic cancer?
A. Most pancreatic cancers have mutations in KRAS, TP53, SMAD4, and
CDKN2A/p16.
B. Telomere shortening is the earliest and prevalent genetic change
identified in the precursor lesions.
C. Underexpression of tumor growth factor-beta (TGF-β) is observed in
some pancreatic cancers.
D. All of the above.
Question 10.15 A 52-year-old man is found to have mildly abnormal liver
function and an elevated serum alpha-fetoprotein (AFP). Workup reveals
prior hepatitis B virus (HBV) infection. Ultrasound reveals a 4-cm lesion in
the left hepatic lobe, and a CT scan reveals no evidence of metastatic disease
or vascular involvement. MRI demonstrates features consistent with
hepatocellular carcinoma (HCC). The patient undergoes partial hepatectomy,
and surgical margins are clear. The pathology report confirms the diagnosis
of HCC. Which of the following approaches should be followed?
A. Adjuvant sorafenib
B. Combination chemotherapy that is doxorubicin based
C. Adjuvant external beam radiation to surgical bed
D. Routine surveillance
Question 10.16 Which of the following criteria may help guide selection of
patients with HCC appropriate for potential liver transplantation?
A. Patients with solitary tumors ≤5 cm, or patients with multifocal disease
with ≤3 tumor nodules each ≤3 cm in size
B. Patients with Child–Pugh B or C cirrhosis
C. Tumors without evidence of macrovascular invasion and distant
metastasis
D. All of the above
Question 10.17 Which of the following statements about staging systems
for HCC is TRUE?
A. The Okuda system takes into account several clinical features that
include tumor size (>50% of liver), ascites (positive or negative),
hypoalbuminemia (<3 g/dL), and hyperbilirubinemia (>3 mg/dL).
B. The Cancer of the Liver Italian Program system uses hepatic tumor
morphology and extent of liver replacement, Child–Pugh score, portal
vein thrombosis, and serum AFP levels.
C. The Barcelona Clinic Liver Cancer scoring system combines
assessment of tumor stage, liver function, and patient symptoms with a
treatment algorithm, and has been shown to correlate well with patient

https://t.me/ALGRAWANY33
outcomes.
D. All of the above.
Question 10.18 Which of the following abnormalities is both a
paraneoplastic syndrome associated with HCC and may also be caused by
end-stage liver failure?
A. Hypoglycemia
B. Erythrocytosis
C. Hypercalcemia
D. Hypercholesterolemia
Question 10.19 An increased risk of developing HCC is associated with
which of the following?
A. Wilson disease
B. Porphyria cutanea tarda
C. Primary biliary cirrhosis
D. All of the above
Question 10.20 Which of the following statements about screening and
prevention is CORRECT?
A. The advent of vaccination for hepatitis B is unlikely to reduce HCC in
endemic areas.
B. A combination of AFP and ultrasound screening is used in high-risk
populations.
C. Aggressive screening programs for HCC have not been shown to
improve survival.
D. Detection of HCC, through surveillance of patients awaiting liver
transplantation, does not increase priority for orthotopic liver
transplantation.
Question 10.21 Which of the following increases the risk of developing
cholangiocarcinoma?
A. Primary sclerosing cholangitis
B. Clonorchis sinensis infestation
C. Chronic portal bacteremia and portal phlebitis
D. All of the above
Question 10.22 A 66-year-old man is noted to have painless jaundice on a
routine follow-up at his primary care physician’s office. Workup reveals a
mass causing biliary obstruction at the hilum. ERCP confirms a high-grade
stricture predominantly involving the left hepatic duct; however, brushings
reveal atypical cells and no malignancy. He is seen at a tertiary care center
and offered surgical management, an en bloc resection of the left hepatic lobe
and extrahepatic bile duct, and a complete periportal lymphadenectomy.
Which of the following statements about management and natural history of
hilar cholangiocarcinoma is/are TRUE?
A. Surgical resection is associated with an operative mortality rate of
30%.
B. Recurrences occur most commonly at the bed of resection, followed by
retroperitoneal lymph nodes. Distant metastases occur in one-third of
cases.
C. Less than 10% of patients have resectable cancer at the time of
diagnosis.
D. All of the above.
Question 10.23 A 70-year-old man presents with 16-lb weight loss and
persistent right upper quadrant pain. CT scan reveals a gallbladder stone and
thickening of the anterior wall of the gallbladder. The patient is treated with
laparoscopic cholecystectomy, and the pathology reveals a moderately
differentiated 2-cm gallbladder adenocarcinoma invading the perimuscular
connective tissue. Margins of resection are negative for tumor. On the basis
of the available information, what is the stage of this cancer?
A. Stage IA
B. Stage IB
C. Stage II
D. Stage IIIA

https://t.me/ALGRAWANY33
Question 10.24 The patient in Question 10.22 recovers from surgery and
seeks a second opinion at a tertiary care center 5 weeks after his
cholecystectomy. A CT scan 2 weeks after surgery shows mild periportal
fullness. What is the most appropriate next step in management?
A. Perform en bloc resection of the gallbladder, resection of segments IVb
and V of the liver, and regional lymph node dissection.
B. No further therapy is warranted; surveillance with CT scans and
laboratories done every 3 months.
C. He requires a second laparotomy to assess the extent of remaining
disease to guide further therapy.
D. Perform ultrasound-guided biopsy of the periportal nodes; if positive,
then fluoropyrimidine-based chemoradiation is indicated.
Question 10.25 Which one of the following statements about adjuvant
therapy for biliary cancers is TRUE?
A. On the basis of randomized data, patients benefit from adjuvant
chemotherapy.
B. Adjuvant radiation is superior to chemotherapy alone.
C. Adjuvant chemotherapy only improves OS for patients with R1
resections.
D. Fluoropyrimidine-based chemoradiation is standard because it is
superior to radiation alone.
Question 10.26 Which of the following statements regarding fibrolamellar
HCC is CORRECT?
A. Fibrolamellar HCC occurs more frequently in men when compared
with women.
B. This variant of HCC is associated with viral hepatitis but not cirrhosis.
C. Lymph node metastasis at the time of presentation is common.
D. Most patients with fibrolamellar HCC present in their sixth decade of
life.
Question 10.27 A 50-year-old man with a history of chronic hepatitis B
infection and Child–Pugh A cirrhosis presents with abdominal pain. He is
otherwise in good health. CT of the abdomen reveals a cirrhotic liver with a
2.5-cm liver mass that rapidly enhances during the arterial phase of contrast
administration and “washout” during the later venous phases. There is no
involvement of the portal vein. AFP is 300 ng/mL. What is the next best step
in his management?
A. Biopsy to obtain a histologic diagnosis.
B. Refer him to a hepatobiliary surgeon.
C. Start him on sorafenib.
D. Refer him to a radiation oncologist.
Question 10.28 Which of the following statements is CORRECT regarding
hepatoblastoma?
A. This is the most common primary cancer of the liver in adults.
B. Hepatoblastoma is a chemoresistant tumor.
C. Patients with this tumor have a poor outcome after liver
transplantation, with a 5-year survival rate of 20%.
D. The peak incidence of hepatoblastoma is within the first 2 years of life.
Question 10.29 Which of the following statements is CORRECT?
A. The hepatitis B X gene product has no known role in causing HCC.
B. NS5A protein product of hepatitis C virus (HCV) genome inactivates
p53.
C. The level of HBV replication is inversely related to the risk of liver
cancer.
D. HBV genotype C is associated with decreased risk of HCC.
Question 10.30 A 45-year-old man with a history of alcoholic cirrhosis
was found by screening ultrasound to have two new lesions in hepatic
segments 7 and 3, measuring 3 cm and 2 cm, respectively. MRI findings were
consistent with HCC. He has been abstinent from alcohol use for the past 2
years. His total bilirubin is 1.5 mg/dL, serum albumin 3 g/dL, and
international normalized ratio (INR) is 1.6. He does not have ascites or
encephalopathy. There is no evidence of metastatic disease, and he is referred
for orthotopic liver transplantation evaluation. He is deemed to be an

https://t.me/ALGRAWANY33
acceptable candidate. While waiting for a donor organ, what should be done
next?
A. Refer him to interventional radiology for ablative therapy.
B. Sorafenib to prevent disease progression while waiting for donor
organ.
C. Cisplatin, interferon alpha (IFNα)-2b, doxorubicin, and 5FU as
neoadjuvant therapy.
D. Observation.
Question 10.31 A 45-year-old woman presents with painless jaundice and
weight loss. The total bilirubin is 17.6 mg/dL, alkaline phosphatase is 568
units/L, and alanine transaminase (ALT) is 138 units/L. Viral hepatitis
serology finding is negative. CT of the chest, abdomen, and pelvis with
contrast is performed and reveals an ill-defined, infiltrative, hypoattenuating
lesion at the hepatic hilum with marked intrahepatic biliary ductal dilatation.
MRI of the liver characterizes the lesion as compatible with
cholangiocarcinoma centered in the hepatic hilum extending to the
confluence of the left medial and lateral hepatic ducts, involving the cystic
duct, proximal extrahepatic duct, and abutting the undersurface of the portal
vein and right hepatic artery. ERCP is performed with common bile duct
stenting. Pathology reports atypical epithelial cells in the brushing.
Hepatobiliary surgery team is consulted and determines that her tumor is
unresectable and she is not a liver transplant candidate due to psychosocial
issues. What is the next step in management once hyperbilirubinemia
resolves?
A. FOLFOX
B. Gemcitabine and cisplatin
C. External beam radiation therapy with concurrent 5FU
D. Palliative care alone including routine stent exchange
Question 10.32 A 52-year-old man is admitted to the hospital due to new
onset of symptomatic ascites and jaundice. He is an alcoholic, and has no
history of medical care before this encounter. The patient is mildly
encephalopathic, with serum total bilirubin 5.6 mg/dL and INR of 2.1. He is
diagnosed with unresectable metastatic HCC and alcoholic cirrhosis after an
extensive workup. The patient’s family arrives, and would like to discuss
treatment options. Your recommendation would be:
A. Gemcitabine and oxaliplatin.
B. Sorafenib.
C. Sunitinib.
D. Hospice or supportive care alone.
Question 10.33 A 63-year-old man with hepatitis C is noted on imaging to
have multiple lesions in the liver, as well as a 2-cm right adrenal mass,
consistent with metastatic HCC. Serum total bilirubin is 1.3, INR is 1.2, and
albumin 3.6. He has no ascites or hepatic encephalopathy. Which following
statement is TRUE regarding systemic therapy?
A. Sorafenib is the only U.S. Food and Drug Administration (FDA)-
approved tyrosine kinase inhibitor for first-line therapy for
unresectable or metastatic HCC.
B. Lenvatinib demonstrates noninferior OS to sorafenib as first-line
therapy for unresectable or metastatic HCC.
C. Ramucirumab is approved as first-line therapy for unresectable or
metastatic HCC.
D. Cabozantinib is approved as first-line therapy for unresectable or
metastatic HCC.

Answers

Question 10.1 The correct answer is D.


Tobacco smoke exposure plays a significant role in the development of
pancreatic adenocarcinoma. It has been estimated that tobacco smoking
contributes to the development of 20% to 30% of pancreatic cancers.
The strongest associations between cigarette smoking and pancreatic
cancer have been observed when the smoking occurred within the
previous 10 years. Smoking cessation can reduce this risk, returning to

https://t.me/ALGRAWANY33
baseline risk level 20 years after quitting. Environmental tobacco smoke
contains the same toxins, irritants, and carcinogens, such as carbon
monoxide, nicotine, cyanide, ammonia, benzene, nitrosamines, vinyl
chloride, arsenic, and hydrocarbons, as primary cigarette smoke. Host
etiologic factors associated with an increased risk of pancreatic cancer
include a history of diabetes mellitus (DM), chronic cirrhosis,
pancreatitis, a high-fat and high-cholesterol diet, and prior
cholecystectomy. Although not all studies have supported a relationship
between DM and pancreatic cancer, a meta-analysis of 20 epidemiologic
studies confirms that the pooled relative risk of pancreatic cancer in
persons with DM for 5 years is double (relative risk, 2.0; 95%
confidence interval [CI], 1.2–3.2) the risk of persons without DM.
Interestingly, the risk of being diagnosed with pancreatic cancer is most
pronounced within the first 2 to 3 years of the diagnosis of diabetes.
This is likely due to diabetes being a paraneoplastic consequence of the
cancer. There is no association with a high-protein diet. People of
African-American descent experience a higher rate of pancreatic cancer
than do those of European ancestry in the United States, with an annual
incidence of 16.7 per 100,000 versus 10.9 per 100,000, respectively.
Death from pancreatic cancer is similarly elevated, with an annual rate
of 14.6 per 100,000 versus 10.6 per 100,000. The diagnosis is slightly,
but significantly, earlier in African-Americans compared with
Caucasians, with a median age of diagnosis of 68 and 73 years,
respectively. In developed countries, the incidence and mortality rates
range from 7 to 9 per 100,000 for men and 4.5 to 6 per 100,000 for
women.
Question 10.2 The correct answer is C.
Of patients with available data in the United States who were diagnosed
with pancreatic cancer from 1996 to 2002, 8% presented with local
disease, 31% presented with regional disease, and 61% had distant
metastases. Therefore, the majority of patients are metastatic at
diagnosis. Five-year survival from all stages of disease is 8%. Pancreatic
cancer tends to occur later in life. Only 10% of patients in Europe
present before the age of 50 years, whereas those aged 50 to 54 years,
experience an incidence of 9.8 per 100,000, and those aged 70 to 74
years, experience an incidence of 57 per 100,000. The median age of
diagnosis with pancreatic cancer in the United States is 70 years.
Pancreatic ductal adenocarcinoma arises from ductal epithelial cells.
Neoplasia arising from these cells progresses from initial PanINs to
invasive carcinomas. The evidence that these lesions are precancerous
includes the observation that after segmental resection of pancreas with
PanINs in the resected tissue, pancreatic cancer can develop in the
pancreatic remnant. The strongest evidence that PanINs are
precancerous lesions is the stepwise progression of genetic mutational
events that correlates with the stepwise progression of worsening
PanINs culminating in adenocarcinoma. PanIN-1 lesions show a
predominance of KRAS mutations and overexpression of HER2/neu. In
PanIN-2 lesions, CDKN2Ap16 mutations are prevalent, and in PanIN-3
lesions, TP53, SMAD4, and BRCA2 mutations predominate. The
mutational pattern of PanIN-3 lesions is equivalent to mutations found
in pancreatic adenocarcinoma. These events are the basis of a current
progression model for pancreatic cancer in which point mutation of
KRAS and overexpression of HER2/neu are initiating early events, loss
of p16 is an intermediate event, and p53, SMAD4, BRCA2 inactivation
follows just before invasion outside of the duct. Approximately 95% of
all cases have KRAS mutation, 90% have CDKN2A mutations, 75% have
TP53 mutations, and 55% have SMAD4 mutations. It is estimated that
10% to 15% of pancreatic cancers are hereditary or have a familial link.
CA 19-9 is a clinically useful tumor marker. The epitope of this
antibody is a sialylated lacto-N-fucopentaose II related to the Lewis-a
blood group antigen. Limitations of CA 19-9 are that it is not specific
for pancreatic cancer, and has been found to be elevated in other tumor
types, such as in biliary tract, colon, and stomach cancers.
Question 10.3 The correct answer is C.
The patient has a stage III, clinical T4N1M0 pancreatic cancer.
According to the eighth edition of the American Joint Committee on
Cancer (AJCC) staging system, this represents unresectable disease. The
T-stage designation classifies T1 to T3 tumors as potentially resectable

https://t.me/ALGRAWANY33
and T4 tumors as locally advanced (unresectable). Tumors with any
involvement of the superior mesenteric, celiac, or common hepatic
arteries are classified as T4. However, tumors that involve the superior
mesenteric, splenic, or portal veins are classified as T3 because these
veins can be resected and reconstructed, provided that they are patent.
Question 10.4 The correct answer is A.
Laparoscopy and multiphase CT have evolved concurrently as methods
to evaluate a pancreatic mass. Both have emerged as highly effective in
evaluating the tumors, but CT as a noninvasive modality supplants the
use of routine laparoscopy. Currently, routine use of laparoscopy is not
warranted unless assessing for suspected peritoneal involvement in a
potentially resectable patient. The cornerstone of diagnostic evaluation
of a pancreatic tumor is the multiphase CT scan, coordinating
intravenous contrast administration with subsequent rapid thin-cut CT
through the pancreas during arterial, portal venous, and parenchymal
phases of enhancement. With this type of CT, extension of the tumor to
the superior mesenteric artery, celiac axis, superior mesenteric
vein/portal vein complex, and contiguous structures can be clearly
determined, as well as an assessment of distant metastasis. Optimally,
CT imaging should precede stent placement and biopsy because of the
possibility of postprocedure inflammation from the biopsy and artifact
from the stent that can confound interpretation of the images. High-
quality MRI may also be used to assess pancreatic cancer, although it
may lack the resolution and vascular detail provided by a triphasic
pancreas protocol CT. Endoscopic ultrasound can image the primary
tumor even when the mass cannot be visualized on CT scan, and is the
best means of obtaining a fine-needle aspiration or fine-needle biopsy of
pancreatic adenocarcinoma. Although preoperative ultrasonography is
useful in assessing tumor characteristics and resectability of pancreatic
adenocarcinoma, it is particularly operator dependent. However, the
procedure cannot provide complete regional and distant staging
information.
Question 10.5 The correct answer is A.
The RTOG performed a prospective randomized trial (RTOG 9704)
comparing gemcitabine with infusional 5FU as the systemic component
of therapy, with all patients also receiving 5FU-based chemoradiation.1
There was no survival difference between patients randomized to
gemcitabine and those who received infusional 5FU. However, among
the 380 patients with resected pancreatic head lesions, survival was
superior for patients randomized to gemcitabine compared with those
who received infusional 5FU (20.6 vs. 15.9 months; hazard ratio [HR]
for death 0.79; 95% CI, 0.63–0.99; p = 0.033). In the CONKO-001 trial,
patients randomized to receive gemcitabine had a median disease-free
survival of 13.9 months (95% CI, 11.4–15.3), whereas those patients
who underwent surgery alone had a median disease-free survival of only
6.9 months (95% CI, 6.1–7.8; p = 0.001).2 However, there was no
statistically significant difference in OS when the initial results of the
study were reported in 2007. The long-term survival outcomes of this
trial were reported in 2013, and the median OS of 22.8 months was seen
in the gemcitabine group, compared with 20.2 months in the observation
group (HR, 0.76; p = 0.01). In addition, 10-year survival rate was
superior in the gemcitabine group (12.2% vs. 7.7%), compared with the
observation group. In the GITSG 9173 trial, 5FU-based chemoradiation
followed by up to 2 years of weekly bolus 5FU was evaluated against
observation alone. A preliminary analysis of survival was reported after
only 43 patients had completed treatment and showed a striking survival
advantage for patients receiving combined modality therapy compared
with survival of patients who underwent surgery alone (median 21.0 vs.
10.9 months, respectively; one-tailed p = 0.03). In ESPAC-1, patients
receiving chemoradiation did worse (median survival of 15.9 months;
HR for death 1.28; 95% CI, 0.99–1.66) than did those not receiving
chemoradiation (median survival of 17.9 months; p = 0.05).3
Conversely, patients who received chemotherapy had a median survival
of 20.6 months (HR for death 0.71; 95% CI, 0.55–0.92) versus 15.5
months for those patients who did not receive chemotherapy, a
statistically significant result (p = 0.009).
Question 10.6 The correct answer is C.

https://t.me/ALGRAWANY33
CONKO-001 established gemcitabine as the adjuvant regimen of choice
due to significantly improved disease-free survival of 7 months
compared with those who only underwent surgical resection.
Unfortunately, OS was not significantly improved.2 The ESPAC-4
regimen of gemcitabine and capecitabine given over 24 weeks versus
gemcitabine alone over 24 weeks showed, for the first time, statistically
significant improved OS of the combination compared with the control
arm (median 28.0 [95% CI 23.5–31.5] vs. 25.5 months [95% CI 22.7–
27.9], HR 0.82 [95% CI 0.68–0.98, p = 0.02]) when initiated within 12
weeks of surgery.4 The subsequent PRODIGE 24-ACCORD trial
showed that, compared with adjuvant gemcitabine, modified (m)
FOLFIRINOX was superior, with a median disease-free survival of 21.6
months compared with just 12.8 months in the monotherapy control arm
(HR 0.58; 95% CI 0.46–0.73; p < 0.001).5 This translated to 39.7% of
patients who received mFOLFIRINOX alive and disease-free at 3 years,
compared with 21.4% in the gemcitabine arm. The median OS was
strongly significant at 54.4 months versus 35.0 months (HR 0.64; 95%
CI 0.48–0.86; p = 0.003). This comes at a cost of significantly increased
grade 3/4 toxicity in 75.9% of patients receiving mFOLFIRINOX versus
52.9% of those who received gemcitabine. To mitigate this and owing to
frequent, severe diarrhea detected early in the course of the trial, the
irinotecan dose was modified from 180 to 150 mg/m2. The trial, in
addition, only enrolled patients with Eastern Cooperative Oncology
Group (ECOG) performance status 0 to 1 and at a maximum age of 75
years. The patient, in this case, meets these criteria and should be
considered for the mFOLFIRINOX regimen, although it must be noted
that gemcitabine plus capecitabine is also reasonable and the two
regimens have never been trialed together. Surveillance to monitor for
disease recurrence would be appropriate only if the patient had ongoing
severe complications of healing from the Whipple resection or other
contraindications lasting beyond 12 weeks from the date of surgery to
make any adjuvant therapy unsafe.
Question 10.7 The correct answer is A.
Malabsorption is a frequent complication seen in patients after Whipple
surgery because of pancreatic enzyme insufficiency. Most patients
require adjustment of the pancreatic enzyme supplement for adequate
control of this symptomatology. In this case, although mFOLFIRINOX
has significant associations with diarrhea, the description of steatorrhea
makes malabsorption from insufficiency the most likely cause. Although
small bowel obstruction and metastatic disease can be seen, the clinical
picture does not fit with these diagnoses. The aim during adjuvant
therapy is to administer the chemotherapy with minimal delays to obtain
the full benefit of therapy.
Question 10.8 The correct answer is B.
The optimal treatment of locally advanced pancreatic cancer remains
controversial. Locally advanced disease is generally incurable, with an
average survival of 7 to 15 months, and all therapies have significant
limitations. Systemic therapy for 2 to 4 months followed by 5FU-based
or capecitabine-based chemoradiation is a reasonable approach for some
patients with locally advanced disease. However, concurrent CRT
approaches have not impacted the survival, compared with patients with
chemotherapy alone. If patients are responding to chemotherapy with
objective radiographic response or CA 19-9 level decline after 2 months
and tolerating therapy well, it is reasonable to continue for 2 more
months.
Question 10.9 The correct answer is C.
The combination of nab-paclitaxel and gemcitabine is the only
gemcitabine combination that showed clinically relevant OS benefit in
patients with advanced pancreatic cancer by a large randomized study.
The OS was superior in patients who received the combination regimen
(8.5 vs. 6.7 months; HR 0.72; p < 0.001), compared with the
gemcitabine-only group. Although erlotinib combined with gemcitabine
also demonstrates statistically improved survival in a phase III trial
(median 6.24 vs. 5.91 months), the combination only improves survival
by 2 weeks, and thus has not been widely clinically adopted.
Question 10.10 The correct answer is B.

https://t.me/ALGRAWANY33
Results from the phase III randomized trial reported significant
improvement in OS for patients receiving FOLFIRINOX compared with
single-agent gemcitabine alone (11.6 vs. 6.8 months; HR 0.57; p <
0.0001).6 Therefore, FOLFIRINOX should be considered for first-line
treatment in younger patients with good performance status and normal
serum bilirubin level. FOLFOX has shown activity in patients with
pancreatic cancer in the second-line setting, but further evaluation of
clinical benefit in the frontline has not been validated.
Question 10.11 The correct answer is A.
BRCA2 is the most commonly mutated tumor suppressor gene in
familial pancreatic cancer followed by PALB2 and ATM. KRAS is an
oncogene, and not a tumor suppressor gene. TP53 gene mutation is
found in more than 70% of all pancreatic cancers, but there is no
evidence to link this mutation with familial pancreatic cancer.
Question 10.12 The correct answer is A.
Non-O blood type has been long known to be associated with increased
risk for lung cancer. A genome-wide association study identified high-
risk single-nucleotide polymorphism for pancreatic cancer in the gene
encoding for the ABO blood type. Patients with HNPCC syndrome or
Lynch syndrome are at higher risk for gastrointestinal malignancies
including pancreatic cancer. KRAS mutation testing has so far not
proven to be effective in differentiating between pancreatic cancer and
other conditions such as pancreatitis and pancreatic adenoma. Hereditary
pancreatitis increases the risk of developing pancreatic cancer by 40%,
and is associated with germline mutations in PRSS1 or SPINK1.
Question 10.13 The correct answer is D.
Data on second-line treatment of patients with pancreatic cancer who
failed first-line gemcitabine have been limited, but in 2016, 5FU with
nanoliposomal irinotecan (nal-Iri) showed significant survival benefit in
the phase III NAPOLI-1 randomized clinical trial when compared with
5FU alone.7 In patients who had previously progressed on a
gemcitabine-based regimen, 5FU/nal-Iri demonstrated a 6.1-month
(95% CI 4.8–8.9) median OS compared with 4.2 months (95% CI 3.3–
5.3; HR 0·67, 95% CI 0.49–0.92; p = 0·012) in the 5FU population. On
the basis of the CONKO-003 study, patients with pancreatic cancer
following progression on first-line gemcitabine had better OS with OFF
(oxaliplatin plus fluorouracil/folinic acid [FF]) compared to FF (5.9 vs.
3.3 months, HR, 0.66; p = 0.01).8 Hence, an OFF chemotherapy regimen
would be an appropriate second-line option in patients who progressed
on the first-line gemcitabine therapy, although it is less preferred than
the irinotecan-containing regimens due to the inferior OS outcome seen
with FOLFOX in the PANCREOX trial. Single-agent erlotinib has not
shown significant clinical benefit for patients with metastatic pancreatic
cancer. Supportive care alone is a reasonable choice in those with
progressive metastatic disease in the absence of evidenced-based
treatment options or patients unwilling or unable to tolerate more
antineoplastic therapy.
Question 10.14 The correct answer is D.
Telomere shortening is thought to predispose to chromosome fusion
(translocation) and missegregation of genetic material during the
mitosis, and later during tumor genesis, telomerase is reactivated.
SMAD4 (DPC4) pathway mediates signals initiated on the binding of
the extracellular proteins TGF-β. The underexpression of TGF-β
receptors results in cellular resistance to the usual suppressive effects of
the TGF-β ligand. The cyclin D oncogene complexes with cyclin-
dependent kinases (CDKs) and phosphorylates the retinoblastoma
protein (tumor suppressor protein) that results in loss of negative
regulatory effect of retinoblastoma protein.
Question 10.15 The correct answer is D.
There is no treatment demonstrating a consistent survival advantage to
support adjuvant therapy for HCC, likely a reflection of the cytostatic
effects of tyrosine kinase inhibitors and lack of active cytotoxic agents.
The phase III STORM trial of adjuvant sorafenib compared with
placebo failed to demonstrate a significant difference in the primary
endpoint of recurrence-free survival or the secondary endpoints of time

https://t.me/ALGRAWANY33
to relapse and OS.9
Question 10.16 The correct answer is D.
The Milan criteria for selection of patients appropriate for liver
transplantation include patients with solitary tumors ≤5 cm, patients with
multifocal disease with less than or equal to three tumor nodules each ≤3
cm in size, patients with Child–Pugh B or C cirrhosis, patients who are
not candidates for primary liver resection, and patients who have tumors
with no evidence of macrovascular invasion.
Question 10.17 The correct answer is D.
Multiple clinical staging systems for hepatic tumors have been
described. The most widely used is the AJCC/TNM. A staging system
based entirely on clinical grounds that incorporate the contribution of
the underlying liver disease was originally developed by Okuda. The
American Association for the Study of Liver Diseases has endorsed the
use of the Barcelona Clinic Liver Cancer system for staging of HCC.
This has now been validated both internally and externally in several
studies.
Question 10.18 The correct answer is A.
Several paraneoplastic syndromes have been described to occur in
patients with hepatocellular cancer. Most of these are biochemical
abnormalities without associated clinical consequences. The most
important ones include hypoglycemia (also caused by end-stage liver
failure); erythrocytosis; hypercalcemia; hypercholesterolemia;
dysfibrinogenemia; carcinoid syndrome; increased thyroxin-binding
globulin; sexual changes such as gynecomastia, testicular atrophy, and
precocious puberty; and porphyria cutanea tarda. The pathogenesis of
hypoglycemia is unclear, but may be related to production of insulin-like
growth factor I by the tumor.
Question 10.19 The correct answer is D.
Chronic viral hepatitis due to both hepatitis B and C account for 75% of
HCC cases worldwide. Primary biliary cirrhosis is associated with an
increased risk of cholangiocarcinoma as well as HCC. Conditions
causing cirrhosis such as Wilson disease, chronic alcohol abuse,
autoimmune hepatitis, and cryptogenic cirrhosis are risk factors for
HCC. Hereditary hemochromatosis and alpha-1-antitrypsin deficiency
also place a patient at high risk. Noncirrhotic conditions such as
hereditary tyrosinemia, porphyria cutanea tarda, galactosemia,
hereditary hemorrhagic telangiectasia, and cigarette smoking are also
associated with HCC.
Question 10.20 The correct answer is B.
A combination of ultrasonography and serum AFP levels is used for the
screening of high-risk populations for HCC. Screening programs are in
place in endemic areas of HCC and in high-risk populations, and recent
data from the Surveillance, Epidemiology, and End Results (SEER)
database show that HCC screening is associated with increased survival
and detection of HCC tumors at earlier stages. Detection of HCC,
through surveillance of patients awaiting liver transplantation, results in
increased priority for orthotopic liver transplantation.
Question 10.21 The correct answer is D.
In most patients, cholangiocarcinoma is sporadic, and no precipitating
factor can be identified. Risk factors that have been associated with the
development of cholangiocarcinoma can be divided into congenital
(choledochal cysts, anomalous pancreatic-biliary tree junction),
autoimmune (primary biliary cirrhosis), infectious (C. sinensis and
Opisthorchis viverrini infestation, chronic portal bacteremia, and portal
phlebitis), and environmental exposures (the radiocontrast agent
thorotrast and, possibly, cigarette smoking).
Question 10.22 The correct answer is B.
Approximately one-third of patients presenting with the suspected
diagnosis of cholangiocarcinoma will have resectable disease. Operative
mortality averages approximately 8%, indicating the high-risk
population that this tumor affects and the complexity of the procedure.
Approximately 10% to 35% of patients survive 5 years after surgical

https://t.me/ALGRAWANY33
resection. Recurrences occur most commonly at the bed of resection,
followed by retroperitoneal lymph nodes. Distant metastases occur in
one-third of cases. The most common sites of metastases are the lung
and mediastinum, followed by the liver and peritoneum. Comparisons of
outcome over time suggest improved outcome in more recent series as a
result of routine inclusion of liver resection. Prognostic factors for
survival include negative microscopic margin status, lymph node
metastases, tumor size, tumor grade, preoperative serum albumin,
hepatic resection, and postoperative sepsis.
Question 10.23 The correct answer is C.
The pathologic stage is T2 NX (stage II) gallbladder cancer. Tumor
invading the perimuscular connective tissue but not extending through
the serosa is classified as T2, according to the eighth edition of the
AJCC Cancer Staging Manual.
Question 10.24 The correct answer is A.
Numerous studies have demonstrated that simple cholecystectomy is
curative for stage I disease (T1, N0). Recent studies have suggested that
the prognosis is different for pT1a and pT1b tumors after simple
cholecystectomy. Invasion of the muscular layer allows access to
lymphatics and vessels, providing the rationale for extended
cholecystectomy in this population. When an extended cholecystectomy
is performed for T2 disease, the 5-year survival has been reported to be
as high as 100%, but probably falls in the range of 70% to 90%. Simple
cholecystectomy alone is associated with a 5-year survival rate of 20%
to 40.5%. Lymph node metastases are seen in 46% of patients with T2
primary tumors, providing another reason in favor of radical repeat
resection after simple cholecystectomy. In series of extended
cholecystectomies, the operative morbidity ranges from 5% to 46%, and
the mortality ranges from 0% to 21%. The risk of resection for each
patient and each type of resection needs to be weighed against the
chance of benefiting from the procedure on the basis of the tumor stage.
Question 10.25 The correct answer is A.
Randomized phase III data from the BILCAP study shows that adjuvant
capecitabine improves OS compared with observation following
macroscopic complete resection of muscle-invasive gallbladder cancer
or cholangiocarcinoma.10 In a small study including 14 patients with
gallbladder cancer treated at the University of Michigan with resection
followed by radiotherapy or CRT, the median radiation dose was 54 Gy,
and approximately half the patients received concurrent chemotherapy.
The median survival was 23 months. Interestingly, there was no
difference in survival between patients with R0 or R1 resection. No
differences were observed in survival or pattern of failure between
patients with gallbladder cancer and bile duct cancer (distal or hilar).
The high risk of systemic spread and locoregional failure associated
with gallbladder cancer that extends beyond the mucosa has led most
cancer centers in the United States to recommend consideration of
adjuvant chemotherapy and radiotherapy. For external beam radiation
therapy, the target volume should include the gallbladder fossa and
adjacent liver, as well as the regional nodal areas.
Question 10.26 The correct answer is C.
Fibrolamellar HCC is a rare histologic variant of HCC. Most patients
present in the third decade of life, and it affects men and women equally.
This variant of HCC is uncommonly associated with prior cirrhosis or
viral hepatitis. In addition, a higher proportion of patients with
fibrolamellar HCC have lymph node metastases at presentation than do
those with the usual HCC.
Question 10.27 The correct answer is B.
This man has a hepatic mass within a cirrhotic liver, which is very
suspicious for HCC. The radiologic features are also highly suggestive
of HCC. If the mass is larger than 2 cm in size with characteristic
appearance of HCC on imaging and AFP is higher than 200 ng/mL, a
biopsy is not essential for management. Patients with a high clinical
suspicion for HCC who are deemed appropriate surgical candidates
should be taken to surgery without a preoperative biopsy.

https://t.me/ALGRAWANY33
Question 10.28 The correct answer is D.
Hepatoblastoma is the most common primary cancer of the liver in
children. The peak incidence is within the first 2 years of life. Surgical
resection is the first line of therapy. However, these tumors are very
chemosensitive, and neoadjuvant chemotherapy can render unresectable
tumors operable. Five-year survival after liver transplantation is
excellent (83%).
Question 10.29 The correct answer is B.
The NS5A protein product of the HCV genome has been demonstrated
to inactivate p53 by sequestration. The hepatitis B x gene product has
been implicated in causing HCC because it is a transcriptional activator
of various cellular genes associated with growth control. The elevated
serum level of HBV DNA, a marker of higher levels of HBV
replication, is associated with a higher risk of HCC. The HBV genotype
C is generally thought to increase the risk of HCC because these
individuals are likely to remain seropositive for hepatitis B e antigen and
thus have higher serum levels of HBV DNA for a longer time.
Question 10.30 The correct answer is A.
This patient is an ideal candidate for locoregional therapy while waiting
for liver transplantation. Due to donor organ shortages, patients may
wait for months before surgery. Transarterial chemoembolization
(TACE) has been shown to reduce dropout from the wait list, reduce
posttransplant recurrence, and occasionally downstage tumors that are
initially beyond transplant criteria. Radiofrequency ablation or TACE
may be used for definitive therapy in patients who are not candidates for
surgical resection or transplantation. Contraindications to TACE include
portal vein thrombosis and poorly compensated cirrhosis. Ascites is a
relative contraindication because it indicates elevated portal vein
pressure, portending a higher risk of worsening liver function with
TACE. Sorafenib would not be a good choice for a patient on the wait
list for surgery because of its vascular endothelial growth factor (VEGF)
inhibition. Neoadjuvant chemotherapy with PIAF (cisplatin, IFNα-2b,
doxorubicin, and 5FU) has been shown in a phase II trial in patients with
advanced HCC to have a 26% response rate, and some patients who
achieved a partial response went on to have surgery. This patient is
already a candidate for surgery, and the toxicity of this regimen may
complicate plans for transplantation. Observation alone would not be
appropriate given the wait time for transplantation and the available
therapies for locoregional control including surgery and intravascular
procedures.
Question 10.31 The correct answer is B.
The UK-ABC-02 trial was a large phase III randomized control trial
conducted after the phase II portion demonstrated an improvement in
progression-free survival (PFS) in patients with locally advanced or
metastatic biliary tract cancers treated with either gemcitabine and
cisplatin or gemcitabine alone.11 The trial redemonstrated the PFS
advantage (8 vs. 5 months, p < 0.001) and met its primary endpoint of
OS with a median OS of 11.7 months in the gemcitabine and cisplatin
arm and 8.1 months in the gemcitabine-alone arm (HR, 0.64; 95% CI,
0.52–0.8; p < 0.001). This combination was tolerated without
substantially increased toxicity. There is no high-quality evidence for
second-line therapy, and, in fact, only 17% of patients in the UK-ABC-
02 trial went on to receive second-line therapy. 5FU-containing
regimens as second-line therapy are supported by a small series of
patients who received FOLFOX4 (4 of 18 patients responded), but high-
quality evidence is lacking to guide second-line and beyond therapy, and
clinical trial enrollment is encouraged. There is mixed literature for
radiation therapy used in the palliative setting for cholangiocarcinoma.
The risk of toxicity is substantial, but a small phase II trial suggested the
median OS may be superior to chemotherapy with stereotactic body
radiotherapy. Larger comparative trials are needed. Given the survival
benefit with gemcitabine and cisplatin, palliative care alone would not
be appropriate unless requested by the patient or the patient is medically
unfit.
Question 10.32 The correct answer is D.

https://t.me/ALGRAWANY33
This patient has Child–Pugh C cirrhosis at presentation and unresectable
HCC. He would not have qualified for enrollment in the SHARP study
with sorafenib because only patients with Child–Pugh A were enrolled.
In clinical practice, patients with Child–Pugh B cirrhosis are often
offered sorafenib therapy if no viable alternatives such as clinical trial
enrollment are available. However, survival is markedly shorter than
that in patients with preserved hepatic function, and there is no evidence
that sorafenib has a meaningful impact on survival in this population. A
phase III trial compared sunitinib with sorafenib in patients with Child–
Pugh A cirrhosis and was stopped early due to poorer survival in the
sunitinib arm. This patient should be offered hospice services or
supportive medical care given his poor prognosis and lack of evidence-
based treatment options.
Question 10.33 The correct answer is B.
The REFLECT phase III noninferiority study randomized patients with
Child–Pugh A, unresectable HCC to first-line sorafenib or lenvatinib
with OS as the primary endpoint.12 The median OS for lenvatinib and
sorafenib was 13.6 and 12.3 months, respectively, demonstrating
noninferior OS with lenvatinib, and leading to FDA approval for this
indication in 2018. Sorafenib was FDA approved in 2007 for
unresectable HCC based on the phase III SHARP study. Ramucirumab
and cabozantinib were both FDA approved in 2019 as second-line
therapies for unresectable or metastatic HCC.

References
1. Regine WF, Winter KA, Abrams R, et al. Fluorouracil-based chemoradiation with either
gemcitabine or fluorouracil chemotherapy after resection of pancreatic adenocarcinoma: 5-year
analysis of the U.S. Intergroup/RTOG 9704 phase III trial. Ann Surg Oncol. 2011;18:1319–1326.
2. Oettle H, Neuhaus P, Hochhaus A, et al. Adjuvant chemotherapy with gemcitabine and long-term
outcomes among patients with resected pancreatic cancer: the CONKO-001 randomized trial.
JAMA. 2013;310:1473–1481.
3. Neoptolemos JP, Stocken DD, Friess H, et al. A randomized trial of chemoradiotherapy and
chemotherapy after resection of pancreatic cancer. N Engl J Med. 2004;350:1200–1210.
4. Neoptolemos JP, Palmer DH, Ghaneh P, et al. Comparison of adjuvant gemcitabine and
capecitabine with gemcitabine monotherapy in patients with resected pancreatic cancer (ESPAC-4):
a multicentre, open-label, randomised, phase 3 trial. Lancet. 2017;389:1011–1024.
Conroy T, Hammel P, Hebbar M, et al. FOLFIRINOX or gemcitabine as adjuvant therapy for
5.
pancreatic cancer. N Engl J Med. 2018;379:2395–2406.
6. Conroy T, Desseigne F, Ychou M, et al. FOLFIRINOX versus gemcitabine for metastatic
pancreatic cancer. N Engl J Med. 2011;364:1817–1825.
7. Wang-Gillam A, Hubner RA, Siveke JT, et al. NAPOLI-1 phase 3 study of liposomal irinotecan in
metastatic pancreatic cancer: final overall survival analysis and characteristics of long-term
survivors. Eur J Cancer. 2019;108:78–87.
8. Oettle H, Riess H, Stieler JM, et al. Second-line oxaliplatin, folinic acid, and fluorouracil versus
folinic acid and fluorouracil alone for gemcitabine-refractory pancreatic cancer: outcomes from the
CONKO-003 trial. J Clin Oncol. 2014;32:2423–2429.
9. Bruix J, Takayama T, Mazzaferro V, et al. Adjuvant sorafenib for hepatocellular carcinoma after
resection or ablation (STORM): a phase 3, randomised, double-blind, placebo-controlled trial.
Lancet Oncol. 2015;16:1344–1354.
10. Primrose JN, Fox RP, Palmer DH, et al. Capecitabine compared with observation in resected biliary
tract cancer (BILCAP): a randomised, controlled, multicentre, phase 3 study. Lancet Oncol.
2019;20:663–673.
11. Valle J, Wasan H, Palmer DH, et al. Cisplatin plus gemcitabine versus gemcitabine for biliary tract
cancer. N Engl J Med. 2010;362:1273–1281.
12. Kudo M, Finn RS, Qin S, et al. Lenvatinib versus sorafenib in first-line treatment of patients with
unresectable hepatocellular carcinoma: a randomised phase 3 non-inferiority trial. Lancet.
2018;391:1163–1173.
___________
Corresponding chapters in DeVita, Hellman, and Rosenberg’s Cancer: Principles & Practice of
Oncology, Eleventh Edition: 54 (Molecular Biology of Pancreas Cancer), 55 (Cancer of the Pancreas),
56 (Molecular Biology of Liver Cancer), 57 (Cancer of the Liver), and 58 (Cancer of the Biliary Tree).

https://t.me/ALGRAWANY33
11 Colorectal and Anal Cancer
Ashley E. Morton and Benjamin R. Tan, Jr.

QUESTIONS

Each of the numbered items below is followed by lettered answers. Select the
ONE lettered answer that is BEST in each case unless instructed otherwise.

Question 11.1 A 30-year-old woman presents with a large abdominal mass


associated with abdominal pain. She underwent a total proctocolectomy for
colon cancer secondary to familial adenomatous polyposis (FAP) 4 years ago
followed by adjuvant chemotherapy for node-positive disease. A recent
esophagogastric endoscopy revealed only a tubulovillous adenoma in the
duodenum, but the computed tomography (CT) scan revealed a 10-cm mass
filling the pelvis. Biopsy showed a desmoid tumor. A TRUE statement
regarding desmoid tumors is:
A. Desmoid tumors represent the second most common cause of death for
patients with FAP.
B. Desmoid tumors are uniformly aggressive and locally invasive.
C. Adjuvant radiation is recommended.
D. Adjuvant chemotherapy is recommended.
Question 11.2 The tumor of the patient in Question 11.1 was deemed
unresectable, and she is now complaining of increased pain. You would
recommend:
A. Doxorubicin with ifosfamide chemotherapy.
B. Immunotherapy with nivolumab.
C. Targeted therapy with sorafenib.
D. Palliative and hospice care.
Question 11.3 A 55-year-old woman underwent a right hemicolectomy for
a cecal mass, 15 years ago. Four years later, she was found to have a hepatic
flexure adenocarcinoma, and a total colectomy was performed. Four years
ago, she presented with postmenopausal bleeding, and was found to have
endometrial adenocarcinoma. She also had multiple skin malignancies,
including sebaceous adenomas and keratoacanthomas. One year ago, she
underwent a Whipple procedure for a duodenal adenocarcinoma invading
into the pancreas. She has three siblings and a paternal uncle with colorectal
cancer all diagnosed in their late 30s. What is the most probable primary
genetic explanation for her inherited colorectal cancer syndrome?
A. Chromosomal instability characterized by the deletion or mutation of a
tumor suppressor gene
B. Chromosomal instability characterized by activation of an oncogene
C. Microsatellite instability (MSI) caused by germline mutations in a
DNA mismatch repair (MMR) gene
D. MSI caused by epigenetic changes associated with hypermethylation in
CpG islands
Question 11.4 Other than genetic counseling, which test would you order
to confirm the diagnosis in the patient in Question 11.3?
A. TP53 mutation and loss of heterozygosity of chromosome 18q
B. APC mutation, including the I1307K allele
C. MSI testing and methylation of CpG islands
D. MSI test and MMR gene mutation including MLH1, MSH2, MSH6, and
PMS2
Question 11.5 The patient in Question 11.3 had a positive test result. What
screening tests and surveillance program would you recommend to her three
daughters aged 24, 22, and 20 years?
A. Colonoscopy now and repeat every 1–2 years and transvaginal
ultrasound (US) for her daughters starting at age 30–35 years
B. Colonoscopy for all and transvaginal US for her daughters starting at
age 30–35 years

https://t.me/ALGRAWANY33
C. Colonoscopy now and repeat every 1–2 years with consideration of
endometrial biopsy every 1–2 years only for her children confirmed to
have the same genetic mutation as the patient; colonoscopy at age 40–
50 years for those with no mutation
D. Colonoscopy and endometrial biopsy at age 30–35 years only for those
confirmed with the same mutation as the patient; colonoscopy at age
40–50 years for those with no mutation
Question 11.6 Which gene is associated with hereditary nonpolyposis
colorectal cancer (HNPCC)?
A. APC
B. MYH
C. STK11
D. MSH2
Question 11.7 A 65-year-old woman presented with intermittent
constipation and diarrhea associated with abdominal cramping. Colonoscopy
revealed a near-obstructing mass and biopsy showed a villoglandular polyp.
She underwent an exploratory laparotomy and resection of a 5 × 5 cm
circumferential necrotic and fungating mass. Pathology revealed a
moderately differentiated adenocarcinoma invading into the muscle (T3) with
4 (N2a) of 30 positive lymph nodes. Margins were negative. CT scan showed
no evidence of metastatic disease. After recovery from her surgery, based on
current evidence, you would recommend:
A. 5-Fluorouracil (5FU) with leucovorin × 6 months
B. Capecitabine with oxaliplatin (CAPOX) × 3 months
C. Irinotecan with 5FU (FOLFIRI ; 5-FU, leucovorin, irinotecan]) × 6
months
D. Oxaliplatin with 5FU (FOLFOX [5FU + leucovorin + oxaliplatin]) × 6
months
E. Oxaliplatin with 5FU (FOLFOX) and bevacizumab × 6 months
Question 11.8 One week after the first dose of chemotherapy, the patient in
Question 11.7 developed a fever of 39°C (102°F) associated with chills. She
also developed diarrhea, mucositis, confusion, and ataxia. Repeat complete
blood count showed a white blood cell count of 0.5 × 103 cells/μL with an
absolute neutrophil count of 100, hemoglobin of 11.7 g/dL, and platelet count
of 42,000. Which one of the following pharmacogenetic conditions would
best explain her clinical course?
A. The patient is homozygous for the thymidylate synthase (TYMS) *3/*3
polymorphism.
B. The patient is homozygous for UGT1A1*28 polymorphism.
C. The patient is heterozygous for the IVS14 + 1 G>A DPYD*2A
mutation.
D. The patient is heterozygous for the ERCC2 Lys751Gln polymorphism.
Question 11.9 A 62-year-old engineer was diagnosed with metastatic cecal
adenocarcinoma to the lung and liver, KRAS wild type. He was initially
treated with oxaliplatin plus infusional 5FU (FOLFOX6) and bevacizumab.
After four cycles (2 months), his CT scan showed progressive disease. You
discussed standard irinotecan-based second-line systemic chemotherapy. A
test for UGT1A1 polymorphism was done, and results revealed him to be
homozygous for UGT1A1*28/*28. Which of the following statements is
TRUE?
A. A patient that is homozygous for the UGT1A1*28 polymorphism
glucuronidate SN38 more efficiently than those with the wild-type *1
genotype and at greater risk for severe neutropenia with irinotecan.
B. A patient that is homozygous for the UGT1A1 *28 polymorphism
glucuronidate SN38 less efficiently than those with the wild-type *1
genotype and at greater risk for severe neutropenia with irinotecan.
C. A patient that is homozygous for UGT1A1*1 polymorphism are at
greater risk for severe neutropenia with irinotecan compared with those
with the UGT1A1 *28 genotype.
D. A patient that is homozygous for UGT1A1*1 or UGT1A1 *28
polymorphisms glucuronidate SN38 with comparable efficiency.
Question 11.10 A 52-year-old teacher presents with a 2-month history of
rectal bleeding. A rectal examination revealed a palpable nonobstructing

https://t.me/ALGRAWANY33
mass 5 cm from the anal verge. Subsequent colonoscopy confirmed a friable,
tethered mass with biopsy that showed moderately differentiated
adenocarcinoma. Transrectal US revealed a T3N1 cancer. CT scans of the
chest, abdomen, and pelvis revealed no metastatic sites. Which of the
following treatment strategies would you recommend next?
A. Total mesorectal excision (TME) alone
B. TME followed by chemotherapy
C. TME followed by chemoradiation
D. Neoadjuvant chemotherapy and radiation
Question 11.11 Which of the following appropriately staged patients with
rectal cancer would be the best candidate for transanal excision?
A. A 48-year-old woman with a 2.5-cm T1 moderately differentiated
rectal adenocarcinoma 4 cm from the anal verge
B. A 65-year-old man with a 4.5-cm T2 well-differentiated
circumferential rectal mass 6 cm from the anal verge
C. A 30-year-old man with a 2-cm well-differentiated T1 mucinous
adenocarcinoma 12 cm from the anal verge
D. A 52-year-old woman with a 3-cm well-differentiated T1N1
adenocarcinoma 5 cm from the anal verge
E. None of the above
Question 11.12 For the patient you have selected for transanal resection in
Question 11.11, pathologic review of the excised specimen showed no
lymphovascular invasion and all margins were negative. Which of the
following options would you recommend?
A. No further therapy with surveillance
B. Short-course (25 Gy/5 fractions) radiation
C. Intracavitary radiation
D. Adjuvant chemotherapy without radiation
E. Adjuvant chemotherapy with radiation
Question 11.13 A 49-year-old woman noted rectal bleeding for 2 months.
She denied any pain, diarrhea, constipation, or weight loss. A colonoscopy
was done that revealed a 2-cm low-lying mass 1 cm from the anal verge. A
transrectal US revealed no lymph nodes. Biopsy showed basaloid squamous
cell carcinoma. CT scans of the abdomen and pelvis did not reveal any
metastatic disease. The BEST curative treatment option for this patient is:
A. Abdominal perineal resection (APR).
B. Short-course (25 Gy/5 fractions) radiation.
C. Neoadjuvant chemoradiation with 5FU.
D. Chemoradiation with 5FU and mitomycin.
E. Nivolumab.
Question 11.14 Two months after treatment, a persistent lesion was noted
during examination. What would you recommend?
A. Proceed with surgery.
B. Proceed with stereotactic radiation.
C. Proceed with carboplatin and paclitaxel.
D. Proceed with further observation.
Question 11.15 A 58-year-old man presented with a 6-month history of
anorexia, fatigue, and a vague right-sided abdominal discomfort. His physical
examination was unremarkable except for mild pallor. Initial laboratory test
revealed a hemoglobin level of 10.7 g/dL with a mean corpuscular volume of
73. He also had mildly elevated alkaline phosphatase and hepatic
transaminases. Colonoscopy revealed a nonobstructing transverse colon
mass. Biopsy demonstrated moderately differentiated adenocarcinoma. CT
scan revealed a 3-cm lesion in the left lobe of the liver and two other lesions
measuring 1.5 to 2 cm in the right lobe of the liver. Positron emission
tomography revealed uptake in the transverse colon and all three known
hepatic lesions. Among the following options, what would be the best option
for this patient based on current studies?
A. Curative-intent resection of the transverse colon primary and all three
hepatic lesions followed by active surveillance
B. Palliative-intent frontline chemotherapy with bevacizumab for

https://t.me/ALGRAWANY33
metastatic colon cancer until progression, followed by palliative
second-line chemotherapy
C. Curative-intent resection of the transverse colon primary and all three
hepatic lesions with perioperative chemotherapy
D. Curative-intent resection of the transverse colon primary and all three
hepatic lesions with radiotherapy
Question 11.16 Which genetic change is associated with resistance to
treatment with cetuximab?
A. KRAS G12C mutation
B. MSI instability high
C. TP53 mutation
D. MLH1 inactivation
Question 11.17 A 54-year-old woman has completed six cycles of first-line
therapy for metastatic KRAS codon 12 mutated right-sided colon cancer with
FOLFOX and bevacizumab and was shown to have stable disease on
restaging scans. Carcinoembryonic antigen (CEA) has also decreased from
135 ng/mL at initial diagnosis to 4.5 ng/mL. She is asking about taking a
break from chemotherapy altogether for the next few months given the stable
disease. When considering observation versus maintenance chemotherapy in
metastatic colorectal cancer, you inform her that:
A. There is no benefit in maintenance therapy with capecitabine and
bevacizumab and observation is preferred.
B. She should switch to FOLFIRI–panitumumab given no response noted
on imaging.
C. Maintenance therapy with capecitabine and bevacizumab will lengthen
the time to progression and is preferred.
D. She needs to complete a total of 12 cycles of oxaliplatin-based regimen
before she can consider observation.
Question 11.18 Which characteristic is not a high-risk feature for stage II
colon adenocarcinoma?
A. T4 perforated tumor
B. Moderately differentiated
C. No involvement of a total of six sampled lymph nodes
D. Lymphovascular invasion
Question 11.19 A 60-year-old woman presents with abdominal discomfort,
bloating, and changes in bowel habits over the past few months. Colonoscopy
reveals ascending colon mass, and pathology is consistent with invasive
adenocarcinoma. Imaging reveals multiple liver lesions and lymph node
metastasis. She undergoes right hemicolectomy for a near-obstructing
ascending colon cancer, and pathology reveals poorly differentiated
adenocarcinoma with positive lymphovascular and perineural invasion, as
well as involvement of 10 out 17 lymph nodes. On the basis of current data,
what treatment regimen would you recommend?
A. 5FU and oxaliplatin plus bevacizumab
B. Trastuzumab and lapatinib
C. 5FU, irinotecan plus cetuximab
D. Nivolumab
Question 11.20 Next-generation sequencing of the tumor resulted for the
patient discussed in Question 11.19. Tumor is microsatellite stable, BRAF
V600E mutated and HER2 negative. She has progressed after 2 months of
first-line therapy. The next best option for systemic therapy would consist of:
A. Nivolumab.
B. Trastuzumab and lapatinib.
C. Best supportive care and hospice.
D. Encorafenib and cetuximab.

Answers

Question 11.1 The correct answer is A.


Desmoid disease is second only to colorectal cancer as a common cause
of death among patients with FAP. There is heterogeneity in the clinical

https://t.me/ALGRAWANY33
presentation of desmoid tumors from asymptomatic abdominal mass to
bowel or ureteral obstruction. Sporadic desmoid tumors may also occur.
Retrospective studies have not confirmed the role of adjuvant radiation
or chemotherapy for desmoid tumors.
Question 11.2 The correct answer is C.
For unresectable tumors, chemotherapy, hormonal therapy, and targeted
agents such as imatinib have been reported in literature as possible
palliative options. Sorafenib, an oral multitargeted tyrosine kinase
inhibitor (TKI), administered at 400 mg daily continuous dose,
conferred a 33% response rate and a 2-year progression-free survival of
81% (compared with 36% in the placebo group) among patients with
progressive, refractory and unresectable or recurrent desmoid tumors.1
Pazopanib, another TKI, also has activity against desmoid tumors.
Aggressive chemotherapy options with doxorubicin, as a single agent, or
combined with dacarbazine or the combination of methotrexate with a
vinca alkaloid had been reported to confer responses, but the toxicities
are higher with combination regimens and benefit is unproven.
Immunotherapy options are being investigated. Because therapeutic
options are available, hospice may not be appropriate at this point.
Question 11.3 The correct answer is C.
This patient meets the Amsterdam criteria for HNPCC with at least three
affected relatives in at least two successive-generation relatives and one
first-degree relative diagnosed with colorectal cancer before age 50
years. Unlike FAP, which is characterized by chromosomal instability
caused by mutations in the APC tumor suppressor gene, HNPCC is
characterized by MSI caused by mutations in the DNA MMR genes.
HNPCC is the most common hereditary syndrome predisposing one to
colorectal cancers, accounting for approximately 2% to 3% of all
colorectal cancer cases. The lifetime risk of developing colon cancer
among patients with HNPCC approaches 80%. HNPCC is also
associated with other malignancies, including endometrial, gastric,
ampullary, biliary, and urinary tract cancers. This patient has Muir–
Torre syndrome characterized by multiple colon cancers and multiple
cutaneous neoplasias, including sebaceous adenomas.
Question 11.4 The correct answer is D.
HNPCC can be confirmed by the demonstration of the MSI-H
phenotype and germline mutation in any of the DNA MMR genes, such
as MLH1 on chromosome 3p, MSH2 on chromosome 2p, MSH6 on
chromosome 2p, PMS1 on 2q, and PMS2 on 7q. Germline mutations
involving MSH2 and MLH1 genes account for more than 60% of the
known mutations present in patients with HNPCC. The National
Comprehensive Cancer Network guidelines recommend the use of
tumor screening with MSI and lack of expression of MMR protein
expression by immunohistochemistry initially, followed by MMR
mutation testing, although proceeding directly to MMR mutation testing
is also acceptable. A negative MMR mutation test result does not rule
out Lynch syndrome or HNPCC.
Question 11.5 The correct answer is C.
Once a germline MMR mutation is identified in a patient, genetic
counseling and testing of at-risk family members are essential. The
patient’s three daughters, along with all her first-degree relatives, have a
50% probability of being a carrier of the mutant gene. In general, for
family members who do not carry the known mutation, there is no need
for intensive surveillance, and routine colorectal screening according to
national guidelines is recommended. For family members who carry the
germline MMR mutation, the National Comprehensive Cancer Network
recommends colonoscopies every 1 to 2 years beginning at age 20 to 25
years or 5 to 10 years younger than the earliest age of diagnosis in the
family, whichever comes first. For women with the MMR mutation,
screening for endometrial cancer with transvaginal US with or without
CA 125 for ovarian cancer screening is not shown to be sufficiently
sensitive or specific in postmenopausal women, but may be considered
at the clinician’s discretion. Transvaginal US is not recommended as a
screening tool in premenopausal women. Endometrial biopsy is both
highly sensitive and specific as a diagnostic procedure, and can be
considered every 1 to 2 years. Women who have completed childbearing

https://t.me/ALGRAWANY33
could opt for prophylactic hysterectomy and bilateral salpingo-
oophorectomy to reduce their risks of developing endometrial and
ovarian cancers.
Question 11.6 The correct answer is D.
Germline mutations in the MMR genes MSH2, MLH1, and MSH6
account for 90% of all patients with HNPCC. APC gene mutation is
commonly associated with FAP. MYH-associated polyposis is a rare
inherited syndrome with increased for colorectal cancer.
Question 11.7 The correct answer is D.
The standard adjuvant therapy for patients with stage III colon cancer
involves the combination of oxaliplatin and 5FU based on the MOSAIC
(Multicenter International Study of Oxaliplatin/5-
Fluorouracil/Leucovorin in the Adjuvant Treatment of Colon Cancer)
and the NSABP-C07 (The National Surgical Adjuvant Breast and Bowel
Project) studies.2,3 Recent updates of the MOSAIC study demonstrated a
significant improvement in 6-year overall survival (OS) among patients
with stage III colorectal cancer treated with FOLFOX compared with
those treated with infusional 5FU and leucovorin. The IDEA
(International Duration Evaluation of Adjuvant Therapy) study did not
confirm the noninferiority of 3 versus 6 months of adjuvant FOLFOX or
CAPOX.4 However, for patients with high-risk features such as T4 or
N2, or both, 6 months of FOLFOX therapy is superior to 3 months of
treatment. Irinotecan-based regimens are not recommended for adjuvant
therapy for patients with stage III, according to the negative results of
three large randomized studies. Current studies on adding bevacizumab
and cetuximab to oxaliplatin regimens are ongoing.
Question 11.8 The correct answer is C.
More than 80% of the administered dose of 5FU is eliminated via the
dihydropyrimidine dehydrogenase (DPD) enzyme. Patients with DPD
deficiency are at high risk for 5FU toxicities, such as severe neutropenia,
neutropenic fever, diarrhea, mucositis, cerebellar ataxia, neurotoxicity,
and even death. The most common mutation associated with DPD
deficiency is the IVS14 + 1 G>A DPYD*2A mutation. Complete DPD
deficiency has been reported in patients homozygous for this allele,
whereas partial DPD deficiency occurs in patients heterozygous for this
allele. More than 40 other mutations and polymorphism have been
identified in the DPD gene, although their functional significance is not
clear. Moreover, instances of low DPD activity have been reported in
patients without any identified DPD mutation or polymorphism. The
TSER*3/*3 genotype is associated with higher levels of thymidine
synthase and lower tumor response to 5FU therapy. Although it is
possible that this patient is homozygous for the UGT1A1*28 gene
resulting in severe neutropenia and diarrhea, the constellation of the
patient’s symptoms, including cerebellar ataxia and mucositis, is more
consistent with 5FU-related toxicities.
Question 11.9 The correct answer is B.
SN-38 is the active metabolite of irinotecan and is 100 to 1,000-fold
more potent than is irinotecan as a topoisomerase I inhibitor.
Glucuronidation is the principal elimination pathway for SN-38. UDP-
glucuronosyltransferase 1 family polypeptide A1 mediates this
glucuronidation of irinotecan encoded by the UGT1A1 gene. Patients are
homozygous for the UGT1A1*28 allele glucuronidate SN-38 less
efficiently than are patients who have one or two wild-type alleles.
Therefore, homozygous patients are exposed to higher plasma
concentrations of SN-38, and are thus at a greater risk for severe
neutropenia.
Question 11.10 The correct answer is D.
The results of the German Rectal Cancer Study confirmed the benefit of
neoadjuvant prolonged course radiation therapy (RT) with 5FU-based
chemotherapy for patients with T3+/N0–2 rectal cancer.5 Although OS
was similar to that in patients treated with neoadjuvant
chemoradiotherapy compared with those treated with postoperative
chemoradiotherapy, local recurrence rates and toxicities were more
favorable with the neoadjuvant approach. Recent data on total
neoadjuvant therapy with the option for a nonoperative management of

https://t.me/ALGRAWANY33
select rectal cancer patients with complete clinical response have also
shown encouraging results.
Question 11.11 The correct answer is A.
Transanal excision should only be considered for select patients with
early-stage rectal cancer with the following characteristics: small rectal
cancers <3 cm; well-to-moderately differentiated T1 tumors within 8 cm
from the anal verge and limited to <30% of the rectal circumference
with no evidence of nodal metastases.
Question 11.12 The correct answer is A.
After transanal resection of appropriately selected patients with early-
stage rectal cancer, no further therapy is recommended.
Question 11.13 The correct answer is D.
The standard therapy for early localized squamous carcinoma of the
anus is chemoradiation with 5FU and mitomycin, which is associated
with excellent cure rates. Salvage APR is reserved for recurrent anal
cancer. Nivolumab is indicated as second-line treatment for metastatic
anal cancer.
Question 11.14 The correct answer is D.
The ACT (Anal Cancer Trial) II study suggests that the optimal time to
assess for complete clinical response for patients with anal cancer
treated with chemoradiation is 26 weeks.6 Many patients who do not
achieve complete response by 11 weeks achieve a complete response by
26 weeks. Thus, surgery at this point would be premature. Further
radiation is also not appropriate at this point. The InterAACT (A
Multicenter Randomized Phase II Advanced Anal Cancer Trial) study
demonstrated that carboplatin and paclitaxel showed equivalent
responses, but with less toxicity compared with cisplatin/5FU in
metastatic, recurrent, or unresectable anal cancer.7
Question 11.15 The correct answer is C.
This patient is potentially curable with an aggressive multidisciplinary
approach. Resection of all tumor sites with close surveillance certainly is
an option for this patient. However, the results of the EORTC (European
Organization for Research and Treatment of Cancer) 40983 study,
showed an improvement in progression-free survival for patients treated
with perioperative chemotherapy with resection.8 Historically, cure rates
of up to 35% have been observed even in patients with metastatic
disease, as long as all tumor sites are resected. Thus, palliative
chemotherapy would not be the best option for this patient. There is
currently no role for radiotherapy for this patient.
Question 11.16 The correct answer is A.
Patients with colorectal cancer who test positive for the KRAS, NRAS, or
BRAF V600E mutations are resistant to treatment with cetuximab.
Question 11.17 The correct answer is C.
On the basis of the phase III CAIRO3 (CApecitabine, IRinotecan,
Oxaliplatin trial) data, after completion of six cycles of CAPOX and
bevacizumab, the patients who transitioned to maintenance therapy with
capecitabine and bevacizumab had a significantly longer time for
progression than did those on observation (8.5 vs. 4.1 months).9 This
study also reported that the patients re-treated with
CAPOX/bevacizumab after progression on maintenance therapy were
noted to have continued improved time to second progression of 19.8
versus 15.0 months. Complete treatment break at this point would not be
preferred, but the patient could make an informed decision based on this
information. Changing to second-line therapy with FOLFIRI plus
panitumumab at this point is an incorrect treatment option because anti-
epidermal growth factor receptor (EGFR) therapy is not beneficial for
patients with KRAS-mutated colorectal cancer. There is no need to go
through all 12 cycles of oxaliplatin at this time if the patient is able to
tolerate the side effects, based on the OPTIMOX-1 (The OPTIMisation
of OXaliplatin) data, because the original chemotherapy may be
restarted at the time of tumor progression.10

https://t.me/ALGRAWANY33
Question 11.18 The correct answer is B.
High-risk features for stage II colon cancers include bowel obstruction,
perforated T4 tumors, poorly differentiated, signet ring or mucinous
tumors, inadequate lymph node sampling (at least 10 is considered
adequate), and lymphovascular invasion.
Question 11.19 The correct answer is A.
FOLFOX and bevacizumab is an approved first-line therapy for
metastatic colon cancer. Although FOLFIRI and Cetuximab is also an
approved first-line regimen, the tumor for this patient is in the right side
of the colon. The FIRE-3 (FOLFIRI plus cetuximab versus FOLFIRI
plus bevacizumab as first-line treatment for patients with metastatic
colorectal cancer) study showed patients with left-sided tumors appear
to have improved survival with anti-EGFR therapy compared with left-
sided tumors.11 Median OS was 38.7 months for left-sided cancers and
16.1 months for right-sided tumors. Bevacizumab seemed to benefit
patients with tumors in either location. Trastuzumab and lapatinib would
not be indicated for HER2 negative tumor. No information is provided
for microsatellite status at time of start of therapy to indicate the benefit
from nivolumab.
Question 11.20 The correct answer is D.
Phase III BEACON supports doublet therapy (Encorafenib plus
Cetuximab) with our without Binimetinib (triplet therapy). Updated
Overall Survival (OS) results showed median OS of 9.3 months with
either the triplet or doublet compared with 5.9 months in the control arm
(investigators’ choice of either cetuximab and irinotecan or cetuximab
and FOLFIRI).12 The overall response rates at the updated analysis were
26% with the triplet, 20% with the doublet, and 2% in the control arm.
The tumor is not microsatellite instability-high (MSI-H), and nivolumab
would not be appropriate. In HER2-positive colon cancers, the phase II
HERACLES (HER2 Amplification for Colo-rectaL cancer Enhanced
Stratification) trial reported 30% ORR with trastuzumab plus lapatinib
in patients with HER2-positive tumors.13 Hospice is certainly a treatment
choice at any time, and could be discussed, but with current data, the
BEACON (Binimetinib, Encorafenib, and Cetuximab Combined to
Treat BRAF-Mutant Colorectal Cancer) regimen would be the next line
of therapy for recommendation.

References
1. Gounder MM, Mahoney MR, Van Tine BA, et al. Sorafenib for advanced and refractory desmoid
tumors. N Engl J Med. 2018;379:2417–2428.
2. André T, Boni C, Mounedji L, et al. Oxaliplatin, fluorouracil, and leucovorin as adjuvant treatment
for colon cancer. N Engl J Med. 2004;350:2343–2351.
3. Kuebler JP, Wieand HS, O’Connell MJ, et al. Oxaliplatin combined with weekly bolus fluorouracil
and leucovorin as surgical adjuvant chemotherapy for stage II and III colon cancer: results from
NSABP C-07. J Clin Oncol. 2007;25:2198–2204.
4. Grothey A, Sobrero AF, Shields AF, et al. Duration of adjuvant chemotherapy for stage III colon
cancer. N Engl J Med. 2018;378:1177–1188.
5. Sauer R, Becker H, Hohenberger W, et al. Preoperative versus postoperative chemoradiotherapy for
rectal cancer. N Engl J Med. 2004;351:1731–1740.
6. James RD, Glynne-Jones R, Meadows HM, et al. Mitomycin or cisplatin chemoradiation with or
without maintenance chemotherapy for treatment of squamous-cell carcinoma of the anus (ACT II):
a randomised, phase 3, open-label, 2 × 2 factorial trial. Lancet Oncol. 2013;14:516–524.
7. Sclafani F, Adams RA, Eng C, et al. InterACT: an international multicenter open label randomized
phase II advanced anal cancer trial comparing cisplatin (CDDP) plus 5-fluorouracil (5-FU) versus
carboplatin (CBDCA) plus weekly paclitaxel (PTX) in patients with inoperable locally recurrent
(ILR) or metastatic disease. J Clin Oncol. 2015;33:TPS792.
8. Nordlinger B, Sorbye H, Glimelius B, et al. Perioperative FOLFOX4 chemotherapy and surgery
versus surgery alone for resectable liver metastases from colorectal cancer (EORTC 40983): long-
term results of a randomised, controlled, phase 3 trial. Lancet Oncol. 2013;14:1208–1215.
9. Simkens LHJ, van Tinteren H, May A, et al. Maintenance treatment with capecitabine and
bevacizumab in metastatic colorectal cancer (CAIRO3): a phase 3 randomised controlled trial of the
Dutch Colorectal Cancer Group. Lancet. 2015;385:1843–1852.
10. Tournigand C, Cervantes A, Figer A, et al. OPTIMOX1: a randomized study of FOLFOX4 or
FOLFOX7 with oxaliplatin in a stop-and-go fashion in advanced colorectal cancer—a GERCOR
study. J Clin Oncol. 2006;24:394–400.
11. Heinemann V, von Weikersthal LF, Decker T, et al. FOLFIRI plus cetuximab versus FOLFIRI plus
bevacizumab as first-line treatment for patients with metastatic colorectal cancer (FIRE-3): a
randomised, open-label, phase 3 trial. Lancet Oncol. 2014;15:1065–1075.
12. Kopetz S, Grothey A, Yaeger R, et al. Encorafenib, binimetinib, and cetuximab in BRAF V600E-
mutated colorectal cancer. N Engl J Med. 2019;381:1632–1643.
13. Sartore-Bianchi A, Trusolino L, Martino C, et al. Dual-targeted therapy with trastuzumab and
lapatinib in treatment-refractory, KRAS codon 12/13 wild-type, HER2-positive metastatic
colorectal cancer (HERACLES): a proof-of-concept, multicentre, open-label, phase 2 trial. Lancet
Oncol. 2016;17:738–746.
___________

https://t.me/ALGRAWANY33
Corresponding chapters in DeVita, Hellman, and Rosenberg's Cancer: Principles & Practice of
Oncology, Eleventh Edition: 61 (Molecular Biology of Colorectal Cancer), 62 (Cancer of the Colon),
63 (Cancer of the Rectum), and 64 (Cancer of the Anal Region).
12 Genitourinary Cancer
Russell K. Pachynski

QUESTIONS

Each of the numbered items below is followed by lettered answers. Select the
ONE lettered answer that is BEST in each case unless instructed otherwise.

Question 12.1 Activating mutations in which of the following genes is seen


in patients with hereditary papillary renal cell carcinoma (HPRC)?
A. VHL
B. MET
C. FLCN
D. SDHB
Question 12.2 A 56-year-old moderately obese woman with a medical
history of chronic hypertension well controlled on a thiazide diuretic presents
to the emergency department with a 1-day history of abdominal pain,
diarrhea, nausea, and fever. General physical examination is significant only
for some mild abdominal tenderness; negative stool guaiac; a white blood cell
count of 14.2/μL; hemoglobin of 14.5 g/dL; a normal platelet count; normal
electrolytes, amylase, lipase, and transaminases; and a creatinine of 0.9
mg/dL. Workup includes an abdominal computed tomography (CT) scan,
which is remarkable for a 1.5-cm enhancing left renal mass in the left lower
pole that is interpreted by the radiologist as a “probable renal cell carcinoma”
without evidence of other metastases. The patient undergoes a laparoscopic
partial nephrectomy. There are no postoperative complications, and she is
back to work 3 weeks later. Postoperative creatinine is 1.3, and pathology
reveals a 2.5-cm renal cell carcinoma (RCC), granular cell type that is
confined to the renal parenchyma. No lymph nodes were recovered. The next

https://t.me/ALGRAWANY33
appropriate step is:
A. Open retroperitoneal lymph node dissection.
B. Adjuvant sunitinib.
C. Adjuvant local radiotherapy.
D. Submission of pathology specimen for second review.
Question 12.3 A 55-year-old woman undergoes partial nephrectomy for
clear cell carcinoma of the kidney, Fuhrman grade IV. The patient does well
for 10 years, at which time she develops a pathologic intratrochanteric
fracture of her left hip. CT scanning of the chest, abdomen, pelvis, and brain
and bone scan reveal no other sites of disease. The most appropriate next step
is:
A. Radiation alone.
B. High-dose interleukin-2 alone.
C. Orthopedic resection of the tumor with reconstruction followed by
radiation.
D. Temsirolimus.
Questions 12.4–5 A 76-year-old man with chronic obstructive pulmonary
disorder and diabetes mellitus presents with back pain and confusion.
Workup reveals extensive metastatic disease in the lungs, bones, and liver
and a 6-cm tumor in the kidney. There is no evidence of cord compression.
Laboratory studies reveal a calcium level of 11.5 mg/dL, lactate
dehydrogenase (LDH) of 600, and a creatinine level of 2.0 mg/dL. After
hydration and zoledronate, his calcium normalizes, hemoglobin is 9.5,
creatinine decreases to 1.7, and Eastern Cooperative Oncology Group
performance status is 2.
Question 12.4 Based on the Memorial Sloan Kettering Cancer Center
(MSKCC) risk factors, this patient would be considered:
A. No risk.
B. Low risk.
C. Intermediate risk.
D. Poor risk.
Question 12.5 The most appropriate therapy in this case is:
A. Interferon-gamma.
B. Ipilimumab plus nivolumab.
C. Sunitinib.
D. Sorafenib.
Question 12.6 A 25-year-old Caucasian man without a medical history
presents with hematuria. Workup reveals bilateral renal cysts, at least one of
them suspicious for malignancy. Family history is significant for a
pheochromocytoma in his father and a pancreatic islet cell tumor and early
death from kidney cancer in a paternal aunt. The most likely familial cancer
syndrome is:
A. Von Hippel–Lindau (VHL) disease.
B. Birt–Hogg–Dubé syndrome.
C. HPRC.
D. Hereditary leiomyomatosis and renal cancer.
Question 12.7 Which of the following associated with loss of VHL
function is most likely to lead to tumor angiogenesis?
A. Modulation of nuclear factor (NF)-κB activity and downregulation of
JUNB
B. Increase in matrix metalloproteinases
C. Increase in hypoxia-inducible factor (HIF)
D. Destabilization of microtubule formation
Question 12.8 Inactivation or alteration in which of the following tumor
suppressor genes is implicated in the pathogenesis of invasive bladder
cancer?
A. TP53
B. RB1
C. PTEN
D. All of the above

https://t.me/ALGRAWANY33
Question 12.9 A patient with hematuria is taken to the operating room,
where an examination under general anesthesia reveals a papillary lesion.
Resection reveals grade III urothelial papillary carcinoma, and multiple
biopsies of the erythematous areas of the bladder all reveal diffuse carcinoma
in situ. Muscle is present in the pathologic specimens, and there is no
evidence for invasive tumor. The appropriate therapy is:
A. Intravesical Bacillus Calmette–Guérin (BCG) vaccine.
B. Intravesical cyclophosphamide.
C. Radiation.
D. Cystectomy.
Question 12.10 The patient receives definitive treatment, as well as with a
follow-up maintenance program; however, 4 months after initiating the
maintenance BCG program, the urologist notes multiple recurrent papillary
lesions. Repeat biopsy reveals urothelial cancer invasive into muscle. CT
scans of the chest, abdomen, and pelvis are unremarkable, creatinine remains
normal at 1.2 mg/dL, and his performance status is excellent. Appropriate
initial therapy at this point is:
A. Reinduction with intravesical BCG.
B. Intravesical chemotherapy with mitomycin C.
C. Partial cystectomy.
D. Cisplatin-based multiagent chemotherapy.
Question 12.11 Which of the following is the most common molecular
abnormality seen in patients with prostate cancer?
A. KRAS mutation
B. BRAF mutation
C. TP53 mutation
D. Chromosomal translocations involving TMPRSS2
Question 12.12 Which of the following is a risk factor for cancer of the
male urethra?
A. Human papillomavirus 6 (HPV-6)
B. Chronic irritation
C. Intravenous drug abuse
D. Caucasian race
Questions 12.13–14 A 62-year-old woman is newly diagnosed with
muscle-invasive bladder cancer. She quit smoking 10 years ago, had a non–
ST-elevated myocardial infarction 4 years ago, underwent coronary artery
bypass surgery and has had no residual cardiac symptoms, takes only a beta-
blocker and a thiazide for hypertension, and has normal laboratory test
results, including a creatinine of 0.9. Cystoscopic biopsy revealed a muscle-
invasive bladder cancer without associated carcinoma in situ (Tcis), and CT of
the chest, abdomen, and pelvis is unremarkable.
Question 12.13 Which of the following statements about radical
cystectomy is CORRECT?
A. An orthotopic neobladder is less effective in women than in men.
B. An abdominal wall diversion will require a urostomy bag.
C. An orthotopic neobladder will require the patient to be willing and able
to perform self-catheterization.
D. Metabolic acidosis is not a significant problem with continent
diversions.
Question 12.14 Which of the following statements about combined
radiation and chemotherapy is most CORRECT?
A. Toxicity profile and tolerability of combined radiation and
chemotherapy are significantly better than that of radical cystectomy.
B. Long-term cancer outcome is similar to cystectomy.
C. It is preferred to cystectomy because of her cardiac history.
D. It will obviate the need for cystectomy.
Question 12.15 Which of the following is CORRECT about neoadjuvant
chemotherapy?
A. Three cycles of methotrexate, vinblastine, doxorubicin, and cisplatin
before cystectomy are a standard of care.

https://t.me/ALGRAWANY33
B. Gemcitabine plus carboplatin should be considered to decrease the risk
of renal failure with cystectomy.
C. It increases the risk of surgical complications of cystectomy.
D. It should always be used with an organ-preservation approach but is
optional if cystectomy is chosen.
Question 12.16 Which of the following genes is most commonly found to
be abnormally altered in invasive (≥T2) urothelial carcinoma of the bladder?
A. ERBB2
B. TP53
C. MDM2
D. FGFR3
Question 12.17 A 71-year-old man without significant medical history
presents with hematuria and flank pain. CT scan reveals a mass at the pelvic–
ureteral junction with associated hydronephrosis, but no associated
lymphadenopathy. Cystoscopy and ureteroscopy reveal an obstructing mass
at the pelvic–ureteral junction, and cytology is diagnostic for urothelial
cancer. Which of the following is the most appropriate therapy?
A. Open nephroureterectomy and bladder cuff resection
B. Open radical nephrectomy with retroperitoneal lymph node dissection
C. Laparoscopic radical nephrectomy without retroperitoneal lymph node
dissection
D. Definitive radiation and combined chemotherapy
Question 12.18 A 51-year-old man with a strong family history of prostate
cancer, normal digital rectal examination, no significant comorbid medical
problems, and a prostate-specific antigen (PSA) of 2.9 seeks advice on
prostate cancer prevention. He is sexually active in a monogamous
relationship and denies any urinary or rectal symptoms. Which is the most
likely to reduce his risk of developing prostate cancer?
A. Reducing his alcohol intake
B. Taking supplemental high-dose vitamin E
C. Avoiding high-fat intake, reducing his body mass index (BMI)
(avoiding obesity), and increasing his physical activity
D. Increasing the calcium/vitamin D intake
Questions 12.19–22 A 71-year-old white man with a history of
hypertension, hyperlipidemia, coronary artery disease, and prior angioplasty
with stent placement, but no prior myocardial infarction, is noted to have an
increase in his PSA from 3.0 to 3.9 ng/mL and then to 4.6 ng/mL over 19
months. He is a semiretired accountant, swims actively three times per week,
and helps care for his mildly demented 95-year-old father. General physical
examination is unremarkable; a rectal examination reveals a mildly enlarged
prostate gland without any palpable nodules.
Question 12.19 The most CORRECT statement about this case is:
A. Biopsy should be discussed because the PSA increase is >0.75
ng/mL/year.
B. Biopsy should not be discussed because PSA is normal for his age.
C. Biopsy should not be discussed because his expected survival makes
treatment not worthwhile, even if prostate cancer is discovered.
D. Biopsy should be discussed because the PSA is >4 ng/mL.
Question 12.20 Biopsy reveals Gleason 8 prostate cancer in six of six
cores. CT scan of the abdomen and pelvis and bone scan are unremarkable.
The most appropriate therapy is:
A. Radical retropubic prostatectomy.
B. Three-dimensional conformal radiotherapy with concomitant androgen
ablation.
C. Interstitial radiotherapy with 125I.
D. All of the above.
Question 12.21 After discussion with a radiation oncologist and urologist,
the patient elects to undergo combined androgen ablation and external beam
radiation therapy. The androgen ablation is administered before the radiation
therapy and continued for 3 months thereafter. Radiotherapy is complicated
only by a mild diarrhea that resolves once the radiation therapy is complete.

https://t.me/ALGRAWANY33
The PSA nadirs at 1.2 ng/mL; however, 9 months after his last dose of the
luteinizing hormone–releasing hormone (LHRH) agonist, the PSA increases
to 2.4 ng/mL and then to 3.6 ng/mL 1 month later. Testosterone level is
normal at 350 ng/mL. The most appropriate next therapeutic and/or
diagnostic maneuver is:
A. Perform magnetic resonance imaging (MRI) of the pelvis to assess for
local recurrence.
B. Reinitiate androgen ablation.
C. Refer the patient to an urologist for salvage prostatectomy.
D. Initiate docetaxel-based chemotherapy.
Question 12.22 The patient is treated with an LHRH agonist along with
bicalutamide. PSA declines to 0.8 ng/mL, but after 10 months, the PSA
begins to slowly increase to a value of 3.7. He continues to feel well and has
minimal urinary symptoms, no bone pain, and no weight loss. The most
appropriate therapy at this point is:
A. Docetaxel-based chemotherapy.
B. Discontinuing bicalutamide.
C. Hospice care.
D. Radionuclide therapy with strontium-98 (Metastron).
Question 12.23 Infestation with which of the following parasites is a risk
factor for developing bladder cancer?
A. Clonorchis sinensis
B. Opisthorchis viverrini
C. Schistosoma haematobium
D. None of the above
Questions 12.24–26 A 51-year-old black male executive with no medical
history undergoes a routine PSA screening evaluation and is found to have a
PSA of 5.5 ng/mL. Biopsy reveals a Gleason 3 + 3 prostate cancer in two of
six biopsy cores. After discussion with a radiation oncologist and urologist,
he elects to receive treatment with a radical retropubic prostatectomy.
Question 12.24 Which of the following statements about the surgery is
most CORRECT?
A. Robotic laparoscopic prostatectomy is associated with a lower
incidence of impotence than open retropubic prostatectomy.
B. The incidence of impotence under the assumption that a bilateral nerve-
sparing procedure can be performed is <10%.
C. Problems with incontinence persist in ~20% of patients.
D. Surgical experience has only a minimal impact on the positive margin
rate.
Question 12.25 Surgical pathology confirms a Gleason score 6 tumor in
both lobes of the prostate. There is a focal surgical positive margin. There is
no evidence of seminal vesicle or lymph node invasion. His postoperative
PSA is undetectable, and he has good continence. The most appropriate next
step is:
A. Adjuvant radiotherapy.
B. Repeat surgical exploration with possible re-excision of the prostatic
bed.
C. Pelvic CT scan.
D. ProstaScint scan.
Question 12.26 The patient maintains an undetectable PSA until 8 years
later, at the age of 59 years, recurrent biochemical disease is noted. After
appropriate discussion, androgen ablation with an LHRH agonist alone is
initiated, and the PSA once again becomes undetectable. The patient
maintains an undetectable PSA while on androgen ablation for 3 years, when
he develops sudden midback pain after lifting his grandson. There are no
associated neurologic signs or symptoms. Bone scan shows marked uptake at
the T8 vertebra, and PSA remains undetectable. The most appropriate
therapeutic or diagnostic maneuver is:
A. Immediate radiotherapy to T8.
B. Therapy with ketoconazole, 400 mg three times daily with
hydrocortisone replacement.

https://t.me/ALGRAWANY33
C. Spinal MRI to rule out cord compression.
D. Bone densitometry to assess for osteoporosis.
Question 12.27 A 75-year-old man with diabetes, hypertension, and
coronary artery disease who is receiving atorvastatin, glyburide, and an
angiotensin-converting enzyme inhibitor is under surveillance after external
beam radiotherapy for a Gleason 3 + 3 prostate cancer that was diagnosed
and treated 10 years earlier when he was found to have a PSA of 4.3 on
routine screening. His PSA level, which had been 0.2 ng/mL, has increased to
0.3, 0.35, and then 0.40 over the period of 18 months. The most appropriate
therapy at this time is:
A. Androgen ablation with an LHRH agonist.
B. Continued active surveillance.
C. High-intensity focused ultrasound to his prostate.
D. High-dose (150 mg) bicalutamide.
Question 12.28 A 65-year-old man has been receiving combined androgen
ablation with leuprolide for 4 years for biochemical recurrence after radical
prostatectomy. His PSA has increased from an undetectable nadir to 1.1
ng/mL on serial measurement over the period of 6 months. The PSA then
continues to increase 2 months later to 2.5, with serum testosterone of 10
ng/mL. Bone scan and CT of the abdomen/pelvis do not reveal any metastatic
disease, and he remains asymptomatic. The most appropriate therapy option
is:
A. Continue current treatment regimen and active surveillance.
B. Docetaxel-based chemotherapy.
C. Start darolutamide.
D. Start high-dose ketoconazole.
Question 12.29 Which of the following individuals has the highest risk of
developing prostate cancer?
A. A 60-year-old Caucasian male with no family history of cancer
B. A 60-year-old African American male with a father with prostate
cancer
C. A 40-year-old Asian male
D. A 40-year-old obese Caucasian male
Question 12.30 Genetic alterations in which of the following is most
common in prostate adenocarcinoma?
A. NRIP1
B. FOXA1
C. PTEN
D. CDK2
Question 12.31 Which of the following statements about the androgen
receptor is most CORRECT?
A. The majority of its activity in prostate cancer is due to its cytoplasmic
effects.
B. Upregulation of androgen receptor expression has been linked to
prostate cancer development.
C. Castration leads to complete inactivation of all androgen receptor–
mediated pathways.
D. Castrate-resistant prostate cancer (CRPC) is associated with
upregulation of androgen receptor expression.
Question 12.32 A 63-year-old uncircumcised man without any significant
medical history presents to his physician with an inability to retract the
foreskin. Examination reveals phimosis, with an underlying painless
ulcerated mass of 1 cm × 2 cm. A 2.5-cm hard node is palpated in the left
inguinal region. Biopsy of the penile lesion reveals squamous cell cancer. In
addition to wide surgical resection of the primary lesion, other appropriate
therapeutic and/or diagnostic maneuvers at this time include:
A. Four-week course of a broad-spectrum antibiotic.
B. Left inguinal lymph node dissection.
C. Bilateral inguinal radiotherapy.
D. Taxane-based chemotherapy.

https://t.me/ALGRAWANY33
Answers

Question 12.1 The correct answer is B.


HPRC is an autosomal-dominant hereditary cancer syndrome
characterized by mutations in the MET proto-oncogene, which encodes
the hepatocyte growth factor/scatter factor receptor tyrosine kinase.
Although MET overexpression has been demonstrated in a number of
epithelial cancers, HPRC is the first cancer syndrome for which
germline MET mutations have been identified. Individuals with this
syndrome are at risk for developing multifocal, bilateral papillary type 1
kidney cancer in their fifth and sixth decades of life, with age-dependent
penetrance of 67% by 60 years of age.
Question 12.2 The correct answer is D.
RCC is currently divided into clear cell and non–clear cell subtypes. The
non–clear cell subtypes have been further divided into papillary,
chromophobe, and collecting duct tumors. Several renal cancers cannot
be accurately subtyped, often because of poor histologic differentiation
such that the originating subtype cannot be easily recognized. Therefore,
a second opinion and pathologic review should be requested. The risk of
recurrence in this case is low, and adjuvant sunitinib has not been shown
to decrease the risk of recurrence. Patients who do have recurrence after
nephrectomy almost always recur systemically; thus, additional local
radiotherapy is not indicated. Likewise, the role of extensive lymph
node dissection is highly controversial and generally not recommended.
In any case, formal retroperitoneal lymph node dissection as is carried
out in testes cancer would certainly not be standard in patients with
localized RCC.
Question 12.3 The correct answer is C.
Patients with renal cancer may develop recurrent disease >5 years after
initial definitive therapy. Development of a single metastatic site is not
unusual. Patients with a single site of metastatic disease and a long
interval between initial diagnosis and recurrent metastatic disease
typically have a good prognosis. Therefore, aggressive surgical resection
including orthopedic stabilization is indicated. Because orthopedic
surgery such as this often leaves microscopic tumors behind in the
surgical field, additional radiotherapy to the site is appropriate.
Radiation alone is insufficient to completely eradicate metastatic renal
cancer and does not repair the underlying structural bone abnormality.
High-dose interleukin-2 is approved for metastatic renal cancer but is
less effective than aggressive local surgery for complete eradication of
disease when possible. Temsirolimus improves survival in patients with
poor prognosis metastatic disease but is not curative.
Question 12.4 The correct answer is D.
Several prognostic factors have been identified that can help explain the
highly heterogeneous natural history of metastatic renal cancer.
Hypercalcemia, poor performance status, elevated LDH, and anemia are
significant prognostic factors in most studies, and the most widely used
prognostic factor model is from the MSKCC. Age, in and of itself, is not
a prognostic factor. The presence of one to two factors is considered
intermediate risk, whereas three or more factors is considered poor risk.
Question 12.5 The correct answer is B.
Phase III data support the use of frontline treatment of intermediate- and
poor-risk metastatic RCC (mRCC) with a combination of the checkpoint
inhibitors ipilimumab and nivolumab. In a randomized, phase III trial
Checkmate 214, the combination of ipilimumab and nivolumab was
superior to treatment with sunitinib in this patient population, with
statistically superior overall survival and progression-free survival, and
an increase in complete responses.1 Sunitinib might be considered in
“favorable-risk” patients as the overall response rates were similar
between ipilimumab plus nivolumab and sunitinib arms. Another first-
line option would be the combination of the axitinib and the checkpoint
inhibitor pembrolizumab, which was superior to sunitinib in a
randomized phase III trial.2 Data suggest that interferon is not effective
in patients with poor prognosis, and phase III data show that sunitinib is
more effective than interferon for metastatic disease in patients with a

https://t.me/ALGRAWANY33
good and intermediate prognosis.
Question 12.6 The correct answer is A.
VHL disease, caused by mutation of the VHL gene, is an autosomal-
dominant cancer syndrome characterized by multiple renal cysts, early-
onset and multiple renal tumors, retinal angiomas and central nervous
system hemangioblastomas, pheochromocytomas, and pancreatic islet
cell tumors. The Birt–Hogg–Dubé syndrome, caused by mutations of the
BHD gene, is characterized by early-onset chromophobe cancers, benign
hair follicle tumors (fibrofolliculomas), and pulmonary cysts. HPRC,
caused by mutations of the MET gene, is characterized by multiple type
I papillary cancers. The hereditary leiomyomatosis and renal cancer
syndrome, caused by mutation of the fumarate hydratase gene, is
characterized by cutaneous and uterine leiomyomas and aggressive type
II papillary cancers.
Question 12.7 The correct answer is C.
VHL is part of the normal oxygen sensing and response system in all
cells and is mutated or altered in >60% of sporadic clear cell carcinomas
of the kidney. Under normoxic conditions, VHL is part of a multiprotein
complex that recognizes the hydroxylated HIF transcription factor, acts
as an ubiquitin ligase, and thus targets it for proteasomal degradation.
Under hypoxic conditions, HIF is not hydroxylated, not targeted to VHL
and thus not degraded, leading to upregulation of multiple genes critical
for the hypoxic response, including vascular endothelial growth factor
and, ultimately, increased angiogenesis. The VHL gene is expressed
constitutively.
Question 12.8 The correct answer is D.
The tumor suppressor genes TP53, RB1 CDKN2A, and PTEN are
implicated in the pathogenesis of invasive bladder cancer.
Question 12.9 The correct answer is A.
Indications for intravesical therapy are multiple recurrent Ta lesions,
especially if they are high grade, or Tcis. It is critical that the pathologic
specimen in patients diagnosed with superficial bladder cancer contains
muscle to determine whether there is an invasive component. Most, but
not all, studies have suggested that intravesical BCG is more effective
than intravesical chemotherapy. Although other chemotherapeutic
agents are being investigated, cyclophosphamide needs to be activated
to 4-hydroxy-cyclosphosphamide in the liver to act as an alkylating
agent, and thus bladder instillation of the parent compound is not
expected to be effective. Radiation or cystectomy, although potentially
useful for invasive carcinoma or refractory bladder cancer, is not
appropriate as initial therapy in this patient.
Question 12.10 The correct answer is D.
Patients with muscle-invasive bladder cancer require definitive local
therapy. Additional intravesical therapy is inappropriate. Partial
cystectomies are only rarely indicated and should be performed in only a
very highly select group, which does not include patients with a history
of Tcis or patients with multiple tumors. The standard definitive therapy
for bladder cancer is cystectomy. Multiagent cisplatin-based
neoadjuvant chemotherapy improves survival for patients with invasive
urothelial bladder cancer and is considered a standard of care.
Question 12.11 The correct answer is D.
Chromosomal translocations fusing the TMPRSS2 androgen-responsive
gene to various ETS family transcription factors, of which ERG is the
most common, occur in ~60% of patients. Prostate cancer is
characterized by a relatively low rate of mutations in KRAS, BRAF, and
TP53, unlike most other tumors.
Question 12.12 The correct answer is B.
Risk factors for male urethral cancers include HPV-16, chronic
irritation, and infection. The incidence of urethral cancer in men with
urethral strictures ranges from 24% to 76%. No racial predisposition has
been noted for urethral cancers.

https://t.me/ALGRAWANY33
Question 12.13 The correct answer is C.
There are several options for urinary diversion after cystectomy, and no
clear evidence has emerged that any is more or less desirable or effective
based on patient gender. Diversion to the abdominal wall with a
urostomy can be performed as a conduit, requiring a urine collection
device, or as a continent diversion, requiring regular catheterization.
Metabolic complications, including metabolic acidosis, can occur with
both conduits and continent diversions. Complications of orthotopic
continent diversions, or neobladders, include intermittent urethral
obstruction and frequent requirement for intermittent or temporary self-
catheterization.
Question 12.14 The correct answer is B.
Organ preservation with combined radiation and chemotherapy has been
investigated in numerous trials, and long-term outcomes are similar to
those observed in cystectomy series. The therapy is, however, an
aggressive and potentially toxic approach in which ~30% of patients will
require cystectomy for incomplete response. It is thus not more tolerable
than cystectomy. The patient’s cardiac history does not preclude major
surgery, and in fact willingness to accept a cystectomy has been an
inclusion criterion for most organ-preservation trials.
Question 12.15 The correct answer is A.
Randomized studies have demonstrated that cisplatin-based combination
chemotherapy provides a survival advantage. The largest phase III
studies used methotrexate, vinblastine, doxorubicin, and cisplatin or a
similar regimen in which the doxorubicin was eliminated (MVC).
Randomized data have demonstrated that there is no increase in surgical
morbidity or complications after neoadjuvant chemotherapy. There are
no data on the neoadjuvant use of carboplatin-based regimens, and data
in the metastatic setting strongly suggest that carboplatin is an inferior
agent in comparison with cisplatin for this disease.
Question 12.16 The correct answer is B.
Alterations, including deletion and mutation in the tumor suppressor
gene TP53, are most common in invasive disease, occurring in 50% to
70% of cases, and are markers of poor prognosis. Activating mutations
of FGFR3 are common in papillary noninvasive cancers, although less
commonly found in T2 tumors (0%–20%). MDM2 and ERBB2 are also
amplified in ~10% of cases, and ERBB2 overexpression can be seen
10% to 50% of patients.
Question 12.17 The correct answer is A.
The standard definitive treatment for renal pelvis urothelial cancers is
radical nephroureterectomy with resection of the bladder cuff at the
ureteral insertion site. Radical nephrectomy without ureter resection is
inappropriate because of the high incidence of undiagnosed or recurrent
disease in the ureter. Definitive radiotherapy would also be
inappropriate because this would be expected to lead to kidney necrosis.
Question 12.18 The correct answer is C.
High-fat diet has been associated with increased prostate cancer
incidence and mortality, and obesity and increased BMI have an
increased risk of death from prostate cancer. There is evidence to
suggest that diet and exercise may slow the course of the disease once
diagnosed. Although dietary studies have suggested that vitamin E
correlates with a decreased incidence of prostate cancer, data from the
phase III intervention study (SELECT) of selenium and vitamin E to
prevent prostate cancer demonstrate that vitamin E use is actually
associated with an increased risk of developing prostate cancer.3 Alcohol
use and calcium/vitamin D intake are not associated with an increased or
decreased risk of prostate cancer.
Question 12.19 The correct answer is A.
The decision to screen for and diagnose prostate cancer always needs to
be accompanied by a discussion between the physician and the patient
regarding risks and benefits of screening. To this end, it should be
recognized that the median survival of an average 70-year-old man
without significant comorbidities is >10 years and that such a patient

https://t.me/ALGRAWANY33
could benefit from the treatment of localized disease. Nonetheless, there
is a risk of treating disease that will never become clinically significant
within the patient’s lifetime. It also needs to be recalled that the
sensitivity and specificity of all PSA-based screening methods are
limited, and there are no absolute “normal” criteria. Therefore, no single
test can determine the absolute need for a biopsy. Predictors of cancer
on biopsy include PSA >4.0 ng/mL, PSA greater than age-adjusted PSA
norms (based on normal increasing PSA with age), free-to-total PSA
ratio, and PSA velocity. Of these, and in this case, the rapid increase in
the PSA is most predictive of malignancy, and this is also predictive of
aggressive disease.
Question 12.20 The correct answer is D.
On the basis of the high Gleason score, the aforementioned rapid PSA
doubling time (PSADT), and the tumor in all cores, this patient is at a
high risk for locally advanced disease and systemic recurrence. As such,
staging CT scan and bone scan are reasonable, although not generally
necessary in patients with lower risk disease. Because of the presence of
high-risk features and the general good health of the patient, watchful
waiting is probably not an appropriate option in this case. Radiotherapy,
with external beam radiotherapy or interstitial brachytherapy, or surgical
prostatectomy is equally appropriate for patients with clinically localized
disease. The choice is dependent on a discussion of expected risks and
benefits.
Question 12.21 The correct answer is B.
Rapidly increasing PSA before diagnosis, high Gleason score, PSA
nadir of >1.0 after radiotherapy, short interval between definitive local
therapy and biochemical recurrence, and rapid PSA increase once
recurrence is identified are all associated with a poor prognosis. The
probability of locally recurrent disease alone is extremely low, and there
is not much value to pelvic MRI for assessing local recurrence. Despite
the poor prognosis, the role of chemotherapy in patients whose
testosterone axis is intact is controversial. Although the timing of
androgen ablation is not clearly defined, some retrospective data suggest
that “early” ablation in high-risk patients provides a long-term survival
advantage over “later” ablation.
Question 12.22 The correct answer is B.
Approximately 10% to 15% of patients receiving treatment with an
antiandrogen will experience an antiandrogen-withdrawal response.
Although the majority of these responses are of brief duration, it is
obviously the easiest therapeutic maneuver. Docetaxel-based
chemotherapy leads to improved survival in CRPC, which is generally
defined as a progressive disease after androgen ablation, an
antiandrogen, and antiandrogen withdrawal. Radioactive nuclides are
occasionally useful for the palliation of diffuse bone pain but have not
been shown to have an impact on survival.
Question 12.23 The correct answer is C.
Over 90% of bladder cancers in Western countries are urothelial
carcinomas. In areas where the parasite S. haematobium is endemic,
squamous cell bladder carcinomas are more common. O. viverrini and
C. sinensis are associated with cholangiocarcinoma and not bladder
cancer.
Question 12.24 The correct answer is C.
Although robotic laparoscopic prostatectomy has been touted as a less-
invasive and potentially more effective approach, there is no reliable
evidence that the risk of long-term complications, including impotence,
is different than that observed in patients undergoing an open procedure
with an experienced surgeon. In both groups, the incidence of impotence
is at least 25%, even in young men with good erectile function before
surgery. The risk of incontinence, when assessed by anonymized,
validated quality-of-life instruments, is on the order of 20%. One of the
most important variables for both pathologic outcome and complications
is the surgeon’s experience.
Question 12.25 The correct answer is A.
Adjuvant radiotherapy has been shown to decrease the risk of recurrence

https://t.me/ALGRAWANY33
in patients with T3 disease. However, overall survival and clinical
metastasis-free survival (MFS) have not been affected by this approach.
Some have thus argued for close monitoring and salvage radiotherapy
on biochemical recurrence in patients such as this. Imaging tests have
not been shown to be helpful in identifying patients most likely to
benefit from adjuvant therapy, and surgical re-exploration is not
indicated either.
Question 12.26 The correct answer is D.
Patients on long-term androgen ablation are at high risk for the
development of osteoporosis and its complications, and an osteoporotic
fracture must be considered in this patient before initiating any therapy
for progressive metastatic disease. If progressive disease were to be
diagnosed, ketoconazole is not an unreasonable second-line hormonal
therapy, but the addition of an antiandrogen would likely be more
appropriate. In the absence of neurologic signs or symptoms, an MRI is
not necessary.
Question 12.27 The correct answer is B.
Patients with a slowly increasing PSA after definitive local therapy may
have a very long natural history, with only a minority of patients having
cancer-related mortality within 10 years. Androgen ablation, especially
in elderly patients, is associated with osteoporosis, muscle loss, and
changes in lipid metabolism that may have significant impact on
cardiovascular comorbidities. High-intensity focused ultrasound for
treatment of possible localized recurrence is still investigational. Single-
agent antiandrogen therapy is not approved in this setting and may lead
to higher mortality in these patients.
Question 12.28 The correct answer is C.
The patient has nonmetastatic CRPC (nmCRPC). The initial diagnostic
test to perform is to confirm the patient is truly at castrate levels of
testosterone (<50 ng/mL), because ~1% of patients do not achieve
complete testosterone suppression with LHRH agonists. Because the
imaging is negative for metastatic disease, it would not be an
appropriate candidate for therapies approved for metastatic CRPC such
as enzalutamide, abiraterone, docetaxel, and sipuleucel-T. Since the
PSADT is short (<2 months), indicating aggressive disease kinetics,
continued surveillance would be less desirable. There are now Food and
Drug Administration (FDA)–approved treatments for nmCRPC that
have been shown to significantly improve MFS compared to placebo
including enzalutamide, apalutamide, and darolutamide. These are
“second-generation” antiandrogen medications that are much more
potent than “first-generation” antiandrogens such as bicalutamide,
flutamide, or nilutamide and increase MFS by ~2 years compared to
placebo.4 With extended follow-up, updated analyses have now shown a
significant improvement in overall survival in addition to MFS with
darolutamide.5 High-dose ketoconazole has been studied in advanced
prostate cancer as it targets a key enzyme (CYP17) in the androgen
synthesis pathway, and although this can result in PSA decline, it has
not been shown to significantly improve overall survival.
Question 12.29 The correct answer is B.
Prostate cancer is rare before the age of 40; thus, additional risk factors
would likely be involved in the etiology of prostate cancer in younger
males. Incidence and mortality rates are significantly lower for Asian
American descent and somewhat lower for Mexican Americans and
other Latinos. African American males have the highest incidence of
prostate cancer, with almost twofold compared with American men of
European descent. African American men are diagnosed at a younger
age with more aggressive disease and worsened clinical outcomes.
There is now also confirmation of a genetic risk component because
genetic polymorphisms, including several on 8q24, have been found to
be closely associated with prostate cancer. In fact, the large difference in
prostate cancer mortality between African Americans and Caucasian
Americans cannot be explained by socioeconomic factors alone. Men
with a first-degree relative with prostate cancer have approximately
twofold to threefold increased risk of developing prostate cancer.
Question 12.30 The correct answer is C.

https://t.me/ALGRAWANY33
PTEN is a tumor suppressor that deactivates PI3K signaling, and its
deletion is seen in ~30% of primary prostate cancers and more
commonly in advanced disease (50%–55%). PTEN plays a prominent
role in tumorigenesis and is used in preclinical models of prostate
cancer. FOXA1 is a transcription factor that is involved in AR and
steroid receptor function and can promote progression to CRPC, but is
altered in ∼5% of cases. CDK2, a cyclin-dependent kinase involved in
the RB pathway, and NRIP1, an AR corepressor, are also less commonly
altered.
Question 12.31 The correct answer is D.
Although the androgen receptor does have cytoplasmic effects, some of
which may be important to prostate cancer oncogenesis, the majority of
its effects occur in the nucleus where it modifies expression of genes
with an androgen-responsive promoter. Upregulation of androgen
receptor expression has been reported to be both necessary and
sufficient for the castrate-resistant state, which is likely an adaptation to
prolonged exposure to a low androgen environment. Such upregulation
is not generally observed de novo and has not been linked to the
development of prostate cancer. Castration dramatically lowers
testosterone, but the levels of other androgenic steroids are sufficient to
cause partial activation of androgen receptor–mediated pathways.
Question 12.32 The correct answer is A.
Penile carcinoma is a rare malignancy that presents most commonly
with phimosis in an uncircumcised patient. Metastatic spread of penile
carcinoma is via superficial inguinal lymphatics, followed by deep
inguinal lymphatics, and then to the pelvic lymphatics. Systemic
metastatic disease almost never develops in the absence of pelvic
lymphadenopathy. Therefore, surgery and, occasionally, radiotherapy
form the cornerstone of treatment for lymph node–positive patients.
However, half of the patients with clinically palpable nodes will have
inflammatory lesions only. Thus, a course of antibiotics before any
further therapy is indicated. If the nodes persist after a course of
antibiotics, biopsy and surgical dissection are indicated. Management of
patients with clinical N0 is more controversial, with some authors
recommending immediate lymph node dissection and others
recommending a watchful waiting approach. Pathologic risk factors as
assessed in the primary lesion may assist in decision-making.
Lymphangiography before a course of antibiotics is generally not
considered helpful.

References
1. Motzer RJ, Rini BI, McDermott DF, et al. Nivolumab plus ipilimumab versus sunitinib in first-line
treatment for advanced renal cell carcinoma: extended follow-up of efficacy and safety results from
a randomised, controlled, phase 3 trial. Lancet Oncol. 2019;20:1370–1385.
2. Rini BI, Plimack ER, Stus V, et al. Pembrolizumab plus axitinib versus sunitinib for advanced
renal-cell carcinoma. N Engl J Med. 2019;380:1116–1127
3. Klein EA, Thompson IM, Tangen CM, et al. Vitamin E and the risk of prostate cancer: the
Selenium and Vitamin E Cancer Prevention Trial (SELECT). JAMA. 2011;306:1549–1556.
4. Fizazi K, Shore N, Tammela TL, et al. Darolutamide in nonmetastatic, castration-resistant prostate
cancer. N Engl J Med. 2019;380:1235–1246.
5. Fizazi K, Shore ND, Tammela T, et al. Overall survival (OS) results of phase III ARAMIS study of
darolutamide (DARO) added to androgen deprivation therapy (ADT) for nonmetastatic castration-
resistant prostate cancer (nmCRPC). J Clin Oncol. 2020;38:5514.
___________
Corresponding chapters in DeVita, Hellman, and Rosenberg’s Cancer: Principles & Practice of
Oncology, Eleventh Edition: 65 (Molecular Biology of Kidney Cancer), 66 (Cancer of the Kidney), 67
(Molecular Biology of Bladder Cancer), 68 (Cancer of the Bladder, Ureter, and Renal Pelvis), 69
(Molecular Biology of Prostate Cancer), and 70 (Cancer of the Prostate).

https://t.me/ALGRAWANY33
13 Skin Cancer
Leonel Hernandez-Aya

QUESTIONS

Each of the numbered items below is followed by lettered answers. Select the
ONE lettered answer that is BEST in each case unless instructed otherwise.

Question 13.1 The following mutations are frequently found in cutaneous


melanomas, EXCEPT:
A. BRAF.
B. NRAS.
C. KIT.
D. NF1.
Question 13.2 Which of the following statements regarding the
presentation of cutaneous melanoma is FALSE?
A. Melanoma can occur in areas of the body without substantial sun
exposure.
B. In the United States, the majority of patients with melanoma present
with distant metastatic disease.
C. Head and neck melanomas have poorer prognosis than trunk or
extremity melanomas.
D. In young adults, the incidence of melanoma is higher in women than
men.
Question 13.3 Risk factors for cutaneous melanoma include all of the
following, EXCEPT:
A. Presence of multiple (>100) melanocytic nevi.
B. Tanning bed use.
C. Germline mutation in CDKN2A.
D. Pregnancy.
Question 13.4 Which of the following statements regarding the
epidemiology of malignant melanoma is FALSE?
A. The incidence of melanoma is increasing more rapidly than that of any
other malignancy.
B. Five-year survival rates for melanoma have increased in the past two
decades.
C. The increased incidence of melanoma can be explained by an increased
incidence in acral sites in non-Caucasian population.
D. Melanoma is the fifth most common U.S. cancer diagnosis.
Question 13.5 Which of the following factors is considered a poor
prognostic feature of malignant cutaneous melanoma?
A. Ulceration of primary tumor
B. Depth of invasion
C. Location in head and neck
D. All of the above
Question 13.6 A healthy 48-year-old woman presented to her
dermatologist with an atypical mole on her left lower extremity. She
underwent a biopsy that reported a 3-mm Breslow thickness primary
cutaneous superficial spreading melanoma, nonulcerated. The lesion was
excised by her local dermatologist. What is the next step in her management?
A. Referral to medical oncology for systemic therapy.
B. Follow-up in a surveillance program.
C. Referral to a surgeon for wide local excision and sentinel lymph node
mapping.
D. Complete lymph node dissection.
Question 13.7 A 50-year-old man presented to his dermatologist with a

https://t.me/ALGRAWANY33
pigmented lesion on his right leg. A shave biopsy confirmed a superficial
spreading melanoma. There was no palpable regional lymphadenopathy. He
was referred to surgery and underwent a wide local excision and sentinel
lymph node biopsy. Pathology showed a 2.2-mm Breslow thickness,
ulcerated, melanoma with two regional lymph nodes microscopically
involved with melanoma. Positron emission tomography/computed
tomography (PET/CT) scan and brain magnetic resonance imaging (MRI) did
not show evidence of metastatic disease (T3b N2a Mx). The tumor is
negative for the BRAF V600E/K mutation. What is the most appropriate
recommendation after surgery?
A. Complete lymph node dissection
B. Adjuvant therapy with a programmed death protein 1 (PD-1) inhibitor
C. Adjuvant therapy with a cytotoxic T-lymphocyte–associated protein 4
(CTLA-4) inhibitor
D. Adjuvant high-dose interferon alfa-2b
Question 13.8 A 40-year-old female presented with a mass in her left
axilla. Past medical history revealed a stage I cutaneous melanoma on the left
shoulder (Breslow depth 0.47 mm, pT1N0) 10 years before. At that time, the
patient had a wide local excision of the primary lesion in her left shoulder,
with no further information available. She has no other comorbidities. On
physical examination, there is palpable lymphadenopathy in left axilla
measuring 5 cm in largest diameter. Resection of the axillary mass confirmed
metastatic melanoma. A CT scan revealed bilateral pulmonary metastases and
liver metastasis, confirming a stage IV M1c metastatic melanoma. Which of
the following is the most appropriate next step in her management?
A. Genetic screening for the RET mutation
B. Immunohistochemical staining for BRAF V600E/K
C. JAK2 mutation analysis
D. KIT mutation analysis
Question 13.9 A 45-year-old man was recently diagnosed with stage IV
melanoma. PET/CT showed metastatic disease to the lung, liver, and bones.
Brain MRI did not show brain lesions. BRAF V600E/K mutation testing was
negative. Which of the following is the most appropriate initial therapy for
this patient?
A. Dacarbazine
B. Temozolomide
C. Interferon alfa-2b
D. Ipilimumab and nivolumab
Questions 13.10–11 A 50-year-old male with metastatic melanoma, on
treatment with ipilimumab plus nivolumab, presented with 3 days of nausea,
vomiting, abdominal pain, and six watery nonbloody bowel movements
daily. He denies history of autoimmune conditions. Abdominal examination
reveals mild diffuse abdominal tenderness. No signs or peritoneal irritation.
Laboratory studies show leukocytosis and a mildly elevated creatinine.
Infectious stool workup with fecal leukocytes, Clostridium difficile antigen
and toxin, cultures, and ova–parasites are unremarkable. CT imaging reveals
bowel wall thickening in descending and sigmoid colon. Sigmoidoscopy and
biopsy reveal inflammatory cell infiltrate with cryptitis.
Question 13.10 Which of the following is the most likely diagnosis?
A. Crohn disease
B. Ulcerative colitis
C. Ischemic colitis
D. Autoimmune colitis
Question 13.11 Which of the following is the most appropriate treatment
for this patient?
A. Close observation
B. Metronidazole
C. Prednisone
D. Trimethoprim/sulfamethoxazole
Question 13.12 Which of the following statements regarding the
combination of ipilimumab and nivolumab as first-line therapy in metastatic
melanoma is FALSE?

https://t.me/ALGRAWANY33
A. Ipilimumab and nivolumab resulted in a progression-free survival
(PFS) of 11.5 months compared to 6.9 months with nivolumab alone
and 2.9 months with ipilimumab alone.
B. Over 50% of patients treated with ipilimumab and nivolumab in the
CheckMate 067 trial experienced grade 3 or 4 adverse events.
C. Combination therapy with ipilimumab and nivolumab showed greater
PFS benefit in PD-L1–positive tumors compared to nivolumab alone.
D. Ipilimumab and nivolumab demonstrated higher response rates
compared to ipilimumab alone.
Question 13.13 A 45-year-old female is diagnosed with metastatic
melanoma. Imaging studies show bilateral pulmonary nodules, liver lesions,
and multiple brain metastases ranging from 5 to 10 mm in size. The patient
does not have neurologic symptoms associated with the brain metastases. Her
tumor is BRAF V600 wild type. Which of the following could be considered
as first-line treatment modality in this otherwise healthy patient with normal
organ function?
A. Whole-brain radiation therapy
B. Ipilimumab and nivolumab
C. Nivolumab
D. Dabrafenib and trametinib
Question 13.14 Which of the following statements regarding adjuvant
therapy is FALSE?
A. In high-risk melanoma, adjuvant interferon-alfa prolongs relapse-free
survival compared to observation.
B. Adjuvant ipilimumab improved recurrence-free survival (RFS) and
overall survival (OS) compared to nivolumab in high-risk stage III
melanoma.
C. Pembrolizumab is an approved therapy for patients with high-risk stage
III melanoma.
D. Combination therapy with dabrafenib and trametinib improved RFS
compared to placebo in stage III melanoma with BRAF V600E/K
mutations.
Question 13.15 Which of the following statements regarding intralesional
therapy with Talimogene laherparepvec (T-VEC) is FALSE?
A. T-VEC can induce durable clinical responses in patients with in-transit
metastasis.
B. T-VEC has lower systemic toxicity than ipilimumab.
C. T-VEC is a genetically modified oncolytic adenovirus.
D. Intralesional T-VEC can cause regressions of noninjected lesions.
Question 13.16 A 42-year-old woman with history of myasthenia gravis
presents with a right-sided upper quadrant pain. CT reveals multiple liver
lesions suspicious for metastatic disease. Liver biopsy reveals malignant
melanoma. The tumor has a BRAF V600E mutation. What is the most
appropriate treatment?
A. Ipilimumab and nivolumab
B. Pembrolizumab
C. Dabrafenib and trametinib
D. Dacarbazine
Question 13.17 A 72-year-old man presented with a 3-week history of
increasing neck pain and difficulty swallowing associated with a rapidly
enlarging mass palpable in the left neck. The patient reported that 3 years
ago, he had stage I melanoma resected from his occipital scalp (Breslow
depth 0.50 mm, Clark II, pT1N0). A core biopsy of the left neck mass
revealed metastatic malignant melanoma. The tumor was positive for a BRAF
V600K mutation. PET scan reveals multiple large hypermetabolic left-sided
cervical, left supraclavicular lymph nodes, and multiple liver and pulmonary
metastases confirming stage IV melanoma. MRI brain is negative. Laboratory
studies showed an elevated lactate dehydrogenase (LDH) at 750 U/L. Which
of the following is the most appropriate treatment for this patient?
A. Imatinib
B. Dacarbazine plus ipilimumab
C. Nivolumab
D. Dabrafenib plus trametinib

https://t.me/ALGRAWANY33
Question 13.18 Which of the following is a common toxicity associated
with BRAF inhibition?
A. Autoimmune colitis
B. Lower extremity edema
C. Squamous cell carcinomas (SCCs)
D. Pneumonitis
Question 13.19 An 80-year-old female with stage IV melanoma with lung
metastasis presents with worsening cough and shortness of breath at rest. She
denies chest pain, orthopnea, or lower extremity edema. She is on her fifth
cycle of nivolumab therapy, and most recent CT scan revealed partial
response to therapy. She reports no fevers, chills, night sweats, weight loss,
abdominal pain, or diarrhea. Medical history is significant for mild chronic
obstructive pulmonary disease (COPD) and hypertension. Oxygen saturation
is 88%. Laboratory studies indicate a normal troponin level, hemoglobin
level of 12.0 g/dL, a leukocyte count of 7,200/μL, a platelet count of 250,000/
μL. Creatinine and liver function tests are normal. Which of the following is
the most likely diagnosis?
A. Community-acquired pneumonia
B. Heart failure
C. Pneumonitis
D. Progression of pulmonary metastasis
Question 13.20 Which of the following mutations is commonly found in
ocular melanomas?
A. GNAQ
B. NRAS
C. KIT
D. BRAF
Question 13.21 A 68-year-old Caucasian man presents with a rapidly
enlarging pink nodule in right lower extremity. Biopsy shows Merkel cell
carcinoma (MCC). CT imaging reveals liver and retroperitoneal metastases.
What is the most appropriate initial therapy in this otherwise healthy patient?
A. Platinum-based chemotherapy
B. Pembrolizumab
C. Ipilimumab
D. Imatinib
Question 13.22 Which genetic change is associated with basal cell
carcinomas (BCCs)?
A. K-RAS mutation
B. Inactivation of the PTCH1 gene
C. Overexpression of EGFR
D. PTEN deletion
Question 13.23 Which of the following treatments are approved for the
treatment of locally advanced and metastatic BCC?
A. Vismodegib
B. Imatinib
C. Sunitinib
D. Sorafenib
Question 13.24 Which of the following statements regarding cutaneous
SCC is FALSE?
A. Actinic keratosis (AKs) lesions are precursors of SCC.
B. Perineural invasion is a poor prognostic factor.
C. Cetuximab is the first-line therapy for advanced disease.
D. Cemiplimab-rwlc is a PD-1 inhibitor approved for advanced SCC.

Answers

Question 13.1 The correct answer is C.


Cutaneous melanomas have high rates of BRAF (40%–50%), NRAS
(20%), or NF1 (15%) mutations. Mucosal and acrolentiginous

https://t.me/ALGRAWANY33
melanomas have lower rates of BRAF mutations (5%–20%) and a
relatively higher rate of KIT mutations (5%–10%). Uveal melanomas
have a distinct set of driver mutations in GNAQ and GNA11, which are
the alpha subunits of G-protein–coupled receptors.
Question 13.2 The correct answer is B.
In the United States, it is estimated that over 80% of patients with
melanoma initially present with localized disease, 9% with regional
disease, and 4% with distant metastatic disease. In the United States and
Australia, the male-to-female ratio of melanoma at diagnosis is 2:1 for
older patients. However, between the age of 15 and 39 years, melanoma
is more common in females (rate ratio, 0.6). The clinical outcome for
patients with melanomas on extremities is better than that for patients
with truncal or head and neck melanomas; thus, the prognostic impact of
sex is difficult to distinguish from the impact of tumor location. There
may still be, however, a prognostic benefit for female sex, independent
of tumor. Location of tumors has prognostic relevance. Head and neck
melanomas have poorer prognosis than trunk or extremity melanomas,
and melanomas on acral sites have poorer prognosis than other
extremity melanomas. Anorectal, female genital, and head and neck
melanomas of mucosal origin have higher mortality risk.
Question 13.3 The correct answer is D.
Physical traits such as blond or red hair, green or blue eyes, and the
presence of multiple (>100) melanocytic nevi have been linked to
increased incidence of melanoma. Ultraviolet (UV) light exposure has
been implicated as a major etiologic factor in the development of
melanoma. UV irradiation can induce DNA damage, inflammation,
immune suppression, and induction of tissue proteases. Animal data
suggest that sun exposure early in life increases the risk of melanoma,
and sunburns early in life have been implicated in melanoma incidence.
Tanning bed use, mainly in adolescence or early adulthood, has been
implicated in the etiology of melanoma. Approximately 5% of
melanomas occur in high-risk families. The most frequent melanoma-
susceptibility gene is a germline mutation in CDKN2A, a tumor
suppressor gene that encodes for p16INK4A and p14ARF. These
proteins control cell-cycle progression and apoptosis and have roles in
correcting DNA damage and cellular senescence. Pregnancy is not
considered a risk factor for the development of primary or recurrent
melanoma. The prior impression of an apparent association of
pregnancy and melanoma may be due to melanoma being the second
most frequent cancer in females of childbearing age.
Question 13.4 The correct answer is C.
Malignant melanoma is the fifth most common U.S. cancer diagnosis.
The actual incidence of melanoma is increasing more rapidly than that
of any other malignancy. The increased incidence of melanoma over the
past few decades can be explained primarily by increased incidence of
cutaneous melanoma in white populations. In nonwhite populations,
there is a much higher proportion of melanomas in acral (subungual,
palmar, plantar) and mucosal locations. However, there is a
disproportionate increase in nonacral cutaneous melanomas in whites
rather than an absolute increase in acral and mucosal melanomas in
nonwhite populations. The 5-year OS rates for melanoma have increased
from 82% in the late 1970s (1975–1977) to 92% in the more recent era
(2002–2006).
Question 13.5 The correct answer is D.
The best predictor of metastatic risk is the depth of invasion. Breslow
thickness is the depth of invasion measured from the granular layer of
the epidermis to the base of the lesion. The current melanoma staging
system of the American Joint Committee on Cancer (AJCC) identifies
tumor (T) stage based on Breslow thickness such that T1 lesions are ≤1
mm thick, T2 lesions are 1.1 to 2 mm thick, T3 lesions are 2.1 to 4 mm
thick, and T4 lesions are >4 mm thick. Many other histologic and
clinical features have relevance for estimating the risk of future
metastasis and mortality. Ulceration of the primary lesion has been
identified as an important negative prognostic factor and is incorporated
in the current staging system. Other prognostic factors include age,
angiolymphatic invasion, mitotic rate, sex, and body site. Head and neck

https://t.me/ALGRAWANY33
melanomas have poorer prognosis than trunk or extremity melanomas,
and melanomas on acral sites have poorer prognosis than other
extremity melanomas.
Question 13.6 The correct answer is C.
The vast majority of melanomas present as clinically localized lesions
without clinical or radiologic evidence of metastatic disease. Most
primary melanomas are diagnosed on histologic assessment of skin
biopsy performed by a dermatologist or a primary care provider. The
patient should be referred to a surgeon for definitive surgical
management that includes both therapeutic resection and pathologic
staging evaluation for regional metastases with a sentinel lymph node
biopsy. Wide excision of the primary melanoma with appropriate
margins is performed to provide local control. The wide excision also
offers an opportunity to evaluate the tissue adjacent to the primary lesion
for microscopic satellites, which, if present, have clinical and prognostic
significance. Melanoma may metastasize by lymphatic or hematogenous
routes. Usually, lymphatic dissemination presents earlier than
hematogenous dissemination. Thus, emphasis is placed on staging the
regional nodes in patients with melanoma. The finding of lymphatic
metastases is associated with a higher risk of systemic disease.
Oncology referral is more appropriate after evaluation by surgery and
staging is completed. Complete lymph node dissection is recommended
for patients with palpable metastatic melanoma in regional lymph nodes.
Question 13.7 The correct answer is B.
This patient has been diagnosed with stage IIIC cutaneous melanoma.
After definitive surgical therapy with wide local excision and sentinel
lymph node biopsy, this patient should be evaluated by medical
oncology to discuss adjuvant systemic therapy. Adjuvant treatment
options for patients with high-risk resected melanoma are rapidly
evolving. Until recently, the traditional systemic therapy approaches
were insufficient. Adjuvant high-dose interferon-alfa was used for many
years with small benefit. A meta-analysis reported improvements in RFS
and OS of <4%. More recently, several prospective randomized trials
demonstrated a significant improvement in RFS in patients with stage III
melanoma treated with checkpoint inhibitors and BRAF-targeted
therapy. Ipilimumab at a high dose (10 mg/kg) showed a significant
improvement in RFS and OS in selected patients with stage III
melanoma and was approved by the Food and Drug Administration
(FDA). However, this regimen is associated with a high risk of severe
immune-mediated adverse events and is no longer recommended in the
adjuvant setting. The PD-1 targeting antibodies pembrolizumab and
nivolumab have been approved as an adjuvant treatment for resected
melanoma based on results from two randomized clinical trials,
Keynote-054 and CheckMate 238. In the Keynote-054 trial,
pembrolizumab was compared to placebo and demonstrated an
improvement in RFS and reduced risk of distant metastases.1 Nivolumab
was superior and less toxic when compared to ipilimumab in the
CheckMate 238 study improving RFS in patients with resected stage
IIIB/C or stage IV.2 For patients with BRAF-mutated stage III
melanoma, BRAF-targeted therapy with dabrafenib plus trametinib for 1
year is also a treatment option. Because this patient is BRAF wild type,
pembrolizumab is the recommended treatment in the adjuvant setting.
Question 13.8 The correct answer is B.
The most appropriate management strategy at this point is testing for the
BRAF V600E/K mutation. This patient has recurrent stage IV melanoma
with pulmonary and liver metastases. Constitutively activating
mutations in the BRAF oncogene have been reported in 33% to 47% of
primary melanomas and up to 55% of metastatic melanomas. The most
common of these mutations results in substitution of the valine residue
at amino acid position 600 to glutamic acid in the BRAF protein
(V600E), locking the kinase into an active conformation. V600K
mutations occur in 20% of tumors. Patients with metastatic melanoma
who have the BRAF mutation are typically younger, and most patients
respond to BRAF inhibitors. BRAF inhibitors have demonstrated
significant clinical benefit in patients with advanced melanoma.
Therefore, BRAF testing is recommended in every patient newly
diagnosed with metastatic melanoma. Activating mutations in RET

https://t.me/ALGRAWANY33
proto-oncogene are associated with other types of cancer, including
medullary thyroid carcinoma. JAK2 mutations are clinically relevant for
the diagnosis of myeloproliferative neoplasms. KIT mutations are more
common in mucosal and acral melanomas, potentially conferring
sensitivity to KIT inhibitors.
Question 13.9 The correct answer is D.
Immunotherapy has emerged as the cornerstone of treatment for
advanced melanoma. The checkpoint inhibitor ipilimumab, a
monoclonal antibody (mAb) directed against CTLA-4, was the first-ever
therapy to demonstrate improved OS in metastatic melanoma, earning
FDA approval for this population in 2011. Two additional inhibitors,
nivolumab and pembrolizumab, both targeting the PD-1 pathway, were
subsequently approved for the same indication in late 2014. A large,
randomized phase III study compared the combination of nivolumab and
ipilimumab to nivolumab alone and ipilimumab alone.3 The patients
treated with combined ipilimumab and nivolumab had better response
rates, and the PFS was significantly longer for the combination (11.5
months), compared to single-agent nivolumab (6.9 months) or
ipilimumab (2.9 months). In this patient with metastatic melanoma, with
multiple sites of metastasis and good performance status, the
combination of ipilimumab plus nivolumab is reasonable. Dacarbazine
and temozolomide should not be considered first-line options in
metastatic melanoma. Interferon alfa-2b is approved for stage III
melanoma in the adjuvant setting. However, interferon has not
demonstrated clinical benefits in metastatic melanoma.
Question 13.10 The correct answer is D.
The most likely diagnosis is autoimmune colitis secondary to
immunotherapy with ipilimumab plus nivolumab. Immunotherapy
triggers an enhanced immune response driven by T-cell activation,
which may lead to autoimmune-related inflammation of normal tissues.
This patient on ipilimumab and nivolumab therapy is presenting with
nausea, vomiting, diarrhea, and abdominal pain, with CT findings
consisting of colitis. Diarrhea is one of the most frequent adverse events
related to ipilimumab plus nivolumab. The term colitis is used to
describe diarrhea associated with abdominal pain, rectal bleeding or
mucous production, or when imaging findings confirm large-bowel
inflammation. Histopathology of ipilimumab-induced colitis reveals a
pattern distinct from inflammatory bowel disease with dysregulation of
gastrointestinal (GI) mucosal immunity. The descending colon is usually
involved, and on microscopy, an inflammatory cell infiltrate with
cryptitis may be evident; granulomas, a hallmark of Crohn disease, have
not been described.
Question 13.11 The correct answer is C.
The most appropriate treatment for this patient with diarrhea and
autoimmune colitis secondary to immunotherapy is prednisone.
Corticosteroids are administered when the severity of an immune-related
adverse event warrants reversal of inflammation. Standard practice in
clinical trials is to grade the side effects according to Common
Terminology Criteria for Adverse Events (CTCAE), and these may also
be used in the clinic. This patient has grade 2 diarrhea and colitis (4–6
loose stools with abdominal pain). In a patient with associated
abdominal pain or if diarrhea frequency exceeds six episodes per day,
steroids should be initiated. Other immunomodulatory medications
(IMMs) used in steroid-refractory cases include the anti–tumor necrosis
factor (TNF)-alpha antibody infliximab, the antimetabolite
mycophenolate mofetil, and the calcineurin inhibitors tacrolimus and
cyclosporine. Sigmoidoscopy-proven colitis with failure of improvement
after 48 to 72 h warrants infliximab administration. In all cases, stool
cultures should be sent to exclude infection, abdominal imaging with x-
ray or CT obtained, and electrolytes monitored. Close observation or
antibiotics are not the mainstay of the treatment of a patient with colitis
secondary to ipilimumab.
Question 13.12 The correct answer is C.
CTLA-4 and PD-1 are nonredundant pathways that regulate adaptive
immunity at different phases of T-cell activation. Dual checkpoint
inhibitors are, therefore, likely to have complementary mechanisms of

https://t.me/ALGRAWANY33
action and synergistic activity. A large, randomized phase III study
(CheckMate 067) compared the combination of nivolumab and
ipilimumab to nivolumab-only and ipilimumab-only monotherapy.3
CheckMate 067 included 945 previously untreated patients with
unresectable stage III or IV melanoma randomized 1:1:1 to nivolumab
alone, nivolumab plus ipilimumab, or ipilimumab alone. The median
PFS was 11.5 months in the ipilimumab/nivolumab group, 6.9 months in
the nivolumab group, and 2.9 months in the ipilimumab group. The
overall response rate (ORR) was 58% with combination therapy, 45%
with nivolumab, and 19% with ipilimumab. CheckMate 067 showed that
OS was significantly improved with combination therapy versus
ipilimumab. In patients with tumors positive for programmed death
ligand 1 (PD-L1), the median PFS was 14.0 months in the nivolumab
plus ipilimumab group and in the nivolumab-alone group, but in patients
with PD-L1–negative tumors, PFS was longer with the combination
therapy than with nivolumab alone (11.2 months [95% CI, 8.0 months to
not reached] vs. 5.3 months [95% CI, 2.8–7.1 months]). Treatment-
related adverse events of grade 3 or 4 occurred in 16.3% of the patients
in the nivolumab group, 55.0% of those in the nivolumab plus
ipilimumab group, and 27.3% of those in the ipilimumab group.
Question 13.13 The correct answer is B.
This patient is presenting with extensive intracranial and extracranial
metastatic melanoma. Approximately 40% of patients with metastatic
melanoma experience brain metastasis. This patient has no neurologic
symptoms associated with the brain metastasis; therefore, whole-brain
radiation could be deferred at this point. Whole-brain radiotherapy is
reserved for large or multiple brain metastases with limited efficacy. In
the CheckMate 204 study, the combination of ipilimumab and
nivolumab demonstrated a significant intracranial clinical benefit in
patients with asymptomatic brain metastasis.4 The combination of
ipilimumab plus nivolumab showed a rate of intracranial objective
response of 55% (26% had a complete response and 30% a partial
response). Similar rates of objective response (50%) and clinical benefit
(56%) were observed for extracranial lesions. This study showed that
combination therapy with ipilimumab and nivolumab has a clinically
meaningful activity and should be considered first line for patients with
asymptomatic, untreated melanoma brain metastases. Single-agent
nivolumab has lower intracranial response rates. Targeted therapy with
dabrafenib and trametinib is not indicated in a patient with BRAF wild-
type melanoma.
Question 13.14 The correct answer is B.
Nivolumab was superior and less toxic when compared to ipilimumab in
the CheckMate 238 study, improving RFS in patients with resected stage
IIIB/C or stage IV.2 CheckMate 238 was a randomized, double-blind,
phase III trial assigning 906 patients who had completed resection of
stage IIIB, IIIC, or IV melanoma to receive either nivolumab at a dose
of 3 mg/kg every 2 weeks or ipilimumab at 10 mg/kg every 3 weeks for
four doses and then every 12 weeks (453 patients), both given for up to
1 year. Treatment with nivolumab was associated with a significant
improvement in RFS and distant metastasis-free survival (DMFS). At a
follow-up of 18 months, the 12-month RFS rate was 70.5% (95% CI,
66.1%–74.5%) in the nivolumab group and 60.8% (95% CI, 56.0%–
65.2%) in the ipilimumab group (HR 0.65; p < 0.001). Treatment-related
grade 3 or 4 adverse event rates were 14.4% in the nivolumab group and
45.9% in the ipilimumab group. Treatment was discontinued because of
any adverse event in 9.7% and 42.6% of the patients in the nivolumab
and ipilimumab groups, respectively. Two deaths (0.4%) related to toxic
effects were reported in the ipilimumab group, and there were none in
the nivolumab group. Additional follow-up is needed to determine the
impact of nivolumab on OS compared to ipilimumab. Based on these
results, in 2017, the FDA approved adjuvant nivolumab for high-risk
patients with stage III melanoma. Adjuvant pembrolizumab is also
approved for patients with stage III melanoma based on the results from
the Keynote-054 trial.1
Question 13.15 The correct answer is C.
T-VEC is a genetically modified herpes simplex virus, type 1 (HSV-1),
a highly lytic, double-stranded DNA virus. T-VEC is injected into

https://t.me/ALGRAWANY33
cutaneous, subcutaneous, or nodal melanoma sites. Local intralesional
therapy with T-VEC can induce durable clinical regressions, usually
with low systemic toxicity. T-VEC is designed to preferentially replicate
in tumors and produce the immune-stimulating cytokine granulocyte
macrophage colony-stimulating factor (GM-CSF). A randomized phase
III trial5 reported clinical responses after intralesional injection of
melanoma metastases with T-VEC, with improved clinical response
rates compared to control patients receiving GM-CSF alone. In that
study, there were regressions of treated and untreated lesions, with a
26% response rate (11% complete response) versus 6% with GM-CSF
control (1% complete response), with durable response rates of 16%
versus 2% (p < 0.0001) and a trend to better survival at interim analysis.
T-VEC produced a ≥50% decrease in 15% of uninjected measurable
visceral lesions, confirming some systemic effect of treatment. The most
common reported side effects were fatigue, nausea, chills, and local
injection site reactions. Studies of T-VEC in combination with immune
checkpoint inhibitors are ongoing.
Question 13.16 The correct answer is C.
In this patient with BRAF-mutated metastatic melanoma and history of
myasthenia gravis, the most appropriate treatment is targeted therapy
with dabrafenib and trametinib. Given her history of autoimmune
disease, checkpoint inhibitor therapy should not be the first line of
therapy in this patient. Myasthenia gravis can be exacerbated during
immunotherapy and can be life-threatening. The combination of
dabrafenib and trametinib is the most appropriate treatment from the
options listed. The COMBI-d phase III study led to the approval of the
combination of dabrafenib and trametinib in 2014 based on improved
response rates (69% vs. 53%, p = 0.0014), median PFS (11.0 vs. 8.8
months, p = 0.0004), and OS (25.1 vs. 18.7 months, p = 0.0107)
compared to dabrafenib alone.6
Question 13.17 The correct answer is D.
This patient is presenting with a rapidly progressive symptomatic
metastatic melanoma with a large mass in neck causing difficulty
swallowing and potentially with risk of airway compromise. From the
options listed, the combination of dabrafenib and trametinib is the most
appropriate initial treatment. Dabrafenib is a BRAF inhibitor, and
trametinib is an MEK inhibitor. MEK is downstream of BRAF in the
MAP kinase signaling pathway. In BRAF-mutated melanomas, the
combination of BRAF/MEK inhibitors demonstrated significant clinical
benefits including OS improvement over single-agent therapy.
BRAF/MEK inhibitors are associated with a high response rate of 70%
in patients with BRAF V600E/K mutations, and responses may be seen
in days to weeks after starting treatment. This approach is preferred to
immunotherapy (ipilimumab, nivolumab) if clinically needed for a rapid
response such as in this symptomatic patient. Imatinib is a tyrosine
kinase inhibitor active against BCR-ABL in chronic myelogenous
leukemia and mutated KIT in GI stromal tumors. A phase II study in
patients with KIT-mutated metastatic melanomas demonstrated a 23%
ORR with imatinib therapy. Unfortunately, most of these responses were
of limited duration.
Question 13.18 The correct answer is C.
A common toxicity of BRAF inhibition is cutaneous SCCs. BRAF
inhibitors are associated with cutaneous SCC, extreme photosensitivity,
and other dermatologic toxicities, which occur much less often with
concurrent MEK inhibitors. Vemurafenib was the first BRAF inhibitor
that demonstrated improved OS and PFS when compared to
chemotherapy. Overall, 38% of patients receiving vemurafenib required
dose modification due to adverse events. Skin complications were
frequently reported. Eighteen percent of vemurafenib-treated patients
developed cutaneous SCC or keratoacanthoma that required simple
excision, whereas 12% experienced grade 2 or 3 photosensitivity skin
reactions. Arthralgia was the most common (21%) noncutaneous side
effect. In another phase II trial, secondary skin lesions were detected in
26% of patients. Compared to vemurafenib, dabrafenib was associated
with less cutaneous SCC or keratoacanthoma (6%), and phototoxic
reactions were rare; however, pyrexia was more common (11%).
Regular dermatologic evaluation is recommended. Autoimmune colitis

https://t.me/ALGRAWANY33
and pneumonitis are immune-related adverse events of checkpoint
inhibitors. Lower extremity edema is a common side effect of imatinib.
Question 13.19 The correct answer is C.
This patient is on therapy with nivolumab for metastatic melanoma. The
most likely diagnosis for her respiratory symptoms is pneumonitis.
Pneumonitis is a relatively uncommon side effect of immune checkpoint
inhibitors. It is more common with the PD-1 inhibitors pembrolizumab
and nivolumab than with ipilimumab (around 2%–5%). The
combination of ipilimumab and nivolumab is associated with the highest
rate of pneumonitis (5%–10%). Although asymptomatic pneumonitis
may be observed and treatment continued, the presence of any
symptoms warrants interruption of immune checkpoint delivery and
initiation of steroid treatment. Oxygen requirement, dyspnea at rest, and
limitation of self-care activities of daily living (ADLs) are indications to
start high-dose corticosteroids, such as methylprednisolone. Consider
adding prophylactic antibiotics for opportunistic infections and
bronchoscopy with biopsy. The patient needs to be reassessed daily. If
not improving after 48 h or worsening, consider additional
immunosuppressive therapy, including infliximab, mycophenolate, or
immunoglobulins.
Question 13.20 The correct answer is A.
The great majority of uveal melanomas have mutually exclusive driver
mutations in the alpha subunits of G-protein–coupled receptors GNAQ
and GNA11. Cutaneous melanomas have high rates of BRAF (40%–
50%), NRAS (20%), or NF1 (15%) mutations. Mucosal and
acrolentigenous melanomas, with low rates of UV radiation exposure,
have lower rates of BRAF mutations (5%–20%) and a relatively higher
rate of KIT mutations (5%–10%).
Question 13.21 The correct answer is B.
For unresectable and metastatic MCC, immune checkpoint inhibitor
therapy with a PD-1/PD-L1 inhibitor is now the standard first-line
therapy. The PD-1 inhibitor pembrolizumab showed a response rate of
56% and 69% survival rate at 24 months in a phase II clinical trial.
Avelumab, pembrolizumab, and nivolumab are checkpoint inhibitors
included as systemic treatment options for MCC. Although response
rates are relatively high with platinum-based chemotherapy, relapses
inevitable occur. Chemotherapy regimens are now considered second
line in this population.
Question 13.22 The correct answer is B.
Studies of BCC have indicated an association with mutations in the
PTCH1 regulatory gene. Loss of heterozygosity at this site is seen in
both sporadic and hereditary BCC. The PTCH1 protein is part of a
receptor complex that regulates the hedgehog (Hh) signaling pathway, a
key regulator of embryonic development and cellular proliferation.
Loss-of-function mutations in PTCH1 allow unopposed smoothened
activity and cellular proliferation.
Question 13.23 The correct answer is A.
For locally advanced BCC or metastatic BCC, systemic therapy with an
Hh pathway inhibitor (vismodegib or sonidegib) is recommended.
Vismodegib is an oral small molecule inhibitor of smoothened homolog
(SMO), a downstream target of PTCH1, and has significant clinical
activity because most BCCs depend on the activation of the Hh
pathway. In a study involving 63 patients with locally advanced BCC
and 33 patients with metastatic BCC, vismodegib 150 mg once daily
was associated with objective responses in 43% of patients with locally
advanced BCC and 21% of patients with locally advanced disease had
complete clinical resolution of tumors.7 Adverse events in the trial were
common, including serious adverse events in 25% of patients and fatal
adverse events in 7% of patients. The most common adverse events in
this and other trials were muscle spasms, alopecia, dysgeusia leading to
weight loss, fatigue, diarrhea, and hyponatremia, and between 12% and
54% of patients discontinued therapy because of adverse effects.
Question 13.24 The correct answer is C.
Cemiplimab-rwlc is now approved for patients with metastatic or locally

https://t.me/ALGRAWANY33
advanced cutaneous SCC. In a phase II study enrolling patients with
metastatic or locally advanced cutaneous SCC and treated with
cemiplimab-rwlc, the ORR was 47%, with 61% of responses durable for
6 months or longer. In the second-line setting, patients with advanced
disease may benefit from cetuximab, the anti-EGFR mAb, which has
been studied in patients with advanced SCC showing modest clinical
activity. AK lesions are precursors of cutaneous SCC, and several
pathologic factors are associated with poor prognosis, including
perineural invasion, positive surgical margins, and regional nodal
metastases.

References
1. Eggermont AMM, Blank CU, Mandala M, et al. Adjuvant pembrolizumab versus placebo in
resected stage III melanoma. N Engl J Med. 2018;378:1789–1801.
2. Weber J, Mandala M, Del Vecchio M, et al. Adjuvant nivolumab versus ipilimumab in resected
stage III or IV melanoma. N Engl J Med. 2017;377:1824–1835.
3. Hodi FS, Chiarion-Sileni V, Gonzalez R, et al. Nivolumab plus ipilimumab or nivolumab alone
versus ipilimumab alone in advanced melanoma (CheckMate 067): 4-year outcomes of a
multicentre, randomised, phase 3 trial. Lancet Oncol. 2018;19:1480–1492.
4. Tawbi HA, Forsyth PA, Algazi A, et al. Combined nivolumab and ipilimumab in melanoma
metastatic to the brain. N Engl J Med. 2018;379:722–730.
5. Andtbacka RHI, Collichio FA, Amatruda T, et al. OPTiM: a randomized phase III trial of
talimogene laherparepvec (T-VEC) versus subcutaneous (SC) granulocyte-macrophage colony-
stimulating factor (GM-CSF) for the treatment (tx) of unresected stage IIIB/C and IV melanoma. J
Clin Oncol. 2013;31:LBA9008.
6. Long GV, Stroyakovsky DL, Gogas H, et al. COMBI-d: a randomized, double-blinded, phase III
study comparing the combination of dabrafenib and trametinib to dabrafenib and trametinib placebo
as first-line therapy in patients (pts) with unresectable or metastatic BRAFV600E/K mutation-
positive cutaneous melanoma. J Clin Oncol. 2014;32:9011.
7. Sekulic A, Migden MR, Oro AE, et al. Efficacy and safety of vismodegib in advanced basal-cell
carcinoma. N Engl J Med. 2012;366:2171–2179.
___________
Corresponding chapters in DeVita, Hellman, and Rosenberg’s Cancer: Principles & Practice of
Oncology, Eleventh Edition: 90 (Cancer of the Skin), 91 (Molecular Biology of Cutaneous Melanoma),
and 92 (Cutaneous Melanoma).
14 Sarcoma
Brian A. Van Tine and Angela C. Hirbe

QUESTIONS

Each of the numbered items below is followed by lettered answers. Select the
ONE lettered answer that is BEST in each case unless instructed otherwise.

Question 14.1 Which of the following inherited syndromes is associated


with the highest risk for the development of soft-tissue sarcoma?
A. Hereditary retinoblastoma
B. Li–Fraumeni syndrome
C. Neurofibromatosis type I
D. All of the above
Question 14.2 Which of the following is TRUE about radiation-induced
sarcomas?
A. Cancer history usually includes breast cancer, lymphoma, and cervical
cancer.
B. They usually occur 10–30 years after radiation exposure.
C. Osteogenic sarcoma, undifferentiated pleomorphic sarcoma (UPS),
angiosarcoma, and lymphangiosarcoma are the usual histologic
subtypes.
D. All of the above.
Question 14.3 Which clonal cytogenetic abnormality is associated with the
CORRECT sarcoma subtype?
A. Ewing sarcoma and t(11;22)(q24;q12)

https://t.me/ALGRAWANY33
B. Synovial sarcoma and t(12;16)(q13;p11)
C. Myxoid liposarcoma and t(X;18)(p11;q11)
D. Alveolar rhabdomyosarcoma and t(17;22)(q22;q13)
Question 14.4 A 20-year-old man presents with right knee pain. X-ray
reveals a “sun-burst” appearance in the distal femur. Biopsy reveals high-
grade osteosarcoma. No distant metastases are identified. Which of the
following is the most appropriate treatment?
A. Limb-sparing resection
B. Limb-sparing resection and adjuvant chemotherapy
C. Definitive radiation
D. Neoadjuvant chemotherapy, limb-sparing resection, and adjuvant
chemotherapy
Question 14.5 In addition to site, which of the following variables is used
to estimate the risk of sarcoma-specific death for a patient?
A. Tumor grade and histology
B. Tumor size and depth
C. Age
D. All of the above
Question 14.6 Which of the following is TRUE regarding the staging of
soft-tissue sarcomas?
A. Staging involves a four-grade system.
B. Stage IV includes N1 disease.
C. High-grade tumors can be classified as stage I depending on the size of
the tumor.
D. Histologic grade, size, depth, and the presence or absence of nodal and
distant metastases are variables used to determine tumor stage.
Question 14.7 A 52-year-old man underwent resection of a 3-cm mass
from the lateral left thigh. Pathology revealed a high-grade leiomyosarcoma,
and the lateral surgical margin was positive. The most appropriate next step
in the treatment of this patient’s cancer would be:
A. Radiation.
B. Adjuvant chemotherapy.
C. Reresection.
D. Observation.
Question 14.8 A 68-year-old woman presented with a purplish nodular
lesion in the occipital scalp. Resection revealed an angiosarcoma measuring 3
cm. Surgical margins were negative. What is the most appropriate next step
in the treatment?
A. Radiologic imaging to look for nodal metastases and referral for
adjuvant radiation
B. Monitoring
C. Adjuvant chemotherapy with an anthracycline
D. Adjuvant chemotherapy with paclitaxel
Question 14.9 A 65-year-old woman presented with abdominal pain and
iron-deficiency anemia. Workup revealed a gastric mass and multiple large
intra-abdominal masses and liver hypodensities. Biopsy of the gastric mass
revealed a spindle cell neoplasm thought to be a leiomyosarcoma. After three
cycles of doxorubicin and ifosfamide, imaging showed disease progression.
The appropriate next step in the management of this patient’s cancer would
be:
A. Docetaxel and gemcitabine.
B. Dacarbazine.
C. Request the pathologist to perform a CD117 (c-Kit)
immunohistochemistry (IHC) with reflex sequencing.
D. Palliative radiation.
Question 14.10 A 55-year-old man presented with a 10-cm mass in the
medial left thigh. Biopsy revealed a high-grade liposarcoma. Imaging
revealed no evidence of distant metastases. What is the most appropriate
treatment?

https://t.me/ALGRAWANY33
A. Definitive radiation
B. Definitive radiation and concurrent doxorubicin
C. Limb-sparing resection followed by adjuvant radiation
D. Preoperative chemotherapy followed by resection
Question 14.11 A 52-year-old man with metastatic unresectable
gastrointestinal stromal tumor (GIST) was treated with imatinib (400
mg/day). Imaging showed initial disease response. However, the disease
progressed after 28 months on therapy. The most appropriate treatment would
be:
A. Erlotinib.
B. Sunitinib.
C. Doxorubicin.
D. Imatinib 800 mg/day.
Question 14.12 Which of the following factors are independent predictors
of poorer disease-specific survival in patients with nonmetastatic soft-tissue
sarcoma?
A. Large tumor size (>10 cm)
B. High-grade histology
C. Older age (>60 years)
D. All of the above
Question 14.13 A 48-year-old woman underwent complete resection of a
9-cm high-grade leiomyosarcoma arising in the lower extremity.
Postoperative adjuvant radiation was administered. Two years later, a chest
computed tomography (CT) scan revealed a new single 3-cm, round,
noncalcified pulmonary nodule. What is the most appropriate next treatment?
A. Complete resection of the lung nodule
B. Radiation
C. Ifosfamide and doxorubicin
D. Docetaxel and gemcitabine
Question 14.14 Which of the following is TRUE about patients with
metastatic or locally recurrent soft-tissue sarcoma?
A. Median survival is 12 months, although 20%–25% of patients are alive
2 years after diagnosis.
B. Complete resection of oligometastases to the lung never results in long-
term survival.
C. Radiation is the preferred treatment of a locally recurrent sarcoma.
D. Combination chemotherapy improves overall survival compared with
single-agent chemotherapy.
Question 14.15 Which of the following is TRUE about chemotherapy
treatment of metastatic soft-tissue sarcoma?
A. Escalating the dose of doxorubicin or ifosfamide does not improve
tumor response rates over standard doses of these agents.
B. Leiomyosarcoma is uniquely sensitive to ifosfamide, whereas synovial
sarcoma is not.
C. Paclitaxel shows broad-spectrum activity.
D. Dacarbazine has modest activity.
Question 14.16 A 30-year-old man presents with a permeative bone tumor
in the distal femur. Open biopsy reveals a UPS. Radiologic imaging does not
find distant metastases. What is the most appropriate treatment?
A. Limb-sparing resection with wide margins
B. Definitive radiation
C. Chemotherapy
D. Preoperative chemotherapy, limb-sparing resection, and adjuvant
chemotherapy
Question 14.17 Which of the following is TRUE regarding osteosarcoma?
A. Approximately 20% of patients with localized high-grade disease
treated with resection and chemotherapy remain disease free 5 years
later.
B. Approximately 60%–80% of patients with localized high-grade disease

https://t.me/ALGRAWANY33
treated with resection and chemotherapy remain disease free 5 years
later.
C. Parosteal (low-grade cortical) osteosarcoma is best treated with
resection and chemotherapy.
D. Periosteal osteosarcoma is best treated with radiation therapy.
Question 14.18 A 45-year-old man presents with a left-sided pelvic pain.
CT reveals a 5-cm mass with appearance of chondroid matrix, arising from
the left side of the pelvic girdle. Bone biopsy reveals chondrosarcoma. Which
of the following is TRUE regarding chondrosarcomas?
A. Most are low-grade tumors.
B. Most are treated with resection and radiation therapy.
C. Children have a better prognosis than adults.
D. Adjuvant chemotherapy has no role in the management of
nonmetastatic disease.
Question 14.19 A 22-year-old woman presents with a giant cell tumor
(GCT) of the distal femur. Appropriate treatment would be:
A. Curettage and debridement.
B. Amputation.
C. Radiation.
D. Preoperative chemotherapy, resection, and adjuvant chemotherapy.
Question 14.20 A 16-year-old female patient presents with a painful
rapidly growing scapular mass. Core needle biopsy reveals Ewing sarcoma.
Staging evaluation shows no evidence of metastatic disease. The most
appropriate therapy is:
A. Resection.
B. Preoperative chemotherapy with vincristine, doxorubicin, and
cyclophosphamide (VAC) alternating with ifosfamide and etoposide
(IE); resection; and adjuvant chemotherapy.
C. Resection and radiation.
D. Preoperative chemotherapy with VAC; resection; and adjuvant
chemotherapy.
Question 14.21 A 25-year-old woman presents with a painless right thigh
mass, which has been slowly growing over a span of 8 years. Core needle
biopsy reveals alveolar soft-part sarcoma. What cytogenetic abnormality
would you expect to see in this tumor?
A. t(11;22)(q24;q12)
B. t(12;16)(q13;p11)
C. der(17)t(X;17)(p11;q25)
D. t(X;18)(p11;q11)
Question 14.22 Which of the following is used to treat alveolar soft-part
sarcoma?
A. Imatinib
B. Sunitinib
C. Dasatinib
D. Nilotinib
Question 14.23 Which of the following sites of soft-tissue sarcomas carries
the best prognosis?
A. Head and neck
B. Extremity
C. Visceral
D. Retroperitoneal
Question 14.24 A 60-year-old woman with advanced soft-tissue sarcoma is
on third-line therapy with pazopanib and develops jaundice. Which of the
following should be done next?
A. Abdominal imaging
B. Laboratory testing
C. Dose reduction
D. Immediately discontinue pazopanib

https://t.me/ALGRAWANY33
Question 14.25 What is the mechanism of action of mesna?
A. Binds and detoxifies ifosfamide metabolites in the kidney and bladder
B. Coats the bladder
C. Inhibition of monoamine oxidase (reducing further chloroacetaldehyde
formation)
D. Rescue impaired hepatic metabolic pathways by acting as an electron
acceptor

ANSWERS

Question 14.1 The correct answer is D.


Patients with Li–Fraumeni syndrome have an increased risk for
developing soft-tissue sarcomas. Neurofibromatosis type I increases the
risk for the development of malignant peripheral nerve sheath tumors.
Patients with retinoblastoma also have an increased risk for developing
soft-tissue and bone sarcomas.
Question 14.2 The correct answer is D.
Radiation-induced sarcomas usually occur 10 to 30 years after exposure
to radiation. The most common cancers in this category include breast,
cervical cancer, and lymphomas. Most radiation-induced sarcomas are
osteogenic sarcoma or UPS, and the prognosis is poor.
Question 14.3 The correct answer is A.
Ewing sarcoma is associated with five known cytogenetic abnormalities,
including t(11;22)(q24;q12). Synovial sarcoma is associated with
t(X;18)(p11;q11), whereas myxoid liposarcoma is associated with
t(12;16)(q13;p11). Alveolar rhabdomyosarcoma is associated with
t(2;13)(q35;q14).
Question 14.4 The correct answer is D.
The current standard of care for the treatment of osteosarcomas is
neoadjuvant chemotherapy, followed by limb-sparing resection and
adjuvant chemotherapy. The percentage of tumor cell necrosis on
pathology from the resection hints to prognosis and helps determine the
adjuvant chemotherapy regimen. Surgery alone or definitive radiation
offers a lower chance of success.
Question 14.5 The correct answer is D.
The Memorial Sloan Kettering Cancer Center (MSKCC) nomogram for
sarcoma-specific mortality incorporates the following variables: age,
tumor size, depth, histology, grade of tumor, and site. Zero points are
assigned for the low-risk features (age 16, size ≤5 cm, superficial depth,
fibrosarcoma histology, and low-grade and upper extremity site). A sum
of the points assigned for each variable helps predict the risk of
sarcoma-specific death.
Question 14.6 The correct answer is B.
Staging of soft-tissue sarcoma requires knowledge of histologic grade,
size, depth, and the presence of metastatic disease and is based on a
three-grade system. N1 disease is considered stage IV disease. Stage I
disease includes only low-grade tumors. High-grade tumors are never
stage I.
Question 14.7 The correct answer is C.
Positive surgical margin predicts a higher risk of local recurrence.
Repeated resection in an attempt to achieve negative surgical margins
would be the next most appropriate step in the management of this
patient. If negative surgical margins are achieved after reresection,
adjuvant radiation would not be required. Adjuvant chemotherapy would
have no clear benefit in this case.
Question 14.8 The correct answer is A.
Soft-tissue sarcoma of the head and neck is associated with a poor
prognosis. Angiosarcoma has a higher risk of nodal metastases and local
recurrence after resection alone. Adjuvant radiation reduces the risk of
local and regional disease recurrence. Although angiosarcomas tend to

https://t.me/ALGRAWANY33
be sensitive to anthracyclines and taxanes, there would be no known
benefit of adjuvant chemotherapy in this case.
Question 14.9 The correct answer is C.
Intra-abdominal leiomyosarcomas and GISTs have similar
characteristics by light microscopy. However, GISTs stain positive for
the CD117 (c-Kit) protein, whereas leiomyosarcomas usually do not.
Leiomyosarcomas frequently respond to chemotherapy, whereas GISTs
do not. However, GISTs are uniquely responsive to c-Kit inhibitors. In
this case, a c-Kit immunostain should be requested to determine whether
the sarcoma is a GIST and not a leiomyosarcoma. Reflex sequencing
should be obtained, as certain mutations respond better than others to
therapy and some GISTs do not have c-Kit or PDGFRA mutations,
predicting a lack of response to the c-Kit inhibitors and indicating the
consideration for a clinical trial. Because this patient has metastatic
disease, systemic therapy would be favored over palliative radiation.
Question 14.10 The correct answer is C.
Resection of the primary tumor with intent to achieve negative surgical
margins followed by adjuvant radiation would be the most appropriate
treatment of a large high-grade extremity liposarcoma. Definitive
radiation with or without concurrent chemotherapy would be less
effective treatment. Preoperative chemotherapy may be used when the
tumor is marginally resectable, but its role in this situation is
controversial.
Question 14.11 The correct answer is D.
Studies have demonstrated that approximately one-third of patients with
GIST resistant to imatinib 400 mg/day will benefit with increasing the
dose of imatinib to 800 mg/day. Should that step fail, sunitinib would be
the next most appropriate therapy. GIST is generally resistant to
chemotherapy drugs, such as doxorubicin. Epidermal growth factor
receptor inhibitors, such as erlotinib, would not be expected to benefit
patients with GIST.
Question 14.12 The correct answer is D.
Independent predictors of prognosis of patients with nonmetastatic soft-
tissue sarcoma include age, histologic subtype, histologic grade, tumor
site, and tumor size. Factors predictive of better disease-specific survival
include younger age, selected histologic subtypes (fibrosarcoma), low
histologic grade, smaller (<5 cm) tumor size, and extremity location.
Nomograms are available that accurately predict the prognosis of
patients with nonmetastatic soft-tissue sarcoma treated with surgery with
or without adjuvant radiation.
Question 14.13 The correct answer is A.
In this patient, the most likely cause of the new lung nodule is a
metastasis from the leiomyosarcoma. Retrospective series have observed
that 20% to 30% of such patients who undergo complete resection of the
lung metastases were alive 5 years later. Factors that predict for a lower
success rate with resection in such patients include more than four lung
nodules, bilateral lung nodules, and a disease-free interval of <12
months. Chemotherapy and radiation provide only palliative intent in
this case.
Question 14.14 The correct answer is A.
For locally recurrent sarcoma without distant metastases, reresection is
the preferred treatment. For patients with metastatic sarcoma, median
survival is 1 year. However, a subset of these patients live for >2 years.
It is important to consider complete resection of oligometastases to the
lung because ~25% of those patients will be alive 5 years later. Nearly
all randomized trials have shown no overall survival benefit with
combination chemotherapy compared to single-agent chemotherapy,
although tumor response rates were consistently higher.
Question 14.15 The correct answer is D.
Several studies have shown higher tumor response rates, but not
improved survival, with larger doses of doxorubicin and ifosfamide,
although both are associated with increased toxicity. Ifosfamide is an

https://t.me/ALGRAWANY33
active agent in synovial sarcoma but has little activity in
leiomyosarcoma. Paclitaxel appears to only be effective in the treatment
of angiosarcoma and Kaposi sarcoma. Perhaps, the first drug identified
to be active in adult soft-tissue sarcoma, dacarbazine, resulted in tumor
response rates of 10% to 30%.
Question 14.16 The correct answer is D.
In contrast with UPS of soft tissue, which is primarily treated with
resection, primary tumor of the bone is treated like high-grade
osteosarcoma with preoperative chemotherapy, limb-sparing resection,
and adjuvant chemotherapy. Outcomes of patients with UPS of bone and
with osteosarcoma are similar when treated this way. Limb-sparing
resection alone or definitive radiation would offer lower chances of
success. Chemotherapy alone would provide palliation, but the chance
of cure is unclear.
Question 14.17 The correct answer is B.
Parosteal osteosarcoma is usually a low-grade tumor that involves the
bone cortex and is associated with a better prognosis than classic
osteosarcoma. Resection is the preferred treatment, and there is no clear
role for chemotherapy. In contrast, periosteal osteosarcoma is an
aggressive, high-grade cortical sarcoma that is best treated like classic
osteosarcoma with resection and chemotherapy. For classic high-grade
osteosarcoma, resection alone is curative in 20%, but the addition of
chemotherapy to resection improves cure rates to 60% to 80%.
Question 14.18 The correct answer is A.
Most chondrosarcomas are low grade and usually occur in patients aged
>40 years. The preferred therapy for chondrosarcoma is resection.
Although chondrosarcomas as a group are known to be resistant to
chemotherapy, mesenchymal and dedifferentiated chondrosarcomas
have a high metastatic potential and appear to benefit from adjuvant
chemotherapy. Children have a worse prognosis than adults.
Question 14.19 The correct answer is A.
GCT of bone is a tumor with a high risk for local recurrence but a low
risk for distal metastases. Appropriate treatment is resection, preferably
by curettage and debridement. Amputation is reserved for bulky local
recurrences. Radiation is reserved for lesions of the spine that can lead
to cord compression. There is no benefit of preoperative or adjuvant
chemotherapy for GCT of bone. There is a role for denosumab for
patients with unresectable GCT.
Question 14.20 The correct answer is B.
A randomized trial of patients with Ewing sarcoma treated with either
VAC alone or alternating with IE given as preoperative and adjuvant
chemotherapy clearly showed an overall survival and progression-free
survival advantage with the alternating regimen of VAC and IE versus
VAC alone in patients with nonmetastatic disease. Resection alone or
with radiation would be significantly less effective than when used in
combination with chemotherapy.
Question 14.21 The correct answer is C.
Alveolar soft-part sarcoma is associated with der(17)t(X;17)(p11;q25).
Ewing sarcoma is associated with five known cytogenetic abnormalities,
including t(11;22)(q24;q12). Synovial sarcoma is associated with
t(X;18)(p11;q11), whereas myxoid liposarcoma is associated with
t(12;16)(q13;p11).
Question 14.22 The correct answer is B.
Sunitinib is an oral small molecule, multitargeted receptor tyrosine
kinase inhibitor that is used in the treatment of alveolar soft-part
sarcoma. Imatinib, dasatinib, and nilotinib are tyrosine kinase inhibitors
that have activity against GIST. Sunitinib is also used to treat GIST.
Question 14.23 The correct answer is B.
Extremity soft-tissue sarcomas have the best prognosis, followed by
visceral and retroperitoneal. Head and neck soft-tissue sarcomas carry
the worst prognosis.

https://t.me/ALGRAWANY33
Question 14.24 The correct answer is D.
Pazopanib has a black box warning for severe and fatal hepatotoxicity.
This can occur at any time during treatment. Liver enzymes should be
followed on weeks 3, 5, 7, and 9, then at months 3 and 4 and as
clinically indicated. Patients should be counseled to immediately contact
their physician and stop the medication should they notice jaundice.
Question 14.25 The correct answer is A.
Mesna functions to reduce the risk of hemorrhagic cystitis by binding
and detoxifying ifosfamide metabolites in the kidney and bladder. The
two proposed mechanisms of action for methylene blue as a treatment of
ifosfamide-induced neurotoxicity include inhibition of monoamine
oxidase by reducing further chloroacetaldehyde formation and rescuing
impaired hepatic metabolic pathways by acting as an electron acceptor.
___________
Corresponding chapters in DeVita, Hellman, and Rosenberg’s Cancer: Principles & Practice of
Oncology, Eleventh Edition: 87 (Molecular Biology of Sarcomas), 88 (Soft Tissue Sarcoma), and 89
(Sarcomas of Bone).
15 Endocrine and Neuroendocrine Tumor
Nikolaos A. Trikalinos

QUESTIONS

Each of the numbered items below is followed by lettered answers. Select the
ONE lettered answer that is BEST in each case unless instructed otherwise.

Question 15.1 Which of the following is TRUE for medullary thyroid


cancer (MTC)?
A. Familial types can be associated with MEN2A and MEN2B and gain-
of-function mutations in the RET proto-oncogene.
B. Tumors might appear solid or hypoechoic on ultrasound (US) and
show microcalcifications.
C. Calcitonin can help diagnose and follow the disease process.
D. All of the above.
Question 15.2 An 82-year-old female is diagnosed with pancreatic
neuroendocrine tumor (NET). On presentation, she has multiple liver lesions,
biopsied and consistent with well-differentiated grade 2 tumor. She is started
on octreotide and does well for about 16 months, when the tumor progresses.
You decide to place her on everolimus. Given her advanced age, she is
hesitant to take a “chemotherapy pill.” Which option is true regarding
everolimus for the treatment of neuroendocrine neoplasms?
A. Prolongs progression-free survival and overall survival
B. Prolongs overall survival
C. Is not effective in neuroendocrine neoplasms of small bowel and lung
D. Can be associated with stomatitis, rash, hyperglycemia, and cytopenias

https://t.me/ALGRAWANY33
Question 15.3 A 48-year-old active male is diagnosed with metastatic low-
grade ileal neuroendocrine neoplasm to the liver and undergoes treatment
with octreotide for 2 years, with occasional liver-directed therapy for
symptom control. He retains a good functional status but ultimately develops
systemic disease, including lung and bone lesions. Somatostatin receptor
positron emission tomography/computed tomography (PET/CT) scan
confirms somatostatin receptor positivity in all lesions. He undergoes
treatment with lutetium-177 (177Lu)-Dotatate for four infusions, followed by
observation. What are the most common immediate side effects he can expect
during treatment with peptide receptor radionuclide therapy (PRRT)?
A. Nausea
B. Transient cytopenias
C. Abnormal liver function tests
D. All of the above
Question 15.4 The same patient in Question 15.3 presents to your office 4
years after completing PRRT. He feels well in general but is more tired than
usual and looks pale. Imaging shows no splenomegaly or cirrhosis and no
major disease growth or new lesions. He has not been on any new
medications and is not abusing alcohol. A complete blood count (CBC) is as
follows:
WBC 1.2 thousand/µL

Hgb 9 g/dL

Mean cell volume (MCV) 112 fL

Plt 45 thousand/µL

Absolute neutrophil count (ANC) 600

Comments on peripheral smear: Basophilic stippling and macrocytosis.


Comprehensive metabolic panel (CMP) is within normal limits.
What should you do next?
A. Advise him not to drink.
B. Check iron, thyroid-stimulating hormone (TSH), and B12 levels; if
normal, perform a bone marrow biopsy.
C. Transfuse packed red blood cells and platelets.
D. Switch to everolimus.
Question 15.5 A 68-year-old male complains of intermittent abdominal
pain, and a contrast-enhanced CT of the abdomen shows a calcified
mesenteric mass and hypodense liver lesions. Biopsy of the mesenteric mass
shows neuroendocrine neoplasm, well differentiated, with a Ki-67 of 2.4%.
His oncologist suggests a liver biopsy and a Gallium Dotatate (Ga-Dotatate)
PET/CT scan to properly stage him. The mesenteric mass is Dotatate avid,
but the liver lesions are not. Which of the following is CORRECT about
somatostatin receptor imaging?
A. A somatostatin receptor PET/CT, such as a Ga-Dotatate PET/CT scan,
is less sensitive than an octreotide scan.
B. Somatostatin receptor imaging can identify patients who might benefit
from PRRT.
C. A “cold” liver lesion on Ga-Dotatate PET/CT means there is no cancer.
D. A “hot” spot on Ga-Dotatate PET/CT is always a neuroendocrine
neoplasm, because this is a very specific test for these tumors.
Question 15.6 Which statement is true about pheochromocytoma (PC) and
paraganglioma (PGL)?
A. They can be associated with mutations in the BRCA and RB gene.
B. Familial syndromes can explain in up to 30% of patients with PC.
C. Mitotane is a useful agent against PC, but not PGL.
D. Surgery should only be performed if it can cure the disease.
Question 15.7 A 53-year-old male is found to have a 2-cm left adrenal
lesion on abdominal CT scan performed for gallstones. He is otherwise
asymptomatic. The lesion measures 5 Hounsfield Units (HU) on unenhanced
CT, whereas magnetic resonance imaging (MRI) shows signal dropout. What
is your recommendation?

https://t.me/ALGRAWANY33
A. Urgent exploratory laparotomy with nephrectomy
B. Fine-needle biopsy
C. Hormonal evaluation and clinical follow-up with repeat imaging
D. Fluorodeoxyglucose (FDG) PET/CT
E. Somatostatin receptor PET/CT
Question 15.8 A 57-year-old patient with chronic gastroesophageal reflux
disease (GERD) and Barrett esophagus has a screening
esophagogastroduodenoscopy (EGD), showing multiple stomach polyps. A
biopsy reveals a well-differentiated neuroendocrine neoplasm, positive for
synaptophysin and chromogranin. A Ki-67 cannot be estimated because of
limited sample, but the pathology report mentions clean margins and a
background of Helicobacter pylori–negative gastritis with loss of gastric
glandular cells and hyperplasia of the antral/pyloric G cells. Which of the
following is CORRECT?
A. Laboratory workup will probably show high gastrin levels.
B. Patient has chronic reflux, which has caused both Barrett esophagus
and a gastric neuroendocrine neoplasm.
C. Patient most probably needs a partial gastrectomy with lymph node
sampling.
D. Patient needs a pancreas protocol CT.
Question 15.9 A 72-year-old female is diagnosed with metastatic
adrenocortical cancer (ACC) when she presents with significant abdominal
pain, ileus, and a blood pressure of 180/110. CT imaging shows multiple
pulmonary nodules with elevation of the serum and urine metanephrines. She
undergoes emergency surgery, followed by mitotane and prednisone
treatment. After 3 weeks on mitotane, she tells you that she is following a
high-fat diet to “starve the cancer.” Three weeks on the new diet, she starts
feeling lightheaded when standing up, nauseous, and now the blood pressure
is closer to 92/45. An emergent chemistry profile shows sodium of 129 and
potassium of 5.9. What is the next best step?
A. Check metanephrine levels.
B. Check mitotane levels.
C. Stop mitotane and re-evaluate in a week.
D. Start hydrocortisone and intravenous (IV) fluids.
Question 15.10 Which of the following is CORRECT regarding
parathyroid tumors?
A. Parathyroid cancer should be suspected in most cases of primary
hyperparathyroidism (PHPT).
B. Standard imaging can help distinguish benign from malignant
parathyroid cancers.
C. Most patients with parathyroid carcinoma die from hypercalcemia.
D. If surgery is not effective, chemotherapy and radiation can control the
disease for many years.
Question 15.11 A 63-year-old patient with metastatic pancreatic
neuroendocrine grade 1 neoplasm and severe flushing is offered long-acting
octreotide. He is worried about starting “chemotherapy” and asks you about
the possible side effects of this treatment. Which answer is most appropriate?
A. “You are not getting chemotherapy, so I would not be worried about
side effects.”
B. “Side effects of octreotide can include nausea, bloating, and
steatorrhea, as well as biliary sludge and gallstones.”
C. “Octreotide can help with the flushing, but it cannot stop the tumor
from growing.”
D. “Octreotide is not chemotherapy, but it can often help shrink your
tumor.”
Question 15.12 A patient with lung neuroendocrine neoplasm has been on
lanreotide for 2 years with good control of tumor growth, but over the past
few months, her chromogranin levels have tripled. A repeat CT of the chest
shows stability. Patient has no new symptoms, but has been recently placed
on amiodarone for arrhythmias and omeprazole for GERD. What is your next
step?
A. Refer the patient for PRRT.
B. Start the patient on everolimus.

https://t.me/ALGRAWANY33
C. Stop amiodarone for 4 days and repeat biochemical testing.
D. Stop omeprazole for 4 days and repeat biochemical testing.
Question 15.13 A patient you follow with carcinoid syndrome has been on
subcutaneous monthly octreotide. His primary is in the duodenum, he has
liver and lung metastases, and his symptomatology is mainly flushing with
intermittent wheezing, but no diarrhea. More recently, he has been
complaining of increased flushing right before his next monthly injection,
and he thinks that the wheezing happens also more frequently around that
time. Imaging shows lesion stability, but his serotonin and chromogranin
levels are slightly elevated. You suspect carcinoid-induced bronchospasm.
Which of the following is CORRECT?
A. Patient should try short-acting octreotide on the past few days of the
month for symptom control.
B. A beta-2 agonist might help with the symptoms until the new injection
is given.
C. Patient should try telotristat ethyl to improve his carcinoid
symptomatology.
D. Patient should be referred for PRRT, as he is progressing.
Question 15.14 Which of the following statements is CORRECT about
parathyroid carcinomas?
A. On presentation, most patients will already have metastases; therefore,
treatment is palliative.
B. Serum parathyroid hormone (PTH) and calcium levels can safely
distinguish parathyroid carcinoma from adenoma.
C. A dedicated neck CT and US showing a heterogeneous mass with
irregular borders is pathognomonic of parathyroid carcinoma.
D. Surgery, if possible, is usually more efficacious than chemotherapy,
embolization, and radiofrequency ablation.
Question 15.15 A 69-year-old female with a history of seizures develops
progressive hoarseness and a left-sided neck mass. Imaging shows a large
left-sided thyroid tumor with regional lymphadenopathy. Fine-needle
aspiration (FNA) is suspicious for cancer. She undergoes a total
thyroidectomy with lymph node dissection, showing a well-differentiated
papillary thyroid cancer measuring 4.5 cm with gross extension involving the
strap muscles (T3b N1), and CT of the chest, abdomen, and pelvis shows no
distant disease (M0). She is placed on thyroid hormone supplementation.
Four weeks after surgery, her thyroglobulin level is 12 ng/mL. She is offered
radioactive iodine (RAI) but is not sure she will benefit from it. Which
statement is CORRECT about her case?
A. There is about a 40% chance that this tumor was initiated by a BRAF
V600E mutation.
B. Patient should debate the usefulness of RAI because she has no
evidence of residual tumor.
C. In the case of RAI, a high-iodine diet can protect the normal tissues
from unintended effects of RAI.
D. During surveillance, TSH levels should be kept on the lower level of
normal (0.5–2 mIU/L) with the help of thyroid hormones to prevent
recurrence.
Question 15.16 The patient in Question 15.15 is followed on a suppressive
TSH regimen for 2 years before imaging identifies multiple pulmonary
metastases, with some easily seen through the airways on bronchoscopy.
Biopsy of one of these confirms the same well-differentiated papillary cancer,
and mutational analysis shows no NTRK or BRAF mutations. She is not
interested in a clinical trial and inquires about oral tyrosine kinase inhibitors
(TKIs), such as lenvatinib or sorafenib. Which answer is CORRECT?
A. She cannot be treated with a TKI because of the prior use of RAI.
B. She should be on lenvatinib, but not on sorafenib, because of the
seizure history.
C. She should not be on a TKI because of tumor airway invasion.
D. She should not be on a TKI because her tumor is lacking NTRK and
BRAF mutations.
Question 15.17 Which of the following statements is CORRECT about
follicular thyroid cancer (FTC)?

https://t.me/ALGRAWANY33
A. It is commonly associated with RET and BRAF mutations.
B. On pathology examination, cells show calcifications, Orphan-Annie
nuclei, and intranuclear pseudoinclusions.
C. Due to the subclinical nature of the disease, most patients present in
advanced stages (IV).
D. None of the above.
Question 15.18 A 40-year-old female with multiple endocrine neoplasia
type 1 (MEN1) syndrome is found to have a 1-cm pancreatic head nodule.
She undergoes an EGD with biopsy, showing a well-differentiated
neuroendocrine neoplasm with a Ki-67 of 2% (grade 1). A Ga-Dotatate
PET/CT scan shows avidity of the lesion with no evidence of distant
metastasis. Her performance status is excellent, and she has no evidence of
hormone hypersecretion. Which of the following is the most appropriate?
A. Patient needs an urgent FDG PET/CT to look for distant metastases.
B. Patient needs neoadjuvant therapy, followed by resection of the
PanNEN.
C. Patient needs resection, followed by adjuvant therapy with lanreotide.
D. Patient can be observed.
Question 15.19 A 28-year-old patient is referred to you from the primary
care physician because of the recent diagnosis of a small pancreatic
neuroendocrine neoplasm. He mentions that 5 years ago, he had a parathyroid
adenoma removed because of hypercalcemia. His father had a small-bowel
carcinoid and multiple renal stones, and his grandfather had a brain tumor.
You suspect a familial syndrome. Which of the following genes is most
probably mutated in his case?
A. RET
B. MEN1
C. CDNK1B
D. TP53
Question 15.20 A 64-year-old male with lupus and Hashimoto thyroiditis
develops hoarseness and a nontender, hard, right-sided neck lump that grows
despite antibiotics and doubles in size within a month. He does not have
fever, weight loss, or night sweats, but complains of some shortness of breath
and wheezing. TSH and calcitonin levels are appropriate (patient is on
thyroid supplementation). Imaging shows multiple lung nodules. What is the
diagnosis for this patient?
A. Nonfunctional goiter
B. Papillary thyroid cancer
C. MTC
D. Thyroid lymphoma

ANSWERS

Question 15.1 The correct answer is D.


MTC comprises about 2% of thyroid malignancies in the United States
and is not associated with exposure to radiation. Instead, MEN2A and
MEN2B familial syndromes are associated with a 95% to 100% chance
of development of MTC. MTC may appear solid and hypoechoic on US,
with some specimens having microcalcifications. MTC does not
concentrate iodine; therefore, iodine scans and RAI are not useful.
Prognosis for combined familial and sporadic MTC is favorable, with
recent studies showing a 5-year survival of 80% to 90% and a 10-year
survival of 70% to 80%. Measurement of serum calcitonin and CEA can
help monitor for residual disease.
Question 15.2 The correct answer is D.
Everolimus is an oral mammalian target of rapamycin (mTOR) inhibitor
that is approved for the treatment of small-bowel and pancreatic NETs.
It has also shown efficacy in NETs of lung origin in the RADIANT-4
study.1 A large phase III study randomized 429 patients with metastatic
NETs and evidence of carcinoid syndrome to treatment with everolimus
plus long-acting octreotide (octreotide LAR) versus placebo plus
octreotide LAR.2 The median progression-free survival was improved
from 11.3 months in the placebo arm to 16.4 months in the everolimus

https://t.me/ALGRAWANY33
arm. However, no study was able to show overall survival benefit for
patients treated with everolimus. Cytotoxic chemotherapy has not
produced any meaningful results in low-grade, small-bowel
neuroendocrine neoplasms but can have an effect in high-grade tumors.
Side effects of everolimus include oral aphthous ulcers, rash,
hyperglycemia, cytopenias, and pneumonitis. Most of these toxicities
require discontinuation or modification of the regimen, but oral ulcers
can respond to dexamethasone oral rinses.
Question 15.3 The correct answer is D.
PRRT has gained popularity in patients with low-grade, well-
differentiated NETs of ileal origin as a second line of treatment. In a
phase III study comparing four cycles of 177Lu-DOTA0, Tyr3 octreotate
given every 8 weeks followed by octreotide LAR 30 mg every 4 weeks
to octreotide LAR 60 mg, there was a significant improvement in the
median progression-free survival from PRRT for patients with midgut
NETs.3 The response rate was 18% in the 177Lu-Dotatate group versus
3% in the control group (p < 0.001). Patients on PRRT might experience
nausea depending on the amino acid infusion used for nephroprotection,
transient cytopenias, and fatigue.
Question 15.4 The correct answer is B.
The patient demonstrates peripheral blood abnormalities, including
macrocytosis, neutropenia, and thrombocytopenia, with basophilic
stippling a few years after treatment with radioisotopes. Getting a
peripheral smear, B12, thyroid, and iron studies is appropriate, and
ultimately, he might benefit from a bone marrow biopsy to rule out
myelodysplasia or leukemia. Myelodysplastic syndrome (MDS) or acute
leukemia can develop in a small subset of patients (<5%) years after
treatment with PRRT, and it should be considered in an otherwise
healthy patient who has no evidence of disease progression on imaging
but develops cytopenias. Everolimus would not be useful in this case, as
it can exacerbate cytopenias and the patient does not have evidence of
disease progression. The lack of major new findings on imaging such as
cirrhosis or splenomegaly, normal CMP, and suspicious smear review
argues against alcohol as the etiology for the hematologic findings.
Question 15.5 The correct answer is B.
Up to 80% of gastrointestinal (GI) NETs and some lung NETs will
express somatostatin receptors. Somatostatin receptor imaging, either
indium-111 (111In)-pentetreotide scintigraphy or Gallium-68 Dotatate
PET/CT scan, is frequently employed in their diagnostic workup.
Gallium-68 Dotatate PET/CT is more sensitive than an octreotide scan
due to its better discriminatory ability. However, despite its high
sensitivity and specificity (both close to 90%), it has its limitations and
can concentrate on normal tissue (such as thyroid) or areas of
inflammation. Demonstration of somatostatin receptors can identify
patients who will potentially respond to 177Lu-Dotatate. A “cold” lesion
on somatostatin receptor imaging does not exclude cancer, as high-grade
neuroendocrine neoplasms or other cancers routinely lack somatostatin
receptors. This patient has a Dotatate-avid primary neoplasm with a
Dotatate-negative liver lesion, and it would be wise to biopsy the liver to
exclude a more aggressive or irrelevant histology.
Question 15.6 The correct answer is B.
PC and their extra-adrenal counterparts, PGLs, can develop in 50% of
patients with MEN2 and MEN3. Up to 30% of patients with PC have
germline mutations associated with various familial syndromes,
including multiple endocrine neoplasia type 2 (MEN2), von Hippel–
Lindau disease (VHL), neurofibromatosis type 1 (NF1), and PC–
paraganglioma syndrome types 1 to 5 (PGL1–PGL5). Mitotane is an
adrenolytic agent used in hypercortisolism states and can treat patients
with adrenocortical tumors, but it has no established role in the
treatment of PCs. Surgery is the treatment of choice for PCs when cure
is considered. For all patients with American Joint Committee on Cancer
(AJCC) stage IV PC or PGL, debulking and metastasectomy may be
considered when there is resectable tumor burden in the context of
slowly progressive disease. Radiation treatment can alleviate local
symptomatology, such as pain or nerve impingement.

https://t.me/ALGRAWANY33
Question 15.7 The correct answer is C.
The patient most probably has a lipid-rich adenoma. For incidentally
identified, nonfunctioning adrenal masses, CT and MRI, as well as
biochemical testing can differentiate between an adrenocortical adenoma
(ACA) and an ACC. ACAs are usually <4 cm, lipid rich, and
homogeneous, with smooth borders, whereas large, heterogeneous
lesions with irregular margins and calcifications are strongly suggestive
of an ACC. The likelihood of malignancy in the setting of an adrenal
mass is <2% for lesions <4 cm, 2% to 6% for lesions 4 to 6 cm, and 25%
for lesions >6 cm. On unenhanced CT, a signal of <10 HU is indicative
of a lipid-rich ACA. Signal dropout on opposed-phase MRI can also be
used to estimate the lipid content of adrenal masses and hence
differentiate lipid-rich ACA from other lesions, but the tumors must be
homogeneous. FDG PET/CT can help characterize some tumors, not
well seen on cross-sectional imaging, but no specific criteria exist at this
time. A somatostatin receptor PET/CT scan has no known utility in the
diagnostic workup of adrenal masses.
Question 15.8 The correct answer is A.
This patient most probably has a completely resected type 1 gastric
neuroendocrine neoplasm associated with atrophic gastritis. In these
patients, elevated gastrin levels can induce hyperplasia of the
antral/pyloric G cells. There is no definite association between NETs,
Barrett esophagus, and chronic gastroesophageal reflux. Barrett
esophagus can predispose to esophageal adenocarcinoma. Gastric NETs
arise from the enterochromaffin-like cells of the stomach and are
categorized in three types. Type 1 gastric carcinoids are most common
(80%) and typically occur in a state of chronic atrophic gastritis that
results in achlorhydria and hypergastrinemia. They usually appear as
multiple small polypoid tumors and are treated with repeat endoscopic
examination, excision, and endoscopic mucosal resection of larger
tumors. Type 2 gastric carcinoid tumors are rare, comprising about 6%
of patients and occur in patients with MEN1 with Zollinger–Ellison
syndrome (ZES). Gastrin-producing tumors may be identified by
pancreatic protocol CT. Type 3 gastric NETs account for ~15% of cases
and occur sporadically without hypergastrinemia. They are solitary,
grow rapidly, and treatment often requires gastrectomy with lymph node
sampling.
Question 15.9 The correct answer is D.
The patient has chosen mitotane treatment for her metastatic ACC.
Mitotane has adrenolytic and steroidogenic inhibitory effects; therefore,
it requires an addition of a corticosteroid to the patient’s regimen. The
metabolism of fludrocortisone is also enhanced. Mitotane requires
careful monitoring because the therapeutic window is tight and
complications may be dose limiting. In this case, the patient is still
loading on mitotane and has switched to a high-fat diet, which might
increase mitotane absorption and thus cause elevated mitotane levels.
The hypotension and electrolyte abnormalities are consistent with
adrenal insufficiency, which should be treated with IV fluids and
corticosteroids. Checking metanephrine levels will not be helpful
because they can be decreased or be normal because of the mitotane
effect. The delay in the results for mitotane levels may prevent the
prompt management of a patient with acute symptoms.
Question 15.10 The correct answer is C.
While PHPT is a rather common disorder, parathyroid carcinomas are
rare causes of PHPT, with a prevalence of <1% of cases of PHPT and
0.005% of all solid malignancies. It is difficult to distinguish adenoma
from carcinoma by imaging, laboratory workup, and physical
examination. Sometimes, an extremely high calcium level (>14 mg/dL)
and a palpable neck mass raise the suspicion of a carcinoma. Most
carcinomas are detected during surgery for hyperparathyroidism, but
even by histopathology, diagnosis of parathyroid carcinoma is
challenging, and ultimately, malignancy is established by local invasion
or distant metastasis. Most patients with parathyroid cancer succumb to
uncontrollable hypercalcemia, not to massive tumor burden. Treatments
for hypercalcemia include vigorous hydration, bisphosphonates,
calcitonin, glucocorticoids, or, in resistant cases, hemodialysis.
Chemotherapy and radiation have minimal efficacy in patients with

https://t.me/ALGRAWANY33
parathyroid cancer, as the malignant cells are resistant to both
therapeutic modalities.
Question 15.11 The correct answer is B.
The patient has a hormone-producing metastatic pancreatic grade 1
neoplasm and is offered a somatostatin analog. Both octreotide and
lanreotide have the potential to inhibit tumor growth and ameliorate the
carcinoid syndrome in more than half of the patients. Somatostatin
analogs, in general, cannot induce an objective tumor response, although
this has been described in a minority of patients. The treatment is
tolerable, with most patients continuing it for months or years. Side
effects include local injection reactions, nausea, bloating, diarrhea,
biliary sludge, cholecystitis, and steatorrhea.
Question 15.12 The correct answer is D.
It is always possible that the patient has subclinical progression, but
imaging and symptoms are not consistent. Therefore, making the
decision to switch treatment to PRRT is not justified. Chromogranin
levels can oscillate with time and are not always indicative of disease
burden, although the three times increase in this patient is probably
significant. Falsely elevated serum levels of chromogranin A may occur
when patients take a commonly prescribed proton-pump inhibitors
(PPIs) or H2 receptor antagonists. A quick way to determine that is to
stop PPIs for at least 3 days and recheck chromogranin levels.
Amiodarone is not known to affect chromogranin levels and has a half-
life of >20 days, so stopping it for 4 days will not have any meaningful
effect.
Question 15.13 The correct answer is A.
Intermittent bronchial obstruction and wheezing can be seen in 10% of
patients with carcinoid syndrome, usually during episodes of flushing.
This patient seems to have otherwise good control of his symptoms for
most of the time, and there is no tumor growth on imaging. We can
assume that the long-acting formulation of octreotide loses its effect
around the time he is due for a new injection. These patients can be
bridged with short-acting octreotide. Carcinoid-associated wheezing
cannot be treated like asthma because the beta-2 agonists that are used to
treat asthma lead to additional release of biogenic amines and peptides.
Telotristat etiprate is an oral inhibitor of tryptophan hydroxylase, a key
enzyme in the synthesis of peripheral serotonin. A recent study
randomized carcinoid patients with four or more bowel movements a
day to receive telotristat etiprate or placebo as double-blind treatment.4
Treatment was associated with decrease in the serotonin metabolite 5-
hydroxyindoleacetic acid (5-HIAA) in urine and decreased bowel
movement frequency while improving self-reported relief of bowel-
related symptoms. However, there is no known use of telotristat etiprate
other than for carcinoid-associated diarrhea. PRRT is currently indicated
for patients with tumors of gastroenteropancreatic origin who have
progressed on somatostatin analogs and have demonstrable somatostatin
receptors on imaging. There is no evidence that this patient is currently
progressing.
Question 15.14 The correct answer is D.
Parathyroid carcinomas are rare tumors, which are usually localized or
locally invasive on presentation. Distal metastases occur in <2% of
newly diagnosed patients; therefore, surgical resection is the mainstay of
treatment. Presentation can be asymptomatic or related to symptoms of
PHPT. Although severe hypercalcemia and extremely high PTH, as well
as a palpable neck mass suggest the diagnosis, there is no single
pathognomonic laboratory or imaging test that can safely distinguish
adenoma from carcinoma and often the diagnosis is made during
surgery. Systemic chemotherapy, embolization, and radiofrequency
ablation produce inferior results to surgery but can be used in select
cases.
Question 15.15 The correct answer is A.
This patient has a left-sided locally advanced papillary thyroid cancer.
She underwent a completion thyroidectomy because of the tumor size
>4 cm and has lymph node involvement, placing her at a high risk for
recurrence. Moreover, she has postoperative thyroglobulin (Tg) levels of

https://t.me/ALGRAWANY33
>10 ng/mL and, therefore, will benefit from RAI treatment. Although
the patent needs thyroid hormone supplementation because of the total
thyroidectomy, a low-iodine diet is recommended 1 to 2 weeks before
scanning or ablative 131I therapy to enhance the uptake and retention of
radioiodine. During surveillance, TSH levels should be kept at <0.1
mIU/L in patients with high risk of recurrence. BRAF V600E mutation
is present in ~40% to 50% of patients with papillary thyroid carcinoma.
Question 15.16 The correct answer is C.
Patient has airway invasion, as seen on bronchoscopy (pulmonary
metastases invading the airways). A prospective study of 140 thyroid
cancer patients (56% medullary, 33% differentiated, and 11% poorly
differentiated) who were treated with a TKI found hemoptysis in nine
patients, which was associated with the presence of airway invasion,
poorly differentiated pathology, history of external beam radiation
therapy (EBRT), and thyroidectomy without neck dissection. Therefore,
an alternative treatment, ideally on a clinical trial, should be sought.
Prior RAI use is not a contraindication to TKI use, and there is no
association between sorafenib of lenvatinib and seizures. Some BRAF
V600E–mutated patients might benefit from dabrafenib, a BRAF
inhibitor. The combination of dabrafenib with the MEK inhibitor
trametinib has been approved for BRAF-mutated anaplastic thyroid
carcinoma. Rearrangements in NTRK1 have been found in about 5% of
papillary thyroid cancers, and the Food and Drug Administration (FDA)
has approved inhibitors such as larotrectinib for any tumor harboring
this mutation. However, neither BRAF nor NTRK mutational status
affects the decision to use lenvatinib or sorafenib.
Question 15.17 The correct answer is D.
Unlike papillary thyroid cancer, FTC is rarely associated with RET or
BRAF mutations but can demonstrate PPARg-PAX8 rearrangements and
RAS mutations, with the latter associated with a worse prognosis.
Papillary, not follicular, cancer cells show Orphan-Annie nuclei, nuclear
grooves, and intranuclear pseudoinclusions. Most tumors are well
differentiated and are usually associated with a good prognosis, and
most patients do not die of their disease. Most patients with FTC and
Hürthle cell carcinoma present with stage II disease, but up to 25%
might present with distant metastases.
Question 15.18 The correct answer is D.
The patient has a well-differentiated small nonfunctioning pancreatic
NET in the setting of MEN1 syndrome. The tumor expresses
somatostatin receptors, as seen on Ga-Dotatate PET/CT scan, and there
is no evidence of distant metastases. An FDG PET/CT can help stage a
pancreatic adenocarcinoma or a high-grade NET but will more often be
negative in low-grade NETs and would likely not be helpful in this case.
There is no established role for neoadjuvant therapy followed by surgery
in small pancreatic NETs, although surgery can be considered for
lesions >1 to 2 cm or growing tumors. Neoadjuvant chemotherapy can
also sometimes be employed in borderline resectable tumors, but the
data are insufficient to support this as a general practice. There is no role
for adjuvant treatment in the setting of optimally resected low-grade
NETs of the pancreas, in contrast to their adenocarcinoma counterparts.
Observation of small pancreatic NETs with dedicated pancreas imaging
is an option for motivated patients.
Question 15.19 The correct answer is B.
The patient most likely has MEN1. MEN1 prevalence is 2 per 100,000
in the general population, and symptoms can present from 5 to 81 years
of age. The MEN1 tumor suppressor gene is located in chromosome 11,
and its product, menin, plays a role in cell division, genomic stability,
and interaction with transcription regulators and histone modification
complexes. Patients with MEN1 can develop pituitary adenomas,
parathyroid adenomas, pancreatic neuroendocrine, and adrenocortical
tumors. In this case, patient has had a parathyroid adenoma, his father
had a small-bowel carcinoid, and his grandfather had a possible pituitary
tumor. All patients with an MEN1 mutation require annual biochemical
and radiologic screening for the detection of anterior pituitary tumors
and insulinomas starting at 5 years, for parathyroid tumors at 8 years,
and gastrinomas and foregut carcinoid tumors at 20 years. RET proto-

https://t.me/ALGRAWANY33
oncogene mutations are seen in MEN2A and MEN2B syndromes,
associated with parathyroid cancer but also MTC and PCs. MEN4
mutations are extremely rare and associated with CDKN1B mutation; the
presentation is MEN1 like. TP53 mutations are encountered in several
cancers and are not specific to pancreatic NETs. Germline TP53
mutations have, however, been associated with the Li–Fraumeni
syndrome and ACCs.
Question 15.20 The correct answer is D.
The patient most likely has an aggressive variety of thyroid lymphoma
in the setting of long-standing Hashimoto thyroiditis. Approximately
50% of patients diagnosed with primary thyroid lymphoma have
underlying Hashimoto thyroiditis. The rapid growth rate is uncommon
for nonfunctional goiter and the more common papillary cancer or FTC,
although exceptions can occur. Given the shortness of breath and
wheezing, the patient should be monitored for any signs of airway
compromise. A normal calcitonin level is not consistent with a diagnosis
of MTC. Treatment for thyroid lymphoma is urgent and usually includes
radiation and chemotherapy.

References
1. Yao JC, Fazio N, Singh S, et al. Everolimus for the treatment of advanced, non-functional
neuroendocrine tumours of the lung or gastrointestinal tract (RADIANT-4): a randomised, placebo-
controlled, phase 3 study. Lancet. 2016;387:968–977.
2. Pavel ME, Hainsworth JD, Baudin E, et al. Everolimus plus octreotide long-acting repeatable for
the treatment of advanced neuroendocrine tumours associated with carcinoid syndrome
(RADIANT-2): a randomised, placebo-controlled, phase 3 study. Lancet. 2011;378:2005–2012.
3. Strosberg J, El-Haddad G, Wolin E, et al. Phase 3 trial of 177Lu-dotatate for midgut neuroendocrine
tumors. N Engl J Med. 2017;376:125–135.
4. Kulke MH, Hörsch D, Caplin ME, et al. Telotristat ethyl, a tryptophan hydroxylase inhibitor for the
treatment of carcinoid syndrome. J Clin Oncol. 2017;35:14–23.
___________
Corresponding chapters in DeVita, Hellman, and Rosenberg’s Cancer: Principles & Practice of
Oncology, Eleventh Edition: 80 (Molecular Biology of Endocrine Tumors), 81 (Thyroid Tumors), 82
(Parathyroid Tumors), 83 (Adrenal Tumors), 84 (Pancreatic Neuroendocrine Tumors), 85 (Carcinoid
Tumors and the Carcinoid Syndrome), and 86 (Multiple Endocrine Neoplasia).
16 Gynecologic Cancer
Matthew A. Powel and David G. Mutch

QUESTIONS

Each of the numbered items below is followed by lettered answers. Select the
ONE lettered answer that is BEST in each case unless instructed otherwise.

Question 16.1 A 35-year-old premenopausal woman of Ashkenazi Jewish


ancestry presents to you because her 60-year-old paternal aunt was recently
diagnosed with ovarian cancer, and she is concerned about her own risk for
ovarian cancer. She has no siblings, and there is no other family history of
cancer. She has two living children and is in good health, and her pelvic
examination is unremarkable. You should advise her that:
A. Ovarian cancer cannot be inherited through the paternal side and that
she does not need any particular screening.
B. A single second-degree relative with cancer at age >50 years does not
confer a significantly increased risk for her.
C. She should visit a genetic counselor.
D. Annual transvaginal ultrasound and CA-125 screening can reduce her
risk of mortality.
E. She should have a prophylactic total abdominal hysterectomy/bilateral
salpingo-oophorectomy (TAH/BSO) if/when she does not want to have
any more children.
Question 16.2 A 35-year-old woman presents to you for recommendations
regarding therapy of her newly diagnosed mucinous ovarian cancer. This was
a 5-cm, grade 1, left-sided mass that was incidentally found at the time of
surgery for endometriosis as part of an infertility workup. The ovary was

https://t.me/ALGRAWANY33
removed, and the operative note states that there was no evidence of tumor on
the external surface of the ovary or elsewhere in the abdomen, but full
surgical staging was not performed. A postoperative computed tomography
(CT) scan of the abdomen and pelvis is unremarkable, and CA-125 is within
normal limits. Pelvic examination is unremarkable. The patient would like to
have children but does not want to compromise her survival. You should
advise her that:
A. She is unlikely to have any residual cancer or a recurrence, and further
surgery or chemotherapy is not needed.
B. She should have a positron emission tomography (PET) scan, and if
there is no uptake, she does not need further surgery or chemotherapy.
C. Because her CT scan and CA-125 are normal, she is unlikely to have
any residual disease, and further surgery is not needed. However,
because the mucinous subtype of ovarian cancer has a very poor
prognosis, she will require three to six cycles of carboplatin/paclitaxel
chemotherapy.
D. She should have complete surgical staging, if possible, via
laparoscopy, with the option of preserving her uterus and contralateral
ovary if there is no further evidence of tumor, which is associated with
a >90% chance of 5-year survival and does not require chemotherapy.
Question 16.3 CA-125 is most useful in what aspect of ovarian cancer
management?
A. Screening
B. Diagnosis
C. Monitoring treatment
D. All of the above
Question 16.4 Which of the following increases a woman’s risk for
developing ovarian cancer?
A. Use of oral contraceptives for >5 consecutive years
B. Nulliparity
C. Breastfeeding
D. Tubal ligation
Question 16.5 A 50-year-old woman presents with a pelvic mass. She is
found to have a tumor of low malignant potential. With TAH/BSO and
staging, there is no gross disease, but the washings were positive on final
pathology, and there was one positive lymph node, making her disease stage
IIIC. What is the most appropriate therapy postoperatively?
A. Intraperitoneal cisplatin and intravenous (IV) taxane
B. IV platinum and a taxane
C. Hormonal therapy with tamoxifen or an aromatase inhibitor
D. No further therapy
Question 16.6 A 50-year-old woman was diagnosed with stage III ovarian
cancer and underwent primary resection followed by chemotherapy. She is
asymptomatic but has an elevated CA-125. Imaging studies do not identify
recurrent disease. Which of the following is the best treatment option?
A. Hormonal therapy
B. Platinum-based chemotherapy
C. Single-agent doxorubicin
D. Single-agent paclitaxel
Question 16.7 Which of the following is a founder BRCA mutation
associated with individuals of Ashkenazi descent?
A. 185delAG
B. 5382insC
C. 617delT
D. All of the above
Question 16.8 A 25-year-old woman has her left ovary removed because of
an ovarian mass. The final pathology shows that this is a moderately
differentiated papillary serous cancer. There was no other evidence of disease
in the abdominal cavity. What is the likelihood that this patient has a positive
pelvic or para-aortic lymph node metastasis?

https://t.me/ALGRAWANY33
A. 5%
B. 15%
C. 30%
D. 50%
Question 16.9 A 55-year-old woman who has just achieved a complete
clinical remission with normal CT scan, pelvic examination, and CA-125
after six cycles of chemotherapy for stage IIIC optimally debulked serous
ovarian cancer presents to you for a second opinion regarding her prognosis
and treatment options at this point. She is in excellent general health and
tolerated chemotherapy well, except for some numbness in her fingers and
toes, which caused her treating oncologist to switch her treatment from
paclitaxel plus carboplatin to docetaxel plus carboplatin after cycle 3. You
should advise her that:
A. The risk of eventual relapse for an optimally debulked patient with a
complete clinical remission is ~30%. No therapy is proven to be of any
further survival benefit at this point.
B. She should have a PET scan. If residual disease is found, she should
have four to six cycles of a non–cross-resistant drug, such as topotecan.
C. Her risk of eventual relapse is ~70%. She should be offered
consolidation therapy with paclitaxel 175 mg/m2 every 3 weeks for 12
months with the expectation of a 30% improvement in survival.
D. Her risk of relapse is ~70%. No therapy is proven to be of any survival
benefit at this point.
Question 16.10 Which of the following statements is TRUE about
granulosa cell tumors of the ovary?
A. They usually occur in premenopausal women.
B. They are usually stage III/IV.
C. Survival of patients with stage I disease is generally good, but they
may relapse later.
D. Survival of patients with stage III/IV disease is favorable, and
chemotherapy is not necessary.
Question 16.11 Which of the following statements is TRUE about germ
cell tumors of the ovary?
A. They occur more often in younger women.
B. They are usually stage III/IV.
C. Appropriate therapy includes TAH/BSO/full surgical staging and
chemotherapy regimens similar to those used in male testicular cancer.
D. The chemotherapy will usually result in infertility.
Question 16.12 A 51-year-old woman presents to you for
recommendations regarding the treatment of her recurrent ovarian cancer.
She was optimally debulked for stage IIIC serous ovarian carcinoma and
completed six cycles of carboplatin plus paclitaxel 36 months ago with a
clinical complete remission. She now has recurrent ascites, which is
histologically positive for tumor compatible with her original primary. CT
scan shows peritoneal carcinomatosis and a pelvic mass. You should advise
her that:
A. Prognosis of recurrent ovarian cancer is poor. She may achieve short-
term benefit from chemotherapy, although hospice is a reasonable
option.
B. Tamoxifen has a 40% chance of response.
C. She has a very high likelihood of disease shrinkage and symptom
palliation with further platinum-based chemotherapy.
D. She has a chance of cure with autologous stem cell transplant.
Question 16.13 A 45-year-old woman has undergone surgical resection
followed by adjuvant chemotherapy for her stage IIIC ovarian cancer. She is
now in complete remission and is interested in further treatment to reduce the
risk of recurrent disease. Which of the following would you recommend?
A. Maintenance treatment with single-agent paclitaxel for 12 months.
B. Clinical surveillance.
C. Maintenance treatment with single-agent topotecan.
D. Maintenance treatment with bevacizumab.
Question 16.14 A 53-year-old woman presents with ascites, omental

https://t.me/ALGRAWANY33
caking, and peritoneal nodularity. Her ovaries are normal. She undergoes
surgery, and all gross tumor is removed. The final diagnosis is papillary
serous primary peritoneal cancer. How should this patient be treated?
A. As if she had a standard ovarian cancer
B. As if she had a peritoneal mesothelioma
C. As if she had breast cancer
D. As a germ cell cancer
Question 16.15 Which of the following precursor lesions is associated with
the highest risk of progression to endometrioid adenocarcinoma?
A. Simple hyperplasia
B. Complex hyperplasia
C. Simple atypical hyperplasia
D. Complex atypical hyperplasia
Question 16.16 Which of the following is TRUE when considering
adjuvant therapy for endometrioid cancer?
A. Clear cell or serous adenocarcinoma histologies are considered high
risk, regardless of stage.
B. Clear cell, but not serous, adenocarcinoma histology is considered high
risk.
C. Grade 2 or higher stage IB endometrioid adenocarcinomas are
considered high risk.
D. Endometrioid adenocarcinomas are low risk, regardless of stage.
Question 16.17 Which of the following drugs has been shown to be
effective when used in combination with immunotherapy in patients with
mismatch repair–proficient metastatic endometrial carcinoma progressing on
prior systemic therapy?
A. Sorafenib
B. Lenvatinib
C. Axitinib
D. Trastuzumab
Question 16.18 A 50-year-old female diagnosed with squamous cell
carcinoma of the cervix was found to have stage IB1 carcinoma. What is the
treatment of choice in this setting?
A. Radiation therapy to pelvis
B. Carboplatin plus paclitaxel followed by hysterectomy
C. Radical hysterectomy with pelvic node dissection
D. Surgical debulking followed by adjuvant chemotherapy
Question 16.19 A 34-year-old female with stage IB1 carcinoma of the
cervix without extensive endocervical extension is interested in fertility
preservation. Which of the following is a reasonable treatment option?
A. Observation since this is an early-stage carcinoma.
B. Brachytherapy plus external beam radiation.
C. Fertility preservation is not possible, and radical hysterectomy with
pelvic node dissection is recommended for all patients.
D. Radical trachelectomy with pelvic node dissection.
Question 16.20 You see a 60-year-old woman with cervical squamous cell
carcinoma extending to the pelvic sidewall and without distant metastatic
disease on her scans. You diagnose her with stage IIIB cervical cancer.
Which of the following is the best treatment option for this patient?
A. Chemotherapy
B. Radical hysterectomy with pelvic node dissection
C. Concurrent chemoradiation and brachytherapy
D. Pelvic exenteration
Question 16.21 Human papillomavirus 16 (HPV16) is commonly
associated with which of the following?
A. Cervical squamous cell carcinoma, but not adenocarcinoma
B. Cervical adenocarcinoma, but not squamous cell carcinoma
C. Both cervical squamous cell and adenocarcinomas

https://t.me/ALGRAWANY33
D. Neither oropharyngeal cancer nor cervical cancers
Question 16.22 Which of the following patients is a suitable candidate with
cervical cancer for treatment with pembrolizumab?
A. A 61-year-old lady with cervical squamous cell carcinoma, stage IIIB,
interested in fertility preservation
B. A 61-year-old female with metastatic cervical squamous cell carcinoma
requiring frontline therapy with programmed death-ligand 1 (PD-L1)
score of 50%
C. A 61-year-old female with metastatic cervical squamous cell carcinoma
requiring second-line therapy with PD-L1 score of 50%, following
progression on chemotherapy
D. A 61-year-old female with metastatic cervical squamous cell carcinoma
requiring frontline therapy with PD-L1 score of 0%, following
progression on chemotherapy

Answers

Question 16.1 The correct answer is C.


This woman should be referred for genetic counseling and possible
genetic testing. Some 40% to 60% of patients of Jewish descent who
have epithelial ovarian cancer carry one of the three founder mutations
in BRCA1 or BRCA2 irrespective of family history, which are inherited
in an autosomal dominant manner, through either the paternal or
maternal side. If this woman carries a BRCA1 mutation, her lifetime risk
of ovarian cancer is 20% to 60%, and her risk of breast cancer is even
higher. Although transvaginal ultrasound and CA-125 screening are
generally recommended for mutation carriers who have their ovaries in
place, there is no good evidence that such screening will decrease
mortality. Oral contraceptives have been suggested to decrease the risk
of ovarian cancer by up to 50%. Removal of the ovaries likely confers
the best method for reducing the risk of ovarian cancer. It remains
controversial whether the uterus should be removed as well.
Question 16.2 The correct answer is D.
Understaging is common, particularly when the preoperative diagnosis
is that of a benign process. Earlier laparoscopic surgical staging series
suggested that up to 30% to 40% of patients thought to have FIGO (The
International Federation of Gynecology and Obstetrics) stage I or II
disease actually had disease in the upper abdomen. The incidence of
extraovarian spread is lower with a grade 1 tumor. Nonetheless,
complete surgical staging, if possible, is advised in this case, because the
recommendation for a stage IA grade 1 mucinous tumor is no
chemotherapy. Although some data have suggested that advanced-stage
mucinous tumors may lower response to chemotherapy compared to
serous tumors, a low-grade, early-stage mucinous tumor does not have a
poor prognosis. However, chemotherapy would be recommended in the
case of any extraovarian spread. In the hands of an experienced surgeon,
laparoscopic staging, including omentectomy and para-aortic lymph
node examination, is an option. Preservation of the uterus and
contralateral ovary is reasonable if no further disease is found at the time
of staging. CA-125 is frequently normal in women with mucinous
ovarian tumors, even when of advanced stage. A PET scan will not
detect microscopic disease, may not be positive in low-grade
malignancies, and is not likely to be helpful.
Question 16.3 The correct answer is C.
CA-125 is not useful for screening because early-stage ovarian cancer
has an elevated CA-125 ~50% of the time. Furthermore, the positive
predictive value of an elevated CA-125 is very low because many other
disorders also cause an elevated CA-125 and the prevalence of the
disease is quite low. Although CA-125 is not particularly helpful in the
diagnosis of ovarian cancer, it is useful in monitoring treatment.
Question 16.4 The correct answer is B.
Any factor that decreases ovulation is also associated with a decreased
risk of ovarian cancer. Therefore, oral contraceptives, multiparity, and
breastfeeding decrease the likelihood that an individual will develop

https://t.me/ALGRAWANY33
ovarian cancer. Several population-based studies show that tubal ligation
decreases the likelihood of developing ovarian cancer.
Question 16.5 The correct answer is D.
There are no data that suggest any adjuvant therapy will improve the
survival in patients with tumors of low malignant potential.
Question 16.6 The correct answer is A.
Elevated CA-125 indicates disease recurrence, and in patients who are
asymptomatic with no other objective evidence of disease recurrence,
the appropriate treatment would be hormonal therapy or observation.
Chemotherapy is considered when patients have symptomatic disease or
there is objective evidence for disease recurrence.
Question 16.7 The correct answer is D.
The majority of inherited disease in the Jewish population is due to one
of the three founder mutations in BRCA1 and BRCA2.
Question 16.8 The correct answer is B.
Up to 15% of patients with gross stage I disease have nodal metastases.
This is why it is very important for these patients to undergo
comprehensive staging.
Question 16.9 The correct answer is D.
Although the majority (almost 80%) of patients with advanced-stage
(III–IV) ovarian cancer will achieve a clinically complete remission with
taxane plus platinum combination therapy, ~70% will ultimately relapse
from a clinical complete remission. Even patients with a surgically
confirmed complete remission (negative second look) have an eventual
relapse rate of ~50%. Although patients with optimally debulked stage
III disease are a relatively favorable subgroup, those optimally debulked
patients who have a larger presurgical tumor burden (IIIC) fare worse
than those with a lesser presurgical tumor burden (IIIA). Maintenance
paclitaxel can improve progression-free survival of women with a
clinical complete response from 21 to 28 months, although it is
associated with no survival benefit and a significant risk of neuropathy.
Little data exist on PET scans in this situation, and consolidation
topotecan has shown no benefit. Although intraperitoneal therapy is a
theoretically attractive option, cisplatin, the most widely used drug,
carries a significant risk of neurotoxicity, and there are no data showing
any survival benefit to its use in the setting of minimal residual disease.
Question 16.10 The correct answer is C.
Sex cord–stromal tumors, of which granulosa cell tumors are the most
common subtype, comprise only ~5% of all ovarian neoplasms. The
peak incidence is in women aged >50 years, although a significant
proportion occurs in premenopausal women. Granulosa cell tumors may
secrete estrogen and be associated with endometrial hyperplasia and
endometrial carcinoma. The majority of patients are diagnosed in stage
I, with 10-year survival of 75% to 95%. However, late relapses may be
observed. Patients with advanced-stage disease have poor outcomes.
Although the rarity of the tumor precludes randomized trial data, most
such patients will be offered chemotherapy, traditionally with
bleomycin, etoposide, cisplatin (BEP), or other regimens used in germ
cell tumors. Another option is the combination of platinum plus taxane,
which may be as effective as BEP.
Question 16.11 The correct answer is A.
Germ cell tumors almost always occur in young women, with a peak
incidence in their early 20s. Some 60% to 70% are stage I at diagnosis.
With the use of platinum-based chemotherapy regimens similar to those
used for men with testicular cancer, even patients with an advanced
stage have a good prognosis. Because most of these tumors occur in
young women, often before they have had children, and because the type
of chemotherapy used will not cause infertility in most young female
patients, the surgical approach is critical. In many patients, the
contralateral ovary and uterus can and should be spared. Pretreatment
infertility consultation should be considered as egg harvesting and
storage is effective and of reasonable cost.

https://t.me/ALGRAWANY33
Question 16.12 The correct answer is C.
Recurrent ovarian cancer is not generally curable with transplant or any
other modality, but patients whose disease recurs with a disease-free
interval of >1 year have a >50% chance of responding to platinum-based
combination therapy and should usually be offered chemotherapy.
Secondary debulking surgery may also be of benefit. Hormonal therapy
in ovarian cancer generally produces response rates of only 10% to 15%
and is usually reserved for patients who cannot tolerate other therapy. It
has also been recommended as the initial salvage therapy in patients
who have an increasing CA-125 as their only manifestation of recurrent
disease.
Question 16.13 The correct answer is B.
Current evidence does not support maintenance chemotherapy with
paclitaxel or topotecan after adjuvant chemotherapy for patients with
advanced disease. The role of maintenance bevacizumab is still
investigational.
Question 16.14 The correct answer is A.
Because primary peritoneal cancers, fallopian tube cancer, and ovarian
cancers are considered to be histologically and molecularly identical, the
treatment recommendation for all three malignancies is the same.
Question 16.15 The correct answer is D.
Endometrial hyperplasia is the precursor lesion for endometrioid
adenocarcinoma, which is the most common form of endometrial
cancer. Of the four subtypes of hyperplasia (simple hyperplasia,
complex hyperplasia, simple atypical hyperplasia, and complex atypical
hyperplasia), complex hyperplasia with atypia is associated with the
highest risk of progressing to endometrial cancer, which occurs in ~29%
of cases.
Question 16.16 The correct answer is A.
Patients with stage IB grade 3 endometrioid carcinoma, serous or clear
cell adenocarcinomas of any stage, and pathologic stage III/IV disease
are all considered high-risk endometrial cancers.
Question 16.17 The correct answer is B.
The combination of lenvatinib and pembrolizumab has been shown to be
active in mismatch repair–proficient endometrial cancers progressing on
systemic therapy.1 Pembrolizumab alone can be used in patients with
high mutation-burden or mismatch repair–deficient tumors.
Question 16.18 The correct answer is C.
Early-stage IB cervical carcinomas are treated with radical hysterectomy
and bilateral pelvic lymphadenectomy or combined external beam
irradiation and brachytherapy. The goal of these treatments is to
eradicate cancer cells in the cervix, paracervical tissues, and regional
lymph nodes. Patients who are treated with radical hysterectomy may
benefit from postoperative radiotherapy or chemoradiation if their
tumors are found to have high-risk disease features.
Question 16.19 The correct answer is D.
Radical vaginal or abdominal trachelectomy and laparoscopic
lymphadenectomy can be considered in carefully selected women with
small stage IB1 (≤2 cm) lesions who are interested in fertility
preservation. Patients with extensive endocervical extension are poor
candidates for trachelectomy. Preoperative magnetic resonance imaging
(MRI) is often obtained in these patients to assess for tumor extension.
Patients following radical trachelectomy are, however, at an increased
risk for miscarriage and preterm delivery.
Question 16.20 The correct answer is C.
Patients with locally advanced cervical cancer are treated with
chemoradiation with concurrent cisplatin, and brachytherapy. Pelvic
exenteration is typically pursued in patients who recur in the pelvis
following definitive intent chemoradiation.
Question 16.21 The correct answer is C.

https://t.me/ALGRAWANY33
HPV16 is commonly associated with both cervical squamous cell and
adenocarcinomas. Together, HPV16 and HPV18 are found in nearly
70% of all cervical cancers.
Question 16.22 The correct answer is C.
Pembrolizumab is approved for use by the Food and Drug
Administration (FDA) for the treatment of metastatic cervical carcinoma
with PD-L1 score >1% progressing on frontline chemotherapy.2

References
1. Makker V, Rasco D, Vogelzang NJ, et al. Lenvatinib plus pembrolizumab in patients with advanced
endometrial cancer: an interim analysis of a multicentre, open-label, single-arm, phase 2 trial.
Lancet Oncol. 2019;20:711–718.
2. Chung HC, Ros W, Delord JP, et al. Efficacy and safety of pembrolizumab in previously treated
advanced cervical cancer: results from the phase II KEYNOTE-158 study. J Clin Oncol.
2019;37:1470–1478.
___________
Corresponding chapter in DeVita, Hellman, and Rosenberg’s Cancer: Principles & Practice of
Oncology, Eleventh Edition: 77 (Ovarian Cancer).
17 Leukemias and Myelodysplastic
Syndrome
Dhruv Bansal and Armin Ghobadi

QUESTIONS

Each of the numbered items below is followed by lettered answers. Select the
ONE lettered answer that is BEST in each case unless instructed otherwise.

Question 17.1 What is true about core-binding factor (CBF) acute myeloid
leukemias (AMLs)?
A. They involve t(8;21) or inv(16).
B. They are associated with a favorable prognosis.
C. They may benefit from addition of gemtuzumab ozogamicin.
D. All of the above.
Question 17.2 You are caring for a 25-year-old woman with AML-M2 and
normal cytogenetics. She receives 7 + 3 induction therapy, and on day 14, the
bone marrow biopsy is ablated. She returns to clinic, and her day-45 bone
marrow shows normal hematopoiesis. However, she relapses after 18 months.
Which of the following likely contributed to her relapse?
A. DNMT3A mutation
B. CEBPA mutation
C. NPM mutation without FLT3-ITD
D. Her age
Question 17.3 Your well-read 30-year-old patient with AML asks you how
you will follow her disease once she is in remission. She asks which is the
most sensitive test for minimal residual disease (MRD). In which order are

https://t.me/ALGRAWANY33
these assays most sensitive (least to most sensitive)?
A. Cytopathology < fluorescence in situ hybridization (FISH) <
polymerase chain reaction (PCR)
B. FISH < cytogenetics < PCR
C. PCR < cytopathology < FISH
D. Cytogenetics < PCR < flow cytometry
Question 17.4 Mutation of which of the following genes is associated with
a worse prognosis in CBF AML?
A. ABL
B. CKIT
C. NPM1
D. GATA-1
Question 17.5 A 52-year-old man presents with AML. On day 2 of
induction therapy, he develops diffuse alveolar hemorrhage. An expected
physical finding on examination would be:
A. Diffuse ecchymosis.
B. Diffuse intravascular coagulopathy.
C. Swollen gums.
D. Cardiac rub.
Questions 17.6–7 You have been following a 75-year-old woman with
mild renal insufficiency and progressive anemia. She has good performance
status and presents to clinic with a 3-day history of progressive fatigue and
new headaches. Her white blood cell (WBC) count is 53,000 with 50% blasts.
Question 17.6 Given her history, what is the expected finding in the
cytogenetics?
A. t(15;17)
B. −7
C. inv(16)
D. t(8;21)
Question 17.7 Which of the following is associated with improved
prognosis in this patient?
A. Probable leukostasis
B. Her age
C. Prior myelodysplastic syndrome (MDS)
D. Good performance status
Question 17.8 Which of the following cytogenetic profiles from an AML
patient is an example of monosomal karyotype?
A. −7
B. +8, inv(3), del(5q)
C. inv(16)
D. −7, −5, t(3;21)
Question 17.9 Which of the following is a poor prognostic factor in adult
acute lymphoblastic leukemia (ALL)?
A. Age <35 years
B. WBC <30,000
C. Diploid chromosomes on karyotyping
D. t(4;11)
Question 17.10 A 70-year-old man presents with newly diagnosed AML-
M4 with eosinophilia. Which of the following factors will have the greatest
impact on the choice of therapy?
A. His age
B. His performance status
C. His cytogenetics
D. Excellent response to 7+3 induction therapy
Question 17.11 You have been following a 54-year-old woman with acute
promyelocytic leukemia (APL) in clinic. After 2 years, she returns to clinic
with fatigue, an elevated WBC, and increased promyelocytes. Peripheral
blood PCR confirms recurrence of her t(15;17) translocation. Before starting

https://t.me/ALGRAWANY33
arsenic salvage therapy, which test should you obtain?
A. Liver function tests
B. Erythrocyte-sedimentation rate
C. D-Dimer
D. Electrocardiogram
Question 17.12 You have been caring for a 65-year-old woman with a
distant history of breast cancer treated with adjuvant cyclophosphamide and
doxorubicin. During the past year, she developed progressive anemia and
thrombocytopenia. She also recently developed leukopenia. The bone
marrow biopsy shows decreased cellularity with dysplastic features and 25%
blasts. Which of the following cytogenetic changes might you expect to find?
A. t(15;17)
B. t(9;21)
C. Complex cytogenetics
D. Trisomy 21
Question 17.13 Which of the following statements is TRUE regarding
elderly patients (age >65 years) with AML?
A. In general, older patients with AML have better outcomes when
compared to younger patients.
B. Treatment with hypomethylating agents leads to the same response rate
as more intensive chemotherapy.
C. Older patients are more likely to have favorable-risk cytogenetics than
younger patients.
D. Treatment with decitabine leads to better response rates but similar
survival compared to low-dose cytarabine.
Questions 17.14–18 You are consulted to see a 20-year-old Hispanic
woman who presented with progressive fatigue during the past week and then
significant epistaxis. Her WBC is 12,000/μL with 40% promyelocytes and a
platelet count of 15,000/μL. International normalized ratio (INR) is 2.7 with a
prothrombin time of 45, partial thromboplastin time of 65, and fibrinogen is
82. On review of her peripheral smear, you observe many promyelocytes
with large granules and multiple Auer rods.
Question 17.14 After review of her peripheral smear, you suspect that she
has APL. While awaiting confirmation of the diagnosis, your initial therapy
should include which of the following?
A. Steroids
B. Cytarabine
C. Fresh-frozen plasma
D. Arsenic trioxide
Question 17.15 Three days after starting idarubicin and all-trans retinoic
acid (ATRA), her coagulopathy has improved. She has shortness of breath in
the morning and rapidly becomes hypoxic over the course of the day. Which
of the following would prove most helpful in treating her hypoxia?
A. High-flow facemask oxygen
B. Furosemide
C. Methylprednisolone
D. Albuterol
Question 17.16 Fifteen days into treatment, she develops a severe
headache. Her neurologic and fundoscopic evaluations are normal. Review of
her morning laboratory tests reveal a WBC of 0.6/μL, hematocrit of 9.8,
platelet count of 25,000/μL, INR of 1.4, and fibrinogen of 190. You obtain a
noncontrast head computed tomography to rule out bleeding, but the most
likely explanation for the headaches is:
A. Relapse.
B. ATRA effect.
C. Idarubicin effect.
D. Transfusion reaction.
Question 17.17 Six months later, you are reviewing her chart. The
complete blood count (CBC) has normalized, and she has tolerated
consolidation therapy. The most recent PCR shows no residual disease.

https://t.me/ALGRAWANY33
However, the patient is at higher risk for relapse because of:
A. Her microgranular variant presentation.
B. Her presenting coagulopathy.
C. Her presenting WBC.
D. Her ethnicity.
Question 17.18 Three years later, she presents to clinic with an elevated
leukocyte count. PCR of peripheral blood confirms the presence of her initial
t(15;17) translocation. Treatment options at this point include which of the
following?
A. Arsenic trioxide
B. Gemtuzumab ozogamicin
C. Autologous transplant after achieving complete remission (CR)
D. All of the above
Question 17.19 A 30-year-old, otherwise healthy woman is diagnosed with
AML. Cytogenetics reveals inv(16). She undergoes induction therapy. Day-
14 bone marrow biopsy shows an ablated marrow. Her day-45 marrow shows
restored cellularity without evidence of disease. Repeat cytogenetics does not
reveal the inv(16) rearrangement. Appropriate consolidation therapy would
be:
A. Allogeneic transplant if a matched sibling donor is available.
B. High-dose cytarabine (HIDAC) 3 g/m2 every 12 hours on days 1, 3,
and 5 for four 28-day cycles.
C. Intermediate-dose cytarabine (IDAC) 300 mg/m2 every 12 hours on
days 1, 3, and 5 for four 28-day cycles.
D. Arsenic 0.15 mg/kg on days 1–5 for four 28-day cycles.
Question 17.20 Which of the following regimens requires graft-versus-
tumor effects to treat residual AML blasts?
A. Busulfan–cyclophosphamide
B. Cyclophosphamide–total body irradiation (TBI)
C. Fludarabine–TBI
D. Fludarabine–busulfan (Flu/Bu4)
Question 17.21 Central nervous system (CNS) prophylaxis should be
considered in which of the following patients?
A. A 25-year-old Hispanic woman with APL who presents with a WBC of
2,500/μL, an INR of 2.5, and fibrinogen of 100
B. A 78-year-old man with AML evolved from MDS
C. A 30-year-old woman with AML who develops headaches while
receiving ondansetron for nausea on day 9 of induction therapy
D. A 20-year-old man with Down syndrome and ALL who presents with
leukocytosis (WBC 120,000/μL)
Question 17.22 Dexamethasone has replaced prednisone in ALL induction
therapy because of improved penetration in which tissue?
A. Testes
B. Spleen
C. Bone marrow
D. Brain
Question 17.23 How long should maintenance therapy for ALL with daily
6-mercaptopurine, weekly methotrexate, and monthly vincristine and
prednisone be continued?
A. 6 months
B. 12 months
C. 24–36 months
D. Until relapse
Question 17.24 Which of the following targeted agents has been shown to
be beneficial in adult Philadelphia chromosome positive ALL?
A. Imatinib
B. Alemtuzumab
C. Gemtuzumab ozogamicin
D. Sunitinib

https://t.me/ALGRAWANY33
Question 17.25 For which of the following patients would you consider
myeloablative stem cell transplantation in CR1?
A. A 30-year-old woman with inv(16) AML-M4 with eosinophilia
B. A 50-year-old man with complex cytogenetics AML-M1
C. A 65-year-old man with complex cytogenetics AML-M1
D. A 50-year-old woman with t(15;17) AML-M3
Question 17.26 Which of the following statements is TRUE regarding the
nucleophosmin1 (NPM1) mutation in AML?
A. In normal karyotype AML, NPM1 mutation without FLT3-ITD confers
a prognosis similar to good-risk cytogenetics.
B. NPM1 mutations are rarely found in patients with AML with normal
karyotype.
C. NPM1 mutation combined with FLT3-ITD is associated with good
prognosis.
D. All the above.
Question 17.27 Mutations that cause genetic alterations contributing to
leukemic pathophysiology are best described as:
A. Driver mutations
B. Passenger mutations
C. Point mutations
D. Missense mutations
Question 17.28 What is the common abnormality seen in patients who
develop therapy-related AML (t-AML) and are treated with topoisomerase
inhibitors in the past for another malignancy?
A. t(15;17)
B. t(8;21)
C. Fusions involving myc
D. Fusions involving KMT2A
Question 17.29 Among normal cytogenetic AMLs, which of the following
is associated with the most favorable prognosis?
A. Mutated NPM1 without FLT3-ITD
B. Wild-type NPM1 without FLT3-ITD
C. Wild-type NPM1 with FLT3-ITD
D. Mutated NPM1 and FLT3-ITD
Question 17.30 Marrow or peripheral blood blast count of 20% is
necessary for the diagnosis, except for AML with:
A. t(8;21)
B. inv(16)
C. t(15;17)
D. All of the above
Question 17.31 Which of the following drugs has been U.S. Food and
Drug Administration (FDA) approved for use in combination with 7 + 3
induction for favorable- and intermediate-risk AML?
A. Inotuzumab ozogamicin
B. Etoposide
C. Gemtuzumab ozogamicin
D. Venetoclax
Question 17.32 Which of the following drugs has been FDA approved for
use in combination with 7+3 induction in patients with FLT3-mutated AML?
A. Sorafenib
B. Midostaurin
C. Gilteritinib
D. Venetoclax
Question 17.33 Which of the following drugs has shown superior overall
survival (OS) over 7+3 in patients with t-AML and AML with
myelodysplasia-related changes?
A. Azacytidine

https://t.me/ALGRAWANY33
B. CPX-351
C. Decitabine
D. FLAG-Ida
Question 17.34 For patients with morphologic findings suspicious for
APL, in addition to correcting coagulopathy, which of the following
treatments should you consider initiating while awaiting genetic
confirmation?
A. Steroids
B. Idarubicin
C. ATRA
D. Arsenic trioxide
Question 17.35 Which of these drugs gained FDA approval for the
treatment of IDH2-mutated relapsed or refractory AML?
A. Enasidenib
B. Ivosidenib
C. Gilteritinib
D. Gemtuzumab
Question 17.36 A frail 70-year-old male presents with IDH1-mutated
AML. Which of these recently approved agents could you consider for
treatment?
A. Enasidenib
B. Ivosidenib
C. Gilteritinib
D. Inotuzamab
Question 17.37 Which of the following agents is associated with
differentiation syndrome?
A. ATRA
B. Ivosidenib
C. Gilteritinib
D. All of the above
Question 17.38 A 62-year-old male with FLT3-mutated AML undergoes
induction with 7+3 in combination with midostaurin. The day-30 bone
marrow reveals refractory disease. Which of the following FDA-approved
treatments would you consider?
A. Sorafenib
B. Gilteritinib
C. Quizartinib
D. Crenolanib
Question 17.39 Why is allogeneic stem cell transplant recommended over
autologous stem cell transplant for the treatment of leukemia?
A. Lower risk of major complications
B. Less toxic preparative regimens
C. Graft-versus-leukemia (GVL) effect
D. Graft-versus-host disease (GVHD)
Question 17.40 Which of the following is the strongest positive predictor
of outcomes after allogeneic transplantation for patients with AML?
A. CR
B. Cytogenetics
C. Molecular profile
D. Conditioning regimen
Question 17.41 Which of the following is an indication for performing an
allogeneic stem cell transplant in AML?
A. t-AML or AML progressing from MDS
B. Primary induction failure
C. Second or subsequent remission
D. All of the above
Question 17.42 After initiation of treatment for B-ALL, which of the

https://t.me/ALGRAWANY33
following is most correlated with outcomes such as OS and disease-free
survival?
A. MRD
B. Blast percentage
C. Cytogenetics
D. Molecular abnormalities
Question 17.43 A 35-year-old male undergoes induction therapy for
Philadelphia chromosome–positive ALL (Ph-ALL), and bone marrow biopsy
after induction therapy reveals morphologic remission but is positive for the
presence of MRD. Which FDA-approved therapy for this scenario would you
recommend in order to achieve MRD-negative status as a bridge to
transplant?
A. Inotuzumab
B. Blinatumomab
C. Tisagenlecleucel
D. Cancer and Leukemia Group B (CALGB) 10403 chemotherapy
regimen
Question 17.44 Which of the following agents is currently approved for
use in relapsed/refractory ALL?
A. Inotuzumab
B. Blinatumomab
C. Tisagenlecleucel
D. All of the above
Question 17.45 What is the characteristic immunophenotype of chronic
leukocytic leukemia (CLL)?
A. CD5+, CD10−, CD23+
B. CD5−, CD10+, CD23−
C. CD5−, CD22+, CD23−
D. CD5−, CD10−, CD79b+
Question 17.46 Which of the following is associated with an aggressive
disease course in CLL?
A. ZAP-70 expression
B. Unmutated IGHV
C. del(17p)
D. All of the above
Question 17.47 A 65-year-old male with 17p deletion CLL presents with
fevers, night sweats, hemoglobin 10 g/dL, and early satiety. He has
splenomegaly on examination, with spleen being palpable about 7 cm below
the left costal margin. What therapy, if any, will you initiate?
A. Fludarabine, cyclophosphamide, rituximab (FCR).
B. Bendamustine plus rituximab.
C. No treatment is necessary.
D. Ibrutinib.
Question 17.48 A 40-year-old male gets initiated on ibrutinib for
symptomatic CLL. A week later, his absolute lymphocyte count has nearly
doubled from 40,000 to 75,000. What do you recommend?
A. Switch to acalabrutinib.
B. Switch to venetoclax in combination with obinutuzumab.
C. Continue ibrutinib.
D. Switch to idelalisib.
Question 17.49 A 62-year-old female presents with CLL and multiple
palpable lymph nodes over 10 cm in size. The absolute lymphocyte count is
42,000. You decide to initiate therapy with venetoclax in combination with
obinutuzumab. What is the next step?
A. Admit for hydration.
B. Initiate venetoclax at a lower dose and follow-up in a week.
C. Initiate venetoclax at standard dose and follow-up in a week.
D. Encourage oral hydration.

https://t.me/ALGRAWANY33
Question 17.50 Which of the following is a common complication of
CLL?
A. Autoimmune hemolytic anemia
B. Hypogammaglobulinemia
C. Skin cancer
D. All of the above
Question 17.51 A 46-year-old Caucasian male presents with a hemoglobin
level of 6.5 and an absolute lymphocyte count of 7,000. A bone marrow
biopsy is performed and reveals the presence of an abnormal B-cell
population positive for CD11c, CD25, CD103, CD123, as well as CD19,
CD20, and CD22. The abnormal B cells are negative for CD5, CD23, and
CD10. BRAF V600E mutation is positive. Based on the immunophenotype,
what initial therapy will you consider?
A. Venetoclax
B. Cyclosporine
C. Cladribine
D. Chlorambucil
Question 17.52 What is not TRUE about treatment with second-generation
tyrosine kinase inhibitors (TKIs) compared to imatinib?
A. They achieve significantly higher complete cytogenetic response
(CCyR) rates at or by 12 months.
B. They produce a faster and deeper decrease in BCR-ABL1 messenger
RNA values as detected by quantitative PCR (qPCR).
C. They have different sensitivity profiles.
D. There are associated with a significant improvement in progression-
free survival (PFS) and OS.
Question 17.53 You are treating a 43-year-old male patient for chronic
myeloid leukemia (CML) with imatinib. He achieved complete hematologic
response at 1 month. Over the next few follow-up visits, a qPCR reveals
progressive decrease in BCR-ABL1 transcripts. You are pleased with the
progress. At the 6-month follow-up, testing reveals a loss of previously
achieved cytogenetic response and a 1 log increase in BCR-ABL1 transcripts
by qPCR. What is the best next step?
A. Initiate hyper-CVAD (cyclophosphamide, vincristine, adriamycin, and
dexamethasone).
B. Refer for an allogeneic stem cell transplant.
C. Switch to a second-generation (TKI).
D. Perform a mutational screening.
Question 17.54 A 64-year-old female presents with a history of breast
cancer, for which she underwent surgical excision followed by adjuvant
chemotherapy with doxorubicin, cyclophosphamide, and paclitaxel about 4
years ago. She now presents with new-onset pancytopenia. You suspect
MDS. Which genetic anomaly are you most likely to discover?
A. 11q23 abnormalities
B. Monosomy 5 or 7
C. 17p deletion
D. 20q deletion
Question 17.55 A 68-year-old female presents with a new diagnosis of
AML, with WBC count 8,000 and 12% circulating blasts. Next-generation
sequencing does not reveal any actionable mutations. Given her age, you do
not feel she is a candidate for intensive chemotherapy. Which of the
following therapies could you consider?
A. Venetoclax and decitabine
B. Sorafenib
C. Enasidenib
D. Ivosidenib
Questions 17.56–58 You are seeing a 72-year-old male with recently
diagnosed MDS. His only symptom is mild fatigue in addition to requiring
about 1 to 2 packed red blood cell transfusions per month for a hemoglobin
level of six.
Question 17.56 You decide to initiate him on an erythropoiesis-stimulating

https://t.me/ALGRAWANY33
agent (ESA). What factor will predict responsiveness to this strategy?
A. Erythropoietin level <500 U/L
B. Recent diagnosis
C. Cytogenetics
D. Age
Question 17.57 The cytogenetics reveals chromosome 5q deletion as the
sole genetic anomaly. Which FDA-approved treatment could you consider
for his management of anemia?
A. Venetoclax
B. Lenalidomide
C. Glasdegib
D. Cytarabine
Question 17.58 He achieves transfusion independence (TI), but 2 years,
later he develops worsening cytopenias. A bone marrow biopsy reveals a
TP53 mutation. Which of these FDA-approved treatments could you
consider?
A. Ibrutinib
B. Azacitidine
C. Cytarabine
D. Venetoclax

Answers

Question 17.1 The correct answer is D.


The favorable t(8;21) translocation results in the juxtaposition of the
initially named AML1 gene with the ETO gene (the AML gene and the
eight-twenty-one gene). These two have been renamed RUNX1 and
RUNX1T1, respectively. RUNX1 is a member of the CBF family of
transcription factors, which regulate differentiation of normal blasts.
This fusion protein creates a dominant-negative protein, which results in
a differentiation block. Inv(16) and the t(16;16) rearrangements likewise
result in dominant-negative effects on CBFs and are associated with a
favorable prognosis. Adding gemtuzumab ozogamicin to chemotherapy
in CD33-positive low-risk AML is associated with improvement in
event-free survival.
Question 17.2 The correct answer is A.
The DNMT3A gene encodes the DNA (cytosine-5)-methyltransferase 3A
enzyme that catalyzes the addition of a methyl group to the cytosine
residue of CpG dinucleotides. In patients with AML with normal
cytogenetics, the presence of DNMT3A mutations is independently
associated with poor outcomes. CEBP/alpha is a transcription factor
involved in myeloid differentiation thought to be activated by ATRA
treatment in APL. Mutations in the CEBPA gene have been noted in
other FAB-AML classifications and have been associated with a
superior outcome compared with the wild-type transcript. NPM1
mutations, in the absence of the FLT3-ITD, have also been associated
with improved prognosis. Age is a strong prognostic factor, with
patients older than 65 years having decreased survival. Her young age
would be a marker of a good prognosis.
Question 17.3 The correct answer is A.
Cytopathology requires >5% blasts on a bone marrow biopsy to
diagnose AML relapse. This results in a sensitivity of 1 in 20. A
cytogenetic review of 30 metaphases results in a 1 in 30 sensitivity.
FISH has a sensitivity of ~1 in 500. PCR techniques allow sensitivity of
~1 in 104. However, both FISH and PCR require screening for defined
cytogenetic changes. Because initial cytogenetic changes typically recur
during AML relapse, this is generally not a significant barrier, unless
patients present with uncommon cytogenetic changes for which FISH
and PCR probes are not commercially available.
Question 17.4 The correct answer is B.
CBF AML includes AML with t(8;21) and inv(16). These leukemias are
generally associated with good prognosis. However, patients whose

https://t.me/ALGRAWANY33
CBF AML has a mutation in CKIT have a worse prognosis than other
CBF AML patients. They may benefit from treatment with a kinase
inhibitor or allogeneic stem cell transplant.
Question 17.5 The correct answer is C.
Diffuse alveolar hemorrhage is a rare complication of induction therapy
in AML, which carries significant mortality risk. It is most commonly
reported in patients with AML and myelomonocytic or monocytic
features (M4 or M5). These myelomonoblasts and monoblasts display a
predilection for tissue invasion, such as in the gums, as well as into the
lungs. This is thought to result from their high expression of adhesion
molecules: intercellular adhesion molecule and vascular cell adhesion
molecule. During initial treatment, activation and death of circulating
blasts may lead to pulmonary inflammation and alveolar hemorrhage.
Because of the rarity of such events, treatment options remain poorly
studied. In small series, high-dose steroids have shown benefit.
Question 17.6 The correct answer is B.
AML evolving from MDS is especially recalcitrant to therapy. It may be
associated with −5 or −7 cytogenetic abnormalities but is most
commonly associated with complex cytogenetics. This patient is
presenting with many poor-risk features (age >65 years, symptoms
suggestive of leukostasis, and underlying organ insufficiency), which
will decrease the probability of response and survival.
Question 17.7 The correct answer is D.
There are many prognostic factors for AML. Of particular importance
are age, cytogenetics, and performance status. Age >65 years has been
associated with worse outcomes, independent of cytogenetics. AML
evolving from prior MDS also confers a worse prognosis. Leukostasis at
presentation presents significant early morbidity and mortality,
especially in an older patient. An elevated peripheral blast count, which
may be associated with leukostasis, is also associated with a poor long-
term prognosis.
Question 17.8 The correct answer is D.
Monosomal karyotype is defined by the presence of two or more
autosomal monosomies or by the presence of one autosomal monosomy
and a structural change (translocation). Monosomal karyotype is
associated with a particularly poor prognosis.
Question 17.9 The correct answer is D.
Age >35 years, poor performance status, African American ethnicity,
and leukocytosis (>30,000/μL B lineage or >100,000/μL T lineage) are
associated with high-risk acute lymphocytic leukemia (ALL).
Cytogenetics including the Ph chromosome t(9;22), t(8;14), and t(4;11)
are both high-risk prognostic factors. Normal diploid chromosomes on
karyotyping are not associated with poor prognosis.
Question 17.10 The correct answer is B.
Treatment of AML in the elderly (>65 years) is difficult because of the
heterogeneous nature of response and the generally high rate of
treatment-related mortality. Thus, decision to undergo standard
induction therapy must be based on a careful assessment of the patient’s
ability to tolerate the therapy and the overall prognosis.
Question 17.11 The correct answer is D.
Arsenic trioxide has demonstrated considerable activity in both initial
and salvage treatment of APL. Arsenic trioxide has been associated with
prolonged QTc syndrome. Therefore, electrocardiograms should be
performed before and during therapy. Electrolytes should also be
monitored and corrected. Additional medications that may prolong the
QTc should be eliminated if possible.
Question 17.12 The correct answer is C.
Prior alkylating therapy increases the risk of MDS and treatment-related
AML. Her recent history of progressive anemia and thrombocytopenia,
with dysplastic features seen in her marrow, are all suggestive of an
underlying MDS. Her blast count >20% meets criterion for AML under

https://t.me/ALGRAWANY33
the current World Health Organization standards. AML evolved from
MDS could carry a loss of chromosome 5 or 7 associated with the prior
MDS, but most often is associated with complex cytogenetics. It also
carries an unfavorable prognosis in multiple studies. t(15;17) is
associated with APL, but not with prior breast cancer or alkylator
therapy. t(9;21) is associated with the Ph chromosome and seen in
chronic myelocytic leukemia or acute lymphocytic leukemia, but is
rarely associated with AML or MDS. Trisomy 21 is associated with
AML-M7 with increased megakaryocytes.
Question 17.13 The correct answer is D.
Older patients with AML have worse outcomes when compared to
younger patients, irrespective of their performance status or
comorbidities. Treatment with the hypomethylating agents, azacitidine
and decitabine, is associated with lower response rates but similar OS
compared to more intensive chemotherapy regimens. Favorable
cytogenetics is less common in older AML patients. Treatment with
decitabine is associated with better response rate, but not with improved
survival, compared to low-dose cytarabine.
Question 17.14 The correct answer is C.
Patients with APL are at risk of early morbidity and mortality because of
disseminated intravascular coagulation and bleeding complications.
Aggressive management of coagulopathy is mandatory, with the goal of
maintaining the fibrinogen >150, INR <1.5, and platelet count >30,000.
Question 17.15 The correct answer is C.
Differentiation syndrome can be a life-threatening complication of APL
and ATRA treatment. As the granule-laden promyelocytes differentiate,
the granular products stimulate pulmonary edema and fluid retention.
Fevers and weight gain are common symptoms of differentiation
syndrome. Treatment includes corticosteroids and management of
coagulopathy. Differentiation syndrome may affect as many as 10% to
25% of patients with APL.
Question 17.16 The correct answer is B.
Headache is a common side effect from ATRA. Treatment options
include dose reduction and symptom management with analgesics.
Unfortunately, patients occasionally require drug discontinuation
because of this side effect.
Question 17.17 The correct answer is C.
The highest risk of relapse in APL is associated with an elevated WBC
>10,000/μL on presentation. Patients with presenting WBC <10,000/μL
achieve CR in ~90% of the time.
Question 17.18 The correct answer is D.
Arsenic trioxide has demonstrated significant efficacy in relapsed APL
and in consolidation protocols. APL promyelocytes commonly express
high levels of CD33, and gemtuzumab ozogamicin (anti-CD33) has
demonstrated efficacy in relapsed APL. APL is one of the few cases in
AML in which autologous transplantation has demonstrated significant
benefit, and it is often used in a salvage consolidation protocol.
Question 17.19 The correct answer is B.
Although most patients with AML will relapse without consolidation
therapy, the appropriate consolidation therapy remains unclear for the
majority of these patients. Young patients with good-risk cytogenetics
represent one of the few cohorts with AML in whom specific
consolidation recommendations are supported by clinical data. A study
sponsored by the CALGB found that young patients with inv(16) had a
78% CR rate at 5 years after four cycles of HIDAC therapy, whereas
57% remained in CR in the IDAC arm and only 16% remained in CR
with conventional 100 mg/m2 dosing. Patients with normal cytogenetics
fared less well as a group with equivalent outcomes (47% vs. 37% CR at
5 years) in the HIDAC and IDAC arms and worse outcomes with
conventional dosing (20%). Patients with unfavorable cytogenetics
should be considered for allogeneic transplant in the first CR. Arsenic
has shown benefit in a consolidation protocol for APL, but it has not

https://t.me/ALGRAWANY33
been evaluated in general AML consolidation protocols.
Question 17.20 The correct answer is C.
Nonmyeloablative conditioning regimens require a GVL effect to
improve the clearance of residual AML blasts not eliminated by the
conditioning regimen. Of all the available choices, C is the only
nonmyeloablative regimen.
Question 17.21 The correct answer is D.
CNS prophylaxis may consist of intrathecal chemotherapy
(methotrexate, cytarabine, corticosteroids), high-dose chemotherapy
(methotrexate, cytarabine, L-asparaginase), or CNS irradiation. Patients
with ALL with increased risk for CNS disease include those with an
elevated WBC count, an elevated lactate dehydrogenase, a traumatic
lumbar puncture, and T-lineage ALL. Although APL may relapse in the
CNS, relapse is uncommon in patients with a presenting WBC <10,000/
μL. CNS evaluation and prophylaxis are not routinely recommended in
patients with APL. There is no role for CNS prophylaxis in elderly
patients with AML without neurologic deficits. Although CNS disease
must be considered in a patient with AML and headaches, a new
headache caused by CNS leukemia on day 9 of induction therapy would
be unusual and treatment could be delayed until proper evaluation is
completed.
Question 17.22 The correct answer is D.
Dexamethasone has improved penetration in the CNS. Given the high
risk of CNS disease in patients with ALL, dexamethasone is preferred to
prednisone.
Question 17.23 The correct answer is C.
Maintenance therapy for ALL should be administered for 2 to 3 years
because attempts to shorten to 12 to 18 months have been associated
with decreased survival.
Question 17.24 The correct answer is A.
Ph-ALL carries a high-risk prognosis, but this has improved with the
ABL TKI imatinib. Second-generation inhibitors such as dasatinib and
nilotinib may also prove of value in the future. Alemtuzumab is an anti-
CD52 monoclonal antibody that has demonstrated significant efficacy in
the treatment of CLL. It has not been evaluated in ALL, which typically
does not express CD52. Gemtuzumab ozogamicin is an anti-CD33
monoclonal antibody cross-linked to a cytotoxin, calicheamicin. It has
been shown to improve outcomes in newly diagnosed favorable-risk
AML when is added to induction chemotherapy and in relapsed AML.
Sunitinib is a nonspecific TKI. It has significant activity against FLT3
and may prove of value in the treatment of AML with FLT3-ITD.
Question 17.25 The correct answer is B.
Stem cell transplantation for AML remains a developing field. Current
studies and meta-analyses have not found benefit in CR1 for patients
with favorable cytogenetics, such as inv(16) and t(15;17). The benefit is
uncertain in patients with normal cytogenetics. Future trials may be able
to assess the benefit of transplant in CR1 in higher risk subpopulations
of normal cytogenetics (e.g., patients with triple DNMT3A, FLT3-ITD,
and NPM1 mutations or ASXL1 mutation). Patients aged <55 years who
have unfavorable cytogenetics such as −5 and −7 and complex
cytogenetics appear to benefit from myeloablative transplantation in
CR1. However, older patients tolerate this approach less well, with
increased treatment-related morbidity and mortality. Decision-making in
older patients must be individualized, and nonmyeloablative transplant
approaches are usually preferred.
Question 17.26 The correct answer is A.
NPM1 gene encodes a protein that functions as a molecular chaperone
between the nucleus and cytoplasm. Mutations involving this gene are
found in 50% to 60% of all AML patients with normal karyotype and
less frequently are seen in patients with abnormal cytogenetics. Aberrant
cytoplasmic localization of NPM1 is associated with point mutations in
the gene. In the absence of FLT3-ITD, the NPM1 mutation is associated
with favorable prognosis.

https://t.me/ALGRAWANY33
Question 17.27 The correct answer is A.
“Driver” mutations cause genetic alterations contributing to leukemic
pathophysiology, whereas “passenger” mutations occur in leukemia
cells and are propagated but are not etiologic to the disease.
Question 17.28 The correct answer is D.
Patients who have received prior chemotherapy for cancer and develop
t-AML often have abnormalities in 11q23, especially those patients
treated with topoisomerase inhibitors such as etoposide or topotecan.
Chromosomal translocations involving band 11q23 result in expression
of a fusion gene containing amino-terminal KMT2A sequences between
exons 8 and 13 fused to a wide variety of partners.
Question 17.29 The correct answer is A.
NPM1 (Nucleophosmin1) encodes a protein that acts as a molecular
chaperone between the nucleus and cytoplasm. It is involved in multiple
cellular processes, including regulation of TP53/ARF pathways,
ribosome biogenesis, and duplication of centrosomes. NPM1 mutations
are found more frequently in AML with normal karyotypes (50%–60%)
and more apt to have FLT3-ITD mutations as well. Among normal
cytogenetic AMLs, the presence of mutated NPM1 in the absence of the
FLT3-ITD is associated with a more favorable prognosis.
Question 17.30 The correct answer is D.
Marrow or peripheral blood blast count 20% is necessary for the
diagnosis of AML, except for patients with t(8;21), inv(16), t(16;16)
(CBF), or t(15;17) (APL), where the presence of the appropriate
cytogenetic/molecular abnormality allows the diagnosis with lower blast
percentage. The t(15;17) is diagnostic of APL.
Question 17.31 The correct answer is C.
The anti–CD33-directed antibody drug conjugate gemtuzumab
ozogamicin delivers the toxin calcheamicin to AML blasts and has been
investigated in a number of large randomized trials in addition to
standard AML induction and consolidation. Overall, these trials have
suggested a survival benefit for patients with favorable- and
intermediate-risk AML, in particular patients with favorable risk, and
this has led to the recent approval of gemtuzumab ozogamicin in AML
by the FDA.
Question 17.32 The correct answer is B.
Based on the results of the RATIFY trial where 717 patients were
randomized to receive induction with or without midostaurin.1
Consolidation was with high-dose ara-C with or without midostaurin
and allogeneic stem cell transplant was allowed in the first CR. Those in
remission continued their midostaurin or placebo for maintenance. OS
and event-free survival were significantly better for the patients
randomized to receive midostaurin, even after censoring patients who
underwent allogeneic stem cell transplantation. This has led to the
approval of midostaurin for the treatment of patients with newly
diagnosed FLT3-mutated AML.
Question 17.33 The correct answer is B.
CPX-351 is a liposomal encapsulated combination of ara-C and
daunorubicin in a 5:1 molar ratio. Preclinical studies had suggested that
ara-C and daunorubicin at molar ratios 1:1 to 10:1 were synergistic with
the highest proportion of synergy and the lowest level of antagonism
occurring at the 5:1 molar ratio. In a recent phase III trial in 309 patients,
aged 60 to 75 years, with previously untreated t-AML, secondary AML
(s-AML), or AML with MDS-related changes, a higher CR rate (47.7%
vs. 33.3%) and OS (median 9.56 vs. 5.95 months) were reported for
CPX-351 compared to 7+3, leading to the approval of this drug.2
Question 17.34 The correct answer is C.
For patients with suspected APL, the realization of persistently high
incidence of early death necessitates high degree of suspicion for this
entity and initiation of therapy with ATRA at the earliest suspicion of its
existence. ATRA rapidly improves coagulopathy associated with APL
and is unlikely to be harmful.

https://t.me/ALGRAWANY33
Question 17.35 The correct answer is A.
Based on the results of a phase I/II study, the inhibitor of the enzyme
isocitrate dehydrogenase-2 (IDH2), enasidenib, has demonstrated
significant activity and acceptable tolerability profile in patients with
relapsed or refractory IDH2-mutated AML, with an overall response rate
of 40% and CR rate of 20%.3
Question 17.36 The correct answer is B.
The inhibitor of the enzyme isocitrate dehydrogenase-1 (IDH1),
ivosidenib, has been associated with an overall response rate of 41.6%
and CR rate of 21.6% in patients with relapsed or refractory IDH1-
mutated AML.
Question 17.37 The correct answer is D.
Enasidenib, ivosidenib, and gilteritinib used for the treatment of AML
and ATRA and arsenic trioxide used for the treatment of APL are agents
that can cause differentiation syndrome. High index of suspicion and
prompt initiation of steroids, typically dexamethasone 10 mg every 12
hours, are the crucial.
Question 17.38 The correct answer is B.
Based on results of phase III ADMIRAL trial, gilteritinib was approved
for monotherapy for patients with relapsed or refractory AML with
FLT3-ITD or TKD mutations.4 Gilteritinib was compared to salvage
chemotherapy and was associated with a significantly longer median OS
of 9.3 months versus 5.6 months in the chemotherapy group. The
percentage of patients who had CR with full or partial hematologic
recovery was 34.0% in the gilteritinib group and 15.3% in the
chemotherapy group.
Question 17.39 The correct answer is C.
Allogeneic transplants can confer an immune GVL effect in which
donor T and natural killer (NK) cells act to eradicate malignant cells that
survive the preparative regimen. Allogeneic hematopoietic stem cell
transplant (allo-HSCT) has the greatest antileukemia effects, but has
substantial risks of graft rejection, GVHD, and infections related to post-
transplant immune deficiency.
Question 17.40 The correct answer is A.
Outcomes of patients in CR are substantially better than for those with
active disease at the time of allogeneic stem cell transplant.
Question 17.41 The correct answer is D.
Allogeneic stem cell transplant is indicated for patients with AML in the
first remission and intermediate- or high-risk category, first remission
and age 60 to 75 if low comorbidity score, therapy-related or secondary
disease, primary induction failure, second or subsequent remission and
relapsed, and active disease based on the availability of donor and
presence of comorbidities. The availability of haploidentical donors with
possibly comparable outcomes, in addition to the use of reduced
intensity/myeloablative regimens, is expanding the use of this valuable
treatment in eligible patients.
Question 17.42 The correct answer is A.
MRD samples following induction, consolidation, and maintenance of
161 patients with non–T-lineage Ph-negative ALL treated in the
international MRC UKALL XII/ECOG 2993 trial were prospectively
analyzed.5 MRD status best discriminated outcome after 10 weeks of
therapy, when the relative risk of relapse was 8.95-fold higher in MRD-
positive patients and the 5-year recurrence-free survival (RFS) was 15%
compared to 71% in MRD-negative patients.
Question 17.43 The correct answer is B.
In a phase II trial of blinatumomab, a novel T-cell engaging CD19/CD3
bispecific antibody in patients with MRD or persistent disease after
induction/consolidation for B-ALL, 16 (76%) of 21 patients became
MRD negative, with an RFS of 78% at a median of 405 days. Although
all the other choices are effective therapies, blinatumomab has the
current FDA approval for this indication.

https://t.me/ALGRAWANY33
Question 17.44 The correct answer is D.
Some of the most significant advances in ALL therapy have been made
in the relapsed setting with three novel immunotherapeutic agents
recently approved for relapsed ALL. Blinatumomab, a CD19 BiTE
(Bispecific T-cell Engagers), is now approved for both MRD-positive
and relapsed/refractory ALL. In TOWER trial, adult patients with
relapsed/refractory B-ALL were randomized to blinatumomab versus
standard-of-care chemotherapy. Treatment with blinatumomab resulted
in better complete response rates (44% remission rate vs. 25%), longer
median duration of remission (7.3 months vs. 4.6 months), and better
median OS (7.7 months vs. 4.0 months).6 Inotuzumab ozogamicin is a
novel anti-CD22 antibody conjugated to calicheamicin. In the phase III
INOVATE study of inotuzumab versus conventional therapy in the
relapsed ALL setting, inotuzumab resulted in significantly higher
remission rates and higher numbers of patients proceeding to transplant,
leading to the approval of this agent in the relapsed ALL setting in
2017.7 Chimeric antigen receptor (CAR)–modified T-cell–directed
therapies such as tisagenlecleucel in CD19-positive malignancies have
shown very high CR rates and ushered in an exciting new era of cellular
therapies.
Question 17.45 The correct answer is A.
CLL cells characteristically express CD5, an antigen normally found on
T cells. CLL cells express the B-cell markers CD19, CD20, CD21,
CD23, and CD24. With the use of sensitive techniques, monoclonal
immunoglobulin can be detected in the serum of some patients, but only
5% to 10% of patients produce large enough quantities to be detected by
serum electrophoresis. C10 positivity could indicate follicular
lymphoma, which typically does not express CD5 and CD23.
Question 17.46 The correct answer is D.
Prognosis in CLL is correlated with the presence of certain
chromosomal abnormalities. When divided into five hierarchical
prognostic categories in order of highest risk, del(17p) was associated
with the worse survival, followed by del(11q), 12q trisomy, no
abnormalities, and del(13q) (as sole abnormality), with median OS times
of 32, 79, 114, 111, and 133 months, respectively. Expression of ZAP-
70 was correlated with IGHV mutation status, with increased expression
of ZAP-70 among unmutated IGHV cases. ZAP-70 and unmutated
IGHV are associated with an aggressive disease course.
Question 17.47 The correct answer is D.
Based on results of phase III trial RESONATE2, comparing ibrutinib to
chlorambucil, ibrutinib received first-line approval based as initial
therapy for patients aged 65 years and older with CLL.8 Response rates,
PFS, and OS were dramatically better with ibrutinib in the first-line
setting (median PFS, not reached vs. 18 months; 24-month OS, 88% vs.
85%). Although the trial was restricted to older or less fit patients, where
chlorambucil is considered a reasonable chemotherapy option, the FDA
awarded a broad label, such that ibrutinib can now be used as first-line
therapy in younger patients. Ibrutinib-based therapy has the advantage
of significant activity across all traditional risk groups in first-line and
relapsed CLL, including del(17p)/M-TP53, del(11q), and unmutated
IGHV CLL.
Question 17.48 The correct answer is C.
A distinctive pattern of response is noted with the use of ibrutinib. There
is rapid reduction in lymphadenopathy with concomitant initial rise in
the absolute lymphocyte count. The latter is related to inhibition of
chemokines and adhesion molecules that cause chemotaxis and
adherence of cells to the stroma. Recognizing this pattern of response is
important as the resulting lymphocytosis should not be considered a sign
of progression.
Question 17.49 The correct answer is A.
Venetoclax is administered once daily and can be associated with mild
toxicities, including gastrointestinal intolerance and neutropenia.
However, it is a highly potent inducer of CLL cell apoptosis, accounting
for the most notable toxicity of tumor lysis syndrome (TLS) that occurs

https://t.me/ALGRAWANY33
if patients are started at too high a dose or if they dose escalate too
rapidly. Therefore, patients are started at a low daily dose of 20 mg and
escalated weekly over 5 weeks to the target 400 mg daily, to mitigate
significant TLS risk. Even with this strategy, patients at high risk for
TLS by virtue of having bulky lymph nodes and high leukemia count
must be aggressively hydrated before and during initiation and
escalation. According to the package insert, these patients should be
hospitalized for therapy initiation and again at first dose escalation.
Question 17.50 The correct answer is D.
A positive direct antiglobulin (Coombs) test is seen in ~25% of cases
with CLL, but overt clinical autoimmune hemolytic anemia (AIHA)
occurs less frequently. Hypogammaglobulinemia occurs in ~50% of
patients with CLL. Approximately 25% of patients with CLL develop
second neoplasms, the most common being skin cancer.
Question 17.51 The correct answer is C.
Pentostatin and cladribine are the nucleoside analogs that are the
mainstay of treatment of hairy cell leukemia (HCL). There is no curative
therapy for HCL, except perhaps allogeneic stem cell transplant, and
some patients may be observed until they develop an indication for
treatment, which is generally related to progressive cytopenias.
Question 17.52 The correct answer is D.
When compared to imatinib, there is no data showing improvement in
PFS or OS with the use of second-generation TKIs. As most CML-
related progressions and deaths occur in the initial 2 years after
diagnosis, a longer follow-up of these studies is unlikely to change these
results.
Question 17.53 The correct answer is D.
If the patient had at least 3 months of treatment, a mutational screening
is advisable and will detect a mutation in 30% to 40% of cases. Factors
suggesting the presence of a mutation include a fast growth in the PCR
values or a quick deterioration of clinical parameters. If a mutation is
found, then the sensitivity to different TKIs should be checked because
different TKIs have different sensitivity profiles. For example, all TKIs,
except ponatinib, are inactive against T315I.
Question 17.54 The correct answer is B.
The risk of therapy-related MDS (t-MDS) varies from 1% (adjuvant
breast cancer chemotherapy studies) to 15% (heavily treated lymphoma
patients). Two types of t-MDS are recognized: type I is related to
alkylating agent exposure where MDS occurs after a latency period of 3
to 5 years in which monosomy 5 or 7 is cytogenetic characteristic of the
disease, and type II t-MDS is related to treatment with topoisomerase II
inhibitors, which generally arises after a short interval from
chemotherapy exposure and rapidly progresses to AML. Chromosome
11q23 abnormalities that involve the MLL gene are characteristic of this
type. Radiation-related MDS has a prognosis similar to de novo MDS.
Question 17.55 The correct answer is A.
Based on the recently published and updated phase Ib study, venetoclax
combined with decitabine or azacitidine in treatment-naive, elderly
patients with AML resulted in a CR + CRi (complete remission with
incomplete hematologic recovery) rate of 73% in patients treated with
venetoclax 400 mg plus hypomethylating agent cohort, including
decitabine or azacitidine.9 Median age was 74 years, with poor-risk
cytogenetics in 49% of patients. This novel combination was effective
and well tolerated in elderly patients with AML.
Question 17.56 The correct answer is A.
ESAs are the most widely used treatment for anemia in lower risk MDS.
Patient selection improves response probability to ESAs and conserves
resources. Using a simple model based on pretreatment endogenous
serum erythropoietin concentration (<100, 100–500, or >500 U/L) and
red blood cell (RBC) transfusion burden (< or ≥2 U/month), one would
distinguish three response categories: high probability of erythroid
response (74%), intermediate (23%), and low (7%).

https://t.me/ALGRAWANY33
Question 17.57 The correct answer is B.
Lenalidomide is approved by the FDA for the treatment of transfusion-
dependent anemia in lower risk MDS patients with a chromosome 5q
deletion [del(5q)] with or without additional cytogenetic abnormalities.
In the lenalidomide registration (MDS-003) phase II clinical trial, the
overall transfusion response rate was 76%, with 67% achieving TI.10 The
median time to response was 4.6 weeks, accompanied by a median rise
in hemoglobin of 5.4 g/dL. Median duration of response exceeded 2
years and is longer in patients with isolate del(5q). Lenalidomide was
administered at a dose of 10 mg daily either continuously or for 21 days
every 4 weeks.
Question 17.58 The correct answer is B.
AZA-001 trial was the first study to demonstrate a survival advantage
for any treatment in higher risk MDS.11 Patients with intermediate- or
high-risk MDS were randomized to treatment with either azacitidine or
conventional care regimens (CCRs) that included either best supportive
care alone, low-dose cytarabine, or intensive AML-type induction
chemotherapy. The median OS was significantly extended in the
azacitidine group compared to CCR (24.5 months vs. 15 months). AML
progression was delayed by >14 months with the azacitidine treatment.
Hematologic response rates also favored azacitidine, with 45%
achieving RBC TI compared to 11% with CCR.

References
1. Stone RM, Mandrekar SJ, Sanford BL, et al. Midostaurin plus chemotherapy for acute myeloid
leukemia with a FLT3 mutation. N Engl J Med. 2017;377:454–464.
2. Lancet JE, Uy GL, Cortes JE, et al. CPX-351 (cytarabine and daunorubicin) liposome for injection
versus conventional cytarabine plus daunorubicin in older patients with newly diagnosed secondary
acute myeloid leukemia. J Clin Oncol. 2018;36:2684–2692.
3. Stein EM, DiNardo CD, Pollyea DA, et al. Enasidenib in mutant IDH2 relapsed or refractory acute
myeloid leukemia. Blood. 2017;130:722–731.
4. Perl AE, Martinelli G, Cortes JE, et al. Gilteritinib or chemotherapy for relapsed or refractory
FLT3-mutated AML. N Engl J Med. 2019;381:1728–1740.
5. Rowe JM, Buck G, Burnett AK, et al. Induction therapy for adults with acute lymphoblastic
leukemia: results of more than 1500 patients from the international ALL trial: MRC UKALL
XII/ECOG E2993. Blood. 2005;106:3760–3767.
6. Kantarjian H, Stein A, Gökbuget N, et al. Blinatumomab versus chemotherapy for advanced acute
lymphoblastic leukemia. N Engl J Med. 2017;376:836–847.
7. Kantarjian HM, DeAngelo DJ, Stelljes M, et al. Inotuzumab ozogamicin versus standard therapy
for acute lymphoblastic leukemia. N Engl J Med. 2016;375:740–753.
8. Burger JA, Tedeschi A, Barr PM, et al. Ibrutinib as initial therapy for patients with chronic
lymphocytic leukemia. N Engl J Med. 2015;373:2425–2437.
9. DiNardo CD, Pratz K, Pullarkat V, et al. Venetoclax combined with decitabine or azacitidine in
treatment-naive, elderly patients with acute myeloid leukemia. Blood. 2019;133:7–17.
10. List A, Dewald G, Bennett J, et al. Lenalidomide in the myelodysplastic syndrome with
chromosome 5q deletion. N Engl J Med. 2006;355:1456–1465.
11. Fenaux P, Mufti GJ, Hellstrom-Lindberg E, et al. Efficacy of azacitidine compared with that of
conventional care regimens in the treatment of higher-risk myelodysplastic syndromes: a
randomised, open-label, phase III study. Lancet Oncol. 2009;10:223–232.
___________
Corresponding chapters in DeVita, Hellman, and Rosenberg’s Cancer: Principles & Practice of
Oncology, Eleventh Edition: 101 (Molecular Biology of Acute Leukemias), 102 (Management of Acute
Leukemias), 103 (Molecular Biology of Chronic Leukemias), 104 (Chronic Myeloid Leukemia), 105
(Chronic Lymphocytic Leukemias), 106 (Myelodysplastic Syndromes).

https://t.me/ALGRAWANY33
18 Lymphomas
Neha Mehta-Shah

QUESTIONS

Each of the numbered items below is followed by lettered answers. Select the
ONE lettered answer that is BEST in each case unless instructed otherwise.

Question 18.1 An 8-year-old child is brought to clinic by his mother due to


a jaw lump that has been increasing in size over the past month. The mother
states that the lump is preventing her child from eating. Physical examination
reveals a child in moderate distress with a left jaw mass that displaces teeth
and is impinging on the trachea. Biopsy reveals sheets of atypical
lymphocytes with areas of necrosis and hemorrhage. Ki-67 is 99%. Flow
cytometry shows B-cell markers as well as CD10 and BCL-6. CD5, BCL-2,
and TdT are absent. Which of the following is the most common
translocation in this malignancy?
A. t(1;14)
B. t(2;5)
C. t(8;14)
D. t(9;22)
Question 18.2 A 45-year-old woman presents with abdominal distension, a
30-lb unintentional weight loss, and back pain. Computed tomography (CT)
of the abdomen shows ascites and a 7-cm retroperitoneal mass as well as
bilateral external iliac and inguinal lymphadenopathy. Positron emission
tomography (PET)/CT shows cervical, axillary, mediastinal, retroperitoneal,
external iliac, and inguinal lymphadenopathy, as well as evidence of
fluorodeoxyglucose (FDG)-avid omental nodules, scattered bony lesions, and
hepatic masses. Excisional biopsy of the inguinal lymph node shows sheets
of large atypical lymphocytes, which stain positive for CD20, CD19, CD10,
MYC (50%), and BCL2 (60%) and negative for CD3, CD5, and CD30.
Which of the following tests would potentially impact your decision
regarding frontline therapy?
A. Immunohistochemistry for Epstein–Barr virus (EBV)–encoded small
RNA in situ hybridization (EBER-ISH)
B. Mutational analysis for TP53
C. Fluorescence in situ hybridization (FISH) for translocations in the
oncogenes for MYC, BCL2, and BCL6
D. Immunohistochemical analysis for cyclin D1
Question 18.3 Which of the following syndromes is associated with a
higher risk of adult lymphoma as opposed to childhood lymphoma?
A. Ataxia-telangiectasia
B. Human immunodeficiency virus/acquired immunodeficiency syndrome
(HIV/AIDS)
C. Wiskott–Aldrich syndrome
D. X-linked lymphoproliferative syndrome
Question 18.4 A 35-year-old Caucasian male presents to his primary doctor
due to 2 months of gradually worsening fevers, chills, and night sweats that
soak through multiple pillows. He recalls having lost some weight over the
past months, involuntarily, but is most bothered by an itchy rash on his
abdomen that has recently started to ulcerate. Physical examination reveals
painless lymphadenopathy. Imaging with a PET/CT shows evidence of FDG-
avid cervical, axillary, retroperitoneal, and inguinal lymphadenopathy.
Excisional biopsy of an axillary lymph node shows pleomorphic large cells
with prominent nucleoli and horseshoe-shaped nuclei. Immunophenotyping
shows the large atypical cells are universally positive for CD30, CD3, and
ALK and negative for CD20. Bone marrow biopsy shows involvement with a
large atypical lymphocyte population with the same immunophenotype. What
is the best systemic treatment option for this patient?
A. Doxorubicin, bleomycin, vinblastine, and dacarbazine (ABVD) for six
cycles

https://t.me/ALGRAWANY33
B. Brentuximab vedotin alone for six cycles
C. Brentuximab vedotin in combination with cyclophosphamide,
doxorubicin, and prednisone (BV-CHP) for six cycles
D. Cyclophosphamide, doxorubicin, vincristine, and prednisone (CHOP)
for six cycles
Question 18.5 A 55-year-old African American man presents to his
primary care physician (PCP) with abdominal pain that has been gradually
increasing over 3 months. He notes that he has been urinating less than
normal. Furthermore, he complains of persistent fevers and drenching night
sweats and notes that he has unintentionally lost 15 lb in the past 2 months.
The patient is less active than his baseline but is able to perform light
housework without difficulty. Physical examination reveals nontender
adenopathy in the bilateral cervical and left axillary regions. PET/CT shows
FDG-avid bilateral cervical, left axillary, and mediastinal lymphadenopathy
as well as a 7-cm retroperitoneal mass, resulting in bilateral hydronephrosis.
Complete blood count (CBC) is within normal limits. Lactate dehydrogenase
(LDH) is 210 IU/L (upper limit of normal 250 IU/L). Bone marrow biopsy
demonstrates no evidence of disease. Which of the following best describes
the patient’s International Prognostic Index (IPI) score?
A. 0
B. 1
C. 2
D. 3
Question 18.6 Human herpesvirus-8 (HHV8) is associated with which of
the following diseases?
A. Burkitt lymphoma
B. Germinal center phenotype diffuse large B-cell lymphoma (DLBCL)
C. Monocytoid B-cell lymphoma
D. Primary effusion lymphoma
Question 18.7 A 50-year-old woman with a history of bilateral textured
breast implants for cosmetic purposes 9 years ago presents with unilateral
breast swelling. Ultrasound of the breast shows a seroma without masses
visualized. There was no evidence of lymphadenopathy. Mammogram
showed no evidence of masses or calcifications. What is the appropriate next
step?
A. Lumpectomy
B. Aspiration of the fluid with analysis for cells expressing CD30
C. Replacement of the breast implant without removal of the capsule
D. Amoxicillin oral antibiotic therapy for 10 days
Question 18.8 Which of the following B-cell lymphomas demonstrates
CD10 positivity and the t(14;18) translocation?
A. Burkitt lymphoma
B. Follicular lymphoma
C. Lymphoplasmacytic lymphoma
D. Mantle cell lymphoma
Question 18.9 A 66-year-old Caucasian male with HIV and CD4 count of
53 is being worked up by his oncologist for a new diagnosis of DLBCL,
which involves diffuse lymphadenopathy, the paranasal sinuses, testes,
adrenal glands, and bone marrow on PET/CT. His LDH is 400 IU/L (upper
limit of normal 250 IU/L). What is the next best step in his management?
A. Initiation of testicular radiation
B. Lumbar puncture and brain magnetic resonance imaging (MRI)
C. Orchiectomy
D. Upper and lower endoscopy
Question 18.10 A 71-year-old man presents to clinic with abdominal pain.
Review of systems is also positive for fatigue and easy bleeding from the
nose and gums. Physical examination reveals splenomegaly. Pertinent labs
include hemoglobin of 12 g/dL, platelets of 16,000, LDH of 400 IU/L (upper
limit of normal 250 IU/L), and albumin of 3.6 g/dL. Peripheral smear reveals
“villous lymphocytes.” Flow cytometry is positive for CD19, CD20, and
CD22 and negative for CD5, CD25, and CD103. Which of the following is a
possible step in initial management?

https://t.me/ALGRAWANY33
A. Single-agent rituximab
B. Autologous stem cell transplant
C. Cladribine
D. Alemtuzumab
Question 18.11 A 51-year-old woman is diagnosed with stage IB mycosis
fungoides, with 25% body surface area involvement with patches of
involvement in the skin. She does not have evidence of disease in the blood
or lymph nodes based on CT of the chest, abdomen, and pelvis or peripheral
blood flow cytometry. Treatment is started with bexarotene. Which of the
following laboratory parameters should be routinely monitored while on
bexarotene?
A. Lipase
B. Thyroid-stimulating hormone (TSH)
C. Free T4
D. Potassium
Question 18.12 A 69-year-old Caucasian man presents with persistent
nausea associated with nonbloody, nonbilious emesis, loss of appetite, and
30-lb weight loss over ~1 year. He also recalls having occasional dark stools.
Physical examination reveals splenomegaly. PET/CT reveals FDG avidity of
intra-abdominal lymph nodes as well as uptake in the stomach and small and
large bowel. Upper and lower endoscopy reveals a nonbleeding gastric ulcer
and diffuse polyps in the colon. Biopsies are performed. Cells express surface
immunoglobulin M (IgM) and IgD along with CD5, CD19, CD20, and cyclin
D1 and are negative for CD23. Cytogenetics reveals t(11;14). Which of the
following statements is accurate regarding the next step in his management?
A. Mutations in the Ig heavy chain are associated with improved
prognosis.
B. Treatment with ibrutinib in the upfront setting is limited because of its
association with peripheral neuropathy.
C. Frontline therapy for elderly patients with good performance status
(PS) includes bendamustine and rituximab and high-dose cytarabine.
D. Maintenance rituximab has been associated with improved overall
survival following autologous transplant in younger fit patients.
Question 18.13 A 46-year-old Caucasian man presents to clinic due to
drenching night sweats associated with persistent, high-grade fevers and 40-
lb weight loss over the past 6 months. Physical examination reveals
lymphadenopathy of the cervical and axillary nodes as well as
hepatosplenomegaly. PET/CT reveals avid uptake throughout the
mediastinum, stomach, and small bowel, with multiple nodes measuring up to
2 cm. FISH reveals BCL-1/IgH fusion gene product. Additional studies show
decreased expression of p21, p27, and p53. Which of the following
statement(s) regarding the illness is CORRECT?
A. Cells arise from antigen-naive B cells of the inner mantle zone.
B. Most patients present in early-stage disease.
C. The condition is more common in females.
D. Tumor cells strongly express surface IgG and IgD as well as CD19 and
CD20.
Question 18.14 A 26-year-old Caucasian man with no past medical history
presents to the emergency department with right knee pain that occurred
suddenly while playing volleyball. He is found to have a pathologic fracture
of the right distal femur. Further imaging with PET/CT reveals localized
uptake of the right femur and tibia. Flow cytometry shows that cells are
positive for CD19, CD20, and CD22. Immunohistochemistry also reveals that
there are large atypical cells that are positive for CD10 and negative for BCL-
6 and MUM-1. FISH for translocations of MYC, BCL2, and BCL6 is
negative. Which of the following statements regarding disease subtype is
accurate?
A. Ig gene shows intraclonal homogeneity.
B. Nuclear factor kappa beta signaling is the primary driver of
proliferation and metastases.
C. Survival is better compared to other subtypes.
D. The subtype is often associated with t(8;14).
Question 18.15 A 63-year-old woman with a past medical history of

https://t.me/ALGRAWANY33
hepatitis B presents to the emergency department with shortness of breath.
She notes that it has been worsening over the past 3 to 4 months but became
unbearable over the past week. Review of systems is positive for involuntary
weight loss of 30 lb over 4 months and pedal edema. Physical examination
reveals pitting edema to the knees bilaterally. PET/CT scan shows diffuse
adenopathy of the mediastinum and the pelvis, with the largest nodes
measuring 2.5 cm. No extranodal disease is appreciated. Excisional biopsy
reveals proliferating large and small lymphocytes. Ki-67 is 85%. Flow
cytometry reveals cells that are positive for CD19, CD20, and CD22 and
negative for CD10, CD5, CD30, and BCL-6. FISH for translocations of
MYC, BCL2, and BCL6 is negative. Which of the following statements
regarding management is accurate?
A. R-CHOP chemotherapy is appropriate.
B. ABVD chemotherapy is preferred as first-line management.
C. Allogeneic stem cell transplant is the treatment of choice if the patient
were to relapse with chemosensitive disease.
D. Patients with hepatitis B cannot be treated safely with rituximab-based
chemotherapy.
Question 18.16 A 55-year-old Chinese man presents to his PCP due to
chronic sinus pain that has been present for 5 months. He also complains of
epistaxis, fevers, drenching night sweats, and 30-lb weight loss. Physical
examination is pertinent for temporal wasting and tenderness to palpation
around the paranasal sinuses as well as cervical adenopathy. Labs reveal
LDH of 425 IU/L (upper limit of normal 250 IU/L). CT of the sinuses reveals
an infiltrating mass in the midline nasal sinus. Immunophenotype reveals
atypical cells that are positive for CD2, CD56, and cytoplasmic CD3 and
negative for CD4, CD8, and surface CD3. Staining for EBER-ISH is positive
in the atypical cells. Circulating EBV is detected in the peripheral blood. The
cytoplasmic granule proteins, granzyme B and TIA-1, are present.
Cytogenetics reveals del(6)(q21;q25). Which of the following statement(s)
regarding treatment is CORRECT?
A. Allogeneic stem cell transplantation is a standard first-line therapy in
fit patients.
B. EBV-related hemophagocytic syndrome is often fatal and treated with
frontline radiation.
C. Localized disease is initially managed with concurrent chemotherapy
and radiation.
D. Patients with disseminated disease that is chemotherapy sensitive are
effectively treated with a combination of R-CHOP and radiotherapy
(RT).
Question 18.17 A 43-year-old woman presents to a dermatologist with a
diffuse skin rash, described as red and itchy with associated thickening of the
palms and soles that has been slowly progressing for the past 4 years.
Physical examination reveals erythematous patches on 10% of the body
surface area. FDG PET reveals the absence of extracutaneous disease. Skin
biopsy reveals Pautrier microabscesses. Immunophenotyping reveals
expression of CD2, CD3, and CD5 and without expression of CD7 or CD30.
Peripheral blood evaluation shows 500 cells/Kcumm with similar
immunophenotyped. What is the next best step in her management?
A. EPOCH chemotherapy combined with pentostatin and fludarabine with
interferon alpha
B. Total skin electron beam RT
C. Narrow band ultraviolet B (UVB) therapy in combination with topical
steroids
D. CHOP chemotherapy
Question 18.18 Which of the following accurately describes an aspect of
primary central nervous system (CNS) lymphoma (PCNSL)?
A. PCNSL primarily occurs in patients with a history of uncontrolled
HIV.
B. Incidence has decreased fivefold from 1985 to 1997 due to
advancement in the treatment of immunosuppressed patients.
C. Therapy with high-dose methotrexate-based regimens can be given for
curative intent.
D. Presents most commonly in the occipital lobe, manifesting as vision
changes and gait abnormalities, but intraocular involvement is

https://t.me/ALGRAWANY33
uncommon.
Question 18.19 Which of the following statement(s) regarding therapy for
relapsed DLBCL is CORRECT?
A. Patients with relapsed DLBCL are only treated for palliative intent.
B. Chimeric antigen T cells are approved for DLBCL for relapse after one
prior therapy.
C. Autologous stem cell transplant for those who are chemoresponsive to
salvage chemotherapy can be given for curative intent.
D. Most relapses from DLBCL occur after 5 years from initial therapy.
Question 18.20 A 71-year-old Caucasian man is brought to his PCP by his
wife, who says that he has been exhibiting increasingly aggressive behavior
over the past 5 months, manifesting as irritability and outbursts of anger, both
of which are outside his norm. She also says that he has had two motor
vehicle accidents in the same time period, due to not noticing traffic lights.
Neurologic examination reveals decreased ability to follow commands,
20/200 vision in both eyes, and left-sided weakness. Brain MRI shows a 4 cm
× 5 cm solitary, nonhemorrhagic mass in the right frontal lobe. Lumbar
puncture reveals predominance of clonal lymphocytes. Biopsy indicates cells
that are positive for BCL-2, BCL-6, and MUM1 as well as CD19, CD20, and
CD22. PET/CT shows no evidence of disease outside the brain. What is the
next best step in his management?
A. High-dose methotrexate and rituximab, with or without Ara-C and
temozolomide
B. R-CHOP chemotherapy with intrathecal methotrexate
C. Six weeks of concurrent chemotherapy with temozolomide and
dexamethasone combined with focal RT
D. Systemic adriamycin and etoposide with high-dose dexamethasone
Question 18.21 Which of the following statements best characterizes
nodular lymphocyte-predominant Hodgkin lymphoma (NLPHL) in contrast
to classic Hodgkin lymphoma (cHL)?
A. Approximately 80% of patients with NLPHL have stage I to II diseases
at the time of diagnosis.
B. Bone marrow involvement is frequent with stage III to IV diseases.
C. Disease presentation manifests predominantly in thoracic nodes before
spreading to the mediastinum.
D. NLPHL cells express CD30 and CD45 as well as B-cell antigens.
Question 18.22 A 24-year-old woman presents to her physician with fever,
neck pain, fatigue, and 10-lb weight loss during the first trimester of
pregnancy. She is concerned about the health of her fetus. Physical
examination reveals nontender cervical adenopathy, leading to a biopsy,
which reveals cells that are positive for CD15 and CD30 and negative for
CD3, CD7, CD20, and CD45. Morphology is pertinent for Reed–Sternberg
cells in a background of inflammatory cells. Which of the following
accurately describes an aspect of care for such patients?
A. BEACOPP (bleomycin, etoposide, doxorubicin, cyclophosphamide,
vincristine, procarbazine and prednisone) chemotherapy is preferred to
ABVD for treatment during the first trimester due to the lack of
antimetabolites in the regimen.
B. MRI scanning is preferred for staging because it is nonteratogenic.
C. Long-term survival of treated pregnant women is inferior to that of
nonpregnant women with the disease.
D. RT with abdominal shielding can be safely used for bulky disease
above the diaphragm.
Question 18.23 A 25-year-old woman presents to the emergency
department with shortness of breath and cough with a 15-lb weight loss in the
past 8 weeks. Chest radiograph demonstrates a large mediastinal mass and a
right pleural effusion. CT of the chest shows a 17-cm anterior mediastinal
mass extending to the bilateral supraclavicular regions with concern for
compression to the superior vena cava. PET/CT shows no evidence of
infradiaphragmatic disease. Biopsy of the mass shows large atypical
lymphocytes, which are positive for CD20, CD79a, and weakly positive for
CD30 and without expression of CD5, CD10, and CD15. There is
compartmentalizing sclerosis seen. Laboratory evaluation shows a normal
CBC and comprehensive metabolic panel (CMP), but LDH is 505 IU/L

https://t.me/ALGRAWANY33
(upper limit of normal 250 IU/L). HIV, hepatitis, and serum pregnancy tests
are all negative. She has an ejection fraction of 70%. Which of the following
treatment options would you recommend?
A. BEACOPP
B. Brentuximab vedotin–AVD (doxorubicin, vinblastin, dacarbazine)
C. Dose-adjusted R-EPOCH (rituximab, etoposide, prednisone,
vincristine, cyclophosphamide, doxorubicin)
D. R-CHOP

ANSWERS

Question 18.1 The correct answer is C.


This patient has Burkitt lymphoma. The classic translocations are t(2;8),
t(8;14), and t(8;22), causing dysregulation of the c-Myc oncogene. The
most common translocation is t(8;14). The t(1;14) occurs in mucosa-
associated lymphoid tissue (MALT) lymphoma. The translocation
t(9;22) is the Philadelphia chromosome associated with B-cell acute
lymphoblastic leukemia (B-ALL) and chronic myeloid leukemia (CML).
The t(2;5) is the NPM1-ALK translocation associated with ALK and is
important in ALK-positive lung cancer and ALK-positive anaplastic
large cell lymphoma.
Question 18.2 The correct answer is C.
The patient has DLBCL, which is further categorized into those with
germinal center or non–germinal center phenotype. Given the high-risk
features, her presentation is concerning for high-grade B-cell lymphoma
or double-hit DLBCL, which is associated with rearrangements in the
oncogenes for MYC t(8;14) with BCL2 t(14;18) and/or BCL6 t(3;14).
Testing for these translocations is most commonly performed by FISH
but can also be performed by karyotype analysis. Patients with double-
hit DLBCL have worse prognosis and are considered for therapy with
dose-adjusted R-EPOCH instead of R-CHOP therapy. Dose-adjusted R-
EPOCH consists of continuous anthracycline-based therapy, and doses
are adjusted based on the depth of the nadir neutrophil and platelet
counts from chemotherapy. Cyclin D1 is associated with mantle cell
lymphoma. TP53 does not have therapeutic implications in DLBCL.
EBER-ISH testing does not affect upfront management of DLBCL but
can have prognostic implications.
Question 18.3 The correct answer is B.
HIV/AIDS increases the risk of adult lymphomas, compared to the other
choices, which are associated with an increased risk of childhood
lymphomas.
Question 18.4 The correct answer is C.
This patient has stage IV anaplastic large cell lymphoma, ALK positive.
ALK is detected in more than half of patients, being more common in
children and is more sensitive to chemotherapy than anaplastic large cell
lymphoma that does not express ALK. ECHELON-2, a randomized,
double-blind, international study in patients with CD30 expressing
peripheral T-cell lymphomas showed that BV-CHP was associated with
improved progression-free survival and overall survival compared to
CHOP.1 This improvement in outcomes was most pronounced in
patients with anaplastic large cell lymphomas. ABVD is an appropriate
treatment for Hodgkin lymphoma. Brentuximab vedotin is approved as a
single agent in the relapsed setting for anaplastic large cell lymphoma,
but this patient should be treated with curative intent with an
anthracycline-containing regimen.
Question 18.5 The correct answer is B.
The IPI score for lymphomas is based on five factors, including age >60,
PS ≥2, LDH > normal, stage III/IV, and ≥2 extranodal sites. Scores of 0
to 1, 2, 3, and 4 to 5 are classified as low risk, low-intermediate risk,
high-intermediate risk, and high risk, respectively. The patient has stage
III/IV disease but is younger than 60, with an Eastern Cooperative
Oncology Group (ECOG) PS of 0, normal LDH, and no extranodal sites.
Therefore, his IPI score is 1 (low risk).

https://t.me/ALGRAWANY33
Question 18.6 The correct answer is D.
Primary effusion lymphoma is associated with HHV8. Endemic Burkitt
lymphoma is associated with EBV, whereas hepatitis C viruses are
associated with monocytoid B-cell lymphoma. There is no established
viral etiology for DLBCL.
Question 18.7 The correct answer is B.
The patient’s clinical presentation is concerning for breast implant–
associated anaplastic large cell lymphoma, which is a rare form of T-cell
lymphoma associated with textured implants that is recognized as an
entity in the 2016 World Health Organization (WHO) classification of
lymphoid malignancies. Cases often present with delayed seromas 7 to
10 years following initial placement of the implant. For patients with
isolated disease in the seroma or limited involvement of the capsule,
primary therapy is surgical with complete en bloc excision of the
capsule and the implant. The disease is characterized by atypical
lymphocytes that express CD30.2
Question 18.8 The correct answer is B.
The two malignancies associated with CD10 positivity and t(14;18) that
result in increased expression of the antiapoptotic protein, BCL-2, are
follicular lymphoma and DLBCL.
Question 18.9 The correct answer is B.
Patients with large cell lymphoma and involvement of testes are
associated with a high risk of involvement in the CNS, which could
include brain parenchymal disease or leptomeningeal disease.
Involvement of the adrenal glands, kidneys, paranasal sinuses, breast,
and bone marrow has also been associated in some series with increased
risk of CNS involvement. A CNS IPI score has been developed to help
risk stratify patients at high risk of CNS involvement and shows that
each of the IPI factors and renal and adrenal involvement is associated
with increased risk of CNS involvement. A CNS IPI score of 4 or higher
is associated with a higher risk of CNS involvement.3 Therefore, in
patients who are at high risk for CNS involvement, initial evaluation
with brain MRI and lumbar puncture with cytology and flow cytometry
to evaluate for CNS involvement is important. The presence of CNS
involvement will impact the treatment plan. Although testicular
radiation may be administered following chemotherapy to prevent
testicular reoccurrence, it is not the next best step. An orchiectomy is not
indicated. Similarly, upper and lower endoscopies are rarely indicated in
DLBCL unless needed to obtain the initial diagnosis.
Question 18.10 The correct answer is A.
The patient has splenic marginal zone lymphoma (SMZL), which is
characterized by circulating villous lymphocytes that express CD19,
CD20, CD22, and BCL-2. The lack of CD103 and CD25 differentiates it
from hairy cell leukemia. Hemoglobin <12 g/dL, high LDH, and
albumin <3.5 g/dL predict a shorter survival of 50%. Possible treatment
options include chemotherapy with alkylating agents in combination
with rituximab, single-agent rituximab, and splenectomy. Upfront
autologous stem cell transplantation would not be a routine approach for
this indolent lymphoma that often responds well to single-agent
rituximab. Cladribine or pentostatin is a treatment option for hairy cell
leukemia. Alemtuzumab is an anti-CD52 antibody used in the treatment
in other hematologic malignancies, including T-cell prolymphocytic
leukemia.
Question 18.11 The correct answer is C.
Bexarotene is an oral retinoid approved in the treatment of cutaneous T-
cell lymphomas. Bexarotene is associated with central hypothyroidism
and hypertriglyceridemia. Therefore, free T4 and triglycerides should be
routinely monitored. Pancreatitis can be associated with other drugs
including brentuximab vedotin or very high triglycerides. TSH is often
low with bexarotene because it causes a central hypothyroidism, and
free T4 should be monitored. Most patients on bexarotene require
thyroid replacement and medications to control hypertriglyceridemia.4
Question 18.12 The correct answer is D.

https://t.me/ALGRAWANY33
The patient has mantle cell lymphoma. Bortezomib, not ibrutinib, is
characterized by its tendency to cause peripheral neuropathy. The
optimal frontline regimen for mantle cell lymphoma is highly debated.
Treatment options include intensive regimens such as alternating
anthracycline-based therapy with cytarabine, hyper-CVAD
(cyclophosphamide, vincristine, adriamycin, and dexamethasone), and
upfront autologous stem cell transplant. Maintenance rituximab
following autologous transplant has been associated with improved
overall survival in younger fit patients. These intensive regimens are
used in younger fit patients. The less intensive regimen of bendamustine
and rituximab is another option that demonstrates high overall response
rates and is often considered for older patients with mantle cell
lymphoma. However, high-dose cytarabine would not be used in elderly
patients. Ibrutinib is approved in the relapsed setting and is not
associated with a high rate of neuropathy. Mutations in the Ig heavy
chain are associated with improved prognosis in chronic lymphocytic
leukemia.
Question 18.13 The correct answer is A.
Mantle cell lymphoma is more common in males and typically presents
as stage IV disease. The tumor cells express IgM and IgD as well as B-
cell–associated antigens. The cells arise from antigen-naive B cells of
the inner mantle zone. Nuclear cyclin D1 is present in all cases and is
important for diagnosis.
Question 18.14 The correct answer is C.
The patient has germinal center B-cell (GCB) DLBCL, based on the
Hans criteria that categorizes DLBCL based on the presence or absence
of CD10, BCL6, and MUM1. Cases that are CD10 positive are always
of germinal center phenotype. GCB is associated with a better prognosis
than the activated B-cell (ABC) subtype DLBCL, which is associated
with nuclear factor kappa beta signaling. Translocation of t(8;14) is
associated with the MYC oncogene.
Question 18.15 The correct answer is A.
For patients with advanced-stage DLBCL, the standard-of-care
treatment is with R-CHOP chemotherapy for those patients who do not
have high-risk translocations. Abbreviated R-CHOP chemotherapy
combined with or without involved field RT is an appropriate treatment
option for low-risk, nonbulky, early-stage DLBCL. Autologous stem
cell transplant is reserved for patient with relapsed chemosensitive
disease. ABVD is the standard regimen for patients with Hodgkin
lymphoma. Patients with hepatitis B can be treated safely with
anthracycline-based chemotherapy for curative intent but are considered
for antiviral prophylaxis with entecavir.
Question 18.16 The correct answer is C.
The patient has extranodal natural killer (NK)/T-cell lymphoma, which
is often associated with EBV. Localized disease can be treated with a
combination of chemotherapy and RT. Use of asparaginase-based
regimens has led to improved outcomes in this patient population.
Treatment failure is common with RT alone. Rituximab is not indicated
in this T-cell lymphoma. Patients with disseminated disease have very
poor prognoses and should be enrolled in clinical trials. The role of
consolidative transplant in this population remains controversial, but
allogeneic transplant in first remission in chemosensitive patients is not
a standard therapy.
Question 18.17 The correct answer is C.
The patient has mycosis fungoides, which is a cutaneous T-cell
lymphoma that often has low level of involvement in the peripheral
blood. For limited disease localized to the skin, treatment options
include topical carmustine, mechlorethamine, oral bexarotene, or
phototherapy using either UVA or UVB. Other options for systemic
disease can include histone deacetylase inhibitors, pralatrexate,
brentuximab vedotin, or mogamulizumab. Because patients can be on
therapy until progression or intolerance, selection of therapies with
minimal cumulative toxicity is important. Phototherapy is often an
initial option with minimal cumulative toxicity. Use of total skin
electron beam radiation is effective but is more often used in more

https://t.me/ALGRAWANY33
extensive disease or advanced cases. Anthracycline-based therapies are
not given with curative intent in this patient population and should be
reserved for select cases.
Question 18.18 The correct answer is C.
The incidence of PCNSL has increased in immunocompetent
individuals, particularly in older individuals. Presentation in the frontal
lobe or multifocal lesions is most common, and patients can present with
personality changes, changes in the level of alertness, headaches, and
signs/symptoms of increased intracranial pressure. Intraocular
involvement occurs in 10% to 20% of cases. Therapy with high-dose
methotrexate is given for curative intent, often in combination with other
agents including rituximab. The optimal dose of high-dose methotrexate
remains debated, but for adequate CNS penetration, methotrexate from
3.5 to 8 g/m2 is often used.
Question 18.19 The correct answer is C.
DLBCL can be treated with curative intent. Most relapses from DLBCL
occur within the first 3 years following initial therapy, and relapses that
occur >5 years from initial therapy are rare. Long-term remissions are
seen with both autologous transplant in chemoresponsive patients and
chimeric antigen T cells. Chimeric antigen T cells are approved for
DLBCL following two lines of therapy and are being studied in the first
relapse.
Question 18.20 The correct answer is A.
The patient has PCNSL for which the backbone of therapy is high-dose
methotrexate combined with rituximab. Regimens that add to this
backbone include those with Ara-C, temozolomide, thiotepa, cytarabine,
vincristine, and/or procarbazine. Intrathecal methotrexate does not
provide sufficient CNS penetration for intraparenchymal brain
involvement. Concurrent radiation and temozolomide is used in other
CNS malignancies but is not used for curative intent in CNS lymphoma.
Concurrent high-dose methotrexate and whole-brain radiation has been
associated with significant encephalomalacia with cognitive decline,
particularly in those over the age of 60 years.
Question 18.21 The correct answer is A.
The atypical cells in NLPHL are CD45 positive and express B-cell
antigens such as CD19, CD20, CD22, CD79a, and PAX5. The cells do
not express CD30. The condition can transform into DLBCL in 2% to
6% of cases. Most patients present with early-stage disease. Bone
marrow involvement is rare, even with more advanced disease. Unlike
the contiguous spread seen in cHL, NLPHL often involves peripheral
nodes in a noncontiguous manner. Inguinal node involvement is
common. Overall, the response to therapy is more favorable, because
>90% of patients have a clinical remission and are alive at 10 years. The
optimal therapy is debated.
Question 18.22 The correct answer is B.
Hodgkin lymphoma is the fourth most common cancer diagnosed during
pregnancy. ABVD chemotherapy is the preferred regimen during
pregnancy because it does not include antimetabolites. Studies have
shown that maternal–fetal mortality in patients treated with ABVD for
Hodgkin lymphoma during pregnancy is not inferior to others.5 MRI is
preferred to CT scanning, in order to avoid exposing the fetus to
ionizing radiation. RT can be used above the diaphragm in the second or
third trimester if the patient has rapid progression of lymphadenopathy,
but chemotherapy-based approaches are preferred.
Question 18.23 The correct answer is C.
This patient has primary mediastinal large B-cell lymphoma, which is a
rare form of non-Hodgkin lymphoma that is most often limited to the
mediastinum and can present with superior vena cava syndrome.
Primary mediastinal large B-cell lymphoma is more common in young
women, and most women are in their third to fourth decades of life at
diagnosis. Pathology classically shows a large B-cell lymphoma with
weak CD30 expression and compartmentalizing sclerosis or thick bands
of fibrosis that separate the tumor cells into lobules of various size.
While outcomes with R-CHOP therapy were historically suboptimal,

https://t.me/ALGRAWANY33
recently studies with dose-adjusted R-EPOCH have shown excellent
results, with over 85% patients remaining disease free without radiation
consolidation. Dose-adjusted R-EPOCH should be considered for
patients with primary mediastinal large B-cell lymphoma. Brentuximab
vedotin–AVD is a treatment option for patients with advanced Hodgkin
lymphomas. BEACOPP is an option for high-risk Hodgkin lymphoma
as well.

References
1. Horwitz S, O’Connor OA, Pro B, et al. Brentuximab vedotin with chemotherapy for CD30-positive
peripheral T-cell lymphoma (ECHELON-2): a global, double-blind, randomised, phase 3 trial.
Lancet. 2019;393:229–240.
2. Clemens MW, Medeiros LJ, Butler CE, et al. Complete surgical excision is essential for the
management of patients with breast implant-associated anaplastic large-cell lymphoma. J Clin
Oncol. 2016;34:160–168.
3. Schmitz N, Zeynalova S, Nickelsen M, et al. CNS international prognostic index: a risk model for
CNS relapse in patients with diffuse large B-cell lymphoma treated with R-chop. J Clin Oncol.
2016;34:3150–3156.
4. Scarisbrick JJ, Morris S, Azurdia R, et al. U.K. consensus statement on safe clinical prescribing of
bexarotene for patients with cutaneous T-cell lymphoma. Br J Dermatol. 2013;168:192–200.
5. Pinnix CC, Osborne EM, Chihara D, et al. Maternal and fetal outcomes after therapy for hodgkin or
non-hodgkin lymphoma diagnosed during pregnancy. JAMA Oncol. 2016;2:1065–1069.
___________
Corresponding chapters in DeVita, Hellman, and Rosenberg’s Cancer: Principles & Practice of
Oncology, Eleventh Edition: 96 (Molecular Biology of Lymphoma), 97 (Hodgkin Lymphoma), 98
(Non-Hodgkin Lymphoma), 99 (Cutaneous Lymphomas), and 100 (Primary Central Nervous System
Lymphoma).
19 Plasma Cell Neoplasms
Ravi Vij and Scott Goldsmith

QUESTIONS

Each of the numbered items below is followed by lettered answers. Select the
ONE lettered answer that is BEST in each case unless instructed otherwise.

Question 19.1 A healthy 59-year-old woman presents to her primary care


physician for a routine annual physical. A complete metabolic panel is
significant for an elevated total protein of 9.7 mg/dL, an albumin of 3.9
mg/dL, a creatinine of 1.0 mg/dL, and a calcium of 9.1 mg/dL. A complete
blood count (CBC) shows a white blood cell count of 7.1, a hemoglobin
(Hgb) of 12.4 g/dL, and a platelet count of 233. Subsequent serum protein
electrophoresis (SPEP) and immunofixation show an immunoglobulin G
(IgG) kappa monoclonal peak of 1.4 g/dL. Serum free light chains show a
total kappa free light chain concentration of 327 mg/dL and a lambda free
light chain concentration of 0.9 mg/dL. Fluorodeoxyglucose-positron
emission tomography/computed tomography (FDG-PET/CT) scan and
magnetic resonance imaging (MRI) of the spine and pelvis are negative for
lytic or hypermetabolic bony lesions or plasmacytomas. A bone marrow
biopsy reveals a monoclonal plasma cell population comprising 70% of the
core biopsy sample. Which of the following management options should be
pursued next?
A. Fusion PET/MRI scanning to assess need for therapy
B. Initiation of antimyeloma therapy
C. Obtain HevyLite Assay
D. Order a next-generation sequencing (NGS) panel to assess for genetic
mutations

https://t.me/ALGRAWANY33
Question 19.2 A 66-year-old man with hypertension and long-standing
tobacco abuse undergoes a screening chest CT that identifies a 4.0 × 5.1 × 3.2
cm heterogeneous mass that appears to be originating from the left fourth rib
and eroding into the left fifth rib. He has been experiencing pain at that
location for 3 months now. His labs are notable for an Hgb of 13.6 g/dL, total
protein of 9.4 g/dL, albumin of 3.6 g/dL, creatinine of 1.1 mg/dL, and
calcium of 9.2 mg/dL. His SPEP demonstrates a monoclonal protein of 1.5
g/dL, identified as IgG lambda by immunofixation. Serum kappa free light
chain concentration is 3.8 mg/dL and lambda free light chain concentration is
17.9 mg/dL. A biopsy of the left rib mass demonstrates a clonal plasma cell
neoplasm, although a bone marrow biopsy obtained from his left iliac crest
demonstrates a normal percentage of plasma cells with a normal
immunophenotype by flow cytometry. Which of the following is the best
next step in his management?
A. Initiate antimyeloma therapy.
B. Obtain a skeletal survey.
C. Obtain a whole-body MRI.
D. Radiation therapy to the rib lesion.
Question 19.3 A 78-year-old man with a history of coronary artery disease
and hypertension is found to have an elevated immunoglobulin A (IgA)
kappa monoclonal protein at 0.9 g/dL. This prompts a bone marrow biopsy,
which demonstrates 15% plasma cells, all clonal. High-risk chromosomal
abnormalities are not identified. His labs are notable for a creatinine of 1.2
mg/dL, a calcium of 9.6 mg/dL, and an Hgb of 13.4 mg/dL. His kappa serum
free light chain concentration is 17 mg/dL and lambda serum free light chain
concentration is 1.6 mg/dL. A 24-hour urine protein electrophoresis identifies
a monoclonal protein of 730 mg/24 h. FDG-PET/CT does not reveal any
bony lytic or metabolically active lesions. Which of the following is the next
best step in his management?
A. Initiate bortezomib, lenalidomide, and dexamethasone.
B. Initiate daratumumab, lenalidomide, and dexamethasone.
C. Perform a radiographic skeletal survey.
D. Repeat SPEP and free light chains in 3 months.
Question 19.4 A 55-year-old woman presents at a local emergency room
with fatigue, myalgias, and light-headedness. Laboratory evaluation reveals a
white blood cell count of 5.5, an Hgb of 6.1 g/dL, and a platelet count of 122.
Chemistry panel shows a creatinine of 1.9 mg/dL and a calcium of 15.1
mg/dL. Workup of her anemia includes a SPEP that shows a monoclonal
protein at a concentration of 4.2 g/dL. Serum immunofixation reveals an IgG
kappa monoclonal protein. Bone marrow biopsy shows a kappa-restricted
plasma cell population comprising 30% of the core biopsy. Cytogenetics
reveals hypodiploidy, and fluorescence in situ hybridization (FISH) studies
show t(4;14). Which of the following best characterizes the risk category of
this patient’s multiple myeloma?
A. Double-hit myeloma due to FISH finding of t(4;14) and hypodiploidy
on cytogenetics
B. High risk due to cytogenetic findings of hypodiploidy
C. High risk due to FISH findings of t(4;14)
D. Standard risk by FISH and cytogenetics
Question 19.5 A 52-year-old woman presents to her oncologist 1 year
following an autologous stem cell transplantation (ASCT) for multiple
myeloma. She has been continued on maintenance therapy following her
transplant and has tolerated this well. She reports that she watched a
television documentary about genomics a week before her appointment and
asks her physician what genomic sequencing has revealed in multiple
myeloma. Which of the following is the best response?
A. Mutations in immunoglobulin heavy chain (IGH) gene locus are
ubiquitous.
B. Myeloma-associated mutations are limited to the protein-coding
regions of the genome.
C. Single-cell sequencing has revealed a consistent pattern of linear
immune cell evolution.
D. The most commonly mutated genes include KRAS and NRAS.
Question 19.6 A 59-year-old man with relapsed multiple myeloma is being
considered for a clinical trial that incorporates venetoclax in a genomically

https://t.me/ALGRAWANY33
targeted fashion. Which of the following cytogenetic abnormalities is likely a
principal inclusion criterion for this trial?
A. del 17p
B. t(4;14)
C. t(11;14)
D. t(14;16)
Question 19.7 An 82-year-old man with a history of diabetes mellitus
complicated by moderate peripheral neuropathy, is recently diagnosed with
multiple myeloma following a workup for new anemia and declining renal
function. The bone marrow biopsy demonstrates 35% clonal plasma cells,
and interphase FISH identifies a t(11;14) chromosomal abnormality. Which
of the following would be the most reasonable initial treatment?
A. Bortezomib, lenalidomide, and dexamethasone
B. Daratumumab, lenalidomide, and dexamethasone
C. Elotuzumab, lenalidomide, and dexamethasone
D. Ixazomib, lenalidomide, and dexamethasone
Question 19.8 A 48-year-old, otherwise healthy man is diagnosed with
multiple myeloma following a workup for anemia. His initial monoclonal
spike is 4.8 g/dL with immunofixation demonstrating an IgG lambda
monoclonal protein. Serum beta-2 microglobulin is 7.0 mg/L, albumin is 3.1
g/dL, and lactate dehydrogenase is within normal limits. Bone marrow biopsy
demonstrates 60% clonal plasma cells, and CD138 selected interphase FISH
identifies the chromosomal abnormalities t(4;14) and del(17p). His renal
function and cardiac function have been unaffected. Which of the following
is the best next step in his management?
A. Induction therapy with bortezomib, cyclophosphamide, and
dexamethasone
B. Induction therapy with carfilzomib, lenalidomide, and dexamethasone
C. Induction therapy with carfilzomib, pomalidomide, and dexamethasone
D. Induction therapy with lenalidomide and dexamethasone
Question 19.9 A 57-year-old resident of the United Kingdom has a new
diagnosis of multiple myeloma made after he presented with a monoclonal
peak of 6.0 g/dL, IgG lambda by serum immunofixation. CBC shows an Hgb
of 8.2 g/dL and a metabolic panel reveals no abnormalities. Bone marrow
biopsy reveals evidence of 55% involvement by a lambda-restricted
monoclonal plasma cell population. He is deemed to be an appropriate
candidate for ASCT. He starts therapy with lenalidomide, cyclophosphamide,
and dexamethasone, which he tolerates well. After two cycles of therapy, his
M-spike improves to 4.0 g/dL, and after four cycles of therapy his M-spike
remains at 4.0 g/dL. Which of the following strategies has been demonstrated
in a clinical trial to provide the best progression-free survival (PFS)?
A. Continue lenalidomide, cyclophosphamide, and dexamethasone for a
total of six cycles then proceed to autologous transplantation.
B. Proceed to ASCT.
C. Switch to bortezomib, cyclophosphamide, and dexamethasone until an
M-protein nadir is achieved and then proceed to autologous
transplantation.
D. Switch to daratumumab-based regimen and abandon plans for
autologous transplantation.
Question 19.10 A 67-year-old woman with Revised International Staging
System (R-ISS) Stage II multiple myeloma with hyperdiploid karyotype
presents for her pretransplant evaluation. She has completed four cycles of
bortezomib, lenalidomide, and dexamethasone induction therapy, and she has
achieved a very good partial response (VGPR). She is scheduled to receive
high-dose melphalan with an ASCT. She mentions that a friend of hers who
lives in France recently underwent tandem ASCT for multiple myeloma
following initial induction with bortezomib, cyclophosphamide, and
dexamethasone. According to the STAMINA study, which of the following is
the best choice of management for this patient?
A. Change therapy to carfilzomib, cyclophosphamide, dexamethasone and
delay ASCT until complete response (CR) is achieved.
B. Proceed with ASCT followed by bortezomib, lenalidomide, and
dexamethasone consolidation if CR is not achieved posttransplant.
C. Proceed with ASCT followed by lenalidomide maintenance if CR is

https://t.me/ALGRAWANY33
not achieved posttransplant.
D. Plan for tandem ASCT if CR is not achieved posttransplant.
Question 19.11 A 57-year-old woman is diagnosed with an IgG kappa
multiple myeloma. Her initial labs reveal a monoclonal peak of 4.2 g/dL,
with kappa free light chains of 160 mg/dL and lambda free light chains of 1.2
mg/dL. A bone marrow biopsy shows 50% involvement by a kappa-restricted
plasma cell population. She begins induction with bortezomib, lenalidomide,
and dexamethasone and attains a VGPR after four cycles of therapy. She
undergoes ASCT and, on workup 100 days posttransplant, a bone marrow
biopsy shows no evidence of persistent plasma cells. SPEP and
immunofixation are negative for any evidence of monoclonal protein. Serum
free light chains are repeated and are normal. She is positive for minimal
residual disease (MRD) by NGS. What would be the appropriate information
to pass onto the patient regarding this finding?
A. MRD positive patients have inferior PFS and overall survival (OS)
when compared with patients achieving stringent CR who are MRD
negative.
B. NGS-based MRD assessment (cloneSEQ®) analyzes a panel of
recurrent mutations in multiple myeloma to detect disease presence.
C. Positive MRD detection by NGS requires confirmation by allele-
specific oligonucleotide quantitative polymerase chain reaction (ASO-
qPCR) technology.
D. The patient should undergo repeat (tandem) ASCT with a goal to
obtain MRD negativity.
Question 19.12 A 62-year-old woman with multiple myeloma completed
induction therapy with bortezomib, lenalidomide, and dexamethasone and is
now 100 days out from her ASCT. Pretreatment bone marrow biopsy
demonstrated 45% clonal plasma cells with cytogenetics demonstrating
del(17p) in 120 of 200 cells analyzed by interphase FISH. Her bone marrow
biopsy at day 100 posttransplant identified 4% plasma cells, and CD138
purified FISH analysis does not identify any chromosomal abnormalities. She
has achieved a CR based on her SPEP. NGS of the bone marrow aspirate
reveals the presence of MRD. Which of the following classes of drugs is a
vital component to this patient’s maintenance strategy?
A. Proteasome inhibitors
B. Immunomodulatory drugs
C. Monoclonal antibodies to CD38
D. Monoclonal antibodies to SLAMF7
Question 19.13 An otherwise healthy 64-year-old woman was diagnosed
with multiple myeloma 18 months ago. She had started induction therapy
with bortezomib, lenalidomide, and dexamethasone, which was unfortunately
delayed because she developed Stevens–Johnson syndrome secondary to the
lenalidomide. Following a 6-week recovery, she reinitiates induction therapy
with bortezomib, cyclophosphamide, and dexamethasone, achieving a partial
response. She undergoes high-dose melphalan with ASCT, achieving a
VGPR and declines any form of maintenance therapy. She now has evidence
of progressive disease with worsening anemia and a rise in M-protein.
Eastern Cooperative Oncology Group (ECOG) performance status is 1, and
she does not have any other active health issues. Which of the following is
the best option for therapy?
A. Daratumumab, pomalidomide, and dexamethasone
B. Daratumumab, bortezomib, and dexamethasone
C. Elotuzumab, pomalidomide, and dexamethasone
D. Isatuximab, pomalidomide, and dexamethasone
Question 19.14 A 62-year-old woman with a history of multiple myeloma
presents for initial evaluation. She was diagnosed 2 years before and
underwent initial induction therapy with bortezomib, lenalidomide, and
dexamethasone, followed by ASCT approximately 18 months ago. She was
not treated with maintenance therapy following transplant due to personal
preference. Nine months after her transplant, she develops worsening anemia
(Hgb 8.7 g/dL) and a rising M-protein level (0.4–3.9 g/dL) and is treated with
daratumumab, bortezomib, and dexamethasone. She achieves a partial
response after two cycles, which persists for 6 months; however, her M-
protein rises again and she has new bony lytic lesions on FDG-PET/CT
imaging. Which of the following carfilzomib-based regimens has been

https://t.me/ALGRAWANY33
demonstrated to improve survival in a prospective phase III trial in patients
with relapsed/refractory multiple myeloma?
A. Carfilzomib (20/27 mg/m2) twice weekly in combination with
lenalidomide and dexamethasone
B. Carfilzomib (20/36 mg/m2) twice weekly in combination with
lenalidomide and dexamethasone
C. Carfilzomib (20/56 mg/m2) twice weekly in combination with
lenalidomide and dexamethasone
D. Carfilzomib (20/70 mg/m2) once weekly in combination with
lenalidomide and dexamethasone
Question 19.15 A 62-year-old woman with relapsed multiple myeloma
presents for recommendations regarding therapy. Four years ago she
underwent induction therapy with bortezomib, lenalidomide, and
dexamethasone, followed by high-dose melphalan with an ASCT. She
achieved a VGPR and remained on lenalidomide maintenance therapy
thereafter. However, after 11 months, while on lenalidomide, she develops
biochemical relapse. Bortezomib is added but her disease continues to
progress. She is switched to carfilzomib, cyclophosphamide, and
dexamethasone, which leads to a partial response that persists for 13 months.
Upon disease progression, as manifested by new bony lesions, therapy is
changed to daratumumab, pomalidomide, and dexamethasone. Again, she
achieves a partial response that lasts for 9 months; however, her most recent
myeloma markers are consistent with disease progression. The patient has a
good performance status without significant organ dysfunction and wishes to
proceed with myeloma-directed therapy. She has no living sibling and did
have an unrelated donor search performed, which identified an unrelated
donor with two human leukocyte antigen (HLA) mismatch (six of eight
match). Which of the following is the best next step in her management?
A. Initiate therapy with elotuzumab, ixazomib, and dexamethasone.
B. Initiate therapy with selinexor plus dexamethasone.
C. Perform a mismatched unrelated donor allogeneic stem cell transplant.
D. Perform a second ASCT.
Question 19.16 A 65-year-old woman with relapsed/refractory multiple
myeloma presents for a second opinion regarding clinical trial options. She
has been treated with bortezomib, carfilzomib, lenalidomide, pomalidomide,
daratumumab, and elotuzumab in various combinations through which her
disease has progressed. She is interested in learning about experimental
therapies targeting B-cell maturation antigen (BCMA). Which of the
following statements is most accurate regarding BCMA and BCMA-targeted
therapies?
A. BCMA is required for differentiation of lymphoid progenitor cells into
mature B cells.
B. BCMA is generally restricted to and highly expressed in mature B cells
and plasma cells.
C. BCMA-targeting therapies are ineffective in patients who have disease
that is refractory to CD38-targeting monoclonal antibodies.
D. BCMA-targeting therapies are only effective when combined with an
immunomodulatory drug.
Question 19.17 A 72-year-old woman with newly diagnosed multiple
myeloma presents for consideration for therapy. She has been diagnosed
following a pathologic fracture of her humerus secondary to a lytic bony
lesion and is noted to have anemia and significant renal impairment with a
creatinine clearance (CrCl) of 17 mL/min that does not improve with
intravenous fluids. In addition to induction therapy for her myeloma, which
of the following would be most appropriate to add to her regimen?
A. Denosumab
B. Ibandronate
C. Teriparatide
D. Zoledronic acid
Question 19.18 A 55-year-old man presents to your clinic for an initial
consultation. He was referred after he was noted to have lower extremity
edema and further workup revealed the presence of a nephrotic syndrome. He
undergoes a renal biopsy, which shows evidence of amyloid deposition in the
glomeruli, positive by Congo red staining. Mass spectrometry confirms

https://t.me/ALGRAWANY33
amyloid light chain (AL) amyloidosis. His labs reveal a lambda free serum
light chain concentration of 140 mg/L and a kappa free light chain
concentration of 2 mg/L. His performance status is rated as an ECOG 2, and
he has symptoms consistent with New York Heart Association (NYHA)
Grade III congestive heart failure. What should be the next step in his
management?
A. ASCT with melphalan 140 g/m2.
B. ASCT with melphalan 200 g/m2.
C. Start cyclophosphamide, bortezomib, and dexamethasone.
D. Start therapy with ixazomib-based therapy.
Question 19.19 A 59-year-old previously healthy man is admitted to the
hospital with progressive paresthesias and muscle weakness that have
developed over the last 6 weeks. Initially he had difficulty pushing down on
the accelerator in his car, but the weakness has progressed to where he can no
longer rise from a chair or grasp objects. In addition to evidence of profound
distal and proximal muscle weakness, on his physical examination, he is
noted to have 3+ lower extremity pitting edema, hyperpigmentation of his
skin, and hepatomegaly. Initial laboratory workup is notable for an Hgb of
17.8 g/dL, total protein of 8.2 g/dL, and albumin of 3.1 g/dL. SPEP shows a
monoclonal protein of 2.9 g/dL, and immunofixation characterizes it as IgG
lambda. In addition to the information known, which of the following lab
abnormalities would be required to make a diagnosis of POEMS
(polyneuropathy, organomegaly, endocrinopathy, monoclonal gammopathy,
skin changes) syndrome?
A. Decreased insulin-like growth factor 1
B. Decreased serum cortisol
C. Elevated thyroid-stimulating hormone level
D. Elevated vascular endothelial growth factor (VEGF) level
Question 19.20 A 71-year-old man with a history of hypertension presents
to the emergency room with confusion. His family states that he had been
complaining of increased tiredness and fatigue that has progressively
worsened over the past 6 months. In addition to the confusion, recently, he
has been sleeping more. His examination is notable for somnolence with a
Glasgow Coma Scale of 11, no identifiable motor deficits, and papilledema
on fundoscopy. His initial lab work identifies an Hgb of 5.1 g/dL, platelets of
64,000/µL, and no circulating blasts. Creatinine is 2.4 mg/dL, total protein is
9.1 g/dL, and albumin is 2.7 g/dL. Quantitative immunoglobulins reveal a
total immunoglobulin M (IgM) of 7,250 mg/dL. CT scan of the head does not
demonstrate any subdural or intracranial bleeding. Serum viscosity measures
at 5.5 cP. Which of the following is the next best step in his management?
A. Bendamustine and rituximab
B. Ibrutinib and rituximab
C. Packed red blood cell transfusion
D. Plasmapheresis

Answers

Question 19.1 The correct answer is B.


The International Myeloma Working Group (IMWG) released an update
to the criteria for diagnosis of multiple myeloma in 2014. Historically
defined by the presence of the CRAB (hypercalcemia, renal failure,
anemia, or bony disease) criteria, denoting the presence of end-organ
damage, the newly updated criteria include a number of radiographic or
biochemical criteria. The defining criteria for multiple myeloma now
include a clonal bone marrow plasma cell population ≥10% or biopsy-
proven bony or extramedullary plasmacytoma and one or more of the
following defining elements:
Evidence of end-organ damage attributed to the plasma cell disorder
Hypercalcemia (>1 mg/dL above upper limit of normal [ULN] or >11
mg/dL)
Renal insufficiency (CrCl <40 mL/min or serum creatinine >2 mg/dL)
Anemia (Hgb >2 g/L below lower limit of normal [LLN] or an Hgb
<10 g/L)
Bone lesions (one or more osteolytic lesions on radiograph, CT, or

https://t.me/ALGRAWANY33
PET/CT)
Any of the following biomarkers of malignancy:
Clonal bone marrow population ≥60%
Involved/uninvolved serum free light chain ratio ≥100 (the involved
free light chain must be ≥100 mg/L)
>One focal lesion on MRI
Because this patient’s bone marrow biopsy reveals a monoclonal
plasma cell population encompassing 70% of the bone marrow, he meets
the diagnosis of multiple myeloma on the basis of the 2014 IMWG
criteria. In addition, this patient’s ratio of involved/uninvolved serum
free light chains is >100, with an involved chain concentration >100
mg/dL. By current definition, the patient has overt multiple myeloma
and will benefit from treatment with an active antimyeloma regimen.
This patient did have appropriate imaging workup performed for
suspected multiple myeloma according to IMWG consensus, which
recommends either low-dose whole-body CT scan or FDG-PET/CT scan
initially, and if negative or inconclusive a subsequent axial or whole-
body MRI. Fusion PET-MRI is an evolving imaging strategy in multiple
myeloma but its role is currently limited to investigational studies. The
HevyLite assay is a novel technique of quantifying intact light and
heavy chain monoclonal proteins. It measures intact immunoglobulin
proteins by measuring unique junctional epitopes between heavy and
light chain constant regions. This is particularly useful for IgA
monoclonal proteins, which often have limited detection by SPEP, but
other roles in multiple myeloma are currently under investigation. NGS
to identify genetic mutations reveals the heterogeneous genomic
landscape of multiple myeloma pathogenesis. However, such panels are
limited to investigational usage at this point and are not generally used
for clinical decision-making.
Question 19.2 The correct answer is C.
This patient has a suspected solitary bone plasmacytoma. It is important
to comprehensively assess for multiple myeloma because a solitary
plasmacytoma can be treated with radiation therapy, whereas systemic
disease warrants systemic antimyeloma therapy. However, although the
patient does not have biochemical evidence of the manifestations of
multiple myeloma, appropriate imaging must be performed to exclude
the presence of other osteolytic lesions or soft tissue masses because the
presence of more than one focal lesion on MRI is considered a
“myeloma-defining event.” Although historically, skeletal surveys with
conventional radiography had been used to assess for bony lesions, they
lack the sensitivity to detect areas of focal bone marrow infiltration and
early osteolytic lesions, in comparison to MRI that has both the
sensitivity and specificity to detect such elements of systemic plasma
cell neoplasms. Per the IMWG consensus guidelines on imaging, the
initial imaging workup of choice for a suspected solitary plasmacytoma
of the bone is a whole-body MRI. FDG-PET/CT is recommended if the
plasmacytoma is extramedullary or if whole-body MRI is unavailable.
Question 19.3 The correct answer is D.
The patient has smoldering myeloma as defined by the IMWG criteria
with >10% clonal plasma cells in the bone marrow and/or serum
monoclonal protein of 3 g/dL or urine monoclonal protein of 500 mg/24
h in the absence of any CRAB criteria. Kyle et al. described the natural
progression of smoldering myeloma in which the cumulative probability
of progression in the first 5 years of diagnosis was 51%.1 The 2018
revised Mayo Clinic risk stratification score, also known as the 20/2/20
score, takes into consideration the risk factors of progression:
Bone marrow plasma cells >20%
M-protein >2 g/dL
Involved/uninvolved free light chain ratio >20
Per the current IMWG guidelines, repeat testing is recommended in 2
to 3 months after diagnosis and then monitoring every 4 to 6 months for
1 year. Skeletal surveys are insufficiently sensitive to identify early
osteolytic lesions or focal bone marrow involvement. The current
IMWG consensus for imaging in the workup of suspected smoldering
myeloma recommends low-dose whole-body CT scan initially; if this is
inconclusive or negative, a subsequent whole-body MRI is

https://t.me/ALGRAWANY33
recommended, which, if negative, should be repeated annually. FDG-
PET/CT may substitute for whole-body CT scans and MRI if the latter
two modalities are unable to be performed.
A score of 0, 1, and ≥2 implied low-, intermediate-, and high risk,
respectively, with a median time-to-progression (TTP) of 110, 68, and
29 months within the validation cohort. By this definition, the patient in
this case has low-risk smoldering myeloma. This scoring system was the
basis for analysis in a recent clinical trial in which patients with
intermediate- or high-risk smoldering myeloma were randomized to
lenalidomide or observation.2 Three-year PFS was 91% in the
lenalidomide arm compared with 66% in the observation arm. An earlier
Spanish study evaluated lenalidomide and dexamethasone (Rd) in which
high-risk asymptomatic patients saw a PFS and OS benefit with Rd
compared to observation, although the risk definitions predated the 2014
revisions to the multiple myeloma definitions. Although such therapy
for intermediate- and high-risk smoldering myeloma may be considered,
there are no studies to support therapy for low-risk smoldering
myeloma.
Question 19.4 The correct answer is C.
Prognosis in multiple myeloma is dependent on several factors,
including age, comorbidities, stage, and disease biology. FISH and
cytogenetic studies are ideal to assess for the presence of genetic
alterations and are crucial to pursue for newly diagnosed patients.
Integrating these factors, a risk classification system for newly
diagnosed multiple myeloma has been developed by the Mayo Clinic,
the Mayo Stratification for Myeloma and Risk-adapted Therapy
(mSMART) criteria, which divides newly diagnosed patients into risk
categories on the basis of cytogenetics, FISH, gene expression profiling
(GEP), and plasma cell labeling index studies. High-risk patients are
defined by a high-risk GEP signature, FISH studies showing gain of 1q,
deletion of 17p, t(4;14), t(14;16), or t(14;20), detection of a p53
mutation, high plasma cell S-phase labeling study, or R-ISS Stage III.
Patients who have two high-risk genetic abnormalities are now labeled
as having “double-hit myeloma” and those with three high-risk genetic
abnormalities have “triple-hit myeloma.” This patient has one high-risk
factor by FISH, specifically the t(4;14). Because the hypodiploidy is not
a high-risk genetic factor, the patient does not have double-hit myeloma.
Standard risk disease encompasses all other abnormalities, including
trisomies, t(11;14) and t(6;14).
Question 19.5 The correct answer is D.
The initial study evaluating the genomic architecture of multiple
myeloma was published in 2011.3 These findings, which evaluated
sequencing of 38 tumor genomes, revealed a lack of consistently
conserved mutations among individual multiple myeloma genomes. The
most frequent mutations included those in NRAS (9/38) and KRAS
(10/38). Frequent mutations were also seen in genes involved in
ribonucleic acid (RNA) processing and protein homeostasis, including
DIS3, XBP1, LRRK2, and FAM46C. The nuclear factor (NF)-kB
pathway is the most frequently affected pathway, with mutations and
rearrangements noted in 11 NF-kB pathway genes. Additional mutations
were seen in histone-modifying enzymes, including HOXA9. Overall,
there are few consistently mutated genes in the multiple myeloma
genome, with the most commonly mutated being NRAS and KRAS. This
finding has been corroborated by numerous subsequent whole-exome
and whole-genome sequencing studies, as well as the prospective
CoMMpass registry, in which around 1,000 patients with newly
diagnosed multiple myeloma have undergone comprehensive whole-
genome, whole-exome, and RNA sequencing. Across these sequencing
studies, mutations occur more frequently in noncoding regions of the
genome, and whereas many of these mutations are likely passenger
mutations, some that disrupt microRNA or small nucleolar RNA likely
have regulatory roles that contribute to pathogenesis. Although the IGH
locus is commonly associated with translocations that drive oncogenic
expression, mutations within that locus have not been described as
pathogenic in multiple myeloma. Single-cell sequencing technology
allows for more granular dissection of the genomic landscape of
multiple myeloma by allowing for clustering of subclones of myeloma
cells. A recent single-cell study of myeloma and smoldering myeloma

https://t.me/ALGRAWANY33
patients described significant intratumoral and intrapatient heterogeneity
in both the mutational profile and subclonal makeup of each subject’s
malignancy, some with a more linear pattern of evolution and others
with branching subclonal evolution, failing to describe a common
pathway of myeloma cell evolution.
Question 19.6 The correct answer is C.
BCL-2 is an antiapoptotic protein that normally prevents cell death by
sequestering proapoptotic proteins involved in the intrinsic apoptosis
pathway. Numerous hematologic malignancies are dependent on BCL-2
overexpression for survival. Venetoclax, a selective, orally bioavailable
inhibitor of BCL-2, has demonstrated significant clinical efficacy in
many of these hematologic malignancies. Overexpression of BCL-2 in
multiple myeloma is heterogeneous. However, preclinical study of
human myeloma cell lines and primary patient samples has
demonstrated that those with t(11;14), which occurs in 10% to 20% of
patients with multiple myeloma, are associated with significant
overexpression of BCL-2 relative to other antiapoptotic proteins. A
phase I study of venetoclax, which had a large proportion of patients
with t(11;14), demonstrated significant efficacy in patients with t(11;14)
with an objective response rate (ORR) of 40%, compared with those
lacking that translocation who had an ORR of 6%.4 These preclinical
and early clinical data have spawned numerous clinical trials evaluating
venetoclax in various combinations with other antimyeloma therapies,
both in a targeted and nontargeted manner. Many of these phase II trials
have demonstrated significant efficacy on the basis of response rate.
However, an interim analysis of the phase III BELLINI trial, which
combines venetoclax with bortezomib and dexamethasone (Vd) versus
placebo and Vd, has been highlighted recently due to a safety signal in
which an interim analysis demonstrated worse OS for patients in the
venetoclax arm, despite significantly improved PFS, the mechanistic
cause of which remains unclear. In an underpowered interim subgroup
analysis of those with t(11;14), PFS was dramatically improved and
there was no difference in OS. The BELLINI trial, which was initially in
a cytogenetically unselected population, and other trials evaluating
venetoclax in multiple myeloma have been amended to include only
those with t(11;14).
Question 19.7 The correct answer is B.
The MAIA trial was a randomized controlled trial comparing
daratumumab, lenalidomide, and dexamethasone (DRd) to Rd in
patients with newly diagnosed multiple myeloma who were deemed
ineligible for ASCT.5 There was a significant improvement in PFS,
which was the primary endpoint, with the addition of daratumumab,
with hazard ratio (HR) for disease progression or death of 0.56 (p <
0.001). There was also improvement in ORR and lowering of MRD.
Although generally well-tolerated, there were significantly more
episodes of neutropenia, pneumonia, and infections overall; and though
underpowered to fully analyze subgroups, the PFS benefit was not
clearly seen in the group with high-risk cytogenetics. OS results are
anticipated.
Bortezomib, lenalidomide, and dexamethasone, or VRd, is a
reasonable front-line treatment regimen for patients who are not eligible
or not planned to go on to immediate ASCT; however, it would not be
the best choice for this patient because of his preexisting neuropathy.
The SWOG S0777 trial was a phase III trial comparing VRd to Rd,
about 6 months of either, in patients who were not intended to undergo
immediate ASCT, followed by maintenance lenalidomide and
dexamethasone in both groups.6 VRd significantly improved PFS
compared with Rd (median PFS 43 months vs. 30 months, respectively)
and improved OS (median OS 75 months vs. 64 months, respectively).
However, grade 3+ neuropathy occurred in 24% of the VRd cohort
compared with 5% of the Rd cohort, which is potentially debilitating
especially in elderly patients. In transplant ineligible patients, the
ELOQUENT-1 trial of elotuzumab plus Rd and TOURMALINE-MM2
trial of ixazomib plus Rd failed to demonstrate a PFS benefit over Rd
alone.
Question 19.8 The correct answer is B.
Several large phase III trials have demonstrated the superiority of triplet

https://t.me/ALGRAWANY33
regimens combining bortezomib with an immunomodulatory imide drug
(IMID) and dexamethasone compared with a doublet of the IMID and
dexamethasone alone in transplant-eligible patients with multiple
myeloma. Carfilzomib is a second-generation proteasome inhibitor with
increased potency relative to bortezomib and comparatively increased
efficacy in the relapsed setting. An initial phase I/II trial in newly
diagnosed multiple myeloma patients in combination with lenalidomide
and dexamethasone (KRd) demonstrated profound initial efficacy with a
best overall response rate of 72%, a stringent CR rate of 39%, and 24-
month estimated PFS of 92%. This combination is endorsed by National
Comprehensive Cancer Network (NCCN) guidelines for the initial
treatment of multiple myeloma. The ongoing phase III FORTE trial has
demonstrated deeper responses in patients receiving KRd for 4 cycles
followed by ASCT and then KRd consolidation for 4 cycles, or 12
cycles of KRd when compared with patients who received carfilzomib,
cyclophosphamide, and dexamethasone (KCd).7
Triplet regimens containing an immunomodulatory drug are favored
over those containing cyclophosphamide on the basis of phase III data
that compared bortezomib, thalidomide, and dexamethasone (VTd) to
bortezomib, cyclophosphamide, and dexamethasone (CVD), with
superior overall response. Lenalidomide is often substituted for
thalidomide due to decreased rates of peripheral neuropathy, and
possibly increased efficacy based on an integrated analysis of multiple
trials comparing VTd to VRd. Pomalidomide is currently being studied
in combination with carfilzomib and dexamethasone in the
relapsed/refractory setting and no data are available for this regimen in
the front-line setting. Although triplet therapy regimes are the current
standard of care, it is possible that quadruplet regimens incorporating
immunotherapy such as daratumumab will become the new standard for
transplant-eligible patients. The CASSIOPEIA trial combined
daratumumab with VTd to VTd alone in a phase III trial of transplant-
eligible patients, with the former associated with improved stringent CR
and MRD negativity rates.8 The ongoing phase II GRIFFIN study
comparing daratumumab plus VRd to VRd alone also demonstrated a
significant stringent CR improvement with the addition of
daratumumab.
Question 19.9 The correct answer is C.
The optimal depth of disease response necessary to proceed with ASCT
has been a matter of controversy. Previously, data have suggested that
patients with a better pretransplant paraprotein nadir have had improved
outcomes. Until recently, prospective data have been lacking. A
retrospective study evaluated the outcomes of patients who had achieved
less than a partial response to initial treatment for their multiple
myeloma. A total of 539 patients were identified who had an ASCT after
less than a partial response to initial chemotherapy. Although additional
pretransplant chemotherapy resulted in deepening responses in 68% of
patients, there was no impact of pretransplant salvage chemotherapy on
treatment-related mortality, PFS, or OS. Recently, however, the
Myeloma XI trial, a large, randomized, multicenter phase III trial out of
the United Kingdom, sought to address, among other questions, whether
“response-adapted” intensification of induction therapy could improve
outcomes.9 Patients received either lenalidomide, cyclophosphamide,
and dexamethasone (CRD) or thalidomide, cyclophosphamide, and
dexamethasone (CTD) as initial induction, four cycles if transplant
eligible and six cycles if transplant ineligible. Those who had a
suboptimal response (minor response or partial response) were
randomized to induction intensification with bortezomib,
cyclophosphamide, and dexamethasone (CVD) until maximum response
or intolerance, or no additional induction therapy. Thereafter, transplant-
eligible patients proceeded with ASCT. Of the 289 patients randomized
to CVD intensification, 42.6% were upgraded to a VGPR or CR. ASCT
improved response rates in both the CVD and no CVD cohorts, although
to a greater depth in the CVD cohort (≥VGPR post-ASCT in 65% vs.
39%, respectively). This translated to a significant difference in the
primary endpoint of PFS with a median PFS of 30 months versus 20
months (HR, 0.60; p < 0.0001) across all patients and 48 months versus
28 months among transplant-eligible patients (HR, 0.50; p < 0.0001).
Based on this trial, changing this patient to CVD followed by transplant
would be advised to aim to achieve a deeper response because the
outcomes were improved compared with proceeding directly with

https://t.me/ALGRAWANY33
ASCT. Continuation of the initial induction regimen in a patient who
appears to have reached a disease plateau is unlikely to provide added
benefit. Although the use of a daratumumab-based regimen as salvage
therapy could be considered, an ASCT thereafter would still be the best
option to provide best outcomes.
Question 19.10 The correct answer is C.
In the STAMINA study (BMT CTN 0702 trial), patients were
randomized 1:1:1 to tandem ASCT followed by lenalidomide
maintenance, single ASCT followed by four cycles of lenalidomide,
bortezomib, and dexamethasone (RVD) consolidation followed by
lenalidomide maintenance, or single ASCT followed by lenalidomide
maintenance.10 The treatment groups were balanced for high-risk disease
as well as induction regimens received. Seventy-three percent of the
patients received a triple-drug induction regimen, 55% having received
RVD and another 14% having received bortezomib, cyclophosphamide,
and dexamethasone (CVD). The primary endpoint of 38-month PFS was
not statistically different between cohorts, 58.5%, 57.8%, and 53.9%,
respectively. There were no significant differences between OS and CR
rates either. Tandem ASCT did not improve outcomes in patients with
high-risk cytogenetics. There was a higher incidence of infection in the
tandem ASCT cohort, but this did not translate to higher treatment-
related mortality or worse quality of life. The rate of nonadherence to
second intervention in the intensive arms was about 30% in each.
The European Myeloma Network 02 trial published phase III trial
data suggesting that tandem ASCT provided a PFS and OS benefit to
patients.11 This may possibly be because patients on that trial did not
receive induction regimens combining a proteasome inhibitor and
immunomodulatory drug. The STAMINA trial did not study whether
changing induction regimens leads to improved outcomes.
Question 19.11 The correct answer is A.
In multiple myeloma, deeper responses have long been associated with
better outcomes. Numerous regimens have the potential to achieve CRs,
although most with such a response will still relapse, implying the
sustained presence of malignancy. MRD assays have the ability to detect
the presence of disease to a much higher degree of sensitivity. Munshi et
al. performed a meta-analysis on 12 trials and identified significantly
better outcomes in MRD-negative patients, with PFS overall (HR, 0.41;
95% confidence interval [CI], 0.36–0.48; p < 0.001) and in studies
specifically looking at patients with a CR (HR, 0.44; 95% CI, 0.34–0.56;
p < 0.001).12 OS was also better in MRD-negative patients overall (HR,
0.57; 95% CI, 0.46–0.71; p < 0.001) and in CR patients (HR, 0.47; 95%
CI, 0.33–0.67; p < 0.001). These findings were confirmed in a recent
large phase III trial (IFM/DFCI 2009) in which MRD negativity
measured by both multiparametric flow cytometry (MFC) and NGS
technologies was highly prognostic of improved PFS and OS, in patients
who received lenalidomide, bortezomib, dexamethasone induction with
or without early ASCT.13
Consensus response criteria, which include assay-specific MRD
outcomes in their definitions, have recently been published. Assays
available to detect MRD include MFC, NGS, and ASO-qPCR. The MFC
and NGS MRD assays are the most commonly used. MFC can detect
one malignant clone out of 104 to 105 cells, and the NGS-based MRD
can detect one malignant cell out of 105 to 106 cells and is generally
considered more sensitive. The NGS-based MRD assay tests for the
presence of a unique clonotypic immunoglobulin region and requires a
sample with myeloma cells present to identify the malignant clonotypic
immunoglobulin for comparison. It does not assess a panel of genetic
mutations like some commercial myeloid or lymphoid malignancy
panels assess, and it does not require confirmation by ASO-qPCR. ASO-
qPCR uses PCR primers complementary to the junctional regions of
rearranged IGH genes to identify clonality at low levels. However, it is
technically demanding as primers adapted to each individual patient are
needed for successful qPCR. Although MRD assays allow for deeper
response detection, how to utilize MRD to guide therapy decisions is the
subject of ongoing clinical trials. Tandem transplantation was
demonstrated to not be beneficial in the recent BMT CTN 0702 trial
among patients even with less than ideal response, although MRD was
not a specific endpoint or hinge-point in this trial.

https://t.me/ALGRAWANY33
Question 19.12 The correct answer is A.
This patient has high-risk cytogenetics with del(17p) at diagnosis. She
did well with her induction therapy and achieved a stringent CR,
although she does have positive MRD. A phase II trial evaluated the
efficacy of a bortezomib, lenalidomide, and dexamethasone (VRd)
maintenance strategy post-ASCT in patients with high-risk cytogenetics
and plasma cell leukemia.14 Patients were treated with lower
maintenance doses of VRd for up to 3 years, followed by single-agent
lenalidomide. In this high-risk cohort, median PFS was 32 months and
3-year OS was 93%. Ninety-six percent achieved a VGPR or better at
best response. This was not different among those who did or did not
have del(17p). In the HOVON-65/GMMG-HD4 trial, newly diagnosed
multiple myeloma patients were randomized to bortezomib-based
induction followed by bortezomib posttransplant maintenance therapy
versus thalidomide posttransplant maintenance.15 Significant
improvement in PFS and median OS was seen in 17p-deleted patients
who were randomized to bortezomib-based maintenance.
A meta-analysis of three large randomized controlled trials
demonstrated a significant improvement in median PFS, 52.8 months in
the lenalidomide maintenance group versus 23.5 months in patients on
placebo or observation. OS was also significantly improved in patients
overall, but a subgroup analysis of patients with high-risk cytogenetics
demonstrated no difference in OS between lenalidomide maintenance
versus observation/placebo. Daratumumab in combination with
lenalidomide versus lenalidomide for maintenance therapy is currently
being investigated in an ongoing phase III SWOG/BMT CTN trial.
Elotuzumab has been studied with lenalidomide as maintenance therapy
in a phase II trial, although this combination remains investigational at
this point.
Question 19.13 The correct answer is B.
Monoclonal antibodies have become powerful additions to the arsenal of
antimyeloma therapies. Daratumumab is a human CD38-targeting IgG
kappa monoclonal antibody. Efficacy was initially demonstrated in the
single-armed phase II SIRIUS trial, in which median PFS and ORR
were 3.7 months and 29.2%, respectively, in heavily treated patients.16
The history of Stevens–Johnson syndrome from lenalidomide would
preclude further treatment with any drug from the immunomodulatory
drug class, namely pomalidomide. In the CASTOR trial, the
combination of daratumumab with bortezomib and dexamethasone
(DVd) was associated with significant PFS benefit compared with
bortezomib plus dexamethasone, with 12-month rate of PFS of 60.7% in
the former and 26.9% in the latter arm (HR, 0.39; p < 0.0001). Patients
included in this trial were not refractory to bortezomib or another
proteasome inhibitor, although two-thirds of patients had had prior
treatment with a proteasome inhibitor.
In POLLUX, a phase III open-labeled, randomized, active-controlled
trial, daratumumab plus lenalidomide and dexamethasone (DRd)
demonstrated a significant PFS benefit compared with lenalidomide and
dexamethasone (Rd) with an HR of 0.37 (p < 0.001) and a significantly
higher overall response.17 The EQUULEUS study was an open-label
trial that enrolled patients previously treated with a proteasome inhibitor
and an IMID, treating them with a combination of daratumumab,
pomalidomide, and dexamethasone (DPd), assessing ORR as the
primary endpoint without a control arm. The ORR was 59.2%.18
Elotuzumab is a humanized monoclonal antibody that targets CS-
1/SLAMF7. It was combined with pomalidomide and dexamethasone in
the ELOQUENT-3 trial, a multicenter phase II trial comparing
elotuzumab, pomalidomide, and dexamethasone versus pomalidomide
and dexamethasone in patients refractory to a proteasome inhibitor and
lenalidomide.19 Elotuzumab, pomalidomide, and dexamethasone had a
median PFS of 10.3 months versus 4.7 months in the control group.
Isatuximab is a chimeric CD38-targeting monoclonal antibody that
targets a unique epitope of CD38 with a different mechanism of action
profile relative to daratumumab, including direct inhibition of the CD38
ectoenzyme. A phase III trial compared isatuximab plus pomalidomide
and dexamethasone (IPd) to Pd. Patients were refractory to lenalidomide
and a proteasome inhibitor. Median PFS was significantly improved in
the IPd arm versus Pd, 11.6 months compared with 6.5 months,
respectively (p = 0.001). There was a trend toward improved OS.

https://t.me/ALGRAWANY33
Question 19.14 The correct answer is A.
The ASPIRE trial was a randomized, open-label, multicenter phase III
study of carfilzomib (administered on days 1, 2, 8, 9, 15, and 16 of a 28-
day cycle with an initial dose of 20 mg/m2 on days 1 and 2 of cycle 1,
and increased to 27 mg/m2 thereafter), lenalidomide, and dexamethasone
(KRd) versus lenalidomide and dexamethasone (Rd) in patients with
relapsed multiple myeloma.20 With a median follow-up of 32 months,
the primary endpoint, PFS, was superior for the three-drug combination
(26.3 months vs. 17.6 months; p = 0.0001). An updated OS analysis
demonstrated an OS benefit for the KRd arm, with median OS of 48.3
months versus 40.4 months, p = 0.0045.
The 20/36 mg/m2 twice-weekly dosing of carfilzomib in combination
with lenalidomide and dexamethasone (Rd) has been studied only in the
front-line setting. A phase I/II trial identified the maximum-
tolerated/planned dose in this untreated population as 36 mg/m2 on the
twice-weekly dosing schedule and demonstrated profound initial
efficacy with a best overall response rate of 72%, a stringent CR rate of
39%, and 24-month estimated PFS of 92%.16 The ENDEAVOR trial was
a large phase III comparison of the second-generation proteasome
inhibitor carfilzomib in combination with dexamethasone to bortezomib
and dexamethasone.21 Neither arm had lenalidomide. It demonstrated
that carfilzomib at a twice-weekly dose of 56 mg/m2 resulted in a PFS of
18.7 months compared with 9.4 months for bortezomib (1.3 mg/m2) and
dexamethasone. The benefit was seen in both patients who had prior
exposure to bortezomib (15.6 months vs. 8.1 months) and no prior
bortezomib exposure (not reached vs. 11.2 months). The phase III
ARROW trial was a randomized trial to evaluate the efficacy of
different dosing strategies of carfilzomib plus dexamethasone,
specifically 70 mg/m2 once weekly versus 27 mg/m2 twice weekly of
carfilzomib. Neither arm included lenalidomide. Patients included were
refractory to their most recent therapy, which included other proteasome
inhibitors. The primary endpoint of PFS was higher in the once-weekly
cohort, median PFS of 11.2 months compared with 7.6 months (HR,
0.69; p = 0.0029).
Question 19.15 The correct answer is B.
Selinexor is an exportin 1 (XPO1) inhibitor approved in combination
with dexamethasone for patients with relapsed/refractory myeloma who
have received at least four prior therapies and whose disease is
refractory to at least two proteasome inhibitors, at least two
immunomodulatory agents, and an anti-CD38 monoclonal antibody.
Approval was based on the STORM trial22 that showed efficacy in
penta-refractory patients with an ORR of 25.3%, median time to first
response of 4 weeks, and median duration of response of 3.8 months.
Median PFS and OS in the entire cohort were 3.7 and 8.6 months,
respectively. Selinexor was associated with significant toxicity,
including cytopenias, fatigue, nausea, anorexia, weight loss,
gastrointestinal toxicity, hyponatremia, and respiratory infections. The
STOMP trial (NCT02343042) is currently evaluating selinexor in
combination with other myeloma therapies including lenalidomide,
pomalidomide, bortezomib, carfilzomib, and daratumumab, with
encouraging efficacy and possibly improved tolerability due to an
attenuated selinexor dosing schedule.
Elotuzumab is a monoclonal antibody that targets CS-1/SLAMF7 and
is approved in combination with an immunomodulatory agent. It is not
approved in combination with ixazomib. Second ASCT remains a viable
option for patients, generally in those who enjoyed a prolonged relapse-
free period who are fit for transplant. Currently, joint guidelines from
the IMWG and other blood and marrow transplantation organizations
state that high-dose therapy with ASCT is appropriate for any patient
who relapsed after primary therapy that included ASCT if the initial
remission duration was more than 18 months. The Myeloma X trial
compared two cohorts where both received reinduction with bortezomib,
doxorubicin, and dexamethasone, with one cohort receiving high-dose
melphalan and ASCT, the other receiving 12 weeks of oral
cyclophosphamide.23 The ASCT cohort experienced a significant
increase in PFS (median PFS 19 months vs. 11 months) and OS (median
OS 67 months vs. 52 months). Two antigen mismatched unrelated donor
transplants are not permitted by the National Marrow Donor Program.

https://t.me/ALGRAWANY33
Question 19.16 The correct answer is B.
BCMA normally functions to maintain the growth and survival of
plasma cells. It binds a proliferation-inducing ligand (APRIL) that is
secreted from various cells in the microenvironment. Activation by
APRIL binding leads to downstream activation of signaling cascades the
increase the expression of antiapoptotic proteins and factors that
promote survival and metastasis. BCMA is selectively induced during
plasma cell differentiation, initially being expressed in postgerminal
center B cells with upregulation as this B cell transitions to the
plasmablast stage and significantly more so on terminally differentiated
long-lived plasma cells. In addition, multiple myeloma cells have been
demonstrated to express BCMA at an even higher level than normal
plasma cells.
BCMA is not involved in normal B-cell homeostasis and is not
expressed before B cells transition to the germinal center. This high
expression and restriction to mainly myeloma cells, plasma cells, and
mature B cells makes an ideal target for immunotherapies with the
potential for maximum efficacy with minimal off-target effects. Indeed,
BCMA has become one of the preferred targets of chimeric antigen
receptor (CAR) T cells, bispecific T-cell engagers (BiTEs), and
antibody–drug conjugates (ADCs) that are currently in clinical
development. BCMA-targeting therapies have been initially studied as
monotherapy in patients who were heavily pretreated, refractory to
proteasome inhibitors, immunomodulatory drugs, and often CD38-
targeting monoclonal antibodies. Despite refractoriness to these
therapies, significant responses have been seen. The ADC belantamab
mafodotin recently was reported to have a 31% overall response rate as
a single agent in patients refractory to at least one drug from each of
those therapeutic classes. In a recent phase I trial evaluating the bb2121
BCMA-targeting CAR-T cell, 55% of patients were refractory to
daratumumab. The response rate was over 70% in this population.
Question 19.17 The correct answer is A.
Osteoclast inhibitors play an important role in the management of
patients with multiple myeloma and lytic lesions, with multiple trials
demonstrating their potential to reduce morbidity due to skeletal-related
events and pain. Denosumab is a monoclonal antibody against receptor
activator of nuclear factor kappa B ligand (RANKL). Unlike
bisphosphonates such as zoledronic acid and pamidronate, its use is not
contraindicated in patients with significant renal impairment that are not
on dialysis. A placebo-controlled randomized noninferiority trial in
multiple myeloma patients with bony lytic lesions compared denosumab
to zoledronic acid, meeting its endpoint of noninferiority in regard to
time to first skeletal-related event. Denosumab was associated with
lower incidence of renal toxicity in this trial, 3% versus 7%. In addition,
an exploratory PFS analysis suggested that denosumab may confer a
PFS benefit relative to zoledronic acid, with median PFS of 46.1 months
versus 35.4 months (HR, 0.82; p = 0.036).
In patients with CrCl ≥30 mL/min, zoledronic acid is still often
preferred due to significant difference in cost. Denosumab is warranted
if zoledronic acid is contraindicated. Teriparatide has not been studied in
the context of multiple myeloma. Ibandronate is not approved for use in
multiple myeloma.
Question 19.18 The correct answer is C.
There is no established standard of care for induction therapy for AL
amyloidosis, and treatment on clinical trials should be prioritized.
Cyclophosphamide, bortezomib, and dexamethasone (CyBorD) is a
commonly used induction regimen. In a retrospective study, up to eight
cycles of CyBorD were found to have an overall hematologic response
rate of 81.4%, including a CR rate of 41.9% and VGPR of 51.4%.
ASCT has been investigated in phase II studies and has been found to
produce long-term durable remissions in AL amyloidosis patients and is
the preferred treatment for transplant-eligible patients. Mayo Clinic
guidelines for transplant eligibility include appropriate physiologic age,
troponin T <0.06 ng/mL, no more than two organs significantly
involved, ECOG ≤2, NYHA functional class I or II, CrCl ≥30 mL/min,
and N-terminal pro-brain natriuretic peptide (NT-proBNP) <5,000 ng/L.
Given this patient’s NYHA class III heart failure symptoms, he is
ineligible for transplantation at this time. Ixazomib was recently

https://t.me/ALGRAWANY33
investigated in combination with dexamethasone in patients with
relapsed/refractory AL amyloidosis in the phase III TOURMALINE-
AL1 trial, compared with physician’s choice of standard therapy.
Ixazomib demonstrated an improvement in a composite primary
endpoint of time-to-vital organ deterioration/death (HR, 0.53; p = 0.011)
without a benefit noted in hematologic responses, a coprimary
endpoint.24
Question 19.19 The correct answer is D.
POEMS syndrome is a rare monoclonal plasma cell dyscrasia that is
characterized mainly by the presence of subacute sensory and motor
neuropathy in conjunction with a number of other manifestations. The
IMWG has defined set criteria that must be met to invoke the diagnosis
of POEMS syndrome as there is notable overlap with some other rare
syndromes. The diagnostic criteria are:
Mandatory major criteria (both required)
Polyneuropathy
Monoclonal plasma cell proliferative disorder (almost always
lambda)
Other major criteria (one required)
Sclerotic bone lesions
Castleman disease
Elevated levels of VEGF
Minor criteria (one required)
Organomegaly (splenomegaly, hepatomegaly, or lymphadenopathy)
Extravascular volume overload (edema, pleural effusion, or ascites)
Endocrinopathy (adrenal, thyroid, pituitary, gonadal, parathyroid,
pancreatic)
Skin changes (hyperpigmentation, hypertrichosis, glomeruloid
hemangioma, plethora, acrocyanosis, flushing, white nails)
Papilledema
Thrombocytosis/polycythemia
The patient in the question required one of the major criteria other
than polyneuropathy and monoclonal plasma cell disorder to fulfill the
requirements for the diagnosis. He already had five of the minor criteria.
VEGF levels correlate with disease activity and have been implicated in
the pathophysiology of POEMS as imbalances between it and other
vascular cytokines have been associated with damage to the vasculature
of the peripheral nervous system. However, anti-VEGF therapy in the
treatment of POEMS has had mixed results. As it pertains to the
endocrinopathies associated with POEMS, thyroid abnormalities and
diabetes are not considered among them as they are overly common.
Question 19.20 The correct answer is D.
This patient likely has Waldenstrom macroglobulinemia
(WM)/lymphoplasmacytic lymphoma (LPL) and has clinical evidence of
hyperviscosity syndrome, which is a medical emergency.
Plasmapheresis to decrease serum IgM levels and reduce serum
viscosity is necessary to reverse the neurologic and ischemic symptoms
associated with hyperviscosity syndrome, often needing to be performed
before a diagnosis is confirmed by bone marrow biopsy.
Even in patients who are significantly anemic, blood transfusions
should be avoided if possible as they increase the serum viscosity.
Rituximab-based therapies can cause an IgM flare, and plasmapheresis
should be considered before such therapy to prevent hyperviscosity
syndrome from occurring or being exacerbated in those who have
elevated IgM levels at baseline, and patients in the trials of
bendamustine/rituximab and ibrutinib/rituximab were permitted to
undergo plasmapheresis before initiating therapy. A multicenter phase II
study of ibrutinib monotherapy demonstrated an overall response rate of
90.5% in patients with relapsed WM, with a 100% ORR in patients with
the MYD88 L265P mutation. In a phase III trial that incorporated
patients with both newly diagnosed and relapsed WM, ibrutinib was
combined with rituximab and compared with placebo plus rituximab.
Ibrutinib plus rituximab had a significantly improved major response
rate (72% vs. 32%; p < 0.001) and a significantly better PFS (HR, 0.20;
p < 0.001). Unlike an earlier phase II trial, only the MYD88 L265P

https://t.me/ALGRAWANY33
mutation was associated with better response rate, with minimal
difference across CXCR4 genotypes, and PFS was comparable across all
mutational profiles. Eight percent of patients with the ibrutinib plus
rituximab combination had IgM flare compared with 47% with
rituximab plus placebo, suggesting that ibrutinib can mitigate the flare
response. Bendamustine and rituximab (BR) is an active regimen in
indolent lymphomas, including WM/LPL. In a phase III trial comparing
BR to R-CHOP in indolent lymphoma, of which 41 patients had
WM/LPL, the ORR was about 95% and median PFS was 69.5% in BR
compared with 28.1% with R-CHOP (HR, 0.33; 95% CI, 0.11–0.64; p =
0.0033).

References
1. Kyle RA, Remstein ED, Therneau TM, et al. Clinical course and prognosis of smoldering
(asymptomatic) multiple myeloma. N Engl J Med. 2007;356:2582–2590.
2. Lonial S, Jacobus S, Fonseca R, et al. Randomized trial of lenalidomide versus observation in
smoldering multiple myeloma. J Clin Oncol. 2020;38:1126–1137.
3. Chapman MA, Lawrence MS, Keats JJ, et al. Initial genome sequencing and analysis of multiple
myeloma. Nature. 2011;471:467–472.
4. Kumar S, Kaufman JL, Gasparetto C, et al. Efficacy of venetoclax as targeted therapy for
relapsed/refractory t(11;14) multiple myeloma. Blood. 2017;130:2401–2409.
5. Facon T, Kumar S, Plesner T, et al. Daratumumab plus lenalidomide and dexamethasone for
untreated myeloma. N Engl J Med. 2019;380:2104–2115.
6. Durie BGM, Hoering A, Abidi MH, et al. Bortezomib with lenalidomide and dexamethasone versus
lenalidomide and dexamethasone alone in patients with newly diagnosed myeloma without intent
for immediate autologous stem-cell transplant (SWOG S0777): a randomised, open-label, phase 3
trial. Lancet. 2017;389:519–527.
7. Gay F, Cerrato C, Petrucci MT, et al. Efficacy of carfilzomib lenalidomide dexamethasone (KRd)
with or without transplantation in newly diagnosed myeloma according to risk status: results from
the FORTE trial. J Clin Oncol. 2019;37:8002.
8. Moreau P, Attal M, Hulin C, et al. Bortezomib, thalidomide, and dexamethasone with or without
daratumumab before and after autologous stem-cell transplantation for newly diagnosed multiple
myeloma (CASSIOPEIA): a randomised, open-label, phase 3 study. Lancet. 2019;394:29–38.
9. Jackson GH, Davies FE, Pawlyn C, et al. Response-adapted intensification with cyclophosphamide,
bortezomib, and dexamethasone versus no intensification in patients with newly diagnosed multiple
myeloma (Myeloma XI): a multicentre, open-label, randomised, phase 3 trial. Lancet Haematol.
2019;6:e616–e629.
10. Stadtmauer EA, Pasquini MC, Blackwell B, et al. Autologous transplantation, consolidation, and
maintenance therapy in multiple myeloma: results of the BMT CTN 0702 trial. J Clin Oncol.
2019;37:589–597.
11. Cavo M, Petrucci MT, Di Raimondo F, et al. Upfront single versus double autologous stem cell
transplantation for newly diagnosed multiple myeloma: an intergroup, multicenter, phase III study
of the European Myeloma Network (EMN02/HO95 MM Trial). Blood. 2016;128:991.
12. Munshi NC, Avet-Loiseau H, Rawstron AC, et al. Association of minimal residual disease with
superior survival outcomes in patients with multiple myeloma: a meta-analysis. JAMA Oncol.
2017;3:28–35.
13. Attal M, Lauwers-Cances V, Hulin C, et al. Lenalidomide, bortezomib, and dexamethasone with
transplantation for myeloma. N Engl J Med. 2017;376:1311–1320.
14. Nooka AK, Kaufman JL, Muppidi S, et al. Consolidation and maintenance therapy with
lenalidomide, bortezomib and dexamethasone (RVD) in high-risk myeloma patients. Leukemia.
2014;28:690–693.
15. Sonneveld P, Schmidt-Wolf IG, van der Holt B, et al. Bortezomib induction and maintenance
treatment in patients with newly diagnosed multiple myeloma: results of the randomized phase III
HOVON-65/GMMG-HD4 trial. J Clin Oncol. 2012;30:2946–2955.
16. Lonial S, Weiss BM, Usmani SZ, et al. Daratumumab monotherapy in patients with treatment-
refractory multiple myeloma (SIRIUS): an open-label, randomised, phase 2 trial. Lancet.
2016;387:1551–1560.
17. Dimopoulos MA, Oriol A, Nahi H, et al. Daratumumab, lenalidomide, and dexamethasone for
multiple myeloma. N Engl J Med. 2016;375:1319–1331.
18. Chari A, Suvannasankha A, Fay JW, et al. Daratumumab plus pomalidomide and dexamethasone in
relapsed and/or refractory multiple myeloma. Blood. 2017;130:974–981.
19. Dimopoulos MA, Dytfeld D, Grosicki S, et al. Elotuzumab plus pomalidomide and dexamethasone
for multiple myeloma. N Engl J Med. 2018;379:1811–1822.
20. Stewart AK, Rajkumar SV, Dimopoulos MA, et al. Carfilzomib, lenalidomide, and dexamethasone
for relapsed multiple myeloma. N Engl J Med. 2014;372:142–152.
21. Dimopoulos MA, Moreau P, Palumbo A, et al. Carfilzomib and dexamethasone versus bortezomib
and dexamethasone for patients with relapsed or refractory multiple myeloma (ENDEAVOR): a
randomised, phase 3, open-label, multicentre study. Lancet Oncol. 2016;17:27–38.
22. Chari A, Vogl DT, Gavriatopoulou M, et al. Oral selinexor–dexamethasone for triple-class
refractory multiple myeloma. N Engl J Med. 2019;381:727–738.
23. Cook G, Ashcroft AJ, Cairns DA, et al. The effect of salvage autologous stem-cell transplantation
on overall survival in patients with relapsed multiple myeloma (final results from BSBMT/UKMF
Myeloma X Relapse [Intensive]): a randomised, open-label, phase 3 trial. Lancet Haematol.
2016;3:e340–e351.
24. Dispenzieri A, Kastritis E, Wechalekar AD, et al. Primary results from the phase 3
TOURMALINE-AL1 trial of ixazomib-dexamethasone versus physician’s choice of therapy in
patients (Pts) with relapsed/refractory primary systemic AL amyloidosis (RRAL). Blood.
2019;134:139.
___________
Corresponding chapter in DeVita, Hellman, and Rosenberg’s Cancer: Principles & Practice of
Oncology, Eleventh Edition: 107 (Plasma Cell Neoplasms).

https://t.me/ALGRAWANY33
20 Cancer Screening
Olivia Aranha

QUESTIONS

Each of the numbered items below is followed by lettered answers. Select the
ONE lettered answer that is BEST in each case unless instructed otherwise.

Question 20.1 In the National Lung Screening Trial (NLST), what was the
percentage of relative reduction in mortality with low-dose computerized
tomography?
A. 6.7%
B. 15%
C. 20%
D. 30%
Question 20.2 Studies of screening for cancer are subject to several types
of bias. Screening studies tend to enroll individuals who are healthier than the
general population. What type of bias is introduced by this so-called healthy
volunteer effect?
A. Lead-time bias
B. Length bias
C. Selection bias
D. Overdiagnosis bias
Question 20.3 The American Cancer Society (ACS) recommends the
following for women at average risk for breast cancer?
A. They should begin having yearly mammograms by age 45, and can
change to having mammograms every other year beginning at age 55.
B. Regular mammograms are not indicated in postmenopausal women.
C. Regular mammograms should continue for as long as a woman is in
good health.
D. If a woman has dense breasts, she should have an annual mammogram,
magnetic resonance imaging (MRI), and breast self-exam.
Question 20.4 In planning a new screening test for ovarian cancer if you
increase the sensitivity, which of the following is TRUE?
A. The specificity is decreased.
B. The specificity is increased.
C. Positive predictive value is decreased.
D. Negative predictive value (NPV) is unchanged.
Question 20.5 In the Mayo Lung Project randomized controlled trial, more
than 9,200 male smokers were randomized to (1) intensive screening: sputum
cytology and chest X-ray (CXR) every 4 months for 6 years or (2) a control
group: same tests performed annually. After nearly 20 years of follow-up,
there were more lung cancers diagnosed in the intensive screening arm versus
the control arm (585 vs. 500) but the intensive screening arm did not show
improvement in lung cancer mortality (4.4 lung cancer deaths per 1,000
person-years in the intensively screened arm vs. 3.9 per 1,000 person-years in
the control arm). Which of the following best explains this?
A. Lead-time bias
B. Overdiagnosis
C. Selection bias
D. Stage shift
Question 20.6 Which of the following is recommended by the cervical
cancer screening guidelines?
A. Screening for cervical cancer should begin at 21 years of age.
B. Women aged 21–29 years should receive cytology screening (with
either conventional cervical cytology smears or liquid-based cytology)
every year.

https://t.me/ALGRAWANY33
C. Human papilloma virus (HPV) testing should be performed in the 21–
29 age group.
D. For women aged 30–65 years, the preferred approach is to be screened
every 3 years with both HPV testing and cytology (co-testing).
Question 20.7 A 50-year-old woman had a total hysterectomy for fibroids
and menorrhagia. What is the recommended approach to cervical cancer
screening for her?
A. Cervical cytology testing should be performed annually indefinitely.
B. Cervical cytology testing should be performed every 3 years until 65
years of age.
C. Cervical cytology testing should be performed every 5 years until 65
years of age.
D. Cervical cytology testing should be discontinued.
Question 20.8 The ACS recommends the following guidelines for
colorectal cancer screening in an average risk adults starting at age 50?
A. Colonoscopy every 20 years
B. Flexible sigmoidoscopy every 5 years
C. Double-contrast barium enema every 10 years
D. Computed tomographic (CT) colonography every 3–5 years
Question 20.9 In analyzing a randomized controlled trial of a screening
test, which is the best indicator of effectiveness?
A. Absolute reduction in mortality
B. Evidence of stage shift
C. Increased 5-year survival
D. Relative reduction in mortality
Question 20.10 The ACS recommends annual screening mammography
and MRI starting at age 30 for women at high risk for breast cancer including
which of the following?
A. Women with a history of fibrocystic breast disease
B. Women with a history of mantle radiation for Hodgkin disease
C. Women with KRAS mutation
D. Women with increased mammographic breast density
Question 20.11 The American Academy of Dermatology has which of the
following skin cancer screening recommendations?
A. A monthly skin self-examinations and a clinical skin examination as
part of a routine cancer-related checkup every 6 months
B. High-risk individuals are persons with multiple nevi or atypical moles
C. Persons at highest risk (i.e., those with a strong family history of
melanoma and multiple atypical nevi) perform frequent self-
examination and seek a professional evaluation of the skin at least once
every 2 years
D. CT scan of the chest and abdomen every 3 years
Question 20.12 In the prostate, lung, colorectal, and ovarian (PLCO)
Cancer Screening Trial, men were randomized to receive annual prostate-
specific antigen (PSA) testing for 6 years or usual care. At the conclusion of
the 13-year follow-up period, it was found that there was a high rate of PSA
testing among men randomized to the control arm. This “drop-in” had what
effect on the results?
A. Increased the positive predictive value (PPV) of the PSA test
B. Increased the relative risk reduction (RRR) of the screening arm when
compared to the control arm
C. Reduced the sensitivity of the PSA test
D. Reduced the statistical power of the study to detect differences in
outcome between the two arms
Question 20.13 A new screening test for breast cancer is tested in a tertiary
care academic medical center in a group of women with BRCA mutations and
is found to have a sensitivity of 60%, specificity of 80%, and PPV of 20. If
the use of the above test is expanded to the general population, what is
expected of the following?
A. The test sensitivity will be lower.

https://t.me/ALGRAWANY33
B. The test specificity will be lower.
C. The PPV will be lower.
D. The test specificity will be higher.

ANSWERS

Question 20.1 The correct answer is C.


In the NLST, approximately 53,000 persons were randomized to receive
three annual low-dose CT (LDCT) scans or single-view posteroanterior
CXRs.1 Eligibility criteria were as follows: age 55 to 74 years, at least a
30 pack-year smoking history, currently smokers or people having quit
within the previous 15 years, and being in relatively good health. With a
median follow-up of 6.5 years, 13% more lung cancers were diagnosed
and a 20% (95% CI, 6.8–26.7; p = 0.004) relative reduction in lung
cancer mortality was observed in the LDCT arm compared to the CXR
arm. An important finding from the NLST was a 6.7% (95% CI, 1.2–
13.6; p = 0.02) decrease in death from any cause in the LDCT group. A
further analysis of the NLST shows that screening prevented the greatest
number of lung cancer deaths among participants who were at the
highest risk but prevented very few deaths among those at the lowest
risk. These findings provide empirical support for risk-based screening
Question 20.2 The correct answer is C.
Selection bias occurs when enrollees in a clinical study differ from the
general population.2 In fact, people who voluntarily participate in
clinical trials tend to be healthier than the general population, perhaps
due to a greater interest in health and healthcare research. Screening
studies tend to enroll individuals healthier than the general population.
This so-called healthy volunteer effect can introduce a powerful bias if
not adequately controlled for by randomization procedures.
Lead-time bias is the interval between the diagnosis of disease at
screening and when it would have been detected because of the
development of symptoms. Length bias is the overrepresentation among
screen-detected cases of those with a long preclinical period (thus, less
rapidly fatal), leading to the incorrect conclusion that screening was
beneficial.
Question 20.3 The correct answers is A.
The ACS recommends that women with an average risk of breast cancer
should begin yearly mammograms at age 45.3 Women should be able to
start the screening as early as age 40 and discuss this with their
healthcare provider. At age 55, women should have mammograms every
other year. Regular mammograms should continue for as long as a
woman is in good health. Breast exams, either from a medical provider
or self-exams, are no longer recommended. MRI screening is only
indicated in women at very high risk for breast cancer.
Question 20.4 The correct answer is A
For a given screening test, sensitivity and specificity are inversely
related. The increase in sensitivity results in a decrease in specificity.
However, it will result in an increase in the PPV.
Question 20.5 The correct answer is B.
Overdiagnosis is a form of length bias and refers to the detection of
tumors that fulfill the histologic criteria for malignancy but are not
biologically destined to harm the patient within the life span. Lead-time
bias is the interval between the diagnosis of disease at screening and
when it would have been detected because of the development of
symptoms. Selection bias occurs when enrollees in a clinical study differ
from the general population. Volunteers who choose to participate in
screening programs tend to be healthier than the general population. In
stage shift, a cancer diagnosis is made at an earlier stage than would
have occurred in the absence of screening.
Question 20.6 The correct answer is A.
The ACS, the American Society for Colposcopy and Cervical Pathology
(ASCCP), and the American Society for Clinical Pathology (ASCP)
issued joint screening guidelines.4 These guidelines recommend that for

https://t.me/ALGRAWANY33
a woman at average risk, screening for cervical cancer should begin at
21 years of age. Women aged 21 to 29 years should receive cytology
screening (with either conventional cervical cytology smears or liquid-
based cytology) every 3 years. HPV testing should not be performed in
this age group, although it can be used to follow up a diagnosis of
atypical squamous cells of undetermined significance (ASC-US).
Women under 21 years of age should not be screened, regardless of their
age of sexual initiation. For women aged 30 to 65 years, the preferred
approach is to be screened every 5 years with both HPV testing and
cytology (co-testing). It is also acceptable to continue screening every 3
years with cytology alone. Women should discontinue screening after
age 65 years if they have had three consecutive negative cytology tests
or two consecutive negative HPV test results within the 10-year period
before ceasing screening, with the most recent test occurring within the
last 5 years.
Question 20.7 The correct answer is D.
Women who have undergone hysterectomy for noncancerous conditions
do not need to undergo cervical cancer screening.
Question 20.8 The correct answer is B.
The ACS, the American College of Gastroenterology, the American
Gastroenterological Association, the American Society for
Gastrointestinal Endoscopy, and the American College of Radiology
have issued joint colorectal cancer guidelines.5 These groups consider
fecal occult blood test (FOBT), fecal immunochemical test (FIT), rigid
and flexible sigmoidoscopies, colonoscopies, and CT colonographies to
all be reasonable screening methodologies. They recommend the
following:
1. Screening modalities be chosen based on personal preference and
access.
2. Average risk adults should begin colorectal cancer screening at age 50
years with one of the following options: annual high sensitivity FOBT
or FIT, a flexible sigmoidoscopy every 5 years, a colonoscopy every
10 years, a double-contrast barium enema every 5 years, or a CT
colonography every 5 years.
Question 20.9 The correct answer is A.
Absolute risk reduction conveys more information than RRR. A large
RRR applied to a very small risk translates to a small number of lives
saved. For example, reducing mortality from two in a million to one in a
million is an impressive sounding 50% risk reduction but only saves one
more life per million patients. Stage shift is the diagnosis of a cancer at
an earlier stage than would have occurred in the absence of screening.
This is necessary but not sufficient for a screening test to be effective in
terms of reducing mortality. Survival time after diagnosis can be inflated
by lead-time bias. Mortality rates are a better measure of effectiveness.
Question 20.10 The correct answer is B.
Annual mammography and MRI starting at age 30 is recommended for
women at high risk due to a known BRCA mutation, women who are
untested but have a first-degree relative with a BRCA mutation, women
who had been treated with radiation to the chest for Hodgkin disease,
and women with an elevated lifetime risk (>20%–25%) based on breast
cancer risk estimation models. It has yet to be determined whether
supplemental imaging reduces breast cancer mortality in women with
increased breast density.
Question 20.11 The correct answer is B.
The American Academy of Dermatology recommends persons at
highest risk (i.e., those with a strong family history of melanoma and
multiple atypical nevi or atypical moles) perform frequent self-
examination and seek a professional evaluation of the skin at least once
per year.
Question 20.12 The correct answer is D.
Sensitivity and specificity are characteristics of a test itself. The PPV is
related to the test sensitivity, test specificity, and population prevalence
of the disease. “Drop-in” or “contamination” by screening in the control
arm potentially decreased the mortality in the control arm, minimizing

https://t.me/ALGRAWANY33
the difference between the groups. Both drop-ins and drop-outs reduce
the statistical power of a clinical trial to detect differences in outcome
between the two arms.
Question 20.13 The correct answer is C.
The sensitivity and specificity are not affected by the population
characteristics. In contrast, the PPV and NPV are influenced by the
disease prevalence in the population being tested. The general
population will have a lower prevalence of breast cancer than a group of
women with known BRCA mutations.

References
1. National Lung Screening Trial Research Team, Aberle DR, Adams AM, Berg CD, et al. Reduced
lung-cancer mortality with low-dose computed tomographic screening. N Engl J Med.
2011;365:395–409.
2. Jansz MS. Selection bias. CMAJ. 2017;189:E673.
3. Oeffinger KC, Fontham ET, Etzioni R, et al. American Cancer Society. Breast cancer screening for
women at average risk: 2015 guideline update from the American Cancer Society. JAMA.
2015;314:1599–1614.
4. Saslow D, Solomon D, Lawson HW, et al. ACS-ASCCP-ASCP Cervical Cancer Guideline
Committee. American Cancer Society, American Society for Colposcopy and Cervical Pathology,
and American Society for Clinical Pathology screening guidelines for the prevention and early
detection of cervical cancer. CA Cancer J Clin. 2012;62:147–72.
5. Wolf AMD, Fontham ETH, Church TR, et al. Colorectal cancer screening for average-risk adults:
2018 guideline update from the American Cancer Society. CA Cancer J Clin. 2018;68:250–281.
___________
Corresponding chapter in DeVita, Hellman, and Rosenberg’s Cancer: Principles & Practice of
Oncology, Eleventh Edition: 37 (Cancer Screening).
21 Cancer Pain
Elizabeth Prsic and William Rafelson

QUESTIONS

Each of the numbered items below is followed by lettered answers. Select the
ONE lettered answer that is BEST in each case unless instructed otherwise.

Question 21.1 A 68-year-old female with metastatic pancreatic cancer has


stable disease following six cycles of 5-FU, leucovorin, irinotecan, and
oxaliplatin (FOLFIRINOX). She is currently receiving sustained-release
morphine 45 mg by mouth every 12 hours, with 10 mg morphine immediate
release by mouth every 4 hours as needed for visceral pain associated with
peritoneal carcinomatosis. She returns monthly for prescription refills and has
no history of aberrant behavior with opioids. Her Opioid Risk Tool score is
low. Which of the following statements is most accurate regarding
management of chronic cancer pain with opioids?
A. For patients receiving greater than 60 mg OME (oral morphine
equivalents), titration with short-acting drugs is better tolerated than
titration with sustained-release drugs.
B. The breakthrough opioid dose should be 25–50% of the patient’s 24-
hour opioid requirement.
C. Morphine can be used safely in patients with renal failure.
D. Anticipating and treating side effects of opioids, including sedation,
nausea, vomiting, and constipation, is important for effective use.
Question 21.2 A 76-year-old male has refractory bone pain associated with
metastatic castrate-resistant prostate cancer. He experienced significant
diarrhea with the initial cycle of docetaxel and does not wish to pursue
further chemotherapy. He otherwise has a good Eastern Cooperative

https://t.me/ALGRAWANY33
Oncology Group (ECOG) performance status of 1 but is limited in activities
due to pain. Hemoglobin and creatinine are normal. Which of the following
options may improve overall survival while alleviating cancer-associated
bone pain?
A. Zoledronic acid
B. Radium-223
C. Calcitonin
D. Dexamethasone
Question 21.3 An 85-year-old female was recently diagnosed with PD-L1
>50% NSCLC. She has an ECOG performance status of 1 and is interested in
pursuing cancer-directed therapies but also values maintaining her quality of
life. You are considering sending her to a palliative care specialist at your
hospital. What should you counsel the family?
A. The American Society of Clinical Oncology recommends palliative
care for patients with advanced solid tumors only if life expectancy is
less than 6 months.
B. Palliative care may prolong survival, while improving the quality of
life.
C. Palliative care focuses on physical symptom support, rather than
treatment of psychological distress, religious, spiritual, or existential
dimensions of the patient’s illness.
D. Palliative care is provided primarily by nurses in the home and
outpatient setting.
Question 21.4 A 45-year-old female recently completed docetaxel and
cyclophosphamide (TC) for BRCA negative, programmed death-ligand 1
(PD-L1) negative triple-negative breast cancer (pT3N0M0). She presents for
follow-up history and physical exam 3 months following completion of
chemotherapy. She reports significant numbness and tingling in a symmetric
stocking and glove distribution in her hands, which has persisted since
completion of her therapy. She has no loss of motor function or functional
ability. What treatment do you recommend?
A. Low-dose opioid
B. Acetaminophen
C. Serotonin-norepinephrine reuptake inhibitor (SNRI)
D. Physical therapy
Question 21.5 A 50-year-old male with metastatic colorectal cancer
presents for inpatient admission due to pain crisis. He has bulky pelvic
disease and associated acute on chronic pain, which has been inadequately
managed with short-acting oxycodone 5 mg PO Q4H PRN. He is admitted
for pain management with morphine patient-controlled analgesia in the
setting of significant disease progression on imaging. His patient-controlled
analgesia (PCA) breakthrough dose was rapidly up titrated on the first day of
his admission, but he has finally obtained analgesia with a total of 200 mg IV
morphine in the past 24 hours. Unfortunately, you now observe multifocal
myoclonus during your evaluation. There is no seizure-like activity, and he
has normal renal and hepatic function. What initial step would you take in the
management of his myoclonus?
A. Discontinue PCA
B. Increase the breakthrough dose of PCA
C. Administer lorazepam 2 mg IV
D. Administer levetiracetam 500 mg IV
Question 21.6 A 68-year-old female with a history of locally advanced,
unresectable pancreatic cancer presents for her third admission in 2 months
for poorly controlled pain. She has tried increasing the potency of her opioids
including morphine, oxycodone, and, most recently, hydromorphone. She is
currently on transdermal fentanyl at 200 μg/hour for long-acting control of
chronic malignancy-associated pain. What would be your next step in
management of her poorly controlled pain?
A. Initiate morphine 4 mg IV Q3H PRN
B. Add morphine sustained-release 90 mg PO BID
C. Consult interventional pain service to evaluate for nerve block
D. Increase transdermal fentanyl patch by 100%
Question 21.7 A 54-year-old female with metastatic rectal cancer and

https://t.me/ALGRAWANY33
complicated perineal wounds presents from home for symptom management.
She was on home hospice, but has had increasing agitation and delirium,
poorly controlled pelvic pain, and requires inpatient hospice care for
symptom control. In addition to appropriate titration of her IV analgesia,
what other treatment would you initiate at this time?
A. Lorazepam 2 mg IV
B. Haloperidol 2 mg IV
C. Dexamethasone 4 mg IV
D. Diazepam 5 mg IV
Question 21.8 A 45-year-old female with a history of cutaneous T-cell
lymphoma with diffuse skin involvement has required 80 mg PO of long-
acting oxycodone Q8H standing, with 40 mg PO of short-acting oxycodone
Q4H PRN for moderate to severe pain. She completed total body irradiation
(TBI) and systemic therapy and has had resolution of her skin lesions. She is
eager to discontinue opioids and asks you how best to proceed. What do you
recommend?
A. Discontinue opioids and monitor for signs and symptoms of
withdrawal
B. Taper opioids quickly
C. Taper opioids slowly
D. Refer patient to pain medicine for management
Question 21.9 A patient with recently diagnosed ER positive/PR positive
Her2/Neu-breast cancer metastatic to bone complains of pain at the site of a
documented scapular metastasis. She has not responded to acetaminophen
650 mg PO Q6H PRN at home, despite around the clock dosing. She has
normal hepatic and renal function. You prescribe morphine 7.5 mg PO Q4H
PRN for her moderate to severe cancer-associated pain. What other
medication would you prescribe for her at this time?
A. Ondansetron 8 mg PO Q8H PRN
B. Metoclopramide 10 mg PO Q6H
C. Diphenhydramine 25 mg PO Q6H PRN
D. Senna 8.6 mg PO daily
Question 21.10 A 68-year-old male with prostate cancer metastatic to bone
and liver presents with worsening abdominal pain, nausea, and lower back
pain. Magnetic resonance imaging (MRI) of the cervical, thoracic, and
lumbar spine reveals progression of lumbar vertebral involvement and tumor
infiltration of the lumbosacral plexus without cord compression. Abdominal
computed tomography (CT) reveals progression of disease in his liver. He
has normal hepatic and renal function. In addition to opioids and antiemetics,
what medical treatment would provide a helpful adjunct for his pain and
nausea?
A. Lorazepam 2 mg PO Q6H PRN
B. Olanzapine 5 mg PO QHS
C. Dexamethasone 6 mg PO Q6H PRN
D. Scopolamine
Question 21.11 A 55-year-old male with recently diagnosed metastatic
prostate cancer presents to discuss pain management related to pelvic ala
involvement. He works as a school bus driver and is interested in avoiding
opioids if possible. He has no relevant past medical history and has normal
hepatic and renal function. Which of the following do you recommend to
him?
A. Naproxen 500 mg PO BID
B. Methadone 5 mg PO TID
C. Tramadol 50 mg PO Q6H
D. Gabapentin 300 mg PO TID
Question 21.12 You are seeing a patient for follow-up of standard risk
multiple myeloma in remission. Last year, he underwent four cycles of
induction therapy with lenalidomide, bortezomib, and dexamethasone,
followed by autologous stem cell transplant (ASCT). He is now day 100 post-
transplant and about to start lenalidomide maintenance. His past medical
history is notable for non-insulin–dependent diabetes mellitus and coronary
artery disease. He has mild pain at the original sites of bony disease in T10
and L2. He rates the pain as 3 out of 10 in severity. He asks for regimens that

https://t.me/ALGRAWANY33
will have the least risk of causing bleeding. What analgesic plan would you
recommend?
A. Naproxen 500 mg PO BID
B. Oxycodone 5 mg PO TID
C. Acetaminophen 975 mg PO TID
D. Celecoxib 200 mg PO once daily
Question 21.13 A 68-year-old man with advanced colon cancer is admitted
to hospice after having no response to second-line 5-FU, leucovorin, and
irinotecan (FOLFIRI). He has metastatic deposits in the liver, with ascites
and significant right upper quadrant pain. He is managed on 30 mg morphine
sustained release every 12 hours, with 15 mg every 1 to 2 hours for
breakthrough. Six weeks later, while on home hospice, he develops terminal
delirium and slips into unconsciousness. His wife notices that he is restless
despite doses of haloperidol, and he seems to be grimacing and
uncomfortable. The wife states that she is reluctant to give additional doses of
morphine sublingually, because she is fearful she will hasten her husband’s
demise. She calls her oncologist for further recommendations. Which of the
following is the most appropriate response?
A. Pain medications should be restricted in the unconscious dying patient
due to the risk of the double effect of opiates.
B. The patient should be aroused through physical maneuvers (e.g., nail
bed press) so to elicit a grading for the pain prior to dosing the
sublingual morphine.
C. Given the concern for uncontrolled pain, is reasonable to start
scheduled low-dose oral oxycodone hourly at home.
D. The patient’s wife should initially treat the patient with a non-opiate
pain medication, such as ketorolac or ibuprofen, prior to escalating to
sublingual morphine.
E. Using a facial pain scale, opiates for breakthrough pain can be safely
administered to the unconscious patient and control symptoms in the
dying process.
Question 21.14 A 20-year-old man with stage II nonseminomatous germ
cell tumor (NSGCT), initially treated with right orchiectomy, undergoes a
retroperitoneal lymph node dissection (RPLND). He is found to have lymph
node involvement (12/16) and extranodal invasion of the primary tumor. Two
cycles of cisplatin and etoposide as adjuvant therapy are recommended.
Patient tolerates well the chemotherapy with minimal nausea and
subsequently receives G-CSF support. Nevertheless, he calls the overnight
coverage on cycle 1 day 7 complaining of severe low back and hip pain. The
aching is so uncomfortable that the patient can only find relief lying flat on
the hardwood floor. He rates his pain as a 6 out of 10 in severity. What is the
best approach to pain management for this patient?
A. 30 mg morphine sustained release every 24 hours, for three doses, then
reassess response
B. Recommend emergency room for advanced imaging to rule out new
bony metastatic disease and for administration of IV pain medications
C. Naproxen 500 mg BO BID
D. Hydromorphone 2 mg PO every 4 hours as needed
E. Oxycodone 5 mg tablets every 8 hours as needed
Question 21.15 You are seeing a 45-year-old female with recently
diagnosed stage T4N0 stage IIC sigmoid colon cancer. She has a history of
chronic back pain for which she is on maintenance, buprenorphine 16
mg/naloxone 4 mg (Suboxone) once daily, due to a history of opioid
dependence. This medication is managed by her primary care physician. She
undergoes colectomy with lymph node dissection. Pathologic analysis with
10 retrieved lymph nodes shows lymphovascular and perineural invasion and
the patient started adjuvant chemotherapy. Two months into the treatment,
patient complains of worsening low back pain. Advanced imaging of the
spine shows the known degenerative disc disease. She requests additional
pain medication to control this. What is the next best step in management?
A. Begin hydromorphone 2 mg PO every 4 hours
B. Begin oxycodone sustained-release 10 mg every 8 hours
C. Begin morphine sustained-release 15 mg every 12 hours
D. Increase suboxone dose by 25%

https://t.me/ALGRAWANY33
E. Make no changes in the pain regimen and notify primary care
physician of worsening low back pain
Question 21.16 A 75-year-old woman who is receiving palliative care for
advanced pancreatic cancer is seeing you today for a sick visit. She
complains of severe mid-back pain that corresponds to disease progression
seen on recent CT scan. She has shortness of breath that is stable and related
to restrictive lung disease from ascites. She is currently taking 90 mg of
morphine extended release PO every 8 hours with scheduled breakthrough
doses of 30 mg morphine immediate release PO every 4 hours as needed
while awake (she sleeps 8 hours a day). She has chronic nausea for which she
takes ondansetron 8 mg every 8 hours. She has a history of depression for
which she takes citalopram 40 mg once daily. Physical exam reveals a
cachectic woman in no apparent distress. Abdominal exam shows moderate
ascites but no significant tenderness; cardiovascular and respiratory
examinations are unrevealing. The decision is made by palliative care consult
to initiate methadone. What is the next best step in management?
A. Obtain an echocardiogram
B. Obtain a sleep study
C. Obtain a chest radiograph
D. Obtain an electrocardiogram
E. Obtain a complete blood count
Question 21.17 Which of the following conditions or active treatments
would be a contraindication to the use of a fentanyl transdermal patch?
A. Obesity
B. SNRI use
C. Daily fever with night sweats
D. Non-steroidal anti-inflammatory (NSAID) use
Question 21.18 You are caring for a 68-year-old female with metastatic
ovarian cancer admitted to the hospital with recurrent small bowel
obstruction, nausea, and pain related to peritoneal carcinomatosis. She finds
the nasogastric tube uncomfortable, but it improves her nausea. Opioid
medications, required to alleviate abdominal pain, cause significant fatigue
and somnolence despite many days of therapy and trials of different agents.
Her primary oncologist states that, given her steep functional decline, it is
unlikely that further cancer-directed therapy will be available to her. What is
your next step in management?
A. Order renal ultrasound and consider nephrology consultation to assess
her renal impairment
B. Encourage aggressive physical therapy
C. Place hospice referral after speaking with primary oncologist
D. Discuss patient’s goals of care
Question 21.19 You are seeing a 65-year-old patient with colon cancer
metastatic to liver and spine for clinic follow-up 4 weeks after initiating 5-
FU, leucovorin, and oxaliplatin (FOLFOX). He has multiple bony lesions, the
most painful of which are in T10, L3, and L4, which is poorly controlled
despite 5 mg oxycodone immediate release every 4 hours around the clock.
He is started on 30 mg oxycodone sustained release every 12 hours, with 5
mg every 4 hours as needed for breakthrough. Following this dose
adjustment, he has minimal breakthrough oxycodone needs. Nevertheless,
patient presents with 4 days of worsening abdominal discomfort and
constipation despite maximum daily dosing of polyethylene glycol (PEG),
senna, bisacodyl suppository as well as lactulose. A rectal exam reveals no
impaction of stool. Physical exam of the abdomen shows a distended but
otherwise soft, nontender abdomen, and normoactive bowel sounds. Follow-
up CT of the abdomen and pelvis reveals stable abdominal disease and no
evidence of bowel obstruction. Patient is uncomfortable but denies nausea.
What is the next best step in management?
A. Refer for MRI of the abdomen and pelvis
B. Administer IV naltrexone
C. Administer SQ methylnaltrexone
D. Recommend trial of oral lactulose
Question 21.20 A 38-year-old woman with anxiety, depression on
trazodone 75 mg PO twice daily is coming to your office for her newly

https://t.me/ALGRAWANY33
diagnosed stage II hormone receptor positive, HER2/Neu negative breast
cancer. She is status post resection of a 0.6 cm intraductal breast carcinoma
with negative sentinel lymph node biopsy and a 21-gene recurrence score of
24. She is prescribed dose-dense doxorubicin and cyclophosphamide (ddAC)
followed by weekly paclitaxel every 2 weeks. After the completion of ddAC,
patient complains of whole-body pain, 10 out of 10 in severity, despite the
oxycodone 2.5 mg and acetaminophen 1,000 mg she has been taking every 8
hours. She is unable to localize the site of pain and is unable to characterize
the pain any further. Examination reveals a comfortable appearing woman in
no distress, with normal neurological exam. Psychiatric examination reveals a
flat affect, depressive mood, but no suicidal ideation or plan. Her distress
thermometer score is 10, scoring mostly in areas of mood and appearance.
During an interview about how she is coping with therapy, she breaks down
in tears. With avoidant eye contact, she just asks that her pain is taken away.
What is the next best step in management?
A. Referral to psychiatry for cognitive behavioral therapy (CBT)
B. Prescribe hydromorphone 1 mg tablets, every 4 hours as needed for
severe pain
C. Referral to emergency room for psychiatric admission
D. Refuse any additional pain medication until patient completes
chemotherapy

Answers

Question 21.1 The correct answer is D.


Side effects of opioid analgesics often limit their effective use. Common
side effects of opioids include sedation, nausea, vomiting, and
constipation. Management of sedation includes reducing individual drug
doses while giving the drug more frequently or switching to an analgesic
with a shorter plasma half-life. In controlled trials, amphetamine,
methylphenidate, and caffeine have been demonstrated to counteract
opioid-induced sedative effects. It is important to reduce or discontinue
all other drugs that might exacerbate the sedative effects of opioid
analgesics. Opioid analgesics produce nausea and vomiting by action
limited to the medullary chemoreceptor trigger zone. The incidence of
nausea and vomiting is markedly increased in ambulatory patients.
Tolerance develops to these side effects with repeated administration.
Switching to an alternative opioid and using an antiemetic together with
the opioid analgesic are ways to treat this adverse effect. Constipation
results from the action of these drugs at multiple sites in the
gastrointestinal (GI) tract and in the spinal cord to produce a decrease in
the intestinal secretions and peristalsis, which leads to dry, hard stools.
When opioid analgesics are started, a regular bowel regimen, including
use of cathartics and stool softeners, should also be instituted.
Methylnaltrexone is a peripherally acting opioid antagonist targeting the
mu opioid receptors in the gut without interfering with analgesia and is
Food and Drug Administration (FDA)–approved for the indication of
opioid-induced constipation.
For patients receiving greater than 60 mg OME, titration with short-
acting drugs are as effective as titration with sustained-release drugs but
better tolerated. Breakthrough opioid dose should generally be 10% to
20% of the patient’s 24-hour opioid requirement. Morphine should be
avoided in patients with renal failure, due to accumulation of neurotoxic
metabolites.
Question 21.2 The correct answer is B.
Radium-223, an alpha-particle-emitting compound, was shown to have
significant efficacy in metastatic castrate-resistant prostate cancer with
significant palliative effect on bone pain survival improvement and no
significant myelosuppression or other adverse events. Additionally,
research supports a significant palliative effect on bone pain. Patients
with advanced prostate cancer and bone pain should be considered for
treatment with this agent.
Bisphosphonate drugs such as zoledronic acid bind to bone
hydroxyapatite, inhibiting osteoclast activity. The major indication for
these drugs is to prevent skeletal morbidity. Analgesia, when occurs,
usually appears within days and may accrue for many weeks with
repeated treatments. The analgesic effect of bisphosphonates appears to

https://t.me/ALGRAWANY33
be dose-dependent. Calcitonin has also been reported anecdotally to be
useful in patients with malignant bone pain, but without survival benefit.
A series of controlled and uncontrolled surveys have demonstrated that
the use of chronic corticosteroid therapy to reduce pain in patients with
prostate cancer improves quality of life. A major pain-related indication
for steroid use includes bone pain. Adverse effects include anxiety,
restlessness, insomnia, hyperglycemia, and delirium. However,
dexamethasone is not associated with improvement in overall survival.
Question 21.3 The correct answer is B.
The American Society of Clinical Oncology recommends palliative care
as an essential component of comprehensive cancer care for all patients
with advanced solid tumors. Early integration of palliative care in the
disease trajectory, along with high-quality comprehensive cancer care,
may even prolong survival, while improving the quality of life. Control
of other symptoms, treatment of psychological distress, and attention to
the religious, spiritual, and existential dimensions of the patient’s illness
experience should be concurrently addressed to maintain the patient’s
quality of life throughout the cancer illness course from diagnosis to
death. Palliative care interdisciplinary teams (which often include
physicians, advanced practitioners, nurses, spiritual care, social workers,
psychologists, and bereavement counselors) specialize in providing this
type of care.
Question 21.4 The correct answer is C.
Duloxetine is a balanced and potent dual reuptake inhibitor of serotonin
and norepinephrine (SNRI) and has been demonstrated to be superior to
placebo in treating painful chemotherapy-induced peripheral
neuropathy. It has been shown to yield improvements in pain, function,
and quality of life at a dose of 60 mg PO daily. SNRIs have a better
toxicity profile compared to tertiary amine tricyclic antidepressants
(TCAs).
Neuropathic pain accounts for 15% to 20% of pain problems that are
difficult to manage. Generally, neuropathic pain is opioid-resistant and
opioid drugs should not be used in this patient population. NSAIDs and
corticosteroids may serve as helpful adjuncts for neuropathic pain.
Acetaminophen is not recommended in this setting. Physical therapy
may benefit patients with phantom limb pain, but generally not for those
with chemotherapy-induced peripheral neuropathy (CIPN).
Question 21.5 The correct answer is C.
Multifocal myoclonus may occur with high doses of all opioid drugs.
Morphine at high doses produces myoclonus, which has not been
directly associated with known active metabolites such as morphine-6-
glucuronide. In patients with myoclonus, the use of benzodiazepines
such as lorazepam has been reported anecdotally effective.
The patient is not experiencing signs or symptoms of opioid
hyperexcitability (opioid hyperalgesia) such as uncontrolled pain,
hypervigilance, total body hyperalgesia, and allodynia, so
discontinuation of the PCA and rotation to another opioid is not
appropriate. As the patient has achieved appropriate analgesia, a dose
increase is not indicated at this time. Whereas myoclonus may occur
with high doses of all opioid drugs, seizures are more commonly
associated with the use of meperidine or naloxone. Since the patient has
no seizure-like activity, the use of an antiepileptic is not indicated.
Question 21.6 The correct answer is C.
A sympathetic block is effective in conditions with vasomotor or
visceromotor hyperactivity. This hyperactivity accompanies many of the
cancer-related pain syndromes such as visceral pain or plexopathies. The
most commonly used sympathetic block is that of the celiac ganglion for
pain due to abdominal malignancies, including cancer of the pancreas,
stomach, duodenum, liver, gallbladder, adrenal gland, and colon.
Nociceptive fibers of the splanchnic, sympathetic, vagal, phrenic, and
somatic nerves converge on the celiac ganglion, which is amenable to a
regional block. Although there has been debate about the usefulness of
this procedure in patients with pancreatic cancer, it should be considered
as an option, together with pharmacologic approaches. A splanchnic
block may also be helpful in the management of patients with intractable
pain. Nerve blocks may be completed by interventional pain, anesthesia,

https://t.me/ALGRAWANY33
or interventional radiology.
Although intravenous morphine would be an appropriate choice for
short-acting analgesia in this patient, breakthrough dose should be
approximately 10% to 15% of 24-hour opioid requirements. Given that
the patient is on 200 μg/hour of transdermal fentanyl, a 4-mg IV
morphine breakthrough dose would be too small for effective analgesia.
Adding a second long-acting opioid is not an appropriate next step in the
management of this patient with poorly controlled pain. A short-acting
IV or PO option should be tried first, prior to increasing long-acting
regimen, preferably with the same agent.
The transdermal route is a convenient way to deliver a potent opioid
on a continuous basis. Serum fentanyl concentrations increase and
steady-state levels are approached at 12 to 24 hours. After patch
removal, the drug persists in the skin, with falling blood levels over 24
hours. When a patient is started on the fentanyl patch, there is up to a
12- to 15-hour delay in the onset of analgesia and a 24- to 72-hour
equilibration period. Without knowing the patient’s short-acting opioid
requirements for appropriate analgesia, increasing her long-acting would
not be the appropriate next step.
Question 21.7 The correct answer is B.
Haloperidol is the treatment of choice to manage hallucinations and
agitated delirium in patients receiving opioid analgesics.
Lorazepam and diazepam can be helpful in the management of severe
anxiety or as adjunct for nausea. However, in the setting of delirium,
benzodiazepines may actually worsen symptoms. Dexamethasone is an
important adjunct in the management of cancer-associated pain,
particularly if related to bone pain, capsular distention, or cerebral
edema. It is also a helpful adjunct in the management of nausea and
cancer-associated fatigue. Dexamethasone does not play a role in the
management of delirium and may worsen agitation. Diazepam may be
helpful in the management of muscle spasms, anxiety, as well as opioid
withdrawal and seizures.
Question 21.8 The correct answer is C.
Opioids should be tapered slowly. The long-term administration of
opioid analgesics is associated with the development of physical
dependence. The sudden cessation of the opioid analgesic produces
signs and symptoms of withdrawal: agitation, tremors, insomnia, fear,
marked autonomic nervous system hyperexcitability, and exacerbation
of pain. Slowly tapering the dose of the opioid analgesic prevents such
symptoms. The appearance of abstinence symptoms from the time of
drug withdrawal is related to the elimination half-life for the particular
drug. For example, with morphine, withdrawal symptoms occur within 6
to 12 hours after drug cessation. For patients who have been on chronic
opioids, a 10% 24-hour opioid dose decrease per month is recommended
to avoid withdrawal.
Opioids, particularly at higher doses, should not be discontinued
suddenly because this may precipitate signs and symptoms of
withdrawal, which can cause significant discomfort and distress.
Similarly, opioids should not be tapered quickly, but rather slowly and
in a stepwise manner under the guidance of a physician. Pain medicine
consultation is not required to wean a patient off opioids, unless there is
concern for underlying opioid use disorder.
Question 21.9 The correct answer is D.
The use of bowel regimens to manage depressed GI motility is critical.
Because nearly all patients have some constipation when receiving
opioids, a bowel regimen should be prescribed along with all opioid
therapies, unless there is a serious contraindication. Furthermore,
patients will continue to experience constipation despite tolerance to
other side effects. In additional to constipation, common side effects of
opioids include nausea, vomiting, sedation, respiratory depression,
multifocal myoclonus, and seizures at higher doses.
Although nausea is not an uncommon side effect of opioids, it is not
standard practice to prescribe antiemetics such as metoclopramide or
ondansetron concurrently with opioids unless symptoms present.
Diphenhydramine plays a role in managing allergic reactions, which are
exceedingly rare with opioids.
Question 21.10 The correct answer is C.

https://t.me/ALGRAWANY33
The use of chronic corticosteroid therapy to reduce pain in patients with
prostate cancer improves quality of life. The major pain-related
indications for corticosteroid use include refractory neuropathic pain,
bone pain, pain associated with capsular expansion (e.g., liver capsule
secondary to hepatic metastases), and headache due to increased
intracranial pressure. In patients with infiltration of the lumbosacral
plexus, corticosteroids provide additive analgesic effects. Lower
corticosteroid doses are efficacious with reduced toxicity.
Dexamethasone 6 to 10 mg intravenously or orally every 6 hours is
reasonable and appears to be just as effective as higher doses, with a
lower rate of anxiety, restlessness, insomnia, hyperglycemia, and
delirium.
Question 21.11 The correct answer is A.
Naproxen and other NSAIDs have analgesic, anti-inflammatory,
antipyretic, and antiplatelet actions. Non-opioid analgesics are generally
thought to produce analgesia by inhibiting activation of peripheral
nociceptors through their prevention of the formation of prostaglandin
E2, a known sensitizer of peripheral receptors to nociceptive stimulation
from tissue injury. NSAIDs differ from one another, both in duration of
their analgesic action and pharmacokinetic profile. Ibuprofen has a
shorter half-life (~2 hours) than naproxen (~13 hours). Because clinical
experience has found that some patients respond better to one NSAID
than another, each patient should be given an adequate trial of one drug
on a regular basis before switching to another. Between 20% to 40% of
patients obtain pain relief with the use of non-opioid analgesics alone.
Risk factors for bleeding associated with NSAID use include advanced
age, higher doses, concomitant administration of corticosteroids, and
history of either ulcer disease or previous GI complications from
NSAIDs. Commonly, proton-pump inhibitors are prescribed, which
have been associated with a significantly decreased frequency of
NSAID-related dyspepsia and fewer ulcers diagnosed by endoscopy.
Methadone should only be considered as a second-line drug for
cancer pain patients who have had prior exposure to opioids. It is an
opioid medication that also acts as an N-methyl-D-aspartate (NMDA)
receptor antagonist, which has a role not only to treat opioid dependence
but also for pain management and as an adjunct for neuropathic pain.
Methadone for the indication of analgesia should be prescribed with
consultation of a pain specialist. Tramadol is a weak opioid, and the
patient has requested to avoid opioids for analgesia, if possible.
Gabapentin is an anticonvulsant used to manage neuropathic pain. It
does not have a role in the first-line for management of mild cancer-
associated pain in the setting of prostate cancer metastatic to bone.
Question 21.12 The correct answer is C.
This patient has mild pain that is not requiring the use of opiates
according to the World Health Organization (WHO) ladder. The most
appropriate management for this patient’s mild pain would be standing
acetaminophen.
Oxycodone is not indicated for this level of pain per WHO ladder,
although it may be reasonable to add a low dose as needed. NSAIDs
represent a reasonable choice in patients with cancer-related pain that is
mild. However, they are contraindicated in patients with increased risk
of bleeding. Moreover, in patients who have a history of cardiovascular
disease, NSAIDs should be avoided because they may increase the risk
of coronary thrombosis. Both nonselective NSAIDs and selective
NSAIDs carry black box warnings for this reason. As this patient will be
started on lenalidomide, his risk for thrombosis is even further
increased, and many patients will be offered aspirin to mitigate it.
Adding NSAIDs to aspirin would further increase platelet dysfunction
and bleeding risk.
Question 21.13 The correct answer is E.
It has been estimated that up to 50% of patients die with terminal pain.
The patient’s pain is clearly uncontrolled as evidenced by facial
grimacing and restlessness. The lack of response to a haloperidol lends
evidence that this is a symptom of pain rather than terminal delirium and
agitation. Facial pain scales, such as the Wong-Baker Faces Pain Rating
Scale, are validated instruments that allow clinicians and caregivers to
monitor and track pain in a patient who is unable to communicate.

https://t.me/ALGRAWANY33
Attempting to arouse a terminally unconscious patient to more
accurately assess pain through an inventory is not likely to be successful
and will only compound the patient’s pain and discomfort. Non-opiate
pain medications such as ketorolac or ibuprofen would not be
appropriate in this situation due to the presence of moderate to severe
pain. In this case sublingual morphine would be the best choice.
Scheduled pain medications hourly is not feasible at home and creates
an extra burden on caregivers who already face risk of burnout from
caregiver fatigue. Should a patient require breakthrough medications
hourly, admission to an inpatient hospice facility would be indicated not
only for optimal symptom control for the dying patient but also for
respite for the caregiver. Oxycodone use should be avoided in
decompensated liver failure, regardless of the etiology, as the
medication is extensively metabolized by CYP enzymes.
Question 21.14 The correct answer is C.
Bone pain is a common side effect from granulocyte-colony stimulating
factor (G-CSF), seen in about 31% of patients. Because many patients
will not explicitly tell their providers or realize the connection between
G-CSF and acute back pain, oncologists must be vigilant. Chemotherapy
with etoposide and cisplatin, aside from being highly emetogenic, is
highly myelosuppressive, requiring the use of G-CSF for primary
prevention of neutropenic fever. Emergency room evaluation and IV
pain medications are generally not required for pain associated with G-
CSF. The only exceptions to this would in case of acute onset of left
upper quadrant abdominal pain with or without guarding or abdominal
rigidity, or if the patient had acute onset of left shoulder pain, which
may be associated with the rare complication of splenic rupture. In
general, NSAIDs such as naproxen are the optimal choice in the setting
of bone pain related to G-CSF administration.1
New metastatic disease after recent staging is unlikely, and testicular
cancer is more likely to have pulmonary visceral metastases (which are
usually painless) than spinal or axial bony metastases. There is
insufficient evidence to recommend any particular antihistamine or
combination of antihistamines, although loratadine is frequently used in
conjunction with NSAIDs based on case reports. In case of persistent,
excruciating pain not responding to these measures, short-acting opiates
such as hydromorphone or oxycodone may be indicated. Generally,
hydromorphone is preferred in the setting of acute or chronic kidney
disease. Initiating extended release opiates for the treatment of acute
pain as in this case is not recommended.
Question 21.15 The correct answer is E.
This patient has chronic pain being managed with buprenorphine.
Because this patient is very likely to be cured of her localized colon
cancer, it is not appropriate for the oncologist to interfere with the pain
contract that has been set up between the patient and her primary care
physician. A diagnosis of cancer does not absolve medical oncologists
from checking prescription monitoring programs, from completing
opiate contracts with their patients, and making appropriate choices for
pain, especially among patients whose burden of disease is low and not
reasonably contributing to their pain.
Question 21.16 The correct answer is D.
Methadone is an effective medication for pain among patients who do
not respond to high doses of traditional opioids. However, it can cause
Q-T prolongation and arrhythmias. Therefore, obtaining an
electrocardiogram is an essential step prior to initiation of therapy, as
well as during therapy. Knowing her baseline Q-T interval is important
prior to changing her regimen. If the corrected Q-T interval (QTc) is
greater than 480 ms, methadone therapy should be held until further
assessment. Q-T prolonging medications, such as ondansetron and
citalopram, should be modified or substituted if possible.
Question 21.17 The correct answer is C.
Fentanyl is a short-acting, potent narcotic medication that is an effective
option for patients with uncontrolled pain who are unable to take
medication by mouth. Heat, whether from an external source or due to
fever and night sweats, may alter the absorption of the drug and lead to
unintentional overdose. The FDA has released a black box warning for

https://t.me/ALGRAWANY33
this issue, noting that patients who are either febrile or are exposing the
patch to heating pads or electric blankets are at risk for accidental
overdose and death.
Morbid obesity is a caution for the use of fentanyl, but not a
contraindication. SNRI use with fentanyl has been reported to increase
the risk of serotonin syndrome, although this is a rare toxicity. NSAIDs
are not contraindicated in patients using fentanyl transdermal patches.
Question 21.18 The correct answer is D.
When treatment approaches are being developed, there must be an open
discussion about advance directives so that the physician has a clear
understanding of the patient’s goals of care. Knowledge of the patient’s
decisions about symptom management, understanding of and decisions
surrounding cancer-directed therapy options, resuscitation, and living
wills, improves the physician’s ability to appropriately and humanely
care for the dying patient with advanced disease.
Question 21.19 The correct answer is C.
This patient has opioid-induced constipation. For patients with three or
more days of severe constipation without response to laxatives or
enemas, it is reasonable to offer methylnaltrexone,2 which has been
shown to produce high rates of laxation within 4 hours of the first
subcutaneous dose. Referral for further imaging would not alleviate the
primary issue, which is opioid-induced constipation. Furthermore, the
patient does not have evidence of a bowel obstruction, which would be a
contraindication to methylnaltrexone administration. Reversing systemic
analgesia and opioid effects with IV naltrexone is not recommended as
this would precipitate withdrawal, acutely worsen pain, and would have
nonselective effects on the GI tract. Because the patient has already
utilized maximum recommended doses of osmotic (PEG, lactulose) and
cathartic (senna, bisacodyl) laxatives, further treatment with lactulose is
unlikely to be effective. Methylnaltrexone should be used with caution
in patients with GI malignancies, diverticulosis, or suspected
obstruction, because there is an increased risk of colitis and perforation
in these subgroups. Side effects of methylnaltrexone more commonly
include abdominal pain, nausea, and flatulence.
Question 21.20 The correct answer is A.
The patient in this scenario is complaining of diffuse pain that is not
concordant with physical examination. In contrast to a patient who is
malingering (feigning pain to obtain medications for secondary gain),
this patient is having a true physical manifestation of psychological
distress. Terms such as psychic pain or existential pain can be used to
describe these scenarios where patients have somatization of their
psychological distress. Although the patient is not actively suicidal,
which would require immediate psychiatric admission, she is at high risk
for worsening distress and pain, indicating the need for an early
intervention with a multimodality approach (social work, psychology) is
indicated. CBT for pain and distress in cancer patients is an important
adjunct to pharmacologic therapy. Rather than escalating opioids, which
may precipitate untoward short- and long-term side effects and sequelae,
a multimodality approach is recommended.3

References
1. Moore DC, Pellegrino AE. Pegfilgrastim-induced bone pain: a review on incidence, risk factors,
and evidence-based management. Ann Pharmacother. 2017;51(9):797–803.
2. Thomas J, Karver S, Cooney GA, et al. Methylnaltrexone for opioid-induced constipation in
advanced illness. N Engl J Med. 2008;358(22):2332–2343.
3. Zaza C, Baine N. Cancer pain and psychosocial factors: a critical review of the literature. J Pain
Symptom Manage. 2002;24(5):526–542.
___________
Corresponding chapter in DeVita, Hellman, and Rosenberg’s Cancer: Principles & Practice of
Oncology, Eleventh Edition: 138 (Management of Cancer Pain).

https://t.me/ALGRAWANY33
22 Oral and Gastrointestinal Complications
Sasha Haarberg and Jennifer Hedgecorth

QUESTIONS

Each of the numbered items below is followed by lettered answers. Select the
ONE lettered answer that is BEST in each case unless instructed otherwise.

Question 22.1 Which of the following neurotransmitters is predominately


responsible for acute emesis?
A. Dopamine
B. 5-HT3
C. Histamine
D. Acetylcholine
Question 22.2 A patient with a past medical history significant for chronic
heart failure is experiencing nausea and vomiting after receiving cisplatin. In
addition to dehydration, lab results show the patient have profound
hypokalemia and hypomagnesemia. Which antiemetic should be used with
caution due to potential for QTc prolongation?
A. Lorazepam
B. Prochlorperazine
C. Olanzapine
D. Famotidine
Question 22.3 Which of the following agents is a selective antagonist of
substance P binding the NK1 receptor?
A. Granisetron
B. Diphenhydramine
C. Prochlorperazine
D. Fosaprepitant
Question 22.4 Which of the following 5-HT3 receptor antagonists differs
from the other agents by having a significantly longer half-life and higher
binding affinity compared to first-generation 5-HT3 antagonists?
A. Dolasetron
B. Granisetron
C. Ondansetron
D. Palonosetron
Question 22.5 A newly diagnosed ER positive/PR positive HER2-breast
cancer patient will receive neoadjuvant chemotherapy with dose-dense AC
(doxorubicin 60 mg/m2 IV on D1 and cyclophosphamide 600 mg/m2 IV on
D1) for four cycles. Which of the combinations below is an appropriate
premedication strategy based on the emetogenic potential of her
chemotherapy regimen?
A. Granisetron + fosaprepitant + dexamethasone
B. Ondansetron + dexamethasone
C. Aprepitant + dexamethasone
D. Prochlorperazine + famotidine + dexamethasone
Question 22.6 A patient with pancreatic cancer presents to clinic to receive
cycle four of FOLFOX (oxaliplatin, fluorouracil, and leucovorin [LV]).
Antiemetics used during previous cycles included dexamethasone,
palonosetron, and aprepitant. However, the patient required rescue
antiemetics and hydration therapy even with the premedications listed above.
Which medication is the best option to add to this patient’s chemotherapy-
induced nausea and vomiting (CINV) regimen?
A. Dronabinol
B. Famotidine
C. Olanzapine

https://t.me/ALGRAWANY33
D. Scopolamine
Question 22.7 Which structure that processes emetogens is often referred
to as the chemoreceptor trigger zone?
A. Area postrema
B. Enterochromaffin cells
C. Lateral reticular formation
D. Nucleus tractus solitarius
Question 22.8 A patient is to receive upper abdominal radiation for
stomach cancer. Which of the following selections correctly matches risk
level and radiotherapy-induced nausea/vomiting (RINV) recommendations
for this patient’s treatment?
A. High risk – Ondansetron + dexamethasone
B. High – Olanzapine _ dexamethasone
C. Moderate – Granisetron +/− dexamethasone
D. Moderate – Prochlorperazine +/− dexamethasone
Question 22.9 Which of the following antiemetic medications is correctly
matched with neurotransmitter it affects for the treatment of CINV?
A. Ondansetron – dopamine
B. Rolapitant – 5HT-3
C. Prochlorperazine – substance P
D. Dronabinol – cannabinoid
Question 22.10 Which of the following patients is at highest risk for
CINV?
A. A 60-year-old male with metastatic non–small cell lung cancer
(NSCLC) to be treated with immunotherapy alone
B. A 30-year-old female with a history of hyperemesis gravidarum with
newly diagnosed triple-negative breast cancer to be treated with
chemotherapy and immunotherapy
C. A 75-year-old women with recurrent renal cell carcinoma to be treated
with a tyrosine kinase inhibitor
D. A 65-year-old man with chronic drug and alcohol use with prostate
cancer to be treated with chemotherapy
Question 22.11 Which medication should be considered for severe or
persistent diarrhea due to its multiple antidiarrheal actions that include
suppression of release of insulin, glucagon, vasoactive intestinal peptide
(VIP), and gastric acid, as well as reduction in motility and pancreatic
exocrine function?
A. Atropine/diphenoxylate
B. Budesonide
C. Loperamide
D. Octreotide
Question 22.12 Which chemotherapy agent is incorrectly associated with
the corresponding most common gastrointestinal (GI) side effect?
A. Capecitabine – diarrhea
B. Irinotecan – diarrhea
C. Ipilimumab – constipation
D. Vincristine – constipation
Question 22.13 A 52-year-old female weighing 75 kg presents to the clinic
for evaluation with severe diarrhea (six stools per day) after receiving three
cycles of ipilimumab and nivolumab in combination for treatment of
metastatic melanoma. Upon imaging, inflammation throughout the colon is
noted by the radiologist. Which treatment is most appropriate?
A. Prednisone 80 mg PO once daily
B. Octreotide 100 mcg IV three times daily
C. Infliximab 375 mg IV every 2 weeks
D. Loperamide 2 mg PO every 2 hours
Question 22.14 Which of the following is likely to cause the most severe
diarrhea?

https://t.me/ALGRAWANY33
A. Capecitabine
B. Single agent continuous infusion 5-FU
C. 5-FU bolus with LV followed by continuous infusion 5-FU
D. Oxaliplatin
Question 22.15 A patient is admitted to the bone marrow transplant unit to
receive an autologous hematopoietic stem cell transplant (HSCT) with high-
dose melphalan as a conditioning regimen. Which of the following is the
most appropriate to prevent mucositis?
A. Cryotherapy (ice chips)
B. Baking soda/salt water rinses
C. Dexamethasone elixir
D. Glutamine
Question 22.16 A patient presents to clinic with a history of chronic graft-
versus-host disease following allogenic-HSCT for her acute myelogenous
leukemia (AML), with the symptoms limited to the oral cavity. Which of the
following is the best treatment option?
A. Dexamethasone elixir
B. Sucralfate
C. Amifostine
D. Pilocarpine
Question 22.17 A 63-year-old male receiving radiation and chemotherapy
for treatment of oropharyngeal cancer reports to clinic with complaints of
severe oral pain and mucositis. Which of the following is the best treatment
option for him?
A. Sucralfate
B. Vicious lidocaine 2%
C. Amifostine
D. Cyrotherapy
Question 22.18 Which of the following is a risk factors for oral mucositis?
A. Smoking
B. Neutropenia
C. Dehydration
D. All of the above

Answers

Question 22.1 The correct answer is B.


Acute CINV is thought to be primarily due to 5-HT–mediated pathways.
5-HT3 receptors are found on vagal afferent fibers within the area
postrema and in the nucleus tractus solitarius. Dopamine was once
thought to be the primary neurotransmitter in acute CINV and still plays
a major role; however, 5-HT is now more commonly accepted at the
most relevant due to effectiveness of CINV treatment with 5-HT3
antagonists. Histamine and acetylcholine have roles in other types of
emesis, such as motion sickness. However, these play a small role in
CINV.
Question 22.2 The correct answer is C.
Olanzapine, although an effective antiemetic, may cause QTc
prolongation and should be used with caution in this patient due to their
history of heart failure and current electrolyte abnormalities. Lorazepam,
prochlorperazine, and famotidine do not have the same risks of QTc
prolongation.
Question 22.3 The correct answer is D.
Fosaprepitant is a selective antagonist of substance P at the NK1
receptor. Granisetron is a 5-HT3 inhibitor, diphenhydramine blocks
histamine from H1-receptors, and prochlorperazine phenothiazine
derivative that blocks dopamine from D1 and D2 receptors in the brain.
Question 22.4 The correct answer is D.
Palonosetron is a second-generation 5-HT3 inhibitor with a half-life of

https://t.me/ALGRAWANY33
about 40 hours in adults. Dolasetron, granisetron, and ondansetron are
first-generation 5-HT3 antagonists with half-lives of 7 to 8 hours, 5 to 9
hours, and 3 to 6 hours, respectively, in IV formulations.
Question 22.5 The correct answer is A.
The chemotherapy regimen this patient will receive is highly emetogenic
based on the Hesketh classification because it contains an anthracycline
(doxorubicin) and cyclophosphamide. The recommended antiemetic
regimens for highly emetogenic chemotherapy should include a 5-HT3
inhibitor such as granisetron, an NK-1 receptor antagonist such as
fosaprepitant, and dexamethasone with or without olanzapine.1
Question 22.6 The correct answer is C.
FOLFOX (oxaliplatin, fluorouracil, and LV) is classified as moderately
emetogenic based on the Hesketh classification, which would include a
5-HT3 inhibitor plus dexamethasone for appropriate premedication.
Despite the treatment, the patient is still experiencing refractory CINV
even with the addition of an NK-1 receptor antagonist (aprepitant).
Olanzapine would be the most appropriate drug to add to the current
regimen due to its high therapeutic index. Olanzapine is an antagonist of
several neurotransmitters, including dopamine and serotonin, and is
effective in preventing both acute and delayed CINV. Dronabinol can be
used for refractory CINV or as a rescue antiemetic; however, it is
considered to be less effective and not as supported by clinical trials.
Famotidine is an inhibitor of histamine at H2 receptors and can decrease
gastric acid secretion. It may be helpful for some of the GI symptoms
experienced with chemotherapy, but not likely to help with refractory
CINV in this case. Scopolamine is an anticholinergic used for motion
sickness and postoperative nausea and vomiting. It may be used in some
cases for refractory CINV, but it is not Food and Drug Administration
(FDA)–approved for this indication and hence other therapies should be
tried first.
Question 22.7 The correct answer is A.
The area postrema is commonly referred to as the chemoreceptor trigger
zone. It is located outside of the blood–brain barrier, so it is accessible to
potential emetogens in both the blood and cerebrospinal fluid (CSF).
The area postrema cannot independently initiate vomiting.
Enterochromaffin are located in the GI tract and play a role in CINV by
chemotherapy directly or indirectly causing the release of local
mediators that stimulate the vagal and splanchnic afferent fibers, leading
to signals to the brain stem. Lateral reticular formation and nucleus
tractus solitarius are other areas in the brain that coordinate vomiting.
Question 22.8 The correct answer is C.
The patient is receiving upper abdominal radiation, which is categorized
as moderate risk for radiotherapy-induced emesis. The antiemetic
guidelines recommend prophylaxis with a 5-HT3 inhibitor plus optional
dexamethasone.2
Question 22.9 The correct answer is D.
Dronabinol is correctly associated with cannabinoid receptors.
Ondansetron is a 5-HT3 inhibitor, rolapitant is an NK-1 receptor
antagonist, and prochlorperazine is a phenothiazine derivative that
blocks dopamine from D1 and D2 receptors in the brain.
Question 22.10 The correct answer is B.
The risk factors for CINV include female sex, younger age, and history
of nausea and vomiting with previous chemotherapy. Other potential
risk factors include history of nausea and vomiting associated with
pregnancy and history of motion sickness. The emetogenic risk is higher
in a 30-year-old woman with history of hyperemesis gravidarium likely
to be treated with a platinum-based combination than in an older man
who is likely to be treated with single agent docetaxel. Immunotherapy
alone and tyrosine kinase inhibitors have lower emetogenic risk.
Question 22.11 The correct answer is D.
Octreotide has many mechanisms of action, which can be effective in
treating diarrhea, including suppression of release of insulin, glucagon,
VIP, gastric acid secretion, reduction in motility and pancreatic exocrine

https://t.me/ALGRAWANY33
function, and increased absorption of water, electrolytes, and nutrients
from the GI tract. Atropine/diphenoxylate inhibits excessive GI motility
and propulsion by inhibiting peristalsis by stimulation of mu and delta
opioid receptors from the diphenoxylate (an analogue of meperidine)
and anticholinergic activity from atropine. Budesonide is an anti-
inflammatory corticosteroid that also has multiple effects but is thought
to inhibit multiple cell types and mediators to control inflammation.
Loperamide is another opioid receptor agonist that acts on mu receptors
in the GI tract to inhibit peristalsis, prolong transit time, and diminish
fluid and electrolyte loss.
Question 22.12 The correct answer is C.
Diarrhea is a common adverse effect of checkpoint-blocking antibodies
and most commonly presents after 6 to 8 weeks of treatment. The
incidence of diarrhea is higher in patients receiving cytotoxic T-
lymphocyte antigen 4 (CTLA-4)–blocking antibodies compared to
inhibition of the programmed cell death protein 1 (PD-1) receptor.
Question 22.13 The correct answer is A.
This patient treated with ipilimumab has grade 2 colitis based on the
number of bowel movements and presence of colitis on imaging. The
management includes withholding ipilimumab and starting prednisone 1
mg/kg/day. Loperamide could be considered if the patient was
experiencing grade 1 diarrhea. Infliximab is recommended for patients
without improvement on corticosteroids.3,4
Question 22.14 The correct answer is C.
Diarrhea is most commonly observed when 5-FU is co-administered
with LV. It is slightly more common with bolus rather than infusional
administration, but it occurs with all administration schedules.
Capecitabine is reported to have a 30% to 40% occurrence of diarrhea.
Diarrhea is not a common toxicity from oxaliplatin.
Question 22.15 The correct answer is A.
Oral cryotherapy leads to an appreciable reduction in oral mucositis in
adults receiving high-dose melphalan before HSCT. None of the other
options have been specifically evaluated in the high-dose melphalan
before HSCT patient setting.5
Question 22.16 The correct answer is A.
The most commonly recommended therapy for the management of
mucosal involvement of oral chronic graft-versus-host disease (GVHD)
is topical high or ultra-high-potency corticosteroids, calcineurin
inhibitors, and analgesics. Patients with symptomatic disease limited to
the oral cavity have been found to benefit from topical steroids such as
dexamethasone elixir (0.5 mg/mL) as a mouth rinse (10 mL) for 2 to 3
minutes at least four times daily.6
Question 22.17 The correct answer is B.
Viscous lidocaine is the best treatment option for this patient with
established symptomatic mucositis. Although both amifostine and
cryotherapy are options for prevention of mucositis, there is no data on
their efficacy in patients with oral pain and mucositis. Sucralfate is not
recommended for patients receiving radiation therapy for head and neck
cancer.5
Question 22.18 The correct answer is D.
There are several patient- and treatment-related risk factors for
experiencing oral mucositis, including smoking, neutropenia, and
dehydration.7,8

References
1. Hesketh PJ, Kris MG, Basch E, et al. Antiemetics: American Society of Clinical Oncology clinical
practice guideline update. J Clin Oncol. 2017;35:3240–3261.
2. Roila F, Herrstedt J, Aapro M, et al. Guideline update for MASCC and ESMO in the prevention of
chemotherapy- and radiotherapy-induced nausea and vomiting: results of the Perugia consensus
conference. Ann Oncol. 2010;21(suppl 5):v232–v243.
3. Weber JS, Kähler KC, Hauschild A. Management of immune-related adverse events and kinetics of
response with ipilimumab. J Clin Oncol. 2012;30:2691–2697.
4. Postow MA. Managing immune checkpoint-blocking antibody side effects. Am Soc Clin Oncol
Educ Book. 2015;76–83.

https://t.me/ALGRAWANY33
Fall-Dickson JM, Berger AM. Oral manifestations and complications of cancer treatment. In:
5.
Shuster JL, Von Roenn JH, eds. Principles and Practice of Palliative Care and Supportive
Oncology. 3rd ed. Lippincott Williams & Wilkins; 2006:210.
6. Wolff D, Anders V, Corio R, et al. Oral PUVA and topical steroids for treatment of oral
manifestations of chronic graft-vs.-host disease. Photodermatol Photoimmunol Photomed.
2004;20:184–190.
7. Barasch A, Peterson DE. Risk factors for ulcerative oral mucositis in cancer patients: unanswered
questions. Oral Oncol. 2003;39:91–100.
8. Dodd MJ, Miaskowski C, Shiba GH, et al. Risk factors for chemotherapy-induced oral mucositis:
dental appliances, oral hygiene, previous oral lesions, and history of smoking. Cancer Invest.
1999;17:278–284.
___________
Corresponding chapters in DeVita, Hellman, and Rosenberg’s Cancer: Principles & Practice of
Oncology, Eleventh Edition: 127 (Nausea and Vomiting), 128 (Diarrhea and Constipation), and 129
(Oral Complications).
23 Oncologic Emergencies
Elizabeth Prsic and Matthew Austin

QUESTIONS

Each of the numbered items below is followed by lettered answers. Select the
ONE lettered answer that is BEST in each case unless instructed otherwise.

Questions 23.1–3 These questions are sequential and refer to the same
clinical case.
Question 23.1 A 48-year-old male presents with a history of progressive
facial swelling and shortness of breath for 1 month. He has a 40-pack-year
smoking history. On examination, the patient has cervical and thoracic
venous distention, worse when supine. Laboratory test results revealed
normal blood counts, basic metabolic profile, and liver functions. Computed
tomography (CT) scan revealed a 6 × 5 cm lung mass with liver metastases.
What is the most likely cause of his condition?
A. Small-cell lung cancer (SCLC)
B. Hodgkin lymphoma
C. Lymphoblastic lymphoma
D. Superior vena cava (SVC) thrombosis
Question 23.2 Biopsy was performed and showed SCLC. Which is the best
initial modality of treatment for this patient?
A. Surgery
B. Radiation therapy
C. Chemotherapy
D. SVC stent

https://t.me/ALGRAWANY33
Question 23.3 What is the most likely location for the primary tumor?
A. Peripheral left lung
B. Peripheral right lung
C. Central left lung
D. Central right lung
Question 23.4 Which among the following is the earliest fundoscopic sign
for increased intracranial pressure (ICP)?
A. Absence of venous pulsations within the center of the optic disk
B. Blurring of the disk margins
C. Disk hemorrhage
D. Foster-Kennedy syndrome
Question 23.5 A 57-year-old man presents with severe back pain and
bilateral lower extremity weakness for 3 days. Magnetic resonance imaging
(MRI) of the spine reveals metastatic lesion at T10 vertebral body with
significant cord compression. What are the most likely primary tumors to
present with malignant spinal cord compression at diagnosis?
A. Prostate cancer and breast cancer
B. Non-Hodgkin lymphoma and lung cancer
C. Non–small cell lung cancer (NSCLC) and breast cancer
D. Breast cancer and colon cancer
Question 23.6 Which of the following signs is common in patients
presenting with increased ICP?
A. Headache reaching maximum intensity in the evening
B. Gait apraxia
C. Immediate relief following vomiting
D. Urinary incontinence
Question 23.7 Which of the following statements regarding rasburicase is
TRUE?
A. Mechanism of action is inhibition of urate oxidase
B. Should be avoided in patients with glucose 6-phosphate deficiency
(G6PD)
C. Rasburicase reduces renal impairment in adult patients with
hyperuricemia
D. Cost of rasburicase is roughly equivalent to IV allopurinol
Question 23.8 A 28-year-old male presents with newly diagnosed acute
myeloid leukemia. He has a white cell count (WBC) of 140,000/mm3,
glucose of 96 mg/dL, sodium of 138 mEq/L, potassium of 4.5 mEq/L,
creatinine of 1.1 mg/dL, blood urea nitrogen (BUN) of 15 mg/dL, uric acid of
4.8 mg/dL, albumin of 5.2 g/dL, and calcium of 10.2 mg/dL. He is started on
induction chemotherapy, aggressive intravenous fluids, and allopurinol.
Twelve hours later, the patient developed cardiac arrest not responsive to
cardiopulmonary resuscitation and died. What is the most likely cause of his
death?
A. Leukostasis
B. Electrolyte imbalance
C. Pulmonary embolism
D. Chemotherapy-induced cardiac arrhythmia
Question 23.9 Which of the following is NOT a component of the Kocher–
Cushing reflex observed in patients with increased ICP?
A. Changes in breathing pattern
B. Headache
C. Hypertension
D. Bradycardia
Question 23.10 A 63-year-old male with metastatic squamous cell
carcinoma of the lung presents with worsening confusion, constipation, and
nausea for 3 days. Laboratory tests revealed hemoglobin of 13.5 g/dL, WBC
of 12,500/mm3, platelets of 155,000/mm3, sodium of 146 mEq/L, potassium
of 4.6 mEq/L, bicarbonate of 28 mEq/L, BUN of 44 mg/dL, creatinine of 1.8
mg/dL, albumin of 2.8 g/dL, and calcium of 13.6 mg/dL. He had recent

https://t.me/ALGRAWANY33
dental evaluation and has no dental problems or recent procedures. What is
the most appropriate initial therapy?
A. Intravenous fluids and bisphosphonate
B. Intravenous fluids and furosemide
C. Intravenous fluids, bisphosphonate, and calcitonin
D. Bisphosphonate and furosemide
Question 23.11 A 59-year-old male with a history of prostate cancer with
widely metastatic bony disease presents to the emergency room with sudden
onset, severe lumbar back pain accompanied by lower extremity weakness,
and difficulties with urination. Urgent MRI imaging reveals an L4 pathologic
compression fracture with retropulsion of the posterior cortex resulting in
severe spinal stenosis and cord compression. He is started on high-dose
corticosteroids in the emergency room. What is the next best step in the
management of this patient?
A. Conventional external beam radiation therapy
B. Linear-accelerator–based stereotactic radiosurgery (SRS)
C. Surgical decompression
D. Docetaxel
Question 23.12 A 72-year-old female with a new diagnosis of metastatic
NSCLC expresses concern for mid-thoracic back pain that has slowly
progressed over a period of weeks. It is worse when sneezing and when she
wakes up in the morning. She denies any neurologic deficits. On
examination, the patient has midline tenderness to palpation in the thoracic
region. MRI shows multiple lesions with increased signal on T2 in the
thoracic and lumbar vertebral bodies consistent with metastatic bony disease.
Within the T10 vertebral body, she is noted to have a metastatic lesion with
posterior epidural extension, canal stenosis, and mild cord compression.
What is the definitive therapy of choice for this patient?
A. Surgical decompression
B. Radiation-based therapy
C. High-dose corticosteroids with prolonged taper
D. Urgent cytoreduction with carboplatin plus paclitaxel
Question 23.13 A 55-year-old male has a new diagnosis of diffuse large B-
cell lymphoma (DLBCL) with widespread, bulky adenopathy and is started
on lymphoma-directed treatment while admitted to the hospital. One day after
initiating therapy, he is noted to have new acute kidney injury, with
hyperkalemia to 5.2 mmol/L, hyperphosphatemia to 4.5 mg/dL, and
hypocalcemia to 7.3 mg/dL. His lactate dehydrogenase (LDH) is elevated to
956 U/L. He is already on aggressive IV hydration and daily allopurinol. A
uric acid is checked and is elevated to 12.2 mg/dL, increased from a
pretreatment value of 5.2 mg/dL 2 days prior. What is the next best step in
managing this patient’s hyperuricemia?
A. Increase allopurinol to BID dosing
B. Start febuxostat
C. Urgent dialysis
D. Rasburicase
Question 23.14 A 72-year-old male presents to the emergency room with
worsening dyspnea. On CT imaging of the chest, he has a 3.2-cm tumor that
exhibits extrinsic mass effect and results in near-complete narrowing of the
SVC. He also has prominent collateral venous structures within the chest as
well as right-sided pathologic hilar adenopathy. He otherwise has subjective
right-sided facial fullness with no hoarseness, stridor, nor dysphagia. On
examination, he has no venous distention of the neck or chest wall and no
facial edema. What is the best next step in the management of this patient?
A. Chemotherapy for rapid tumor debulking
B. Radiation therapy to the right-sided lung mass
C. Endovascular stenting and angioplasty
D. Obtain biopsy to establish tissue diagnosis
Question 23.15 A 27-year-old female without significant past medical
history presents with 3 weeks of progressive neck swelling, hoarseness, and
dysphagia. She has also recently experienced night sweats, fatigue, and
unintentional weight loss of 10 lb. Imaging by chest X-ray reveals superior
mediastinal widening and pleural effusion. Which of the following should be

https://t.me/ALGRAWANY33
avoided until diagnostic work up is complete?
A. Elevating head of bed
B. Thoracentesis
C. Corticosteroids
D. Supplemental oxygen
Question 23.16 A 47-year-old female with newly diagnosed metastatic
high-grade neuroendocrine carcinoma presents to the emergency room with
worsening anemia. On examination, she appears ill and is disoriented.
Laboratory values include a hemoglobin of 6.5 g/dL, platelet count of 55K,
an LDH of 885 U/L, and an undetectable haptoglobin. Coombs testing is
negative, and a peripheral smear reveals four to six schistocytes per high-
powered field. Liver function studies are notable for an indirect
hyperbilirubinemia and she has normal renal function. ADAMTS13 and
C3/C4 testing are normal. What is the definitive therapy for this patient?
A. Start urgent plasma exchange
B. Start eculizumab, a monoclonal antibody that inhibits complement
C. Start high-dose corticosteroids with a prolonged taper
D. Start therapy directed at underlying malignancy
Question 23.17 A 49-year-old male with history of metastatic rectal cancer
presents with urinary incontinence, saddle anesthesia, diminished rectal tone,
and worsening right lumbosacral pain. He has bulky pelvic disease and bone
involvement of the lumbar and sacral vertebrae. What is the most likely
diagnosis based on his presenting history?
A. Malignant cord compression
B. Cauda equina syndrome
C. Brown-Sequard syndrome
D. Unilateral sciatic nerve compression
Question 23.18 A 65-year-old female with a 90-pack-year smoking history
presents to the emergency room with persistent cough and one episode of
mild hemoptysis. Chest radiograph showed a 2.0-cm spiculated mass. Basic
metabolic panel is notable only for sodium of 119 mmol/L. The patient
appears euvolemic, with normal vital signs and no other significant past
medical history. Complete blood count and physical and neurologic
examination are unremarkable. Her urine osmolality and urine sodium are
elevated. In addition to complete staging imaging and referral for biopsy with
interventional radiology, which of the following would you recommend
initially?
A. Discharge home with oral salt administration and close Na+ monitoring
B. Admit to hospital for water restriction, IV or oral salt administration,
and Na+ monitoring
C. Admit to hospital and aggressively diuresis with furosemide
D. Admit to neurologic intensive care unit (ICU) for 100 mL of 3% NaCl
and monitoring
Question 23.19 Which of the following statements is TRUE in regard to
the etiologies of cancer-associated elevated ICP?
A. Despite cancer being a hypercoagulable state, patients are not at an
increased risk for dural venous sinus thrombosis.
B. Increased cerebrospinal fluid (CSF) production is a common cause for
elevated ICP.
C. It is uncommon to see brain metastases in melanoma and lung cancer.
D. Meningeal carcinomatosis can result in impaired CSF reabsorption and
result in normal pressure hydrocephalus (NPH).
Question 23.20 A 55-year-old male with history of limited-stage SCLC
presents to the emergency department with intractable headaches despite
treatment with acetaminophen and ibuprofen over the past 24 hours. He now
has intractable nausea and vomiting. On fundoscopic examination he is noted
to have papilledema. CT brain imaging shows a lesion within his fourth
ventricle, with associated dilatation of the lateral and third ventricles. What is
the definitive treatment for this patient?
A. Neurosurgical consultation for external ventriculostomy
B. Radiation-based therapy to culprit lesion
C. Cisplatin/etoposide for rapid cytoreduction

https://t.me/ALGRAWANY33
D. Interventional radiology embolization

Answers

Question 23.1 The correct answer is A.


Malignant disease is the most common cause of SVC syndrome (SVCS).
Among malignancies causing SVCS, the most common is SCLC,
followed by squamous cell lung cancer and non-Hodgkin lymphoma in
most series due to their commonly central location. DLBCL and
lymphoblastic lymphoma are the most common among lymphomas that
cause SVCS. Hodgkin lymphoma commonly involves the mediastinum,
but rarely causes SVCS. Breast cancer is the most common metastatic
cancer that causes SVCS. Nonmalignant conditions causing SVCS are
becoming increasingly common because of the use of vascular devices,
which can result in SVC thrombosis.1
Question 23.2 The correct answer is C.
Chemotherapy alone or in combination with thoracic irradiation therapy
is the standard treatment for SCLC and is effective in rapidly improving
the symptoms of SVCS. No significant difference in response rates to
chemotherapy or radiation has been detected in most studies. Relief of
SVCS typically occurs within 7 to 10 days after initiation of therapy.
Surgery is not an appropriate option for this patient with extensive-stage
SCLC. An SVC stent may be used in case of lack of response or
relapsed obstruction after initial therapy. Although radiation could be
used for immediate symptomatic relief, combination chemotherapy is
also very effective and preferable as initial therapy, avoiding the large
radiation field and also addressing the metastatic lesions.
Question 23.3 The correct answer is D.
The SVC extends from the junction of the right and left innominate
veins to the right atrium and is completely encircled by lymph node
chains draining from the right thoracic cavity and lower part of the left
thorax. Approximately 80% of the tumors causing SVCS are located in
the right lung. In the absence of mediastinal lymph node enlargement,
the most likely cause of the SVC is direct extension from the centrally
located right-sided tumor.
Question 23.4 The correct answer is A.
Absence of venous pulsations within the center of the optic disk is a
common early finding associated with increased ICP. Papilledema with
blurring of the optic disk margins or small hemorrhages occur at later
time points. Foster-Kennedy syndrome, optic nerve atrophy as a result
of sphenoid wing meningioma and contralateral papilledema from
increased ICP, is a rarely seen late event.2
Question 23.5 The correct answer is B.
Although the majority of patients with malignant spinal cord
compression (MSCC) have a documented history of malignancy,
approximately 20% of patients present with this complication at the time
of initial diagnosis. In fact, only 1 in 500 patients with MSCC have this
complication as part of the initial oncologic presentation.
The most common types of cancer associated with MSCC include
breast cancer, prostate cancer, lung cancer, and lymphoma. The
cumulative incidence of MSCC is disease-specific and highest in
multiple myeloma (8%), prostate cancer (7%), nasopharyngeal cancer
(6.5%), and breast cancer (5.5%). Although breast cancer is a common
cause of MSCC, this manifestation rarely occurs at presentation. Rather,
breast cancer–associated MSCC is more common due to late axial bone
metastases late in the disease trajectory. Colon cancer is a less common
cause of MSCC. MSCC as the primary manifestation of malignancy is
most common seen in non-Hodgkin lymphoma, myeloma, and lung
cancer.
Question 23.6 The correct answer is C.
Headache is the most common complaint of patients with ICP.
Classically, pain is severe and resistant to common analgesics. Pain
typically is most severe upon waking in the morning, most likely due to

https://t.me/ALGRAWANY33
decreased venous drainage in the supine position while sleeping. Despite
these classically associated symptoms of ICP, the majority of patients
have nonspecific tension-type or migraine-like headaches. Interestingly,
patients frequently report immediate relief from their headache
following vomiting. In patients with chronic disturbance of spinal fluid
reabsorption such as NPH, a characteristic clinical syndrome includes
cognitive decline, urinary incontinence, and gait apraxia. Although this
is a relatively uncommon neurologic condition, it is not considered an
oncologic emergency and is not associated with increased ICP.
Question 23.7 The correct answer is B.
Rasburicase is a recombinant urate oxidase and not an inhibitor of this
enzyme. Urate oxidase is an enzyme that catalyzes the conversion of
uric acid to allantoin, which is a more soluble catabolite. This enzyme is
present in most mammals but not in humans.
Rasburicase should be avoided in patients with G6PD deficiency
because it may cause hemolysis as a result of the increase in the
byproduct hydrogen peroxidase. This deficiency is more common in
those of African or Mediterranean ancestry. Although the incidence of
laboratory tumor lysis syndrome (TLS) is significantly lower with the
use of rasburicase in adult patients, this has not consistently been shown
for the incidence of clinical TLS. Other than laboratory parameters,
outcomes were not significantly different between rasburicase and
allopurinol in adults. Importantly, a course of rasburicase is about
15,000 times more expensive than a course of oral allopurinol, and 15 to
20 times more expensive than a course of intravenous allopurinol.
Therefore, cost must be factored into decision-making.3
Question 23.8 The correct answer is B.
Despite vigorous hydration, the most likely cause of death soon after
initiating chemotherapy is TLS. Many patients with TLS develop life-
threatening complications, and a quarter of patients die during
hospitalization.4 Among the metabolic complications of TLS,
hyperkalemia poses the greatest immediate threat and represents the
most important cause of early mortality. Although release of potassium
from dying cells is the principal cause of hyperkalemia, falling
adenosine triphosphate (ATP) levels before cell lysis may lead to
potassium leakage. This may explain the initial rise in serum potassium
seen in laboratory TLS. Leukostasis, pulmonary embolism, and cardiac
arrhythmia are possible causes that are far less likely in this patient.
Interestingly, higher mortality is reported in patients with solid tumors
rather than hematologic malignancies, likely a consequence of less
prophylaxis and reduced awareness.
Question 23.9 The correct answer is B.
The Kocher–Cushing reflex (or Cushing reflex) is characterized by
changes in breathing pattern, arterial hypertension, and bradycardia.
Despite being the most common symptom in patients with increased
ICP, headaches are not a component of this eponymous triad.
Question 23.10 The correct answer is C.
This patient has symptomatic hypercalcemia (nausea, vomiting,
constipation, disorientation) and requires prompt reduction in the
calcium levels. Hypercalcemia is the most common paraneoplastic
syndrome, occurring in up to a third of patients with advanced cancer.
Together with hydration, bisphosphonates are the cornerstone of therapy
for malignancy-associated hypercalcemia. The bisphosphonate
zoledronate is commonly used and produces normalization of serum
calcium in 4 to 10 days, with therapeutic duration of 4 to 6 weeks in the
majority of patients. Calcitonin can cause rapid decline in serum calcium
but is short-acting. Due to risk of tachyphylaxis, calcitonin should not be
continued for longer than 72 hours.3 Calcitonin is appropriate to add to
IV hydration and bisphosphonate in the acute setting, given the need for
rapid decrease in serum calcium due to symptomatic hypercalcemia.
After adequate hydration, 20 to 40 mg of IV furosemide can enhance
calcium excretion. However, furosemide is not part of initial
management of hypercalcemia. Bisphosphonates inhibit both normal
and pathologic bone resorption via direct and indirect effects on
osteoclasts. Because the patient is symptomatic and the effect of
bisphosphonates may take a few days, he will require an additional

https://t.me/ALGRAWANY33
short-acting agent.
Question 23.11 The correct answer is C.
The patient’s history is concerning for MSCC given acute onset of
symptomatology and accompanying neurologic deficits. This diagnosis
is confirmed with MRI. Initial management of MSCC includes
corticosteroids to reduce vasogenic cord edema and facilitate pain
management; however, long-term benefit in terms of survival is limited.5
In carefully selected patients, surgery results in improved outcomes
compared to radiation-based therapies.6 Indications for surgical
decompression include radio-insensitive tumors, involvement of a
previously radiated segment, painful radiculopathy despite radiation,
cord compression resulting from a pathologic fracture, rapid neurologic
deterioration, and spinal instability. Surgical decompression is palliative
and can relieve paraplegia and pain for a period of time.7 Given the
acute onset of symptoms, rapid decline, and a clear pathologic fracture
resulting in cord compression, surgery is his best option.
Question 23.12 The correct answer is B.
This patient presents with a history concerning for MSCC given her
known diagnosis of metastatic lung cancer and worsening unexplained
back pain. Often, pain from the epidural mass is worsened by maneuvers
that increase the intrathoracic pressure, including sneezing and
coughing, and is worse when in the recumbent position. MRI for this
patient shows multilevel vertebral body disease, with a focal area of
epidural extension and cord compromise. Given the multifocal nature,
without rapid neurologic compromise and absence of a pathologic
fracture, radiation-based therapy would be preferred in this context. The
efficacy of radiation therapy depends on the primary tumor, with
lymphoma, myeloma, and seminoma regarded as highly responsive,
whereas NSCLC, renal, thyroid, gastrointestinal cancers, sarcomas, and
melanomas are less sensitive.7 Options for radiotherapy include
conventional external beam radiation therapy, which is the most
common modality, versus stereotactic body radiotherapy. Although
corticosteroids are considered frontline therapy for MSCC, the benefits
of transient and the treatment requires additional measures such as local
or systemic therapy.
Question 23.13 The correct answer is D.
This patient meets criteria for TLS based on his electrolyte abnormalities
(hyperkalemia, hyperphosphatemia, hypocalcemia) in combination with
worsening renal function.8 This syndrome is most often seen following
cytotoxic chemotherapy in hematologic malignancies. Incidence of TLS
has been reported as high as 42% for patients with high-grade non-
Hodgkin lymphoma.9 Rapid cell turnover and cell death lead to release
of potassium, phosphate, and uric acid, causing hyperkalemia,
hyperphosphatemia, hyperuricemia, and secondary hypocalcemia. Uric
acid nephropathy occurs when high levels of uric acid deposit in renal
tubules, resulting in obstruction and a reduction in glomerular filtration.
For prophylaxis, allopurinol is typically started 1 to 2 days prior to
induction for adult patients at high risk of TLS. Treatment duration is
dependent on ongoing risk.3 If uric acid becomes elevated despite
prophylaxis, it is essential to prevent renal damage by lowering levels.
This is best accomplished by rasburicase, a recombinant urate oxidase,
which has been shown to be efficacious within hours of administration.
Although increasing allopurinol may provide some additional benefit,
effects are often delayed 48 to 72 hours. If a patient has significant renal
impairment or treatment-refractory electrolyte abnormalities, dialysis
may be required, but not supported by the laboratory values presented.
There is no role for febuxostat in the management of TLS.
Question 23.14 The correct answer is D.
This patient has radiographic findings of extrinsic SVC compression,
but clinically does not have findings concerning for SVCS. The presence
of collateral venous structures within the chest suggests this lesion has
gradually worsened with time. Given the clinical stability of the patient,
it is important to establish a pathologic diagnosis, because many of the
treatments in SVCS are disease-specific. Options for biopsy include
bronchoscopy, mediastinoscopy, or CT-guided biopsy, and there is no
evidence to suggest these procedures are unsafe in patient with SVCS.10

https://t.me/ALGRAWANY33
The decision to use radiation or chemotherapy depends on tissue
histology. Chemotherapy alone or in combination with radiation is the
treatment of choice for SCLC.11 In NSCLC, radiation has a slightly
better efficacy in relieving the obstruction compared to chemotherapy.12
Invasive procedures such as endovascular stenting or angioplasty are
often reserved for patients who are clinically deteriorating and need
immediate relief of the obstruction.
Question 23.15 The correct answer is C.
Lymphoma involving the mediastinum may cause SVCS, most
commonly primary mediastinal B-cell lymphoma or lymphoblastic
lymphoma. The presence of dysphagia, hoarseness, or stridor are
adverse prognostic factors for patients with lymphoma presenting with
SVCS. Other primary mediastinal malignancies that may cause SVCS
include thymomas and germ cell tumors. Hodgkin lymphoma commonly
involves the mediastinum, but it rarely causes SVCS. During the
diagnostic process, the patient can benefit from bed rest and oxygen
administration. Positioning may also help relieve symptoms for some
patients. Some clinicians advocate the use of diuretics or corticosteroids
if the patient is uncomfortably symptomatic. Ideally, the use
corticosteroids should be delayed until after pathologic confirmation of
the cause of SVCS, as they may hamper efforts for an accurate
diagnosis, especially with lymphomas.
Question 23.16 The correct answer is D.
The patient has a thrombotic microangiopathy, characterized by anemia
and thrombocytopenia and supported by evidence of hemolysis,
including elevated LDH and low haptoglobin. Intravascular hemolysis is
confirmed by evidence of schistocytes on peripheral blood smear. This
is most likely a manifestation of the patient’s underlying malignancy,
that at present is presumably advanced and untreated.13 Negative
Coombs testing makes an autoimmune hemolytic anemia less likely.
Normal ADAMTS13 and complement levels make thrombotic
thrombocytopenic purpura (TTP) and atypical HUS (aHUS) less likely.
Plasma exchange is the definitive treatment for TTP and eculizumab, a
complement inhibitor, and is the treatment for aHUS. Corticosteroids are
indicated for autoimmune hemolytic anemia and would provide no
therapeutic benefit to this patient.
Question 23.17 The correct answer is B.
Cauda equina syndrome is characterized by an asymmetric painful
lumbosacral polyradiculopathy, a patchy sensory deficit corresponding
to multiple lumbar and sacral nerve roots, and bladder and bowel
incontinence. In a patient with cancer, this syndrome raises suspicion for
leptomeningeal carcinomatosis. The presence of signs and symptoms
referable to intracranial disease such as headache and asymmetric
cranial neuropathies facilitates the diagnosis of leptomeningeal
involvement. Pain is the most common presenting symptom in patients
with MSCC. Neurologic symptoms such as lower extremity weakness
and paraplegia typically evolve within weeks to months of the onset of
back pain. Few patients experience diminished sensation below the level
of compression at presentation. Symptoms of neurogenic bladder
dysfunction such as nocturia, hesitancy, and urinary retention may also
present. A classic Brown-Sequard syndrome characterized by leg
weakness and loss of proprioception on the side of cord infiltration and
loss of pain and temperature sensation on the opposite side is rarely
seen, but incomplete variants exist. Infiltration of the sciatic nerve must
be distinguished from malignant epidural compression of a root or the
cauda equina. With unilateral involvement, bladder and bowel
symptoms are absent.
Question 23.18 The correct answer is B.
In patients with cancer, hyponatremia secondary to inappropriate
secretion of antidiuretic hormone (ADH) (or syndrome of inappropriate
antidiuretic hormone [SIADH]) occurs as a paraneoplastic syndrome, or
as a complication of therapy. SIADH is often associated with solid
tumors of the lung, breast, head, and neck.3 SIADH is defined as a hypo-
osmolar or dilutional hyponatremia with excessive natriuresis. This
patient presents with asymptomatic, euvolemic hyponatremia, which
likely developed over weeks to months, rather than hours to days in the

https://t.me/ALGRAWANY33
setting of SIADH. The rapidity of correction is guided by clinical
presentation and the pace with which the hyponatremia developed.
Given this, her sodium correction should proceed over several days.
Ideally, she would be managed with judicious water restriction and IV
or oral salt administration. Sodium monitoring should be performed
regularly. Given she is at normal risk of osmotic demyelination
syndrome due to serum sodium concentration >105 mmol/L, no history
of alcoholism, advanced liver disease, hypokalemia, or malnutrition, the
goal of sodium correction is 4 to 8 mmol/L/day. For the initial treatment
of hyponatremia in asymptomatic patients with serum sodium ≦125,
which likely developed over weeks, when patient is euvolemic, 20 to 40
mg of furosemide may be considered along with intravenous normal
saline. Diuresis alone would not be an appropriate next step in
management. Given the need for frequent sodium monitoring, outpatient
management is not appropriate. Furthermore, ICU admission for
hypertonic saline is not required because the patient is not symptomatic,
serum sodium >115 mmol/L, and hyponatremia did not appear to have
developed acutely.
Question 23.19 The correct answer is D.
In patients with advanced cancer, leptomeningeal carcinomatosis can
directly interfere with CSF absorption at the subarachnoid granulations.
Under these conditions, the spinal fluid reaches a new equilibrium and
gives rise to a condition called normal pressure hydrocephalus (NPH),
with classic radiographic findings, including ventricular enlargement out
of proportion to age-related cortical atrophy. These patients often have a
triad of clinical findings, including cognitive decline, micturition, and
gait apraxia. Patients with cancer are hypercoagulable and are at
increased risk for dural venous sinus thrombosis, causing venous
outflow obstruction, and “slit-like” ventricles. Increased CSF production
is a rare cause of increased ICP. Lung cancer and melanoma are
particularly prone to seed the CNS.14
Question 23.20 The correct answer is A.
The patient’s clinical presentation is concerning for elevated ICP, likely
due to the underlying SCLC. He has acute obstructive hydrocephalus,
which constitutes a neurosurgical emergency.2 In patients with
significant symptoms or signs of herniation, placement of an external
ventriculostomy will provide immediate relief. Long-term options
additionally include placement of a ventriculoperitoneal shunt or
placement of a third ventriculostomy. The former is often avoided in the
setting of leptomeningeal disease to avoid peritoneal seeding. Both
radiation-based therapies and chemotherapy play an important role in
the management of SCLC, including those with cerebral brain
metastases. Neither, however, would provide the immediate relief
needed for this patient. There is no role for embolization-based
procedures in acute obstructive hydrocephalus.

References
1. Higdon ML, Atkinson CJ, Lawrence KV. Oncologic emergencies: recognition and initial
management. Am Fam Physician. 2018;97:741–748.
2. Esquenazi Y, Lo VP, Lee K. Critical care management of cerebral edema in brain tumors. J
Intensive Care Med. 2017;32:15–24.
3. Pi J, Kang Y, Smith M, Earl M, Norigian Z, McBride A. A review in the treatment of oncologic
emergencies. J Oncol Pharm Pract. 2016;22:625–638.
4. Durani U, Shah ND, Go RS. In-hospital outcomes of tumor lysis syndrome: a population-based
study using the national inpatient sample. Oncologist. 2017;22:1506–1509.
5. Al-Qurainy R, Collis E. Metastatic spinal cord compression: diagnosis and management. BMJ.
2016;353:i2539.
6. Patchell RA, Tibbs PA, Regine WF, et al. Direct decompressive surgical resection in the treatment
of spinal cord compression caused by metastatic cancer: a randomised trial. Lancet. 2005;366:643–
648.
7. Ropper AE, Ropper AH. Acute spinal cord compression. N Engl J Med. 2017;376:1358–1369.
8. Cairo MS, Bishop M. Tumour lysis syndrome: new therapeutic strategies and classification. Br J
Haematol. 2004;127:3–11.
9. Hande KR, Garrow GC. Acute tumor lysis syndrome in patients with high-grade non-Hodgkin’s
lymphoma. Am J Med. 1993;94:133–139.
10. Wilson LD, Detterbeck FC, Yahalom J. Clinical practice. Superior vena cava syndrome with
malignant causes. N Engl J Med. 2007;356:1862–1869.
11. Wurschmidt F, Bunemann H, Heilmann HP. Small cell lung cancer with and without superior vena
cava syndrome: a multivariate analysis of prognostic factors in 408 cases. Int J Radiat Oncol Biol
Phys. 1995;33:77–82.
12. Rowell NP, Gleeson FV. Steroids, radiotherapy, chemotherapy and stents for superior vena caval
obstruction in carcinoma of the bronchus: a systematic review. Clin Oncol (R Coll Radiol).
2002;14:338–351.

https://t.me/ALGRAWANY33
Gordon LI, Kwaan HC. Cancer-and drug-associated thrombotic thrombocytopenic purpura and
13. hemolytic uremic syndrome. Semin Hematol. 1997;34:140–147.
14. Lassman AB, DeAngelis LM. Brain metastases. Neurol Clin. 2003;21:1–23, vii.
___________
Corresponding chapters in DeVita, Hellman, and Rosenberg’s Cancer: Principles & Practice of
Oncology, Eleventh Edition: 112 (Superior Vena Cava Syndrome), 113 (Increased Intracranial
Pressure), 114 (Spinal Cord Compression), and 115 (Metabolic Emergencies).
INDEX

A
Abemaciclib, 15, 19
ACA. See Adrenocortical adenoma (ACA)
ACC (adrenocortical cancer), 157, 162, 163
Acetaminophen, 16, 20
for cancer pain management, 239, 245–246
ACT II study, 116
Actinic keratosis (AKs) lesions, 137, 145
Acute emesis, neurotransmitters for, 249, 253
Acute myeloid leukemias (AMLs), 65, 69
allogeneic transplantation
indication for performing, 183, 193
outcomes after, 183, 193
CKIT gene mutation in, 177, 187
complication of induction therapy in, 177, 188
consolidation therapy in, 179, 190
core-binding factor, 176, 187
cytopathology, 176, 187
diagnosis of, 180, 185, 192, 196
DNMT3A gene mutation in, 176, 187
in elderly, treatment of, 178, 188, 189
evolving from myelodysplastic syndrome, 188
favorable- and intermediate-risk AML, drugs used in, 181, 192
FLT3-mutated AML, 181–182, 192, 193
IDH1-mutated AML, agents for treatment of, 182, 193
IDH2-mutated relapsed or refractory AML, drugs used in, 182, 193
monosomal karyotype, 177, 188
with myelodysplasia-related changes, 182, 192
normal cytogenetic, 181, 192
nucleophosmin1 mutation in, 181, 191, 192
prognostic factors for, 177, 188
residual blasts, treatment of, 180, 190
stem cell transplantation for, 180, 191
therapy-related, 181, 182, 191, 192
venetoclax and decitabine combination therapy in, 185, 196
Acute obstructive hydrocephalus, 266
Acute promyelocytic leukemia (APL), 178, 189
arsenic salvage therapy in, 178, 189
ATRA and headache, 179, 189

https://t.me/ALGRAWANY33
differentiation syndrome in, 179, 189
fresh-frozen plasma in, 178–179, 189
initiation of therapy with ATRA, 182, 192
relapsed, arsenic trioxide in, 179, 190
risk of relapse in, 179, 190
Adenocarcinoma, 84, 89
distal esophageal, 72, 77
endometrioid, 170, 174
esophageal, 71, 76
gastroesophageal junction, 72, 77
of lung. See Lung adenocarcinoma
pancreatic, 93, 94, 100–101, 102
prostate, 125, 132
small bowel, 85, 89–90
Adrenocortical adenoma (ACA)
lipid-rich, 156–157, 162
metastatic, mitotane treatment for, 157, 163
Adrenocortical cancer (ACC), 157, 162, 163
Adult acute lymphoblastic leukemia (ALL)
B-ALL, treatment for, 183, 194
CNS prophylaxis in, 180, 190
induction therapy in, 180, 190
maintenance therapy for, 180, 191
Philadelphia chromosome–positive, 183, 194
poor prognostic factor in, 177, 188
relapsed/refractory ALL, agents approved for, 183, 194
targeted agents in, 180, 191
Afatinib, 50, 56
AIHA (autoimmune hemolytic anemia), 195
AKs (actinic keratosis) lesions, 137, 145
Alanine aminotransferase (ALT), 20
Alemtuzumab, 180, 191
ALK (anaplastic lymphoma kinase), 18, 22
ALL. See Adult acute lymphoblastic leukemia (ALL)
All-trans retinoic acid (ATRA), 179, 189
Allogeneic hematopoietic stem cell transplant (allo-HSCT), 183, 193
indication for performing, 183, 193
Allopurinol, 257, 262, 264
Alpelisib, 16, 20
side effect of, 65, 69
ALT (alanine aminotransferase), 20
Alternative hypothesis
randomized clinical trials and, 10, 13
sample size calculations, 10, 13
Alveolar rhabdomyosarcoma, 151
Alveolar soft-part sarcoma, 149–150, 154
treatment of, 150, 154
American Academy of Dermatology, 232, 235
American College of Gastroenterology, 234
American College of Radiology, 234
American Gastroenterological Association, the American Society for Gastrointestinal Endoscopy, 234
American Society for Clinical Pathology (ASCP), 234
American Society for Colposcopy and Cervical Pathology (ASCCP), 234
American Society of Clinical Oncology, 242
5-aminolevulinic acid, 26, 31
AMLs. See Acute myeloid leukemias (AMLs)
Anal cancer, early localized, therapy for, 112, 116
Anaplastic gliomas, non-1p/19q-deleted, 26, 32
Anaplastic large cell lymphoma
breast implant–associated, 201, 207
stage IV, 201, 206
Anaplastic lymphoma kinase (ALK), 18, 22
Anaplastic oligodendrogliomas, 26, 32
chemotherapy in, 26, 32
radiation in, 26, 32
Androgen receptor, prostate cancer and, 125, 132
Angiosarcoma, and risk of nodal metastases, 147, 152
Anthracycline, 19
Antiandrogen-withdrawal response, 123, 130
Antiapoptotic protein, 2, 6
Antibody–drug conjugate, 18, 22
APC gene mutation, 115
APL. See Acute promyelocytic leukemia (APL)
Apoptosis, 6
APRIL (a proliferation-inducing ligand), 226
Area postrema, 250, 254
ARROW trial, 225
Arsenic trioxide, 179, 189, 190
ARTIST trial, 80
ASCCP (American Society for Colposcopy and Cervical Pathology), 234
ASCP (American Society for Clinical Pathology), 234
Aspartate transaminase (AST), 20
ASPIRE trial, 225
AST (aspartate transaminase), 20
Astrocytoma, 24, 30
ATRA (all-trans retinoic acid), 179, 189
ATRX mutations, 24, 30
Autoimmune colitis, 135, 140–141, 143
Autoimmune hemolytic anemia (AIHA), 195
Autologous stem cell transplantation, 212, 220
AZA-001 trial, 197

B
B-cell lymphomas, 201, 207
B-cell maturation antigen (BCMA), 216, 226

https://t.me/ALGRAWANY33
-targeted therapies, 216, 226
Bacillus Calmette–Guérin (BCG) vaccine, intravesical, 121, 127
Barcelona Clinic Liver Cancer scoring system, 95, 104
Barrett esophagus, 71, 76
high-grade dysplasia in, 72, 77
Basal cell carcinomas (BCCs)
advanced and metastatic, treatment of, 137, 144
PTCH1 gene, inactivation of, 137, 144
BCG (Bacillus Calmette–Guérin) vaccine, intravesical, 121, 127
BCL-2, 220–221
BCMA. See B-cell maturation antigen (BCMA)
BCS. See Breast-conserving therapy (BCS)
BELLINI trial, 221
Benign CNS tumor, 25, 31
Bevacizumab, 26, 27, 32, 33
for bilateral vestibular schwannoma, 28, 34
Bevacizumab-IRDye 800CW, 26, 31
Bexarotene, 201, 208
Bicalutamide, 45
Bilateral vestibular schwannoma, 28, 34
targeted therapy for, 28, 34
Biliary cancers, adjuvant therapy for, 97, 105
Binimetinib, 113, 117–118
Birt–Hogg–Dubé syndrome, 127
Bisphosphonate, 258, 263
for cancer pain management, 242
pain associated with bone metastases, 64, 69
Bladder cancer
cisplatin-based multiagent chemotherapy in, 121, 128
combined radiation and chemotherapy in, 122, 128
intravesical therapy in, 121, 127
muscle-invasive, 121, 128
Schistosoma haematobium in, 124, 130
tumor suppressor genes in pathogenesis of, 121, 127
urinary diversion after cystectomy, 122, 128
Blinatumomab, 183, 194
“Blinding” in randomized clinical trials, 9, 12
Blood pressure monitoring, 18, 22
Body mass index (BMI), 76
Bone metastases, 64, 69
Bone pain
alleviating cancer-associated, 236, 242
granulocyte-colony stimulating factor and, 239–240, 246–247
Bortezomib, 17, 21, 208, 215, 224–225
BRAF mutation, 3, 7, 24, 30
Brain metastases, 28, 34
Brain tumors
benign or low-grade, 26, 31
proton therapy in, 26, 31
Breast cancer
ACS guidelines for MRI screening of, 61, 66
adjuvant tamoxifen, and, 63, 68
annual mammography and MRI at age, 231, 235
approval for immunotherapy in, 65, 69
BRCA1-associated, 60, 66
breast-conserving surgery, contraindication to, 62, 68
chemotherapy regimen in, 250, 253
core needle biopsy for diagnosis of, 61, 67
Cowden syndrome and, 61, 66
cytogenetic changes, 178, 189
ductal carcinoma in situ, 62, 67
frontline therapy for hormone-positive metastatic, 64, 69
Gail model, 62, 67
by germline mutations, 60, 66
HER2-positive, preoperative chemotherapy for, 63, 68
inflammatory, 64, 69
Li–Fraumeni syndrome and, 60, 66
local recurrences after breast-conserving therapy, 62, 68
luminal A subtype, 60, 66
male, 64, 69
metastatic, 19
oncotype DX testing and endocrine therapy in, 61, 66
oophorectomy before menopause and risk of, 61, 67
pain management in, 237, 238, 243, 244–245
risk factors in, 61, 66, 230, 233
routine surveillance for early stage cancer patients, 64, 68
screening test for, 232, 235
staging, 62, 67
tamoxifen and raloxifene, contraindications to use of, 61, 67
taxanes, use of, 63, 68
therapy-related MDS, 185, 196
triple-negative, 16, 20
Breast-conserving therapy (BCS)
contraindication to, 62, 68
local recurrences after, 62, 68
Brown-Sequard syndrome, 265
Buprenorphine, for pain management, 247
Burkitt lymphoma, 86, 91, 199, 206
t(8;14) translocation in, 199, 206

C
CA-125, and ovarian cancer, 168, 172
CA 19-9, tumor marker, 100
Calcitonin, 236, 242, 258, 263
Cancer. See also specific cancers

https://t.me/ALGRAWANY33
associated elevated increased intracranial pressure, 260, 265–266
circulating tumor DNA, 2, 4
“driver” mutation, 1, 4
targeted therapy in, 2, 5
Cancer Genome Atlas, 3, 7, 77
Cancer of the Liver Italian Program system, 95, 104
CAR (chimeric antigen receptor) T-cell therapy, 18, 21
Carboplatin, 16
Carcinoids, 84, 89
Carcinoid syndrome, 158, 164
Carfilzomib, 213, 215, 221
Carney triad, associated with GIST, 84, 88
CASPIAN trial, 57
CASSIOPEIA trial, 222
CASTOR trial, 224
Castrate-resistant prostate cancer (CRPC), 125, 131
Cauda equina syndrome, 259, 265
CBT (cognitive behavioral therapy), 241, 248
CD117 (c-Kit) protein, 147, 152
CDH1 gene, mutation in, 74, 79
CDK inhibitors. See Cyclin dependent kinase (CDK) 4/6 inhibitors
CEBPA gene, 187
Cemiplimab-rwlc, 137, 145
Central nervous system (CNS) prophylaxis, 180, 190
Cervical cancer
approach to screening for, 231, 234
early-stage IB, radical hysterectomy in, 170, 175
locally advanced, 171, 175
screening guidelines for, 231, 234
stage IB1, radical trachelectomy in, 171, 175
treatment with pembrolizumab, 171, 175
Cervical squamous cell and adenocarcinomas, 171, 175
Cetuximab, 113, 117–118
for colorectal cancer, 112, 117
in head and neck SCC, 38, 43
CheckMate 204 study, 142
CheckMate 238 trial, 142
Chemoreceptor trigger zone, 250, 254
Chemotherapy-induced nausea and vomiting (CINV), 250, 253–254
dronabinol associated with cannabinoid receptors in, 250, 254
5-HT3 receptors and, 249, 253
risk factors for, 250–251, 254
Chemotherapy-induced peripheral neuropathy (CIPN), 243
Child–Pugh A cirrhosis, 107
Child–Pugh B cirrhosis, 107
Child–Pugh C cirrhosis, 107
and unresectable HCC, 98, 107
Chimeric antigen receptor (CAR) T-cell therapy, 18, 21
cytokine release syndrome and, 18, 21
cHL (classic Hodgkin lymphoma), 204, 210
Cholangiocarcinoma
hilar, 96, 104
risk of developing, 96, 104
Chondrosarcomas, 149, 153
Chromoplexy, defined, 2, 5
Chromothripsis, defined, 2, 5
Chronic corticosteroid therapy, for pain management, 238, 245
Chronic leukocytic leukemia (CLL)
characteristic immunophenotype of, 184, 194
complication of, 184, 195
ibrutinib-based therapy for, 184, 195
prognosis in, 184, 194
symptomatic, 184, 195
Chronic myeloid leukemia (CML), 185, 196
CINV. See Chemotherapy-induced nausea and vomiting (CINV)
CIPN (chemotherapy-induced peripheral neuropathy), 243
Circulating tumor cells (CTCs), 3, 7
Circulating tumor DNA (ctDNA), 2, 4
as cancer stage increases, 3, 7
Cirrhotic liver, 106
Cisplatin, 19, 37, 41
Classic Hodgkin lymphoma (cHL), 204, 210
Clinical trials. See also specific trials
intention-to-treat analysis of clinical trial, 11, 13
of investigational drugs, 11, 14
phase I trials, 8, 12
preclinical trial, 8, 12
CLL. See Chronic leukocytic leukemia (CLL)
CML (chronic myeloid leukemia), 185, 196
Cognitive behavioral therapy (CBT), 241, 248
Cold compress, 16, 19
Colitis
autoimmune, 135, 140–141, 143
grade 2, 251, 255
Colon cancer
methylnaltrexone in, 241, 247–248
pain management in, 239, 240, 246, 247
Colony-stimulating factor-1 (CSF-1), 6
Colorectal cancer (CRC). See also Hereditary nonpolyposis colorectal cancer (HNPCC)
advanced, 15, 19
BRAF V600E in, 113, 117–118
FOLFOX and bevacizumab, first-line therapy for, 113, 117
guidelines for screening, 231, 234
high-risk features for stage II, 113, 117
maintenance therapy with capecitabine and bevacizumab, 113, 117
microsatellite instability and, 109–110, 114

https://t.me/ALGRAWANY33
pain management in, 237, 243
resistant to treatment with cetuximab, 112, 117
stage III, adjuvant therapy for, 110, 115
COMBI-d phase III study, 143
Common Terminology Criteria for Adverse Events (CTCAE), 141
Confounding factor, 10, 13
Conjunctivitis, as adverse effect, 17, 20–21
CONKO-001 study, 101
CONKO-003 study, 103
Core-binding factor acute myeloid leukemias, 176, 187
Cowden syndrome, 84, 88
and risk of breast cancer, 61, 66
CPX-351, 182, 192
CRITICS trial, 80
Crizotinib, 56
for non-small-cell lung cancer, 47, 54
CROSS trial, 78
CRPC (castrate-resistant prostate cancer), 125, 131
CRS (cytokine release syndrome), 18, 21
CTCAE (Common Terminology Criteria for Adverse Events), 141
CTCs (circulating tumor cells), 3, 7
ctDNA. See Circulating tumor DNA (ctDNA)
Cyclin dependent kinase (CDK) 4/6 inhibitors
adverse effects of, 15, 19
for metastatic breast cancer, 19
Cysteine cathepsin proteases, 6
Cytarabine, 17, 20–21
Cytokine release syndrome (CRS), 18, 21
Cytokines, tumor-derived, 3, 6
Cytotoxic T-lymphocyte–associated protein 4 (CTLA-4), 141

D
Dabrafenib, 28, 34
Daratumumab, 215, 224–225
DCIS (ductal carcinoma in situ), 62, 67
De-ESCALaTE trial, 41–42
Denosumab, 216, 226–227
Deoxyribonucleic acid (DNA)
circulating tumor. See Circulating tumor DNA (ctDNA)
variant, 1, 4
Desmoid tumors, 109, 114
unresectable, targeted therapy with sorafenib for, 109, 114
Dexamethasone, 16, 19, 20, 213, 215, 222, 224–226, 238, 245, 251–252, 255
in ALL induction therapy, 180, 190
Dexrazoxane, 16, 19
Diarrhea, 15, 19, 251, 255
checkpoint-blocking antibodies and, 251, 255
incidence of, 251, 255
octreotide and, 251, 254
Differentiation syndrome, 179, 189
agents associated with, 182, 193
Diffuse alveolar hemorrhage, induction therapy in AML and, 177, 188
Diffuse large B-cell lymphoma (DLBCL), 199, 206, 258, 261
advanced stage, 203, 208–209
germinal center B-cell, 202, 208
R-CHOP chemotherapy for, 203, 208–209
therapy for relapsed, 204, 209
Dihydropyrimidine dehydrogenase (DPD), 111, 115–116
Diphenhydramine, 16, 20, 238, 244
Distal esophageal adenocarcinoma, 72, 77
Distal femur, permeative bone tumor in, 149, 153
Diuretics, for fluid retention, 19
DLBCL. See Diffuse large B-cell lymphoma (DLBCL)
DNA. See Deoxyribonucleic acid (DNA)
DNA mismatch repair (MMR) gene, germline mutations in, 109–110, 114
genetic counseling and testing of at-risk family members, 110, 115
DNMT3A gene, 176, 187
Docetaxel, 16, 19
Doxorubicin, 18, 22
DPD (dihydropyrimidine dehydrogenase), 111, 115–116
“Driver” mutation, 1, 4, 181, 191
Dronabinol, 250, 253
“Drop-in” by screening, 232, 235
Ductal carcinoma in situ (DCIS), 62, 67
radiation therapy after lumpectomy in, 62, 67
Duloxetine, 243
Duodenum, tumor in, 84, 89

E
E-cadherin mutation testing, 74, 79
EATL (enteropathy-associated T-cell lymphoma), 85–86, 90
ECHELON-2 trial, 206
EGF (epidermal growth factor), 3, 6
EGFR (epidermal growth factor receptor), 36, 41
EGFR gene, 23, 24, 29, 30
EGFR T790M mutation, 2, 5
Electrocardiogram
before arsenic salvage therapy, 178, 189
for peripheral T-cell lymphoma, 17, 21
Electrolyte imbalance, 257, 262
Electron radiotherapy, 27, 33
ELOQUENT-1 trial, 213, 221
ELOQUENT-3 trial, 224
Elotuzumab, 226

https://t.me/ALGRAWANY33
Empiric seizure prophylaxis, 28, 34
EMR (endoscopic mucosal resection), 72, 77
Enasidenib
differentiation syndrome and, 182, 193
for IDH2-mutated relapsed or refractory AML, 182, 193
Encephalitis, 18, 21
Encorafenib, 113, 117–118
ENDEAVOR trial, 225
Endocrine and neuroendocrine tumors. See specific tumors
Endometrial carcinoma
combination of lenvatinib and pembrolizumab in, 170, 174–175
mismatch repair–proficient metastatic, 170, 174–175
Endometrial hyperplasia, 170, 174
Endometrioid adenocarcinoma, 170, 174
Endometrioid cancer, 170, 174
adjuvant therapy for, 170, 174
stage IB grade 3, 170, 174
Endoscopic mucosal resection (EMR), 72, 77
Enfortumab vedotin, 18, 22
Enteropathy-associated T-cell lymphoma (EATL), 85–86, 90
Entrectinib, 50, 56
EORTC 26053 CATNON trial, 32
EORTC 26062-22061 phase III trial, 32–33
EORTC 40983 study, 117
Ependymoma
adjuvant radiotherapy in, 24, 30
radiation in, 24, 30
standard of care for newly diagnosed, 24, 30
Epidermal growth factor (EGF), 3, 6
Epidermal growth factor receptor (EGFR), 36, 41
Epithelial mesotheliomas, 51, 56
EQUULEUS study, 224
Erlotinib, 15, 19
Esophageal adenocarcinoma, 71, 76
Esophageal cancer
incidence of, 71, 76
perioperative chemotherapy in, 73, 78
European Myeloma Network 02 trial, 223
Everolimus, 155, 161
Ewing sarcoma, 149, 154
and cytogenetic abnormalities, 146, 151
Excisional biopsy, 67
Existential pain, 248
Extracorporeal lithotripsy, 27, 33
Extranodal natural killer (NK)/T-cell lymphoma, 203, 209
Extravasation, vinca alkaloids, 16, 19
EXTREME trial, 42
F
Facial pain scale, 239, 246
Fallopian tube cancer, 18, 22
platinum-based chemotherapy in, 18, 22
Familial paraganglioma, 88
Famotidine, 16, 20, 250, 253–254
Fentanyl, 244
contraindication to use of, 240, 247
Fibrolamellar HCC, 97, 106
Field defect, 23, 29
FISH (fluorescence in situ hybridization), 6, 176, 187, 199, 206
FLAURA trial, 55
FLOT4-AIO trial, 78
FLT3-ITD mutation, 3, 6
Fluid retention syndrome, 19
Fluorescence in situ hybridization (FISH), 6, 176, 187, 199, 206
5-fluorouracil (5-FU), 19
diarrhea and, 251, 255
and leucovorin, 17, 20, 75, 79–80
FOLFIRINOX
in adenocarcinoma of pancreas, 93, 94, 101, 102
modified, 93, 101, 102
FOLFOX (oxaliplatin, fluorouracil, and LV), 250, 253–254
in pancreatic cancer, 102
Follicular lymphoma, 201, 207
Follicular thyroid cancer (FTC), 159, 165
FORTE trial, 221
Fosaprepitant, 249, 253
Foster-Kennedy syndrome, 261
Frameshift mutations, 1, 4
FTC (follicular thyroid cancer), 159, 165
Fulvestrant, 20
in breast cancer, 64, 69
Fumarate hydratase gene, 127
Furosemide, 16
Fusion PET-MRI, 218

G
Gabapentin, 245
Gail model, and breast cancer, 62, 67
Gallbladder cancer
and cholecystectomy, 96–97, 105
T2 NX (stage II), 96, 105
Gastric cancer, 71, 76
combination chemotherapy for, 75, 80
diagnostic modalities in patients with, 74, 79

https://t.me/ALGRAWANY33
early-onset diffuse, 74, 79
early-stage, 72, 76–77
gene mutations in, 74, 79
hereditary diffuse, 74, 78
intestinal, 74, 79
Lynch syndrome–associated, 74, 79
Gastric neuroendocrine neoplasm, 157, 162–163
Gastroesophageal cancer, 72, 77
Gastroesophageal junction (GEJ), 72, 77
Gastroesophageal reflux disease (GERD), 71, 76
Gastrointestinal stromal tumor (GIST), 82, 87, 152
familial and genetic syndromes with, 84
fluorodeoxyglucose-positive metastatic, 82–83, 87
imatinib for, 148, 152
imatinib-resistant, 83, 87–88
regorafenib for, 84, 88
small intestinal, 86, 91
succinate dehydrogenase-deficient, 84, 88
sunitinib therapy for, 83, 88
GBM. See Glioblastoma (GBM)
GCT (giant cell tumor), 149, 153
GEJ (gastroesophageal junction), 72, 77
Gemcitabine, and cisplatin, 98, 107
Gemtuzumab ozogamicin, 179, 180, 190, 191
favorable- and intermediate-risk AML, 181, 192
Genetic counseling and genetic testing, 167, 172
Genomic rearrangements, 23, 29
GERD (gastroesophageal reflux disease), 71, 76
Germ cell tumors of ovary, 169, 174
German Rectal Cancer Study, 116
Germinal center B-cell (GCB) DLBCL, 202, 208
Giant cell tumor (GCT), 149, 153
Gilteritinib, differentiation syndrome and, 182, 193
GIST. See Gastrointestinal stromal tumor (GIST)
Glioblastoma (GBM), 3, 7
alterations in EGFR, RB1, P53 signaling pathways, 25, 30
elderly patients, standard of care for, 27, 32
hypofractionated radiotherapy in, 27, 32
IDH1/2 mutations, 24, 30
as malignant adult CNS cancer, 25, 31
prognostic factor with increased response to alkylating agents in, 24, 30
recurrent, antiangiogenic agent for, 27, 32
recurrent, irradiation in, 27, 32
second-line therapy in, 27, 33
somatic gene alteration in mesenchymal subgroup of, 24, 30
standard of care for elderly patients, 27, 32
tumor-treating fields on, 27, 33
Gliomas. See also specific gliomas
compound enhancing on imaging, 26, 31
low-grade, 26, 32
non-1p/19q-deleted anaplastic, 26, 32
Glucocorticoids, for immune-related adverse events, 17, 21
Goals of care of patient, 241, 247
Grade I meningioma, 28, 33
Graft-versus-host disease (GVHD), 193, 251–252, 255
Granulosa cell tumors, of ovary, 169, 173
GVHD (graft-versus-host disease), 193, 251–252, 255

H
Hairy cell leukemia (HCL), 184–185, 195
Haloperidol, 238, 244
Hashimoto thyroiditis, 85, 89, 160, 166
HCC. See Hepatocellular carcinoma (HCC)
HCL (hairy cell leukemia), 184–185, 195
HCV (hepatitis C virus), 98, 106
HDGC (hereditary diffuse gastric cancer), 74, 78
Head and neck melanomas, 134, 139
Head and neck squamous cell carcinoma (HNSCC)
adjuvant chemotherapy in, 39, 43
cetuximab in, 38, 42
combinations of chemotherapy agents, 37–38, 42
concurrent chemotherapy and radiation therapy in, 38, 42
epidermal growth factor receptor in, 36, 41
HPV-related, 37, 41
immunotherapy for, 39, 43
induction chemotherapy in, 37, 39–40, 42, 44
locally advanced, 39, 44
risk factors for, 36, 41
second primary cancers in, 39, 44
treatment options for, 37, 41–42
Headache, 179, 189
Healthy volunteer effect, 230, 233
Heart rate monitoring, 18, 22
Helicobacter pylori
eradication, 72, 76–77
infection, 71, 76
Hemoglobin A1c (HbA1c), 16, 20
Hemorrhagic cystitis, prevention of, 18, 22
Hepatitis C virus (HCV), 98, 106
Hepatoblastoma, 97, 106
Hepatocellular carcinoma (HCC)
clinical suspicion, 97, 106
fibrolamellar, 97, 106
hypoglycemia and, 95, 104
increased risk of developing, 96, 104

https://t.me/ALGRAWANY33
Milan criteria for selection of patients for liver transplantation, 95, 103
paraneoplastic syndromes with, 95, 104
routine surveillance for, 95, 103
screening and prevention, 96, 104
staging systems for, 95, 104
Hepatotoxicity, 16, 20
HER2 testing, 78, 79
HERACLES trial, 118
Hereditary diffuse gastric cancer (HDGC), 74, 78
Hereditary leiomyomatosis and renal cancer syndrome, 120, 127
Hereditary nonpolyposis colorectal cancer (HNPCC), 79
diagnosis of, 110, 114
MSH2 gene and, 110, 115
Hereditary papillary renal cell carcinoma (HPRC), 119, 126
HevyLite assay, 218
HIF (hypoxia-inducible factor), 120, 127
High-grade dysplasia, in Barrett esophagus, 72, 77
High-intensity focused ultrasound, 27, 33
Hilar cholangiocarcinoma, 96, 105
Hippocampal avoidance during WBRT, 28, 34
HNPCC. See Hereditary nonpolyposis colorectal cancer (HNPCC)
HNSCC. See Head and neck squamous cell carcinoma (HNSCC)
Hodgkin lymphoma, 204, 210, 261, 264
classic, 204, 210
nodular lymphocytepredominant, 204, 210
in pregnancy, 204, 210
Hormonal therapy in ovarian cancer, 168, 173
Hospice care, 98, 107
HPRC (hereditary papillary renal cell carcinoma), 119, 126
HPV. See Human papillomavirus (HPV)
5-HT3 antagonists, 249, 253
Human herpesvirus-8 (HHV8), primary effusion lymphoma and, 200, 207
Human papillomavirus (HPV)
head and neck squamous cell carcinoma, 37, 41
human papillomavirus 16 (HPV16), 171, 175
oropharyngeal carcinoma, 37, 41–42
Hyaluronidase, 16, 19
Hypercalcemia
parathyroid carcinoma and, 157, 163
symptomatic, 258, 263
Hypercholesterolemia, 52, 57
Hypogammaglobulinemia, 195
Hypotension, chimeric antigen receptor T-cell therapy and, 21
Hypothyroidism, as immune-mediated toxicity, 39, 43
Hypoxia-inducible factor (HIF), 120, 127

I
IBC (inflammatory breast cancer), 64, 69
Ibrutinib, 208
IDH1/2 mutations, 24, 30
Ifosfamide, 18, 22
IGCLC (International Gastric Cancer Linkage Consortium), 78
Ileal tumor, 84, 85, 89–90
Imatinib, 82, 83, 87–88, 180, 191
tyrosine kinase inhibitors vs., 185, 195
Immune checkpoint inhibitors, toxicity of, 17, 21
Immune-related adverse events (IRAEs), 17, 21
Immune system deficiency, 3, 6
Immunotherapy
in breast cancer, 65, 69
for head and neck squamous cell carcinoma, 39, 43
in melanomas, 134–135, 140
IMpower133 trial, 57
IMRT (intensity-modulated radiation therapy), 39, 44
Inflammatory breast cancer (IBC), 64, 69
Inherited colorectal cancer syndrome, 109–110, 114
Inotuzumab ozogamicin, 183, 194
Intensity-modulated radiation therapy (IMRT), 39, 44
Intention-to-treat (ITT) analysis of clinical trial, 11, 13
INTERAACT study, 116
Intergroup 0099 trial, 43
Intergroup Trial (INT 0116), 79
Interim analysis, 9, 13
not planning in design stage, 10, 13
purposes of, 9, 12
Intermittent bronchial obstruction, 158, 164
International Gastric Cancer Linkage Consortium (IGCLC), 78
International Myeloma Working Group (IMWG), 218
International Prognostic Index (IPI) score, 200, 206–207
Intestinal lymphomas, primary vs. secondary, 85, 90
Intra-abdominal leiomyosarcomas, 152
Intracranial pressure, increased
cancer-associated elevated, 260, 265–266
headache and, 257, 262
Kocher–Cushing reflex and, 257, 262
signs of, 257, 261, 262
Intravesical therapy, for bladder cancer, 121, 127
IPI (International Prognostic Index) score, 200, 206–207
Ipilimumab, for metastatic melanoma¸135, 142
IRAEs (immune-related adverse events), 17, 21
Irinotecan, 18, 21
and neutropenia, 111, 116
Isocitrate dehydrogenase (IDH) mutation, 23, 29
low-grade gliomas, 26, 32
oligodendrogliomas, 24, 30

https://t.me/ALGRAWANY33
Isolated axillary metastases, 65, 69
ITT (intention-to-treat) analysis of clinical trial, 11, 13
Ivosidenib
differentiation syndrome and, 182, 193
for IDH1-mutated AML, 182, 193

J
JAK2 V617F mutation, 3, 6
Jejunal tumor, 84, 85, 89–90

K
KDM5A overexpression, 24, 30
Kocher–Cushing reflex, 257, 262–263
KRAS-mutation, 3, 7

L
LACE (Lung Adjuvant Cisplatin Evaluation), 51, 56
Large cell lymphoma, 207
Large-scale genomic profiling, 4
Laryngeal squamous cell carcinoma
adjuvant chemotherapy in, 39, 43
concurrent chemotherapy and radiation therapy, 36, 38, 41, 43
LDCT (low-dose computerized tomography), 230, 233
Lead-time bias, 230, 233–234
Leiomyosarcomas
high-grade, 147, 148, 151, 152
intra-abdominal, 147, 152
recurrence, 17, 151
Lenalidomide, 186, 196, 213, 215, 222
Length bias, 230, 233–234
Leptomeningeal carcinomatosis, 260, 265–266
Leucovorin, 17, 19, 20
Leukemia
allogeneic stem cell transplant, 183, 193
graft-versus-leukemia effect, 183, 193
Leuprolide, 45
Lidocaine, vicious, 252, 255
Li–Fraumeni syndrome, 84, 88
and breast cancer, 60, 66
soft-tissue sarcoma and, 146, 151
Liposarcoma, high-grade, 147–148, 152
Liver transplantation, 98, 106
Lorazepam, 237, 243
Lorlatinib, 18, 22, 49, 55
Loss of heterozygosity, 1, 4, 23, 29
on chromosome 17, 24, 30
Low-dose computerized tomography (LDCT), 230, 233
Low-grade gliomas, 26, 32
Lower extremity edema, 143
LPL (lymphoplasmacytic lymphoma), 217, 228
Lumbar puncture, 201, 207
Luminal A breast cancer, 60, 66
Lung adenocarcinoma
acquired alectinib resistance in, 49, 55
dabrafenib and trametinib for, 49, 55
diagnosis of, 47, 54
EGFR T790M gatekeeper mutation in, 47, 54
platinum-based doublet chemotherapy and pembrolizumab for, 48, 54
resistance to erlotinib, 47, 54
stage IV, 48, 54
Lung Adjuvant Cisplatin Evaluation (LACE), 51, 56
Lung cancer, 15, 19
malignant spinal cord compression, 261–262
Lung neuroendocrine neoplasm, 158, 164
Lung squamous carcinoma, 3, 7
Lupus and Hashimoto thyroiditis, 160, 166
Lymph node metastasis, positive pelvic or para-aortic, 168–169, 173
Lymphoblastic lymphoma. 264
Lymphoma, 84, 89. See also specific lymphomas
adult, 200, 206
childhood, 200, 206
International Prognostic Index score for, 200, 206–207
intestinal, primary vs. secondary, 85, 90
involving mediastinum, 259, 264
malignant spinal cord compression, 261–262
Lymphoplasmacytic lymphoma (LPL), 217, 228
Lynch syndrome–associated gastric cancer, 74, 79

M
Magnetic resonance imaging (MRI), 65, 69
MAIA trial, 221
Malabsorption after Whipple surgery, 102
Male breast cancer, 64, 69
Male urethral cancers, risk factor for, 121, 128
Malignant adult primary central nervous system (CNS) tumor, 25, 31
Malignant mesotheliomas
histologic subtype of, 51, 56
immunohistochemistry markers in, 51, 57
indicator of poor prognosis in, 51, 57
Malignant spinal cord compression (MSCC)
management of, 258, 263
primary tumors to, 257, 261–262

https://t.me/ALGRAWANY33
MALT (mucosal-associated lymphoid tissue) lymphoma, 85, 89
Mantle cell lymphoma, 202, 208
MAPS (Mesothelioma Avastin Plus Pemetrexed-cisplatin Study) trial, 57
Maximum tolerated dose (MTD), 8, 12
MCC. See Merkel cell carcinoma (MCC)
MDS. See Myelodysplastic syndrome (MDS)
Mediastinal B-cell lymphoma, 264
Medullary thyroid cancer (MTC), 155, 161
Medulloblastoma
adjuvant chemotherapy, 24, 29
classification of, 23, 29
as malignant CNS cancer in children, 25, 31
platinum-based chemotherapy in, 24, 29
radiotherapy in, 24, 29
reduced survival rate in, 23, 29
surgical debulking in, 24, 29
WNT-activated subgroup of, 23, 29
Melanomas
adjuvant therapy in, 134, 135–136, 139–140, 142
autoimmune colitis and, 135, 140–141
BRAF V600E/K mutation in, 134, 140
cutaneous, 133, 138
KIT gene mutations in, 133, 138
pregnancy and, 133, 138
risk factors for, 133, 138
stage IIIC, 134, 139–140
immunotherapy in, 134–135, 140
intralesional therapy in, 136, 142
malignant, 133, 138–139
malignant cutaneous, 134, 139
metastatic
combination therapy in, 135, 141
extensive intracranial and extracranial, 135, 141–142
RAF-mutated, 136, 143
targeted therapy with dabrafenib and trametinib in, 136, 143
therapy with nivolumab for, 136–137, 143–144
ocular, 137, 144
oncology referral and lymph node dissection, 134, 139
stage IIc, 28, 34
stage IV, 134–135, 140
uveal, 144
Memantine, 28, 34
Memorial Sloan Kettering Cancer Center (MSKCC) nomogram, 151
Meningiomas, as benign CNS tumor, 25, 31
Merkel cell carcinoma (MCC), 137, 144
immune checkpoint inhibitor therapy with pembrolizumab, 137, 144
Mesna, 18, 22
mechanism of action of, 150, 154
Mesothelioma Avastin Cisplatin Pemetrexed Study (MAPS) trial, 57
Mesotheliomas. See also specific mesotheliomas
mutations in BRCA-associated protein 1 tumor suppressor gene in, 51, 57
Meta-analysis, identifying relevant studies in, 10, 13
Metalloproteinases, 6
Metaphase karyotyping, 6
Metastases, 84, 89. See also specific entries
Methadone, 245, 247
Methotrexate, high-dose, 17, 20
Methylene blue, 26, 31
Methylnaltrexone, 241, 247–248
mFOLFOX treatment, for colorectal cancer, 15
MGMT promoter methylation, 24, 27, 30, 33
Microsatellite instability (MSI), 23, 29, 109–110, 114
Midostaurin, 181–182, 192
Missense mutations, 1, 4
Mitotane, 162, 163
MMR gene mutation, 110, 114
Monosomal karyotype, 177, 188
Morbid obesity, 247
Morphine, 243–244
MOSAIC study, 115
MRI (magnetic resonance imaging), 65, 69
MSCC. See Malignant spinal cord compression (MSCC)
MSI (microsatellite instability), 23, 29, 109–110, 114
MSKCC (Memorial Sloan Kettering Cancer Center) nomogram, 151
MTC (medullary thyroid cancer), 155, 161
MTD (maximum tolerated dose), 8, 12
Mucosal-associated lymphoid tissue (MALT) lymphoma, 85, 89
Mucositis
cryotherapy in, 252, 255
prevention of, 251, 255
risk factors for, 252, 255
treatment for, 252, 255
Muir–Torre syndrome, 114
Multi-institutional RTOG 1205 trial, 32
Multifocal myoclonus, 237, 243
Multiple endocrine neoplasia type 1 (MEN1) syndrome, 159, 165–166
Multiple myeloma
antimyeloma therapy for, 211, 218
autologous stem cell transplantation, 212, 220
clinical trial in, 212, 220–221
daratumumab, lenalidomide, and dexamethasone for, 213, 221
defining criteria for, 218
FISH and cytogenetic studies, 212, 219–220
genomic architecture of, 212, 220
minimal residual disease and, 214, 223
osteoclast inhibitors for, 226–227

https://t.me/ALGRAWANY33
prognosis in, 219–220
relapsed/refractory, 215, 225
Revised International Staging System (R-ISS) Stage II, 213–214
risk classification system, 212, 219–220
Mutual exclusivity, 3, 7
MYC amplification, 23, 29
Mycosis fungoides, 203, 209
stage IB, 201, 208
Myeloablative stem cell transplantation in CR1, 180, 191
Myelodysplastic syndrome (MDS)
azacitidine for, 187, 197
erythropoiesis-stimulating agent in, 186, 196
therapy-related, 185, 196
transfusion-dependent anemia in, 186, 196
Myelodysplastic syndrome (MDS), 65, 69, 161, 188
Myeloma-defining event, 218
Myeloma XI trial, 222
Myxoid liposarcoma, 151

N
NAPOLI-1 randomized clinical trial, 103
Naproxen, 245, 246–247
National Cancer Database (NCDB), 89
National Comprehensive Cancer Network (NCCN), 114, 221
National Lung Screening Trial (NLST), 230, 233
Natural killer (NK) cells, 3, 6
NCDB (National Cancer Database), 89
Necrotic cell death, 5
Negative-feedback loops, 2, 5
Neoadjuvant chemotherapy, 122, 128
NET. See Neuroendocrine tumor (NET)
Neuroendocrine tumor (NET), 85, 90, 155, 161
peptide receptor radionuclide therapy in, 155–156, 161
peripheral blood abnormalities in, 156, 161–162
somatostatin receptor imaging in, 156, 162
well-differentiated small nonfunctioning pancreatic, 159, 165
Neurofibromatosis type 1 (NF1), 24, 30, 88
soft-tissue sarcoma and, 146, 151
Neuropathic pain, 243
Neuropathy, incidence of, 17, 21
NF1 mutations, 28, 34
NF2 mutation, 28, 34
Niraparib, 18, 22
Nivolumab, 135, 142
NK (natural killer) cells, 3, 6
NK1 receptor, 249, 253
NLPHL (nodular lymphocytepredominant Hodgkin lymphoma), 204, 210
NLST (National Lung Screening Trial), 230, 233
Nodular lymphocytepredominant Hodgkin lymphoma (NLPHL), 204, 210
Non-1p/19q-deleted anaplastic gliomas, 26, 32
Non-opioid analgesics, 245
Non-small-cell lung cancer (NSCLC)
adjuvant platinum-based chemotherapy in, 49, 55
cisplatin or carboplatin combined with pemetrexed for, 48, 54
EGFR-mutated, 55
locally advanced, 49, 50, 55, 56
metastatic, 16–17
pain management in, 237, 242–243
pemetrexed maintenance therapy in, 48, 54
radiation-based therapy for, 258. 263
recurrent, 16, 19
RET fusion–positive, 16, 20
ROS1-rearranged metastatic, treatment of, 47, 54
ROS1 translocations in, 50, 56
stage T2aN1M0 (IIB), 51, 56
Noninferiority trials, 9, 12
hypothesis testing for, 11, 13
primary objective of, 11, 13
Nonsense mutations, 1, 4
Nonsteroidal anti-inflammatory drugs (NSAIDs), 245–246
Normal pressure hydrocephalus (NPH), 260, 265–266
NS5A protein product of HCV genome, 97, 106
NSABP-C07 study, 115
NSAIDs (nonsteroidal anti-inflammatory drugs), 245–246
NSCLC. See Non-small-cell lung cancer (NSCLC)
Null hypothesis, randomized clinical trials and, 10, 13

O
Octreotide, 251, 254
Okuda system, 95, 104
Olanzapine, 249, 250, 253
Oligodendroglioma
anaplastic, 26, 32
calcification on magnetic resonance imaging, 25, 31
1p/19q codeletion, 24, 26, 30, 32
Oncogenes, 1, 4
Ondansetron, 16, 20
Opioid analgesics
long-term administration of, 238, 244
side effects of, 236, 242
Opioid-induced constipation, 247–248
Oral cavity SCC
stage II (T2N0M0), close monitoring in, 40, 44–45
stage IVA (T4aN1M0), surgery for, 38, 42

https://t.me/ALGRAWANY33
Oropharyngeal carcinoma, HPV-related, 37, 41–42
Osimertinib, 50, 55
Osteosarcoma, 149, 153
treatment of, 146, 151
OTL38, 26, 31
Ovarian cancer, 167, 172
advanced-stage, 169, 173
Ashkenazi Jewish ancestry, 167, 168, 172, 173
BRCA1 and BRCA2 mutations and, 168, 173
CA-125 in, 168, 172
clinical surveillance, 170, 174
elevated CA-125, and hormonal therapy in, 168, 173
germ cell tumors of ovary, 169, 174
goals of care of patient, 241, 247
granulosa cell tumors of ovary, 169, 173
lymph node metastasis, 168–169, 173
nulliparity, risk for, 168, 172
recurrent, platinum-based combination therapy in, 169, 174
screening test for, 230, 233
stage IA grade 1 mucinous tumor, 167–168, 172
tumors of low malignant potential, 168, 173
Ovarian suppression, 63, 68
Overdiagnosis, 231, 233–234
Oxaliplatin, 15, 19
Oxycodone, 246

P
Paclitaxel, 63, 68
Pain management, cancer. See specific entries
Palbociclib, 15, 19
in breast cancer, 64, 69
Palliative care, 237, 242–243
Palonosetron, 249, 253
Pancoast tumor of lung, 50, 56
Pancreatic adenocarcinoma
adjuvant therapy for, 93, 100–101
Whipple resection for, 93, 101
Pancreatic cancer, 92, 95, 99, 103
adjuvant therapy for, 93, 100–101
combination therapy in, 94, 102
factors are associated with an increased risk of, 92, 99
familial, BRCA2 gene mutation in, 94, 102
gemcitabine-based chemotherapy for, 93, 102
KRAS mutation and development of, 92, 100–101
locally advanced, 93, 102
multiphase multidetector helical CT for, 93, 100
nab-paclitaxel and gemcitabine in, combination of, 94, 102
pain management in, 236, 237, 242, 243–244
palliative care for, 240, 247
risk factors for, 94, 102–103
second-line treatment for, 94, 103
stage III, T4N1M0, 92, 101
telomere shortening and, 95, 103
tobacco smoke and, 99
tumor growth factor-beta, underexpression of, 95, 103
Pancreatic enzyme supplementation, 93, 102
Pancreatic grade 1 neoplasm, 157–158, 163
Pancreatic intraepithelial neoplasms (PanINs), 92, 99–100
PANCREOX trial, 103
Papillary thyroid cancer, 158–159, 164–165
Paraganglioma (PGL), 156, 162
Paraneoplastic syndromes, 58
Parathyroid carcinomas, 157, 158, 163, 164
Parosteal osteosarcoma, 149, 153
Parotid gland cancer, 39, 44
“Passenger” mutations, 181, 191
Pazopanib, hepatotoxicity from, 150, 154
PC. See Pheochromocytoma (PC)
PCNSL. See Primary central nervous system lymphoma (PCNSL)
PCR. See Polymerase chain reaction (PCR)
PDGFR amplification, 24, 30
Pembrolizumab, 16–17, 20, 75, 80
Pemetrexed, 16–17, 20
Penile carcinoma, 125, 132
Peptide receptor radionuclide therapy (PRRT), 155–156, 161
Persistent grade 2 peripheral neuropathy, 15, 19
Peutz–Jeghers syndrome, 89
PGL (paraganglioma), 156, 162
Phase I trials, 8, 12
appropriate dose for further studies, 10, 13
primary purpose of, 9, 12
Phase II trials, 9, 12
response rate and survival endpoint in, 9, 13
Phase III trials, 9, 12
Pheochromocytoma (PC), 156, 162
familial syndromes in, 156, 162
PHPT (primary hyperparathyroidism), 157, 163
Plasmapheresis, 217, 228
Platinum-based chemotherapy, 16–17, 20
Pleural mesothelioma
chemotherapy regimens for, 52, 57
unresectable, 51, 57
Pneumonitis, 136–137, 143–144
POEMS syndrome, 216, 227–228
POLLUX trial, 224

https://t.me/ALGRAWANY33
Poly (ADP-ribose) polymerase (PARP) inhibitors, 65, 69
Polymerase chain reaction (PCR), 176, 187
based methods, 3, 6
Porphyria cutanea tarda, 96, 104
Positive direct antiglobulin (Coombs) test, 195
Preclinical studies, 14
Preclinical trial, 8, 12
Prednisone, 135, 141, 251, 255
Preexisting resistance mutation, 2, 5
Pregnancy, Hodgkin lymphoma during, 204, 210
Primary biliary cirrhosis, 96, 104
Primary central nervous system (CNS) tumor. See also Brain tumors; Gliomas
calcification on magnetic resonance imaging, 25, 31
environmental exposure increasing, 25, 31
with Epstein–Barr virus infection, 25, 31
Epstein–Barr virus infection and, 25, 31
magnetic resonance imaging, 25, 31
malignant adult, 25, 31
radiation in, 25, 31
radiographic changes, 25, 31
Primary central nervous system lymphoma (PCNSL), 203, 209
methotrexate and rituximab for, 204, 209–210
Primary CNS lymphoma, 25, 31
Primary effusion lymphoma, 200, 207
Primary hyperparathyroidism (PHPT), 157, 163
Primary mediastinal large B-cell lymphoma, 205, 210
R-EPOCH therapy for, 205, 210
Primary peritoneal cancer, 170, 174
Principle of self-seeding, 3, 6
PRODIGE 24-ACCORD trial, 101
Programmed death-ligand 1 (PD-L1) expression, 16–17, 20
Prophylactic corticosteroids, for fluid retention, 19
Prophylactic cranial irradiation, 28, 34
Prostate cancer. See also Castrate-resistant prostate cancer (CRPC)
adjuvant radiotherapy in, 124, 130
androgen ablation and risk of osteoporosis, 124–125, 131
androgen ablation in, 123, 129
androgen receptor and, 125, 132
antiandrogen-withdrawal response, 123, 130
chromosomal translocations involving TMPRSS2 in, 121, 128
definitive local therapy, 124–125, 131
high-fat diet and, 122–123, 129
high-risk features and therapy, 123, 129
metastatic castrate-resistant, -associated bone pain, 236, 242
pain management in, 238–239, 245
predictors of cancer on biopsy, 123, 129
PTEN genetic alterations in, 125, 132
risk factors of, 125, 131
robotic laparoscopic prostatectomy in, 124, 130
Prostate, lung, colorectal, and ovarian (PLCO) Cancer Screening Trial, 232
Prostate-specific antigen (PSA) testing, 232
Proteasome inhibitors, 214, 224
Protein-coding genome, 1, 4
Proton pump inhibitors, 19
PRRT (peptide receptor radionuclide therapy), 155–156, 161
PSA (prostate-specific antigen) testing, 232
Pseudo-progression, postchemoradiation, 25, 31
Pseudo-response, 31
Psychic pain, 248
Pulmonary carcinoid tumors, 49, 55

Q
QTc prolongation, antiemetic for, 249, 253

R
RADIANT-4 study, 161
Radiation-induced sarcomas, 146, 151
Radiation Therapy Oncology Group (RTOG)
91-11, 41, 43
0522 trial, 43
1016 trial, 41–42
9704, 93, 100–101
Radical cystectomy, 122, 128
Radiofrequency ablation, 98, 106
Radiotherapy-induced nausea/vomiting (RINV), 250, 254
Radium-223, 236, 242
RAINBOW trial, 80
Ramucirumab, 75, 80
Randomization
“compliance” of, 8, 12
purpose of, 8, 12
Randomized clinical trials (RCTs), 8, 12
“blinding” in, 9, 12
designing, 10, 13
noncompliance in, 10, 13
RANK-ligand inhibitors, for pain associated with bone metastases, 64, 69
RAS proteins, 2, 5
Rasburicase, 257, 262, 264
RATIFY trial, 192
Rb gene, 23, 29
RB1 mutation, 1, 4
RCC. See Renal cell carcinoma (RCC)
RCTs. See Randomized clinical trials (RCTs)
REAL-2 trial, 80

https://t.me/ALGRAWANY33
Rectal cancer
metastatic, pain management in, 238, 244
staging of, 111, 116
T3+/N0-2, 111, 116
transanal excision, 111–112, 116
REFLECT phase III noninferiority study, 107
REGARD study, 80
Regorafenib, 84, 88
Renal cell carcinoma (RCC), 119, 126
ipilimumab plus nivolumab in, 120, 127
Memorial Sloan Kettering Cancer Center risk factors, 120, 126
orthopedic resection of, 120, 126
prognostic factors in, 120, 126
recurrent metastatic disease, 120, 126
temsirolimus in, 120, 126
Renal pelvis urothelial cancers, 122, 129
Resistance mutation, pre-existing, 2, 5
RESONATE2 trial, 195
Retinoblastoma, 146, 151
Rhabdomyosarcoma, 18, 22
Ribociclib, 15, 19
RINV (radiotherapy-induced nausea/vomiting), 250, 254
Rituximab, 201, 207–208
Romidepsin, 17, 21
RTOG. See Radiation Therapy Oncology Group (RTOG)

S
Sacituzumab govitecan, 16, 20
Salivary gland cancers
expression of androgen receptor in, 40, 45
histologic types, 45
Sarcomas. See also specific sarcomas
Ewing sarcoma, and cytogenetic abnormalities, 146, 151
radiation-induced, 146, 151
sarcoma-specific mortality, 147, 151
SCC. See Squamous cell carcinoma (SCC)
Schistosoma haematobium, bladder cancer by, 124, 130
SCLC. See Small cell lung cancer (SCLC)
Scopolamine, 250, 254
SEER (Surveillance, Epidemiology, and End Results), 96, 104
Selection bias, 230, 233–234
Self-seeding, principle of, 3, 6
Selinexor, 215, 225
Selpercatinib, 16, 20
Seminoma, 53, 58
Serotonin-norepinephrine reuptake inhibitor (SNRI), 237, 243
Serous cystadenocarcinoma of the ovary, 3, 7
Sex cord–stromal tumors, 173
SHH pathway, 23, 29
SIADH (syndrome of inappropriate antidiuretic hormone), 53, 58, 259–260, 265
Siewert type I cancer, 72, 77
SIRIUS trial, 224
Sjögren disease, 89
Skin cancer, 195
screening recommendations, 232, 235
Small bowel tumor, 84, 85, 89–90
Small-cell lung cancer (SCLC)
bone marrow examination, 53, 59
chemotherapy in, 52, 58
combination chemotherapy for, 256, 261
genomic alterations in, 53, 58
genomic characterization of, 50, 56
limited-stage, 28, 34, 48, 55, 260, 266
location for primary tumor, 256, 261
neurosurgical consultation for external ventriculostomy, 260, 266
PCI in extensive-stage, 52, 58
platinum-etoposide chemotherapy plus immunotherapy in, 52, 57
and SVC syndrome, 256, 261
Smoldering myeloma, 212, 219
SMZL (splenic marginal zone lymphoma), 201, 207–208
SN-38, and neutropenia, 111, 116
SNRI (serotonin-norepinephrine reuptake inhibitor), 237, 243
Sodium thiosulfate, 16, 19
Soft-tissue sarcomas, 146, 151
of head and neck, 147, 152
locally recurrent, 148, 153
metastatic, 148, 153
chemotherapy treatment of, 148, 153
nonmetastatic, disease-specific survival, 148, 152
sites of carries, 150, 154
staging of, 147, 151
SOFT trial, 68
SOLAR-1 trial, 69
Somatic coding alterations, range per cancer genome, 1, 4
Somatic gene alteration in glioblastoma, 24, 30
Somatic mutations, 24, 30
Somatostatin receptor imaging, 156, 162
Sorafenib, 28, 34
Splenic marginal zone lymphoma (SMZL), 201, 207–208
Squamous cell carcinoma (SCC)
cutaneous, 137, 145
head and neck. See Head and neck squamous cell carcinoma (HNSCC)
as toxicity of BRAF inhibition, 136, 143
STAMINA study, 222–223
Stem cell transplantation. See specific entries

https://t.me/ALGRAWANY33
Stereotactic proton-based radiation therapy (RT), 29
STK11 gene, 84, 89
STOMP trial, 103, 225
Succinate dehydrogenase (SDH)-deficient GIST, 84, 88
Sunitinib, 180, 191
for alveolar soft-part sarcoma, 150, 154
associated hypothyroidism, 88
Super-enhancer, 23, 29
Superior vena cava (SVC), 259, 264
Superiority trials, 9, 12, 13
Surveillance, Epidemiology, and End Results (SEER), 96, 104
Suspected solitary bone plasmacytoma, 218–219
SVC (superior vena cava), 259, 264
SWOG S0777 trial, 213, 221
SWOG/BMT CTN trial, 224
Sympathetic block, for pain management, 243–244
Symptomatic hypercalcemia, 258, 263
Syndrome of inappropriate antidiuretic hormone (SIADH), 53, 58, 259–260, 265
Synovial sarcoma, 151

T
T-cell lymphoma
cutaneous, 208, 209
pain management in, 238, 244
peripheral, 17, 21
T cells, 3, 6
TACE (transarterial chemoembolization), 106
Talimogene laherparepvec (T-VEC), 136, 142
Tamoxifen, 19
for breast cancer, 63, 68
TAMs (tumor-associated macrophages), 3, 6
Telotristat etiprate, 164
Temozolomide (TMZ), 26, 27, 32
TEXT trial, 68
Thrombotic microangiopathy, 264–265
Thymoma, postoperative radiotherapy in, 53, 58
Thyroid function tests, 83, 88
Thyroid lymphoma, 160, 166
Tisagenlecleucel, 183, 194
TKIs. See Tyrosine kinase inhibitors (TKIs)
TLS. See Tumor lysis syndrome (TLS)
TMZ (temozolomide), 26, 27, 32
Tocilizumab, 21
Tofacitinib, 28, 34
TOURMALINE-AL1 trial, 227
TOWER trial, 194
TP53 gene
mutation, 1, 4, 23, 24, 29, 30
low-grade gliomas, 26, 32
urothelial carcinoma of bladder, alterations in, 122, 129
variant with twofold increase in risk, 25, 30
Tramadol, 245
Transarterial chemoembolization (TACE), 106
Transforming growth factor (TGF)-β, 23, 29
Trastuzumab, 73, 78
risk factors for cardiac dysfunction with, 63, 68
TSC (tuberous sclerosis complex), 28, 34
TTFs. See Tumor-treating fields (TTFs)
Tuberous sclerosis complex (TSC), 28, 34
Tumor-associated macrophages (TAMs), 3, 6
Tumor-derived cytokines, 3, 6
Tumor lysis syndrome (TLS), 195, 262
criteria for, 263–264
incidence of, 264
Tumor necrosis, inflammation from, 2, 5
Tumor suppressor genes, 1, 4
genetic variants of, 25, 30
mutation of, 23, 29
Tumor-treating fields (TTFs), 27, 33
adverse side effects of, 27, 33
factors affecting efficacy of, 27, 33
Turcot syndrome, 84, 88
20/2/20 score, 219
Tyrosine kinase inhibitors (TKIs), 159, 165
imatinib vs., 185, 195

U
UK-ABC-02 trial, 107
Ulceration of primary lesion, 134, 139
Ultrasonography, 96, 104
Undifferentiated pleomorphic sarcoma (UPS), 149, 153
Urate oxidase, 262
Urethral cancers, male, risk factor for, 121, 128

V
VAC (vincristine, doxorubicin, and cyclophosphamide), 149, 154
Vemurafenib, 143
Veterans Administration Laryngeal Study Group Trial, 43
VHL (Von Hippel–Lindau) disease, 28, 34, 120, 127
VHL gene, 120, 127
Villous lymphocytes, 201, 207–208
Vinca alkaloids extravasation, 16, 19
Vincristine, 16, 19

https://t.me/ALGRAWANY33
Viral hepatitis, chronic, 104
Vismodegib, 137, 144
Von Hippel–Lindau (VHL) disease, 28, 34, 120, 127

W
Waldenstrom macroglobulinemia (WM), 217, 228
Warm compress, for vinca alkaloid extravasation, 16, 19
Watchful waiting, 28, 33
WBRT (whole-brain radiotherapy), 28, 34, 142
Wheezing, carcinoid-associated, 158, 164
Whole-body MRI, 211–212, 218–219
Whole-brain radiotherapy (WBRT), 28, 34, 142
Wilson disease, 96, 104
WM (Waldenstrom macroglobulinemia), 217, 228
Wong-Baker Faces Pain Rating Scale, 246

X
Xerostomia, 39, 44

Z
ZES (Zollinger–Ellison syndrome), 163
Zoledronic acid, 236, 242
Zollinger–Ellison syndrome (ZES), 163

You might also like